Sei sulla pagina 1di 189

CONFLICT

 OF  LAWS  DIGESTS  AND  NOTES1   the  action  of  the  claimants.  
 
ATTY.  ARIS  L.  GULAPA   Condition  No.  14  subverts  the  public  policy  on  transfer  of  venue  of  proceedings  
  of  this  nature,  since  the  same  will  prejudice  rights  and  interests  of  innumerable  
  passengers  from  different  places  of  the  country  who,  under  Condition  No.  14,  
IV.  JURISDICTION  AND  CHOICE  OF  LAW   will  have  to  file  suits  against  Sweet  Lines  only  in  the  City  of  Cebu.  
   
  ALG:  But  the  lawyers  are  neither  of  low  income  nor  less  literate.  
GR:  A  state  does  not  have  jurisdiction  in  the  absence  of  some  reasonable  basis    
for   exercising   it,   whether   the   proceedings   are   in   rem,   quasi   in   rem,   or   in   FACTS:  
personam.  To  be  reasonable,  the  jurisdiction  must  be  based  on  some  minimum   Respondents   Atty.   Leovigildo   Tandog   and   Rogelio   Tiro,   bought   tickets   for  
contacts   that   will   not   offend   traditional   notions   of   fair   play   and   substantial   Voyage   90   at   the   branch   office   of   petitioner,   Sweet   Lines   Inc.,   a   shipping  
justice.   company  transporting  inter-­‐island  passengers  and  cargoes,  at  Cagayan  de  Oro  
  City.   Tandog   and   Tiro   were   to   board   Sweet   Lines’   vessel,   M/S   "Sweet   Hope"  
EXC   1:   Principle   of   Forum   Non   Conveniens—A   court,   even   though   it   has   bound  for  Tagbilaran  City  via  the  port  of  Cebu.  
jurisdiction,   will   not   entertain   the   suit   if   it   believes   itself   to   be   seriously    
inconvenient   forum,   provided   that   a   more   convenient   forum   is   available   to   Upon   learning   that   the   vessel   was   not   proceeding   to   Bohol   (since   many  
plaintiff.   passengers  were  bound  for  Surigao),  Tandog  and  Tiro,  per  advice,  went  to  the  
  branch   office   for   proper   relocation   to   M/S   "Sweet   Town".   Because   the   said  
*Most  convenient  forum:     vessel  was  already  filled  to  capacity,  they  were  forced  to  agree  "to  hide  at  the  
1.  The  parties  may  conveniently  resort  to  it   cargo  section  to  avoid  inspection  of  the  officers  of  the  Philippine  Coastguard."  
2.  It  is  in  a  position  to  make  an  intelligent  decision  as  to  the  law  and  the  facts    
3.   It   has   or   is   likely   to   have   the   power   to   enforce   its   decision   (principle   of   Tandog   and   Tiro   alleged   that   they   were   exposed   to   the   scorching   heat   of   the  
effectiveness)   sun  and  the  dust  coming  from  the  ship's  cargo  of  corn  grits  during  the  trip  and  
  that   the   tickets   they   bought   at   Cagayan   de   Oro   City   for   Tagbilaran   were   not  
  honored  and  they  were  constrained  to  pay  for  other  tickets.  Hence,  they  sued  
CHOICE  OF  FORUM   Sweet  Lines  for  damages  and  for  breach  of  contract  of  carriage  in  the  alleged  
  sum  of  P10,000.00  before  CFI  of  Misamis  Oriental.  
   
1.  SWEET  LINES  INC.  V.  TEVES   Sweet  Lines  moved  to  dismiss  the  complaint  on  the  ground  of  improper  venue  
  based  on  the  condition  printed  at  the  back  of  the  tickets:  14.  It  is  hereby  agreed  
DOCTRINES:   GR:   Venue   may   be   changed   or   transferred   from   one   province   to   and   understood   that   any   and   all   actions   arising   out   of   the   conditions   and  
another  by  agreement  of  the  parties  in  writing,  based  on  Rule  4,  §3,  of  the  Rules   provisions   of   this   ticket,   irrespective   of   where   it   is   issued,   shall   be   filed   in   the  
of  Court.   competent   courts   in   the   City   of   Cebu.   The   motion   was   denied.   MR   was   filed   but  
  was   also   denied.   Hence,   this   instant   petition   for   prohibition   for   preliminary  
EXC   #1:   Such   an   agreement   will   not   be   held   valid   where   it   practically   negates   injunction,   'alleging   that   the   respondent   judge   Teves   has   departed   from   the  
accepted   and   usual   course   of   judicial   proceeding"   and   "had   acted   without   or   in  
                                                                                                                excess  or  in  error  of  his  jurisdiction  or  in  gross  abuse  of  discretion.  
1
    Digests  lifted  from  the  3C  2012-­‐2013  case  digests.    Edited  and  annotated  according    
to  class  discussion.  
 
CONFLICT  OF  LAWS                                                                                    AV  DE  TORRES   1  
ATTY.  ARIS  L.  GULAPA                                            AY  2015-­‐2016  
ISSUE:    
May   a   common   carrier   engaged   in   inter-­‐island   shipping   stipulate   thru   a   Also,   it   should   also   be   stressed   that   these   companies   are   franchise   holders   of  
condition   printed   at   the   back   of   passage   tickets   to   its   vessels   that   any   and   all   certificates   of   public   convenience   and   therefore,   posses   a   virtual   monopoly  
actions  arising  out  of  the  contract  of  carriage  should  be  filed  only  in  a  particular   over   the   business   of   transporting   passengers   between   the   ports   covered   by  
province  or  city,  in  this  case  the  City  of  Cebu,  to  the  exclusion  of  all  others?   their   franchise.   They   may   thus   dictate   their   terms   of   passage,   leaving  
  passengers  with  no  choice  but  to  buy  their  tickets  and  avail  of  their  vessels  and  
HELD:   facilities.  
There  was  a  valid  contract  of  carriage  entered  into  by  Sweet  Lines  and  Tandog    
and   Tiro,   the   best   evidence   thereof   is   the   passage   tickets.   All   the   essential   Lastly,   bulk   of   those   who   board   these   inter-­‐island   vessels   come   from   the   low-­‐
elements   of   a   valid   contract   (consent,   cause   or   consideration   and   object)   are   income   groups   and   are   less   literate,   and   who   have   little   or   no   choice   but   to  
present.   avail  of  petitioner's  vessels.  
   
However,  in  this  case,  with  respect  to  the  14  conditions  printed  at  the  back  of   2.   Condition   No.   14   subverts   the   public   policy   on   transfer   of   venue   of  
the  passage  tickets,  these  are  commonly  known  as  "contracts  of  adhesion,"— proceedings  of  this  nature,  since  the  same  will  prejudice  rights  and  interests  
certain  contracts  where  almost  all  the  provisions  of  which  have  been  drafted   of   innumerable   passengers   from   different   places   of   the   country   who,   under  
only   by   one   party—the   validity   and/or   enforceability   of   which   will   have   to   be   Condition   No.   14,   will   have   to   file   suits   against   Sweet   Lines   only   in   the   City   of  
determined   by   the   peculiar   circumstances   obtaining   in   each   case   and   the   Cebu.  
nature  of  the  conditions  or  terms  sought  to  be  enforced.    
  For,   although   venue   may   be   changed   or   transferred   from   one   province   to  
In   recognition   of   the   character   of   contracts   of   this   kind,   the   protection   of   the   another   by   agreement   of   the   parties   in   writing,   based   on   Rule   4,   §3,   of   the  
disadvantaged   is   expressly   enjoined   by   the   NCC:   Art.   24.   In   all   contractual   Rules  of  Court,  such  an  agreement  will  not  be  held  valid  where  it  practically  
property   or   other   relations,   when   one   of   the   parties   is   at   a   disadvantage   on   negates  the  action  of  the  claimants.  The  philosophy  underlying  the  provisions  
account   of   his   moral   dependence,   ignorance   indigence,   mental   weakness,   on  transfer  of  venue  of  actions  is  the  convenience  of  the  plaintiffs  as  well  as  his  
tender  age  and  other  handicap,  the  courts  must  be  vigilant  for  his  protection.   witnesses  and  to  promote  the  ends  of  justice.  
   
In   line   with   that,   the   court   ruled   that   Condition   No.   14   should   be   held   as   void   Considering  the  expense  and  trouble  a  passenger  residing  outside  of  Cebu  City  
and  unenforceable  for  the  following  reasons:   would  incur  to  prosecute  a  claim  in  the  City  of  Cebu,  he  would  most  probably  
  decide  not  to  file  the  action  at  all.  The  condition  will  defeat  the  ends  of  justice.  
1.   It   is   unjust   and   unfair   to   bind   passengers   to   the   terms   of   the   conditions    
printed  at  the  back  of  the  passage  tickets.   On  the  other  hand,  Sweet  Lines  has  branches  or  offices  in  the  respective  ports  
  of   call   of   its   vessels   and   can   afford   to   litigate   in   any   of   these   places.   Hence,   the  
There  is  an  acute  shortage  in  inter-­‐island  vessels  plying  between  the  country's   filing  of  the  suit  in  the  CFI  of  Misamis  Oriental  will  not  cause  inconvenience  or  
several   islands,   and   with   that,   the   facilities   they   offer   leave   much   to   be   prejudice  Sweet  Lines.  
desired,   thus,   passengers   literally   scramble   to   whatever   accommodations    
may  be  availed  of,  even  through  circuitous  routes,  and/or  at  the  risk  of  their   Public  policy  is  that  principle  of  the  law,  which  holds  that  no  subject  or  citizen  
safety  and  this  was  precisely  the  experience  of  Tandog  and  Tiro.  Under  these   can   lawfully   do   that   which   has   a   tendency   to   be   injurious   to   the   public   or  
circumstances,   it   is   hardly   just   and   proper   to   expect   the   passengers   to   against  the  public  good.  Under  this  principle,  the  freedom  of  contract  or  private  
examine   their   tickets   for   conditions   that   may   be   printed   much   charge   them   dealing  is  restricted  by  law  for  the  good  of  the  public.  
with  having  consented  to  the  conditions,  so  printed,  especially  if  there  are  a    
number  of  such  conditions  in  fine  print,  as  in  this  case.  
 
CONFLICT  OF  LAWS                                                                                    AV  DE  TORRES   2  
ATTY.  ARIS  L.  GULAPA                                            AY  2015-­‐2016  
2.  HSBC  V.  SHERMAN   over   all   disputes   arising   under   this   guarantee.”   The   company   failed   to   pay   its  
G.R.  No.  72494  |  August  11,  1989   obligation.   Thus,   HSBC   demanded   payment   but   for   failure   to   pay,   HSBC   filed  
  the  above-­‐mentioned  complaint.  
Doctrines:  GR:  Choice  of  forum  should  govern.    
EXC   #2:   When   due   process   demands   that   the   choice   of   forum   be   liberally   Private  respondents  filed  a  MTD  on  the  ground  of  lack  of  jurisdiction  over  the  
construed.   subject   matter   and   over   their   persons.   The   trial   court   denied   the   MTD,   ruling  
  that  there  is  nothing  in  the  Guarantee  which  says  that  the  courts  of  Singapore  
A  State  does  not  have  jurisdiction  in  the  absence  of  some   reasonable  basis  for   shall   have   jurisdiction   to   the   exclusion   of   the   courts   of   other   countries   or  
exercising   it,   whether   the   proceedings   are   in   rem,   quasi   in   rem,   or   in   personam.   nations   and   that   jurisdiction   over   the   persons   of   defendants   is   acquired   by  
To   be   reasonable,   the   jurisdiction   must   be   based   on   some   minimum   contacts   service  of  summons  and  copy  of  the  complaint  on  them.  There  has  been  a  valid  
that  will  not  offend  traditional  notions  of  fair  play  and  substantial  justice.   service  of  summons  on  both  defendants  and  in  fact  the  same  is  admitted  when  
  said   defendants   filed   a   'Motion   for   Extension   of   Time   to   File   Responsive  
The  defense  of  private  respondents  that  the  complaint  should  have  been  filed  in   Pleading.’   Private   respondents   then   filed   before   CA   a   petition   for   prohibition  
Singapore   is   based   merely   on   technicality.   They   did   not   even   claim,   much   less   with  preliminary  injunction  and/or  prayer  for  a  restraining  order.  CA  rendered  a  
prove,   that   the   filing   of   the   action   here   will   cause   them   any   unnecessary   decision   granting   the   injunction   stating   that:   “The   loan   was   obtained   by   a  
trouble,  damage,  or  expense.  On  the  other  hand,  there  is  no  showing  that  HSBC   company   incorporated   in  Singapore.   The   loan   was   granted   by   the  
filed  the  action  here  just  to  harass  private  respondents.   Singapore  Branch  of  HSBC.  The  Joint  and  Several  Guarantee  was  also  concluded  
  in  Singapore.   The   loan   was   in   Singaporean   dollars   and   the   repayment   thereof  
Av:  Respondent  must  allege  and  prove  that  the  filing  of  the  action  in  the  forum   also   in   the   same   currency.   The   transaction,   to   say   the   least,   took   place   in  
1.   will   cause   unnecessary   trouble,   damage,   or   expense   on   his   part   and   that   2.   it   Singporean   setting   in   which   the   law   of   that   country   is   the   measure   by   which  
was  filed  to  harass  him.   that  relationship  of  the  parties  will  be  governed.”  
   
FACTS:   ISSUE:  
A   complaint   for   collection   of   a   sum   of   money   was   filed   by   Hongkong   and   Whether  Philippine  courts  have  jurisdiction  over  the  suit.  
Shanghai  Banking  Corporation  (HSBC)  against  private  respondents  Jack  Robert    
Sherman  and  Deodato  Reloj,  before  the  RTC  QC.   HELD:  
  Yes.  While  it  is  true  that  "the  transaction  took  place  in  Singaporean  setting"  and  
In   1981,   Eastern   Book   Supply   Service   PTE,   Ltd.   (company),   a   company   that   the   Joint   and   Several   Guarantee   contains   a   choice-­‐of-­‐forum   clause,   the  
incorporated   in   Singapore   applied   with,   and   was   granted   by,   the   Singapore   very   essence   of   due   process   dictates   that   the   stipulation   be   liberally  
branch   of   HSBC   an   overdraft   facility.   As   a   security   for   the   repayment   of   sums   construed.   One   basic   principle   underlies   all   rules   of   jurisdiction   in   International  
advanced   through   the   aforesaid   overdraft   facility,   private   respondents   and   a   Law:   A   State   does   not   have   jurisdiction   in   the   absence   of   some   reasonable  
certain   Robin   de   Clive   Lowe,   all   of   whom   were   directors   of   the   company,   basis  for  exercising  it,  whether  the  proceedings  are  in  rem  quasi  in  rem  or  in  
executed  a  Joint  and  Several  Guarantee  in  favor  of  HSBC  whereby   the   former   personam.   To   be   reasonable,   the   jurisdiction   must   be   based   on   some  
agreed   to   pay,   jointly   and   severally,   on   demand   all   sums   owed   by   the   minimum   contacts   that   will   not   offend   traditional   notions   of   fair   play   and  
COMPANY   to   petitioner   BANK   under   the   aforestated   overdraft   facility.   The   substantial  justice.  
Joint   and   Several   Guarantee   provides   that:   “This   guarantee   and   all   rights,    
obligations  and  liabilities  arising  hereunder  shall  be  construed  and  determined   The   instant   case   presents   a   very   odd   situation.   In   the   ordinary   habits   of   life,  
under   and   may   be   enforced   in   accordance   with   the   laws   of   the   Republic   of   anyone   would   be   disinclined   to   litigate   before   a   foreign   tribunal,   with   more  
Singapore.   We   hereby   agree   that   the   Courts   of   Singapore   shall   have   jurisdiction   reason   as   a   defendant.   However,   in   this   case,   private   respondents   are  
Philippine   residents   (a   fact   which   was   not   disputed   by   them)   who   would   rather  
 
CONFLICT  OF  LAWS                                                                                    AV  DE  TORRES   3  
ATTY.  ARIS  L.  GULAPA                                            AY  2015-­‐2016  
face  a  complaint  against  them  before  a  foreign  court  and  in  the  process  incur   Although   the   Joint   and   Several   Guarantee   prepared   by   HSBC   is   a   contract   of  
considerable   expenses,   not   to   mention   inconvenience,   than   to   have   a   adhesion   and   that   consequently,   it   cannot   be   permitted   to   take   a   stand  
Philippine  court  try  and  resolve  the  case.  The   defense   of   private   respondents   contrary   to   the   stipulations   of   the   contract,   substantial   bases   exist   for   HSBC’s  
that   the   complaint   should   have   been   filed   in   Singapore   is   based   merely   on   choice  of  forum,  as  discussed  earlier.  
technicality.  They   did   not   even   claim,   much   less   prove,  that  the  filing  of  the    
action  here  will  cause  them  any  unnecessary  trouble,  damage,  or  expense.  On    
the   other   hand,   there   is   no   showing   that   HSBC   filed   the   action   here   just   to  
CHOICE  OF  LAW  
harass  private  respondents.    
   
Further,   the   parties   did   not   thereby   stipulate   that   only   the   courts   of   3.  BELLIS  V.  BELLIS  
Singapore,   to   the   exclusion   of   all   the   rest,   has   jurisdiction.   Neither   did   the   G.R.  No.  L-­‐23678  |  June  6,  1967  
clause   in   question   operate   to   divest   Philippine   courts   of   jurisdiction.   In    
International   Law,   jurisdiction   is   often   defined   as   the   right   of   a   State   to   DOCTRINE:   Whatever   public   policy   or   good   customs   may   be   involved   in   our  
exercise   authority   over   persons   and   things   within   its   boundaries   subject   to   System   of   legitimes,   Congress   has   not   intended   to   extend   the   same   to   the  
certain   exceptions.   Thus,   a   State   does   not   assume   jurisdiction   over   travelling   succession  of  foreign  nationals.  Article  16,  a  specific  provision,  must  prevail  over  
sovereigns,   ambassadors   and   diplomatic   representatives   of   other   States,   and   Article  17,  a  general  one.  
foreign  military  units  stationed  in  or  marching  through  State  territory  with  the    
permission   of   the   latter's   authorities.   This   authority,   which   finds   its   source   in   Av:  Thus,  even  if  under  Article  17,  the  prohibitive  law  on  deprivation  of  legitimes  
the   concept   of   sovereignty,   is   exclusive   within   and   throughout   the   domain   of   cannot  be  rendered  null  or  ineffective  by  laws  promulgated  in  Texas,  Article  16  
the  State.  A  State  is  competent  to  take  hold  of  any  judicial  matter  it  sees  fit  by   (which  provides  that  CIAO  should  be  governed  by  the  national  law  of  the  person  
making   its   courts   and   agencies   assume   jurisdiction   over   all   kinds   of   cases   whose  succession  is  under  consideration)  should  prevail.  
brought  before  them.    
  FACTS:  
While   in   the   main,   the   motion   to   dismiss   fails   to   categorically   use   with   Amos  Bellis  was  a  citizen  and  resident  of  Texas  at  the  time  of  his  death.         He  
exactitude   the   words   'improper   venue'   it   can   be   perceived   from   the   general   had  5  legitimate  children  with  his  first  wife,  Mary  Mallen,  whom  he  divorced.  
thrust   and   context   of   the   motion   that   what   is   meant   is   improper   venue,   The   He  had  3  legitimate  daughters  with  his  second  wife,  Violet,  who  survived  him,  
use  of  the  word  'jurisdiction'  was  merely  an  attempt  to  copy-­‐cat  the  same  word   and   another   3   illegitimate   children   with   another   woman.   Before   he   died,   he  
employed   in   the   guarantee   agreement   but   conveys   the   concept   of   venue.   At   executed   2   wills—the   first   one   disposing   of   his   Texas   properties,   the   other  
any  rate,  this  issue  is  now  of  no  moment  because  the  Court  holds  that  venue   disposing   his   Philippine   properties.  In   his   will,   which   he   executed   in   the  
here  was  properly  laid  for  the  same  reasons  discussed  above.   Philippines,   he   directed   that   after   all   taxes,   obligations,   and   expenses   of  
  administration  are  paid  for,  his  distributable  estate  should  be  divided,  in  trust,  
The   respondent   Court   likewise   ruled   that:   “In   a   conflict   problem,   a   court   will   in  the  following  order  and  manner:  a)  $240,000.00  to  his  first  wife  Mary  Mallen  
simply   refuse   to   entertain   the   case   if   it   is   not   authorized   by   law   to   exercise   b)  $120,000.00  to  his  three  illegitimate  children  Amos  Bellis,  Jr.,  Maria  Cristina  
jurisdiction.   And   even   if   it   is   so   authorized,   it   may   still   refuse   to   entertain   the   Bellis,  Miriam  Palma  Bellis,  or  $40,000.00  each,  and  c)  After  foregoing  the  two  
case  by  applying  the  principle  of  forum  non  conveniens.”  However,  whether  a   items  have  been  satisfied,  the  remainder  shall  go  to  his  seven  surviving  children  
suit  should  be  entertained  or  dismissed  on  the  basis  of  the  principle  of  forum   by  his  first  and  second  wives.  
non   conveniens   depends   largely   upon   the   facts   of   the   particular   case   and   is    
addressed   to   the   sound   discretion   of   the   trial   court.   Thus,   the   CA   should   not   Maria  Cristina  Bellis  and  Miriam  Palma  Bellis,  filed  their  respective  oppositions  
have  relied  on  such  principle.   to   the   project   of   partition   on   the   ground   that   they   were   deprived   of   their  
 
 
CONFLICT  OF  LAWS                                                                                    AV  DE  TORRES   4  
ATTY.  ARIS  L.  GULAPA                                            AY  2015-­‐2016  
legitimes   as   illegitimate   children   and,   therefore,   compulsory   heirs   of   the   However,   it   is   evident   that   whatever   public   policy   or   good   customs   may   be  
deceased.   involved  in  our  System  of  legitimes  (that  is  the  public  policy  that  a  compulsory  
  heir  cannot  be  deprived  of  his  legitime),  Congress  has  not  intended  to  extend  
The  lower  court  issued  an  order  overruling  the  oppositions  and  approving  the   the  same  to  the  succession  of  foreign  nationals.  For  it  has  specifically  chosen  
executor’s   final   account,   report   and   administration,   and   project   of   partition.   to   leave,  inter   alia,   the  amount  of   successional   rights,   to   the   decedent's  
Relying   upon   Art.   16   of   the   Civil   Code,   it   applied   the   national   law   of   the   national  law.  Specific  provisions  must  prevail  over  general  ones.  A  provision  in  
decedent,  which  in  this  case  did  not  provide  for  legitimes.   a   foreigner's   will   to   the   effect   that   his   properties   shall   be   distributed   in  
  accordance  with  Philippine  law  and  not  with  his  national  law,  is  illegal  and  void,  
ISSUE:   for   his   national   law   cannot   be   ignored   in   regard   to   those   matters   that   Article  
Whether  such  illegitimate  children  of  Bellis  be  entitled  to  successional  rights   10—now  Article  16—of  the  Civil  Code  states  said  national  law  should  govern.  
   
HELD:   4.  TAYAG  V.  BENGUET  CONSOLIDATED  
No.   The   said   illegitimate   children   are   not   entitled   to   their   legitimes.   Under    
Texas   law,   there   are   no   legitimes.   Even   if   the   other   will   was   executed   in   the   DOCTRINE:  Benguet  Consolidated  is  a  domestic  corp.  subject  to  the  unrestricted  
Philippines,  his  national  law,  still,  will  govern  the  properties  for  succession  even   jurisdiction  of  the  local  courts.  Its  shares  of  stocks  are  not  immune  from  lawful  
if  it  is  stated  in  his  testate  that  it  shall  be  governed  by  the  Philippine  law.   court   orders.   The   actual   situs   of   shares   of   stock   is   in   the   Philippines,   the  
  corporation  being  domiciled  here.  
Art.16,  Par.  2  renders  applicable  the  national  law  of  the  decedent,  in  intestate    
and   testamentary   successions,   with   regard   to   four   items:   (a)   the   order   of   FACTS:  
succession,   (b)   the   amount   of   successional   rights,   (c)   the   intrinsic   validity   of   Idonah   Perkins   died   in   New   York   and   left   stock   certificates   covering   32,000  
provisions  of  will,  and  (d)  the  capacity  to  succeed.   shares   of   Benguet   Consolidated   under   the   possession   of   County   Trust   Co.   of  
  New  York,  the  domiciliary  administrator  of  the  estate.  
ART.16  Real  property  as  well  as  personal  property  is  subject  to  the  law  of    
the  country  to  where  it  is  situated.   Ancillary   administration   proceedings   were   instituted   in   Manila   and   Renato  
  Tayag  was  later  appointed  as  ancillary  administrator.  A  dispute  arose  between  
However,   intestate   and   testamentary   successions,   both   with   respect   to   the   domiciliary   and   ancillary   administrator   as   to   who   was   entitled   to   the  
the  order  of  successions  and  to  the  amount  of  successional  rights  and  to   possession   of   the   stock   certificates.   So   that   claims   of   Perkins’   local   creditors  
the   intrinsic   validity   of   testamentary   provisions,   shall   be   regulated   by   the   could   be   satisfied,   the   lower   court   ordered   the   domiciliary   administrator   to  
national   law   of   the   person   whose   succession   is   under   consideration,   surrender   the   stock   certificates   but   the   domiciliary   refused   to   comply.   This  
whatever   may   be   the   nature   of   the   property   and   regardless   of   the   prompted   the   ancillary   administrator   to   petition   to   the   court   that   the   stock  
country  wherein  said  property  may  be  found.   certificates   be   considered   lost   and   cancelled,   and   that   Benguet   be   ordered   to  
  issue   new   certificates   to   it.   The   CFI   granted   the   petition.   From   such   order,  
As   a   counter-­‐argument,   appellants   invoked   Art.   17,   Par.   3,   of   the   Civil   Code,   Benguet   Consolidated   appealed   contending   that   it   cannot   be   declared   lost  
stating—   because  they  are  in  actual  existence  and  is  in  the  possession  of  Country  Trust  in  
  New  York.  
Prohibitive   laws   concerning   persons,   their   acts   or   property,   and   those    
which  have  for  their  object  public  order,  public  policy  and  good  customs   ISSUE:  
shall   not   be   rendered   ineffective   by   laws   or   judgments   promulgated,   or   Whether  the  lower  court  had  the  power  to  issue  the  questioned  order?  
by  determinations  or  conventions  agreed  upon  in  a  foreign  country.    
 
 
CONFLICT  OF  LAWS                                                                                    AV  DE  TORRES   5  
ATTY.  ARIS  L.  GULAPA                                            AY  2015-­‐2016  
HELD:   contacts   between   Phil.   law   and   Phil.   courts   on   the   one   hand,   and   the  
Yes.  The  power  of  the  ancillary  administrator  to  gain  control  and  possession  of   relationship  between  the  parties  on  the  other.  
all   the   assets   of   the   decedent   within   the   jurisdiction   of   the   Philippines   is    
undisputed.  The  administration  extends  to  all  assets  of  the  decedent  within  the   FACTS:  
state   where   it   was   granted   and   the   administrator   of   one   state   has   no   power   Pakistan   Intl   Airlines   (PIA)   executed   2   separate   contracts   of   employments   in  
over  assets  in  another  state.   Manila,  one  with  Farrales  and  the  other  with  Mamasig.  The  pertinent  portions  
  of  the  contract  state  that  (1)  the  agreement  is  for  a  period  of  3  years,  but  can  
The   court   has   the   authority   to   require   that   the   ancillary   administrator’s   be  extended  by  the  mutual  consent  of  the  parties;  (2)  notwithstanding  anything  
authority  over  the  stock  certificates  be  respected.  Benguet  Consolidated  is  a   to   contrary   as   herein   provided,   PIA   reserves   the   right   to   terminate   this  
domestic  corp.  subject  to  the  unrestricted  jurisdiction  of  the  local  courts.  Its   agreement   at   any   time   by   giving   the   EMPLOYEE   notice   in   writing   in   advance  
shares  of  stocks  are  not  immune  from  lawful  court  orders.  The  actual  situs  of   one   month   before   the   intended   termination   or   in   lieu   thereof,   by   paying   the  
shares  of  stock  is  in  the  Philippines,  the  corporation  being  domiciled  here.   EMPLOYEE  wages  equivalent  to  one  month's  salary;  (3)  this  agreement  shall  be  
  construed   and   governed   under   and   by   the   laws   of   Pakistan,   and   only   the   Courts  
Since   the   domiciliary   persistently   refuses   to   deliver   that   owned   by   the   of   Karachi,   Pakistan   shall   have   the   jurisdiction   to   consider   any   matter   arising  
decedent   to   the   ancillary,   there   was   nothing   arbitrary   in   considering   them   as   out  of  or  under  this  agreement.  
lost  and  requiring  issuance  of  new  certificates  in  lieu  thereof.    
  1  year  and  4  months  before  the  expiration  of  the  contracts  of  employment,  PIA  
Benguet’s   contention   that   its   by-­‐laws   should   be   followed   requiring   a   final   court   sent  separate  letters  to  Mamasig  and  Farrales,  advising  them  that  their  services  
resolution   on   ownership   first   before   issuance   of   a   new   certificate   is   also   as   flight   stewardesses   would   be   terminated.   Farrales   and   Mamasig   filed   a  
without   merit.   There   is   no   question   of   ownership   since   Country   Trust   did   not   complaint   for   illegal   dismissal   and   non-­‐payment   of   company   benefits   and  
even   appeal.   Even   assuming   it   did,   the   command   of   a   court   decree   prevails   bonuses.   PIA   contended   that   F   &   M   were   habitual   absentees   and   had   the   habit  
over  a  by-­‐law.   of  bringing  in  from  abroad  sizeable  quantities  of  personal  effects.  
   
5.  PAKISTAN  INTERNATIONAL  AIRLINES  V.  OPLE   ISSUE:  
  Whether   the   provisions   in   the   contract   that   the   agreement   shall   be   governed  
CHOICE  OF  LAW   by  the  laws  of  Pakistan  and  that  only  the  courts  of  Karachi,  Pakistan  shall  have  
  jurisdiction   over   any   controversy   arising   out   of   the   agreement,   may   be   given  
DOCTRINE:  GR:  Lex  loci  intentionis  governs.   effect  
   
EXC:  (1)  When  the  contract  is  a  labor  contract  because  it  is  imbued  with  public   HELD:  
interest   and   (2)   when   there   are   multiple   substantive   contacts   between   Phil.   No.  The  first  clause  cannot  be  invoked  to  prevent  the  application  of  Phil.  labor  
laws  and  courts  and  the  parties.   laws  and  regulations  to  the  subject  matter  of  the  case.  The  ER-­‐EE  relationship  
  between  PIA  and  F&M  is  affected  with  public  interest  and  the  applicable  Phil.  
The  first  clause  cannot  be  invoked  to  prevent  the  application  of  Phil.  labor  laws   laws  and  regulations  cannot  be  rendered  illusory  by  the  parties  agreeing  upon  
and   regulations   to   the   subject   matter   of   the   case.   The   ER-­‐EE   relationship   some  other  law  to  govern  their  relationship.  The  second  clause  cannot  also  be  
between   PIA   and   F&M   is   affected   with   public   interest   and   the   applicable   Phil.   invoked   because   the   circumstances   of   the   case   shows   multiple   substantive  
laws  and  regulations  cannot  be  rendered  illusory  by  the  parties  agreeing  upon   contacts   between   Phil.   law   and   Phil.   courts   on   the   one   hand,   and   the  
some  other  law  to  govern  their  relationship.  The  second  clause  cannot  also  be   relationship   between   the   parties   on   the   other:   contract   was   executed   and  
invoked   because   the   circumstances   of   the   case   shows   multiple   substantive   partially  performed  in  the  Phils.,  F&M  are  Filipino  citizens  and  PIA  is  licensed  to  

 
CONFLICT  OF  LAWS                                                                                    AV  DE  TORRES   6  
ATTY.  ARIS  L.  GULAPA                                            AY  2015-­‐2016  
do   business   in   the   Phils.,   and   F&M   were   based   in   the   Phils.   in   between   their   next  flight  to  LA,  the  mother  and  daughter  couldn’t  be  accommodated  because  
flights.  All  the  above  contacts  point  to  the  Philippine  courts  and  administrative   it   was   fully   book.   They   were   constrained   to   book   in   another   flight   and  
agencies  as  a  proper  forum  for  the  resolution  of  contractual  disputes  between   purchased  2  tickets  from  American  Airlines  at  $918.  
the   parties.   The   challenged   portion   of   the   employment   agreement   cannot   be    
given   effect   so   as   to   oust   Philippine   agencies   and   courts   of   the   jurisdiction   In   the   Philippines,   petitioners   filed   an   action   for   damages   based   on   breach   of  
vested   upon   them   by   Philippine   law.   Finally,   and   in   any   event,   PIA   did   not   contract   of   air   carriage   before   the   RTC   Makati.   The   RTC   ordered   the   airline  
undertake   to   plead   and   prove   the   contents   of   Pakistan   law   on   the   matter;   it   company  to  pay  the  ticket  costs,  as  well  as  moral  damages  and  attorney’s  fees.  
must   therefore   be   presumed   that   the   applicable   provisions   of   the   law   of    
Pakistan  are  the  same  as  the  applicable  provisions  of  Philippine  law.   However,  the  CA  held  that  moral  damages  are  recoverable  only  where  there  is  
  fraud   or   bad   faith   (in   a   breach   of   contract   of   carriage).   Since   it   is   a   matter   of  
As  to  the  dismissal  of  Farrales  &  Mamasig,  they  were  illegally  dismissed  and  are   record   that   overbooking   of   flights   is   a   common   and   accepted   practice   of  
entitled  to  3  years  backwages  without  qualification  or  deduction.  PIA’s  right  to   airlines   in   the   US   and   is   specifically   allowed   under   the   Code   of   Federal  
procedural  due  process  was  observed  as  it  was  given  the  opportunity  to  submit   Regulations   by   the   Civil   Aeronautics   Board,   no   fraud   nor   bad   faith   could   be  
a   position   paper   and   present   evidence.   Also,   the   provisions   of   the   employment   imputed  on  respondent  TWA.  TWA  was  remiss  in  not  informing  petitioners  that  
contract   must   not   be   contrary   to   law,   morals,   good   customs,   public   order,   the  flight  was  overbooked.  But  there  was  no  bad  faith  in  placing  the  petitioners  
public   policy.   The   employment   contract   prevents   security   of   tenure   of   F&M   in  the  waitlist  along  with  48  passengers.  
from  accruing.    
  ISSUE:  
6.  ZALAMEA  VS.  COURT  OF  APPEALS  AND  TRANSWORLD  AIRLINES   Whether   there   was   bad   faith   on   the   part   of   TWA,   considering   that   TWA  
G.R.  No.  104235  |  November  18,  1993   contends  that  overbooking  of  flights  is  a  common  and  accepted  practice  in  the  
  US  
DOCTRINE:  The  principle  of  lex  loci  contractus  which  require  that  the  law  of  the    
place  where  the  airline  ticket  was  issued  should  be  applied  by  the  court  where   HELD:  
(1)   the   passengers   are   residents   and   nationals   of   the   forum   and   (2)   the   ticket   is   Yes,   there   was   fraud   or   bad   faith.   The   US   law   or   regulation   allegedly  
issued  in  such  State  by  the  defendant  airline.     authorizing   overbooking   has   never   been   proved.   Foreign   laws   do   not   prove  
  themselves   nor   can   the   courts   take   judicial   notice   of   them.   Like   any   other  
FACTS:   fact,   they   must   be   alleged   and   proved.  Written  law  may  be  evidenced  by  an  
Petitioner-­‐spouses   Zalamea   and   their   daughter   purchased   3   airline   tickets   from   official  publication  thereof  or  by  a  copy  attested  by  the  officer  having  the  legal  
the  Manila  agent  of  TransWorld  Airlines  for  a  flight  to  NY-­‐LA.  The  tickets  were   custody  of  the  record,  or  by  his  deputy,  and  accompanied  with  a  certificate  that  
at  a  discount  of  75%  and  the  daughter  was  a  full  fare.   such   officer   has   custody.   The   certificate   may   be   made   by   a   secretary   of   an  
  embassy   or   legation,   consul   general,   consul,   vice-­‐consul,   or   consular   agent   or  
While   in   NY,   they   received   a   notice   of   the   reconfirmation.   On   the   appointed   by  any  officer  in  the  foreign  service  of  the  Philippines  stationed  in  the  foreign  
date,   they   checked   in   at   10am   for   their   11am   flight   but   were   placed   on   the   country  in  which  the  record  is  kept,  and  authenticated  by  the  seal  of  his  office.  
waitlist.   The   daughter   appeared   as   No.   13   on   the   waitlist   while   the   two    
Zalameas  were  listed  as  No.  34,  showing  a  party  of  two.  Out  of  the  42  names,   TWA  relied  solely  on  the  statement  of  its  customer  service  agent,  Ms.  Lather,  in  
the  first  22  names  were  eventually  allowed  to  board,  including  the  father.  The   her   deposition.   Aside   from   such,   there   is   no   official   publication   of   said   code  
others  weren’t  able  to  fly.  As  it  were,  those  holding  full-­‐fare  tickets  were  given   presented  as  evidence.  Respondent  court’s  finding  that  overbooking  is  allowed  
first  priority.  The  father  later  discovered  that  he  was  holding  his  daughter’s  full-­‐ has  no  basis.  
fare   ticket.   Those   with   discounted   tickets   were   denied   boarding.   Even   in   the    
Even   if   the   claimed   U.S.   Code   of   Federal   Regulations   exist,   the   same   isn’t  
 
CONFLICT  OF  LAWS                                                                                    AV  DE  TORRES   7  
ATTY.  ARIS  L.  GULAPA                                            AY  2015-­‐2016  
applicable   in   accordance   with   the   principle   of   lex   loci   contractus   which    
require  that  the  law  of  the  place  where  the  airline  ticket  was  issued  should  be   Aniceto   claims   that   upon   their   arrival   at   the   LA   Airport   for   their   flight,   they  
applied  by  the  court  where  the  passengers  are  residents  and  nationals  of  the   proceeded   to   United's   counter   where   they   were   attended   by   Linda.   Linda  
forum  and  the  ticket  is  issued  in  such  State  by  the  defendant  airline.  Since  the   examined   their   tickets,   punched   something   into   her   computer   and   then   told  
tickets   were   sold   and   issued   in   the   Philippines,   the   applicable   law   would   be   them   that   boarding   would   be   in   fifteen   minutes.   When   the   flight   was   called,  
Philippine  law.   the  Fontanillas  proceeded  to  the  plane.  To  their  surprise,  the  stewardess  at  the  
  gate   did   not   allow   them   to   board   the   plane,   as   they   had   no   assigned   seat  
Even   on   the   assumption   that   overbooking   is   allowed,   respondent   TWA   is   still   numbers.  They  were  then  directed  to  go  back  to  the  "check-­‐in"  counter  where  
guilty   of   bad   faith   in   not   informing   its   passengers   beforehand   that   it   could   Linda   subsequently   informed   them   that   the   flight   had   been   overbooked   and  
breach   the   contract   of   carriage   even   if   they   have   confirmed   tickets   if   there   was   asked   them   to   wait.   The   Fontanillas   tried   to   explain   to   Linda   the   special  
overbooking.   Respondent   TWA   should   have   incorporated   stipulations   on   circumstances  of  their  visit.  However,  Linda  told  them  in  arrogant  manner,  "So  
overbooking   on   the   tickets   issued   or   to   properly   inform   its   passengers   about   what,   I   can   not   do   anything   about   it."   Subsequently,   three   other   passengers  
these   policies   so   that   the   latter   would   be   prepared   for   such   eventuality   or   with   Caucasian   features   were   graciously   allowed   to   baord,   after   the   Fontanillas  
would  have  the  choice  to  ride  with  another  airline.  TWA  was  also  guilty  of  not   were   told   that   the   flight   had   been   overbooked.   The   plane   then   took   off   with  
informing  its  passengers  of  its  alleged  policy  of  giving  less  priority  to  discounted   the  Fontanillas’  baggage  in  tow,  leaving  them  behind.  
tickets.    
  The  Fontanillas  then  complained  to  Linda,  who  in  turn  gave  them  an  ugly  stare  
7.  UNITED  AIRLINES  V.  COURT  OF  APPEALS   and   rudely   uttered,   "it’s   not   my   fault.   It’s   the   fault   of   the   company.   Just   sit  
  down   and   wait."   When   Mr.   Fontanilla   reminded   Linda   of   the   inconvenience  
DOCTRINE:   According   to   the   doctrine   of   lex   loci   contractus,   as   a   general   rule,   being   caused   to   them,   she   bluntly   retorted,   "Who   do   you   think   you   are?   You  
the   law   of   the   place   where   a   contract   is   made   or   entered   into   governs   with   lousy   Flips   are   good   for   nothing   beggars.   You   always   ask   for   American   aid."  
respect   to   its   nature   and   validity,   obligation   and   interpretation.   This   has   been   After   which   she   remarked   "Don’t   worry   about   your   baggage.   Anyway   there   is  
said   to   be   the   rule   even   though   the   place   where   the   contract   was   made   is   nothing   in   there.   What   are   you   doing   here   anyway?   I   will   report   you   to  
different  from  the  place  where  it  is  to  be  performed,  and  particularly  so,  if  the   immigration.  You  Filipinos  should  go  home."  Such  rude  statements  were  made  
place  of  the  making  and  the  place  of  performance  are  the  same.   in  front  of  other  people  in  the  airport  causing  the  Fontanillas  to  suffer  shame,  
  humiliation   and   embarrassment.   The   chastening   situation   even   caused   the  
The   fact   of   rewriting   airplane   tickets   does   not   change   the   nature   of   the   original   younger   Fontanilla   to   break   into   tears.   After   some   time,   Linda,   without   any  
contract  of  carriage  entered  into  by  the  parties  in  the  place  where  it  was  issued.   explanation,   offered   the   Fontanillas   $50.00   each.   She   simply   said   "Take   it   or  
  leave  it."  This,  the  Fontanillas  declined.  The  Fontanillas  then  proceeded  to  the  
FACTS:   United   Airlines   customer   service   counter   to   plead   their   case.   The   male  
Aniceto   Fontanilla   purchased   from   United   Airlines,   through   the   Philippine   employee  at  the  counter  reacted  by  shouting  and  left  without  saying  anything.  
Travel   Bureau   in   Manila   three   "Visit   the   U.S.A."   tickets   for   himself,   his   wife   and    
his  minor  son  Mychal  for  four  routes.  All  flights  had  been  confirmed  previously   On   the   other   hand,   the   United   Airlines   claims   that   the   Fontanillas   did   not  
by   United   Airlines.   The   Fontanillas   then   proceeded   to   the   US   as   planned.   While   initially  go  to  the  check-­‐in  counter  to  get  their  seat  assignments  for  the  flight.  
there,  Aniceto  bought  two  additional  coupons  for  his  family  at  United's  office  in   They   instead   proceeded   to   join   the   queue   boarding   the   aircraft   without   first  
Washington  Dulles  Airport.  After  paying  the  penalty  for  rewriting  their  tickets,   securing   their   seat   assignments   as   required   in   their   ticket   and   boarding  passes.  
the   Fontanillas   were   issued   tickets   with   corresponding   boarding   passes   with   Having  no  seat  assignments,  the  stewardess  at  the  door  of  the  plane  instructed  
the   words   "CHECK-­‐IN   REQUIRED,"   set   to   leave   from   Los   Angeles   to   San   them   to   go   to   the   check-­‐in   counter.   When   the   Fontanillas   proceeded   to   the  
Francisco.   check-­‐in   counter,   Linda   Allen,   the   United   Airlines   Customer   Representative   at  
the  counter  informed  them  that  the  flight  was  overbooked.  She  booked  them  
 
CONFLICT  OF  LAWS                                                                                    AV  DE  TORRES   8  
ATTY.  ARIS  L.  GULAPA                                            AY  2015-­‐2016  
on   the   next   available   flight   and   offered   them   denied   boarding   compensation.   foreign  statute  of  limitations.  
Linda  vehemently  denies  uttering  the  derogatory  and  racist  words  attributed  to    
her  by  the  Fontanillas.   EXC  TO  THE  EXC  TO  THE  EXC:  The  court  of  the  forum  will  not  enforce  any  foreign  
  claim  obnoxious  to  the  forum's  public  policy.  
The  incident  prompted  the  Fontanillas  to  file  case  for  damages  before  the  RTC    
of   Makati   however   the   latter   dismissed   the   complaint.   On   appeal,   the   CA   ruled   A  "borrowing  statute"  directs  the  state  of  the  forum  to  apply  the  foreign  statute  
in  favor  of  the  Fontanillas.  Hence,  United  Airlines  sought  redress  with  the  SC.   of  limitations  to  the  pending  claims  based  on  a  foreign  law.  
   
ISSUE:   FACTS:  
Whether  the  CA  erred  in  applying  US  laws  in  the  case  at  bar   On   June   6,   1984,   Bienvenido   M..   Cadalin,   Rolando   M.   Amul   and   Donato   B.  
  Evangelista,   in   their   own   behalf   and   on   behalf   of   728   other   overseas   contract  
HELD:   workers   (OCWs)   instituted   a   class   suit   by   filing   an   "Amended   Complaint"   with  
Yes.   The   court   held   that   the   CA   erred   in   applying   the   laws   of   the   US   as   the   Philippine   Overseas   Employment   Administration   (POEA)   for   money   claims  
Philippine   law   is   the   applicable   law.   Although,   the   contract   of   carriage   was   to   arising  from  their  recruitment  by  AIBC  and  employment  by  BRII.  
be  performed  in  the  US,  the  tickets  were  purchased  through  United's  agent  in    
Manila.   It   is   true   that   the   tickets   were   "rewritten"   in   Washington,   D.C.   BRII   is   a   foreign   corporation   with   headquarters   in   Houston,   Texas,   and   is  
however,   such   fact   did   not   change   the   nature   of   the   original   contract   of   engaged   in   construction;   while   AIBC   is   a   domestic   corporation   licensed   as   a  
carriage  entered  into  by  the  parties  in  Manila.   service  contractor  to  recruit,  mobilize  and  deploy  Filipino  workers  for  overseas  
  employment  on  behalf  of  its  foreign  principals.  
According  to  the  doctrine  of  lex  loci  contractus,  as  a  general  rule,  the  law  of  the    
place   where   a   contract   is   made   or   entered   into   governs   with   respect   to   its   The   amended   complaint   principally   sought   the   payment   of   the   unexpired  
nature  and  validity,  obligation  and  interpretation.  This  has  been  said  to  be  the   portion  of  the  employment  contracts,  which  was  terminated  prematurely,  and  
rule  even  though  the  place  where  the  contract  was  made  is  different  from  the   secondarily,   the   payment   of   the   interest   of   the   earnings   of   the   Travel   and  
place  where  it  is  to  be  performed,  and  particularly  so,  if  the  place  of  the  making   Reserved   Fund,   interest   on   all   the   unpaid   benefits;   area   wage   and   salary  
and  the  place  of  performance  are  the  same.  Hence,  the  court  should  apply  the   differential  pay;  fringe  benefits;  refund  of  SSS  and  premium  not  remitted  to  the  
law  of  the  place  where  the  airline  ticket  was  issued,  when  the  passengers  are   SSS;  refund  of  withholding  tax  not  remitted  to  the  BIR;  penalties  for  committing  
residents   and   nationals   of   the   forum   and   the   ticket   is   issued   in   such   State   by   prohibited   practices;   as   well   as   the   suspension   of   the   license   of   AIBC   and   the  
the  airline.   accreditation  of  BRII.  
   
8.  CADALIN  V.  POEA  ADMINISTRATOR   ISSUE:    
G.R.  No.  L-­‐104776  |  December  5,  1994   1.   Whether   it   is   the   Bahrain   law   on   prescription   of   action   based   on   the   Amiri  
  Decree  No.  23  of  1976  (1  year)  or  a  Philippine  law  on  prescription  that  shall  be  
DOCTRINE:  GR:  A  foreign  procedural  law  will  not  be  applied  in  the  forum.   the  governing  law.  
  2.  Whether  the  prescriptive  period  governing  the  filing  of  the  claims  is  3  years,  
EXC:  Law  on  prescription  of  actions  is  sui  generis  and  may  be  viewed  procedural   as  provided  by  the  Labor  Code  or  10  years,  as  provided  by  the  NCC.  
or  substantive  depending  on  its  characterization.   3.  Whether  it  is  the  overseas-­‐employment  contracts,  which  became  the  law  of  
  the  parties,  or  the  Amiri  Decree,  which  is  more  favorable  and  beneficial  to  the  
EXC   TO   THE   EXC:   The   characterization   is   irrelevant   when   the   country   of   the   workers,  that  shall  be  enforced.  
forum   has   a   "borrowing   statute."   The   country   of   the   forum   will   apply   the    

 
CONFLICT  OF  LAWS                                                                                    AV  DE  TORRES   9  
ATTY.  ARIS  L.  GULAPA                                            AY  2015-­‐2016  
HELD:   application  in  this  jurisdiction  of  Sec.  156  of  the  Amiri  Decree  No.  23  of  1976.  
1.   Philippine   law   will   govern.   Art.   156   of   the   Amiri   Decree   No.   23   of   1976   The  courts  of  the  forum  will  not  enforce  any  foreign  claim  obnoxious  to  the  
provides:   A   claim   arising   out   of   a   contract   of   employment   shall   not   be   forum's  public  policy.  To  enforce  the  one-­‐year  prescriptive  period  of  the  Amiri  
actionable   after   the   lapse   of   one   year   from   the   date   of   the   expiry   of   the   Decree  No.  23  of  1976  as  regards  the  claims  in  question  would  contravene  the  
contract.   public   policy   on   the   protection   of   labor.   The   1987   Constitution   emphasized  
  that:   The   state   shall   promote   social   justice   in   all   phases   of   national  
As  a  general  rule,  a  foreign  procedural  law  will  not  be  applied  in  the  forum.   development   (Sec.   10).   The   state   affirms   labor   as   a   primary   social   economic  
Procedural  matters,  such  as  service  of  process,  joinder  of  actions,  period  and   force.  It  shall  protect  the  rights  of  workers  and  promote  their  welfare  (Sec.  18).  
requisites   for   appeal,   and   so   forth,   are   governed   by   the   laws   of   the   forum.   The  State  shall  afford  full  protection  to  labor,  local  and  overseas,  organized  and  
This  is  true  even  if  the  action  is  based  upon  a  foreign  substantive  law.   unorganized,   and   promote   full   employment   and   equality   of   employment  
  opportunities  for  all  (Art.  XIII,  Sec.  3).  
However,  a  law  on  prescription  of  actions  is  sui  generis  in  Conflict  of  Laws  in    
the   sense   that   it   may   be   viewed   either   as   procedural   or   substantive,   2.   3   years.   The   claimants   are   of   the   view   that   the   applicable   provision   is   Art.  
depending  on  the  characterization  given  such  a  law.   1144,   NCC,   which   provides:   The   following   actions   must   be   brought   within   ten  
  years   from   the   time   the   right   of   action   accrues:   (1)   Upon   a   written   contract;   (2)  
Thus  in  Bournias  v.  Atlantic  Maritime  Company,  the  American  court  applied  the   Upon  an  obligation  created  by  law;  (3)  Upon  a  judgment.  
statute  of  limitations  of  New  York,  instead  of  the  Panamanian  law,  after  finding    
that   there   was   no   showing   that   the   Panamanian   law   on   prescription   was   NLRC,   on   the   other   hand,   believes   that   the   applicable   provision   is   Art.   291   of  
intended  to  be  substantive.  Being  considered  merely  a  procedural  law  even  in   the  LC,  which  in  pertinent  part  provides:  Money  claims-­‐all  money  claims  arising  
Panama,  it  has  to  give  way  to  the  law  of  the  forum  on  prescription  of  actions.   from   employer-­‐employee   relations   accruing   during   the   effectivity   of   this   Code  
  shall  be  filed  within  three  (3)  years  from  the  time  the  cause  of  action  accrued,  
However,   the   characterization   of   a   statute   into   a   procedural   or   substantive   otherwise  they  shall  be  forever  barred.  
law   becomes   irrelevant   when   the   country   of   the   forum   has   a   "borrowing    
statute."  Said  statute  has  the  practical  effect  of  treating  the  foreign  statute  of   The   claims   in   the   cases   at   bench   all   arose   from   the   employer-­‐employee  
limitation  as  one  of  substance.  A  "borrowing  statute"  directs  the  state  of  the   relations,  which  is  broader  in  scope  than  claims  arising  from  a  specific  law  or  
forum  to  apply  the  foreign  statute  of  limitations  to  the  pending  claims  based   from   the   collective   bargaining   agreement.   The   contention   of   the   POEA  
on   a   foreign   law.   While   there   are   several   kinds   of   "borrowing   statutes,"   one   Admin,   that   the   3-­‐year   prescriptive   period   under   Art.   291   of   the   LC   applies  
form  provides  that  an  action  barred  by  the  laws  of  the  place  where  it  accrued,   only  to  money  claims  specifically  recoverable  under  said  Code,  does  not  find  
will  not  be  enforced  in  the  forum  even  though  the  local   statute  has  not  run   support  in  the  plain  language  of  the  provision.    
against  it.  Sec.  48  of  our  Code  of  Civil  Procedure  is  of  this  kind,  which  provides:    
If  by  the  laws  of  the  state  or  country  where  the  cause  of  action  arose,  the  action   3.   The   overseas-­‐employment   contracts   should   be   read   as   adopting   the  
is  barred,  it  is  also  barred  in  the  Philippines  Islands.   provisions   of   the   Amiri   Decree   No.   23   of   1976   as   part   and   parcel   thereof.   NLRC  
  applied   the   Amiri   Decree   No.   23   of   1976,   which   provides   for   greater   benefits  
Sec.  48  has  not  been  repealed  or  amended  by  the  NCC.  Art.  2270  of  said  Code   than  those  stipulated  in  the  overseas-­‐employment  contracts  of  the  claimants.  It  
repealed  only  those  provisions  of  the  Code  of  Civil  Procedures  as  to  which  were   was  of  the  belief  that  "where  the  laws  of  the  host  country  are  more  favorable  
inconsistent  with  it.  There  is  no  provision  in  the  NCC,  which  is  inconsistent  with   and   beneficial   to   the   workers,   then   the   laws   of   the   host   country   shall   form  
or  contradictory  to  Section  48  of  the  Code  of  Civil  Procedure.   part   of   the   overseas   employment   contract."   It   quoted   with   approval   the  
  observation   of   the   POEA   Administrator   that   "...in   labor   proceedings,   all  
However,   n   the   light   of   the   1987   Constitution,   however,   Section   48   cannot   be   doubts   in   the   implementation   of   the   provisions   of   the   Labor   Code   and   its  
enforced   ex   proprio   vigore   (by   its   own   force)   insofar   as   it   ordains   the   implementing  regulations  shall  be  resolved  in  favor  of  labor."  
 
CONFLICT  OF  LAWS                                                                                    AV  DE  TORRES   10  
ATTY.  ARIS  L.  GULAPA                                            AY  2015-­‐2016  
  A  basic  policy  of  contract  is  to  protect  the  expectation  of  the  parties.  Such  party  
AIBC   and   BRII   claimed   that   NLRC   acted   capriciously   and   whimsically   when   it   expectation   is   protected   by   giving   effect   to   the   parties'   own   choice   of   the  
refused  to  enforce  the  overseas-­‐employment  contracts,  which  became  the  law   applicable  law.  The  choice  of  law  must,  however,  bear  some  relationship  to  the  
of  the  parties.  They  contend  that  the  principle  that  a  law  is  deemed  to  be  a  part   parties   or   their   transaction.   There   is   no   question   that   the   contracts   sought   to  
of  a  contract  applies  only  to  provisions  of  Philippine  law  in  relation  to  contracts   be   enforced   by   claimants   have   a   direct   connection   with   the   Bahrain   law  
executed  in  the  Philippines.   because  the  services  were  rendered  in  that  country.  
     
The  overseas-­‐employment  contracts,  which  were  prepared  by  AIBC  and  BRII   N.B.:   Benefits:   OT   pay-­‐25%   daytime,   50%   nighttime;   Work   on   rest   day   pay-­‐
themselves,  provided  that  the  laws  of  the  host  country  became  applicable  to   150%;   Holiday   pay-­‐150%;   1   year   continuous   service-­‐21   days   leave;   5   years  
said   contracts   if   they   offer   terms   and   conditions   more   favorable   that   those   continuous   service-­‐28   days;   30   days   termination   notice   or   compensation  
stipulated  therein.  It  was  stipulated  in  said  contracts  that:   equivalent  to  the  amount  of  wages  payable  to  the  worker.  
   
...the   benefits   provided   to   Employee   hereunder   are   substituted   for   and   9.  ASIAVEST  MERCHANT  BANKERS  V.  COURT  OF  APPEALS  
in   lieu   of   all   other   benefits   provided   by   any   applicable   law,  provided   of   G.R.  No.  110263  |  July  20,  2001  *Division  decision  
course,  that  total  remuneration  and  benefits  do  not  fall  below  that  of  the    
host   country   regulation   or   custom,   it   being   understood   that   should   DOCTRINES:  GR:  Upon  proving  through  evidence  the  existence  and  authenticity  
applicable   laws   establish   that   fringe   benefits,   or   other   such   benefits   of  the  foreign  judgment,  said  foreign  judgment  enjoys  presumptive  validity  and  
additional   to   the   compensation   herein   agreed   cannot   be   waived,   the  burden  then  fell  upon  the  party  who  disputes  its  validity  to  prove  otherwise.  
Employee  agrees  that  such  compensation  will  be  adjusted  downward  so    
that   the   total   compensation   hereunder,   plus   the   non-­‐waivable   benefits   Requisites  for  recognizing  a  valid  judgment  rendered  by  foreign  tribunals:  [FSP]  
shall  be  equivalent  to  the  compensation  herein  agreed.”   (1)  it  is  convincingly  shown  that  there  has  been  an  opportunity  for  a  full  and  fair  
  hearing  before  a  court  of  competent  jurisdiction;  (2)  that  the  trial  upon  regular  
Any   ambiguity   in   the   overseas-­‐employment   contracts   should   be   interpreted   proceedings   has   been   conducted,   following   due   citation   or   voluntary  
against  AIBC  and  BRII,  the  parties  that  drafted  it.  The  interpretation  of  obscure   appearance   of   the   defendant   and   under   a   system   of   jurisprudence   likely   to  
words   or   stipulations   in   a   contract   shall   not   favor   the   party   who   caused   the   secure   an   impartial   administration   of   justice;   and   (3)   that   there   is   nothing   to  
obscurity  (Art.  1377,  NCC).   indicate  either  a  prejudice  in  court  and  in  the  system  of  laws  under  which  it  is  
  sitting  or  fraud  in  procuring  the  judgment.  
The  parties  to  a  contract  may  select  the  law  by  which  it  is  to  be  governed.   In    
such  a  case,  the  foreign  law  is  adopted  as  a  "system"  to  regulate  the  relations   A   foreign   judgment   is   presumed   to   be   valid   and   binding   in   the   country   from  
of  the  parties,  including  questions  of  their  capacity  to  enter  into  the  contract,   which   it   comes,   until   a   contrary   showing,   on   the   basis   of   a   presumption   of  
the  formalities  to  be  observed  by  them,  matters  of  performance,  and  so  forth.   regularity  of  proceedings  and  the  giving  of  due  notice  in  the  foreign  forum.  
   
Instead   of   adopting   the   entire   mass   of   the   foreign   law,   the   parties   may   just   FACTS:  
agree   that   specific   provisions   of   a   foreign   statute   shall   be   deemed   Asiavest   Merchant   Bankers,   a   corporation   organized   under   the   laws   of  
incorporated  into  their  contract  "as  a  set  of  terms."  By  such  reference  to  the   Malaysia,   initiated   a   collection   suit   before   the   High   Court   of   Malaya   in   Kuala  
provisions  of  the  foreign  law,  the  contract  does  not  become  a  foreign  contract   Lumpur   against   Philippine   National   Construction   Corporation   (PNCC),   a  
to  be  governed  by  the  foreign  law.  The  said  law  does  not  operate  as  a  statute   corporation   duly   incorporated   and   existing   under   Philippine   laws.   Asiavest  
but  as  a  set  of  contractual  terms  deemed  written  in  the  contract     sought  to  recover  the  indemnity  of  the  performance  bond  it  had  put  up  in  favor  
  of  PNCC  to  guarantee  completion  of  the  Felda  Project  and  the  non-­‐payment  of  

 
CONFLICT  OF  LAWS                                                                                    AV  DE  TORRES   11  
ATTY.  ARIS  L.  GULAPA                                            AY  2015-­‐2016  
the  loan  it  extended  for  the  completion  of  another  project.  The  High  Court  of   and   their   successors   in   interest   by   a   subsequent   title.   The   judgment   may,  
Malaya  rendered  judgment  in  favor  of  Asiavest.   however,  be  assailed  by  evidence  of  want  of  jurisdiction,  want  of  notice  to  the  
  party,   collusion,   fraud,   or   clear   mistake   of   law   or   fact.   In   addition,   under   Sec.  
Following   unsuccessful   attempts   to   secure   payment   from   PNCC   under   the   3(n),  Rule  131  of  the  Revised  Rules  of  Court,  a  court,  whether  in  the  Philippines  
judgment,   Asiavest   initiated   a   complaint   before   the   RTC   to   enforce   the   or  elsewhere,  enjoys  the  presumption  that  it  was  acting  in  the  lawful  exercise  of  
judgment   of   the   High   Court   of   Malaya.   PNCC   moved   to   dismiss   the   case   its  jurisdiction.  Hence,  once  the  authenticity  of  the  foreign  judgment  is  proved,  
contending   that   the   foreign   judgment   should   be   denied   recognition   or   the   party   attacking   a   foreign   judgment,   is   tasked   with   the   burden   of  
enforcement   for   want   of   jurisdiction,   want   of   notice,   collusion   and/or   fraud,   overcoming   its   presumptive   validity.   Hence,   once   the   authenticity   of   the  
and  there  is  a  clear  mistake  of  law  or  fact.  The  RTC  and  CA  dismissed  the  case.   foreign   judgment   is   proved,   the   party   attacking   a   foreign   judgment,   is   tasked  
  with  the  burden  of  overcoming  its  presumptive  validity.  
ISSUE:    
Whether  the  foreign  judgment  should  be  given  recognition  and  enforcement  in   In   the   instant   case,   Asiavest   sufficiently   established   the   existence   of   the   money  
the  Philippines.   judgment   of   the   High   Court   of   Malaya   by   the   evidence   it   offered.   Vinayak  
  Prabhakar  Pradhan,  as  sole  witness,  testified  that  he  is  in  active  practice  of  the  
HELD:   law  profession  in  Malaysia;  that  the  writ  of  summons  was  served  on  PNCC  and  
Yes.   Generally,  in  the  absence  of  a  special  compact,  no  sovereign  is  bound  to   on   Cora   S.   Deala,   its   financial   planning   officer;   that   certain   Messrs.   entered  
give   effect   within   its   dominion   to   a   judgment   rendered   by   a   tribunal   of   their   conditional   appearance   for   PNCC   questioning   the   regularity   of   the   service  
another   country;   however,   the   rules   of   comity,   utility   and   convenience   of   of  the  writ  of  summons  but  subsequently  withdrew  the  same  when  it  realized  
nations   have   established   a   usage   among   civilized   states   by   which   final   that  the  writ  was  properly  served;  that  the  matter  was  then  heard  before  the  
judgments   of   foreign   courts   of   competent   jurisdiction   are   reciprocally   High  Court  of  Kuala  Lumpur  in  a  series  of  dates  where  PNCC  was  represented  
respected  and  rendered  efficacious  under  certain  conditions  that  may  vary  in   by   counsel;  and   that   the   end   result   of   all   these   proceedings   is   the   judgment  
different  countries.   sought   to   be   enforced.   In   addition,   Asiavest   offered   the   following   documentary  
  evidence:  (a)  A  certified  and  authenticated  copy  of  the  Judgment  promulgated  
In   this   jurisdiction,   a   valid   judgment   rendered   by   a   foreign   tribunal   may   be   by  the  Malaysian  High  Court,  etc.    
recognized   insofar   as   the   immediate   parties   and   the   underlying   cause   of    
action   are   concerned   so   long   as:   (1)   it   is   convincingly   shown   that   there   has   Having  thus  proven  through  evidence  of  the  existence  and  authenticity  of  the  
been   an   opportunity   for   a   full   and   fair   hearing   before   a   court   of   competent   foreign  judgment,  said  foreign  judgment  enjoys  presumptive  validity  and  the  
jurisdiction;   (2)   that   the   trial   upon   regular   proceedings   has   been   conducted,   burden   then   fell   upon   the   party   who   disputes   its   validity,   herein   PNCC,   to  
following  due  citation  or  voluntary  appearance  of  the  defendant  and  under  a   prove  otherwise.  
system  of  jurisprudence  likely  to  secure  an  impartial  administration  of  justice;    
and  (3)  that  there  is  nothing  to  indicate  either  a  prejudice  in  court  and  in  the   PNCC   failed   to   sufficiently   discharge   the   burden   that   fell   upon   it–to   prove   by  
system  of  laws  under  which  it  is  sitting  or  fraud  in  procuring  the  judgment.   clear   and   convincing   evidence   the   grounds,   which   it   relied   upon   to   prevent  
  enforcement   of   the   Malaysian   High   Court   judgment.   The   reasons   or   grounds  
A  foreign  judgment  is  presumed  to  be  valid  and  binding  in  the  country  from   relied   upon   by   PNCC   in   preventing   enforcement   and   recognition   of   the  
which   it   comes,   until   a   contrary   showing,   on   the   basis   of   a   presumption   of   Malaysian  judgment  primarily  refer  to  matters  of  remedy  and  procedure  taken  
regularity   of   proceedings   and   the   giving   of   due   notice   in   the   foreign   forum.   by   the   Malaysian   High   Court   relative   to   the   suit   for   collection   initiated   by  
Under  Sec.  50(b),  [now  Sec.  48]  Rule  39  of  the  Revised  Rules  of  Court,  which   petitioner.   Needless   to   stress,   the   recognition   to   be   accorded   a   foreign  
was   the   governing   law   at   the   time   the   instant   case   was   decided,   a   judgment,   judgment   is   not   necessarily   affected   by   the   fact   that   the   procedure   in   the  
against   a   person,   of   a   tribunal   of   a   foreign   country   having   jurisdiction   to   courts  of  the  country  in  which  such  judgment  was  rendered  differs  from  that  of  
pronounce  the  same  is  presumptive  evidence  of  a  right  as  between  the  parties   the   courts   of   the   country   in   which   the   judgment   is   relied   on.   Ultimately,  
 
CONFLICT  OF  LAWS                                                                                    AV  DE  TORRES   12  
ATTY.  ARIS  L.  GULAPA                                            AY  2015-­‐2016  
matters   of   remedy   and   procedure   such   as   those   relating   to   the   service   of   the  judgment  of  the  foreign  court  simply  because  our  rules  provide  otherwise.  
summons   or   court   process   upon   the   defendant,   the   authority   of   counsel   to    
appear  and  represent  a  defendant  and  the  formal  requirements  in  a  decision   All   in   all,   PNCC   had   the   ultimate   duty   to   demonstrate   the   alleged   invalidity   of  
are  governed  by  the  lex  fori  or  the  internal  law  of  the  forum,  i.e.,  the  law  of   such  foreign  judgment,  being  the  party  challenging  the  judgment  rendered  by  
Malaysia  in  this  case.   the   High   Court   of   Malaya.   But   instead   of   doing   so,   PNCC   merely   argued   that  
  the  burden  lay  upon  petitioner  to  prove  the  validity  of  the  money  judgment.  
GROUNDS  RELIED  UPON:    
a)   Improper   service   of   summons—In   this   case,   it   is   the   procedural   law   of   N.B.:  Rule  39,  Sec.  48.  Effect  of  foreign  judgments  or  final  orders.  —  The  effect  
Malaysia   where   the   judgment   was   rendered   that   determines   the   validity   of   the   of   a   judgment   or   final   order   of   a   tribunal   of   a   foreign   country,   having  
service   of   court   process   on   PNCC   as   well   as   other   matters   raised   by   it.   As   to   jurisdiction  to  render  the  judgment  or  final  order  is  as  follows:  
what  the  Malaysian  procedural  law  is,  remains  a  question  of  fact,  not  of  law.  It    
may   not   be   taken   judicial   notice   of   and   must   be   pleaded   and   proved   like   any   (a)  In  case  of  a  judgment  or  final  order  upon  a  specific  thing,  the  judgment  or  
other  fact.  §24   and   25   of   Rule   132   of   the   Revised   Rules   of   Court  provide  that  it   final  order,  is  conclusive  upon  the  title  to  the  thing,  and  
may   be   evidenced   by   an   official   publication   or   by   a   duly   attested   or   (b)  In  case  of  a  judgment  or  final  order  against  a  person,  the  judgment  or  final  
authenticated   copy   thereof.   It   was   then   incumbent   upon   PNCC   to   present   order   is   presumptive   evidence   of   a   right   as   between   the   parties   and   their  
evidence   as   to   what   that   Malaysian   procedural   law   is   and   to   show   that   under   successors  in  interest  by  a  subsequent  title.  
it,  the  assailed  service  of  summons  upon  a  financial  officer  of  a  corporation,    
as  alleged  by  it,  is  invalid.  It  did  not.     In   either   case,   the   judgment   or   final   order   may   be   repelled   by   evidence   of   a  
  want   of   jurisdiction,   want   of   notice   to   the   party,   collusion,   fraud,   or   clear  
b)  Judgment  is  tainted  with  evident  collusion,  fraud  and  clear  mistake  of  fact   mistake  of  law  or  fact.  (50a)  
or   law—Fraud   to   hinder   the   enforcement   within   the   jurisdiction   of   a   foreign    
judgment   must   be   extrinsic,   i.e.,   fraud   based   on   facts   not   controverted   or   Rule   131,   Sec.   3.  Disputable   presumptions.  —  The   following   presumptions   are  
resolved  in  the  case  where  judgment  is  rendered,  or  that  which  would  go  to  the   satisfactory  if  uncontradicted,  but  may  be  contradicted  and  overcome  by  other  
jurisdiction  of  the  court  or  would  deprive  the  party  against  whom  judgment  is   evidence:  
rendered  a  chance  to  defend  the  action  to  which  he  has  a  meritorious  defense.    
Intrinsic  fraud  is  one  which  goes  to  the  very  existence  of  the  cause  of  action  is   (n)   That   a   court,   or   judge   acting   as   such,   whether   in   the   Philippines   or  
deemed   already   adjudged,   and   it,   therefore,   cannot   militate   against   the   elsewhere,  was  acting  in  the  lawful  exercise  of  jurisdiction;  
recognition   or   enforcement   of   the   foreign   judgment.   Evidence   is   wanting   on    
the  alleged  extrinsic  fraud.  Hence,  such  unsubstantiated  allegation  cannot  give   Rule   132,   Sec.   24.   Proof   of   official   record.  —  The   record   of   public   documents  
rise  to  liability  therein.   referred   to   in   paragraph   (a)   of   Section   19,   when   admissible   for   any   purpose,  
  may  be  evidenced  by  an  official  publication  thereof  or  by  a  copy  attested  by  the  
c)   Contrary   to   Constitutional   prescription   that   every   decision   must   state   the   officer   having   the   legal   custody   of   the   record,   or   by   his   deputy,   and  
facts   and   law   on   which   it   is   based—Lastly,  there  is  no  merit  to  the  argument   accompanied,  if  the  record  is  not  kept  in  the  Philippines,  with  a  certificate  that  
that   the   foreign   judgment   is   not   enforceable   in   view   of   the   absence   of   any   such  officer  has  the  custody.  If  the  office  in  which  the  record  is  kept  is  in  foreign  
statement  of  facts  and  law  upon  which  the  award  in  favor  of  the  petitioner  was   country,   the   certificate   may   be   made   by   a   secretary   of   the   embassy   or   legation,  
based.  The  lex  fori  or  the  internal  law  of  the  forum  governs  matters  of  remedy   consul   general,   consul,   vice   consul,   or   consular   agent   or   by   any   officer   in   the  
and  procedure.  Considering  that  under  the  procedural  rules  of  the  High  Court   foreign   service   of   the   Philippines   stationed   in   the   foreign   country   in   which   the  
of   Malaya,   a   valid   judgment   may   be   rendered   even   without   stating   in   the   record  is  kept,  and  authenticated  by  the  seal  of  his  office.  (25a)  
judgment  every  fact  and  law  upon  which  the  judgment  is  based,  then  the  same    
must   be   accorded   respect   and   the   courts   in   this   jurisdiction   cannot   invalidate  
 
CONFLICT  OF  LAWS                                                                                    AV  DE  TORRES   13  
ATTY.  ARIS  L.  GULAPA                                            AY  2015-­‐2016  
10.  GARCIA  V.  RECIO   that  the  divorce  issued  in  Australia  was  valid  and  recognized  in  the  Philippines.  
G.R.  No.  138322  |  October  2,  2001  *En  banc  decision   The   Australian   divorce   had   ended   the   marriage;   thus,   there   was   no   more  
  martial  union  to  nullify  or  annul.  
DOCTRINE:   EXC:   When   the   law   requires   proof   that   it   is   validly   obtained   in    
conformity  with  the  foreign  law.   ISSUES:  
  1.  Whether  the  divorce  between  respondent  and  Editha  Samson  was  proven.  
Av:  Such  as  in  cases  of  divorce,  where  Art.  26,  FC,  requires  that  the  divorce  be   2.  Whether  Rederick  was  proven  to  be  legally  capacitated  to  marry  Grace.  
proven  to  be  validly  obtained  by  the  alien  spouse.  Thus,  the  divorce  cannot  be    
presumed  to  be  valid.   HELD:  
  1.  Yes.  At  the  outset,  we  lay  the  following  basic  legal  principles:  Philippine  law  
 
A   divorce   obtained   abroad   by   an   alien   may   be   recognized   in   our   jurisdiction,   does   not   provide   for   absolute   divorce;   hence,   our   courts   cannot   grant   it. A  
provided   such   decree   is   valid   according   to   the   national   law   of   the   foreigner.   marriage   between   two   Filipinos   cannot   be   dissolved   even   by   a   divorce  
 
However,  the  divorce  decree  and  the  governing  personal  law  of  the  alien  spouse   obtained   abroad,   because   of   Articles   15  and   17  of   the   NCC. In   mixed  
 
who  obtained  the  divorce  must  be  proven.  Our  courts  do  not  take  judicial  notice   marriages   involving   a   Filipino   and   a   foreigner,   Art.   26 of   the   FC   allows   the  
of   foreign   laws   and   judgment;   hence,   like   any   other   facts,   both   the   divorce   former   to   contract   a   subsequent   marriage   in   case   the   divorce   is   "validly  
 
decree  and  the  national  law  of  the  alien  must  be  alleged  and  proven  according   obtained   abroad   by   the   alien   spouse   capacitating   him   or   her   to   remarry." A  
to  our  law  on  evidence.   divorce  obtained  abroad  by  a  couple,  who  are  both  aliens,  may  be  recognized  
  in   the   Philippines,   provided   it   is   consistent   with   their   respective   national  
FACTS:   laws.  
Rederick  Recio,  a  Filipino,  was  married  to  Editha  Samson,  an  Australian  citizen,    
in  Malabon,  Rizal  in  1987.  They  lived  together  as  husband  and  wife  in  Australia.   Grace  argues  that  the  divorce  decree,  like  any  other  foreign  judgment,  may  be  
In  1989,  a  decree  of  divorce,  purportedly  dissolving  the  marriage,  was  issued  by   given   recognition   in   this   jurisdiction   only   upon   proof   of   the   existence   of   (1)   the  
an  Australian  family  court.  In  1992,  Rederick  became  an  Australian  Citizen.   foreign  law  allowing  absolute  divorce  and  (2)  the  alleged  divorce  decree  itself.  
  She  adds  that  Rederick  miserably  failed  to  establish  these  elements.  Rederick,  
In  1994,  Grace  Garcia,  a  Filipina,  and  Rederick  were  married  in  Cabanatuan  City.   on   the   other   hand,   argues   that   the   Australian   divorce   decree   is   a   public  
In   their   application   for   a   marriage   license,   Rederick   was   declared   as   "single"   document–a   written   official   act   of   an   Australian   family   court.   Therefore,   it  
and  "Filipino."   requires  no  further  proof  of  its  authenticity  and  due  execution.  
   
In  March  1998,  Grace  filed  a  Complaint  for  Declaration  of  Nullity  of  Marriage  on   Rederick   is   getting   ahead   of   himself.   Before   a   foreign   judgment   is   given  
the   ground   of   bigamy,   claiming   that   she   learned   of   Rederick’s   marriage   to   presumptive   evidentiary   value,   the   document   must   first   be   presented   and  
Editha  only  in  1997.  In  his  Answer,  Rederick  averred  that,  as  far  back  as  1993,   admitted   in   evidence.  A   divorce   obtained   abroad   is   proven   by   the   divorce  
he   had   revealed   to   Grace   his   prior   marriage   and   its   subsequent   dissolution   in   decree   itself.   Indeed   the   best   evidence   of   a   judgment   is   the   judgment  
1989.  Thus,  he  was  legally  capacitated  to  marry  Grace  in  1994.   itself.  The  decree  purports  to  be  a  written  act  or  record  of  an  act  of  an  official  
  body  or  tribunal  of  a  foreign  country.  
In  1998,  while  the  suit  for  the  declaration  of  nullity  was  pending,  Rederick  was    
able   to   secure   a   divorce   decree   from   a   family   court   in   Sydney   because   his   Under  Sections  24  and  25  of  Rule  132,  a  writing  or  document  may  be  proven  as  
marriage   with   Grace   had   "irretrievably   broken   down."   Thus,   Rederick   prayed   in   a   public   or   official   record   of   a   foreign   country   by   either   (1)   an   official  
his   Answer   that   the   Complaint   be   dismissed   on   the   ground   that   it   stated   no   publication  or  (2)  a  copy  thereof  attested  by  the  officer  having  legal  custody  of  
cause  of  action.  The  trial  court  declared  the  marriage  dissolved  on  the  ground   the  document.  If  the  record  is  not  kept  in  the  Philippines,  such  copy  must  be  (a)  
accompanied  by  a  certificate  issued  by  the  proper  diplomatic  or  consular  officer  
 
CONFLICT  OF  LAWS                                                                                    AV  DE  TORRES   14  
ATTY.  ARIS  L.  GULAPA                                            AY  2015-­‐2016  
in   the   Philippine   foreign   service   stationed   in   the   foreign   country   in   which   the   The   court   a   quo   erred   in   finding   that   the   divorce   decree   ipso   facto   clothed  
record   is   kept   and   (b)   authenticated   by   the   seal   of   his   office.   These   Rederick   with   the   legal   capacity   to   remarry   without   requiring   him   to   adduce  
aforemetioned   rules   on   evidence   were   not   complied   with   since   Rederick   only   sufficient  evidence  to  show  the  Australian  personal  law  governing  his  status;  or  
presented  the  divorce  decree.   at   the   very   least,   to   prove   his   legal   capacity   to   contract   the   second   marriage.  
  Neither  can  we  declare  the  marriage  null  and  void  on  the  ground  of  bigamy.  It  
Fortunately   for   Rederick’s   cause,   when   the   divorce   decree   of   1989   was   may   turn   out   that   under   Australian   law,   Rederick   was   really   capacitated   to  
submitted   in   evidence,   Grace’s   counsel   objected,   not   to   its   admissibility,   but   marry  Grace  as  a  direct  result  of  the  divorce  decree.  Hence,  the  most  judicious  
only   to   the   fact   that   it   had   not   been   registered   in   the   Local   Civil   Registry   of   course  is  to  remand  this  case  to  the  trial  court  to  receive  evidence,  if  any,  which  
Cabanatuan   City.  The   trial   court   ruled   that   it   was   admissible,   subject   to  Grace’s   show  Rederick’s  legal  capacity  to  marry  Grace.  
 
qualification. Hence,   it   was   admitted   in   evidence   and   accorded   weight   by   the    
judge.   Indeed,   the   failure   to   object   properly   rendered   the   divorce   decree   N.B.:   Article   15.  Laws   relating   to   family   rights   and   duties,   or   to   the   status,  
admissible  as  a  written  act  of  the  Family  Court  of  Sydney,  Australia.   condition   and   legal   capacity   of   persons   are   binding   upon   citizens   of   the  
  Philippines,  even  though  living  abroad.  (9a)  
Compliance   with   Arts.   11,   13,   and   52   of   the   FC   (1)   file   a   sworn   application   for   a    
marriage  license  with  the  proper  local  civil  registrar  which  shall  specify  that  if   Article   17.  …   Prohibitive   laws   concerning   persons,   their   acts   or   property,   and  
previously  married,  how,  when  and  where  the  previous  marriage  was  dissolved   those  which  have  for  their  object  public  order,  public  policy  and  good  customs  
or  annulled;  2)  furnish  a  death  certificate  of  the  deceased  spouse  or  the  judicial   shall   not   be   rendered   ineffective   by   laws   or   judgments   promulgated,   or   by  
decree  of  annulment  or  declaration  of  nullity  of  his  or  her  previous  marriage;  3)   determinations  or  conventions  agreed  upon  in  a  foreign  country.  (11a)  
recording  of  judgment  of  annulment  or  of  absolute  nullity  of  the  marriage,  the    
partition  and  distribution  of  the  properties  of  the  spouses,  and  the  delivery  of   Art.  21.   When   either   or   both   of   the   contracting   parties   are   citizens   of   a   foreign  
the   children's   presumptive   legitimes   in   the   appropriate   civil   registry   and   country,   it   shall   be   necessary   for   them   before   a   marriage   license   can   be  
registries   of   property)   is   not   necessary.   Rederick   was   no   longer   bound   by   obtained,  to  submit  a  certificate  of  legal  capacity  to  contract  marriage,  issued  
Philippine   personal   laws   after   he   acquired   Australian   citizenship   in   1992.   by  their  respective  diplomatic  or  consular  officials.  
Naturalization  is  the  legal  act  of  adopting  an  alien  and  clothing  him  with  the    
political  and  civil  rights  belonging  to  a  citizen.  Naturalized  citizens,  freed  from   Stateless   persons   or   refugees   from   other   countries   shall,   in   lieu   of   the  
the   protective   cloak   of   their   former   states,   don   the   attires   of   their   adoptive   certificate   of   legal   capacity   herein   required,   submit   an   affidavit   stating   the  
countries.   By   becoming   an   Australian,   respondent   severed   his   allegiance   to   circumstances  showing  such  capacity  to  contract  marriage.  (66a)  
the  Philippines  and  the  vinculum  juris  that  had  tied  him  to  Philippine  personal    
laws.   Art.   26.   All   marriages   solemnized   outside   the   Philippines,   in   accordance   with  
  the   laws   in   force   in   the   country   where   they   were   solemnized,   and   valid   there   as  
2.   No.   Rederick   merely   presented   a   decree   nisi   or   an   interlocutory   decree–a   such,   shall   also   be   valid   in   this   country,   except   those   prohibited   under   Articles  
conditional   or   provisional   judgment   of   divorce.   The   legal   capacity   to   contract   35  (1),  (4),  (5)  and  (6),  36,  37  and  38.  (17a)  
marriage   is   determined   by   the   national   law   of   the   party   concerned.   The    
certificate   mentioned   in   Art.   21   of   the   FC   would   have   been   sufficient   to   Where   a   marriage   between   a   Filipino   citizen   and   a   foreigner   is   validly  
establish   the   legal   capacity   of   Rederick,   had   he   duly   presented   it   in   court.   A   celebrated   and   a   divorce   is   thereafter   validly   obtained   abroad   by   the   alien  
duly   authenticated   and   admitted   certificate   is   prima   facie   evidence   of   legal   spouse   capacitating   him   or   her   to   remarry,   the   Filipino   spouse   shall   have  
capacity   to   marry   on   the   part   of   the   alien   applicant   for   a   marriage   license.   capacity  to  remarry  under  Philippine  law.  (As  amended  by  Executive  Order  227)  
However,   based   on   the   records   submitted,   there   is   absolutely   no   evidence   that    
proves  Rederick’s  legal  capacity  to  marry  Grace.  
 
 
CONFLICT  OF  LAWS                                                                                    AV  DE  TORRES   15  
ATTY.  ARIS  L.  GULAPA                                            AY  2015-­‐2016  
Rule   132,   Section   19.   Classes   of   Documents.  —  For   the   purpose   of   their   the  fact  that  he  is  a  subject  of  a  foreign  judgment,  in  order  to  file  a  petition  for  
presentation  in  evidence,  documents  are  either  public  or  private.   recognition  of  foreign  judgment.  This  he  may  do  first,  by  proof  of  a  valid  divorce  
  (existence,   authenticity,   and   validly   obtained   in   conformity   with   the   national  
Public  documents  are:   law).   After   which,   the   divorce   decree   serves   as   a   presumptive   evidence   of   his  
  right.   By   virtue   of   being   a   subject   of   the   foreign   divorce   decree/judgment  
a)   The   written   official   acts,   or   records   of   the   official   acts   of   the   sovereign   proven,  he  may  file  a  suit  for  recognition  of  foreign  judgment.    
authority,   official   bodies   and   tribunals,   and   public   officers,   whether   of   the    
Philippines,  or  of  a  foreign  country;   FACTS:  
  Gerbert  Corpuz  was  a  former  Filipino  citizen  who  acquired  Canadian  citizenship  
Section  24.  Proof  of  official  record.  —  The  record  of  public  documents  referred   through  naturalization  in  2000.  In  2005,  Gerbert  married  Daisylyn  Sto.  Tomas,  a  
to   in   paragraph   (a)   of   Section   19,   when   admissible   for   any   purpose,   may   be   Filipina,   in   Pasig   City.  Due   to   work   and   other   professional   commitments,  
evidenced   by   an   official   publication   thereof   or   by   a   copy   attested   by   the   officer   Gerbert  left  for  Canada  soon  after  the  wedding.  He  returned  to  the  Philippines  
having   the   legal   custody   of   the   record,   or   by   his   deputy,   and   accompanied,   if   sometime  in  April  2005  but  was  shocked  to  discover  that  his  wife  was  having  an  
the  record  is  not  kept  in  the  Philippines,  with  a  certificate  that  such  officer  has   affair   with   another   man.   Gerbert   returned   to   Canada   and   filed   a   petition   for  
the  custody.  If  the  office  in  which  the  record  is  kept  is  in  foreign  country,  the   divorce,  which  was  granted  in  December  2005  and  took  effect  a  month  later.  
certificate   may   be   made   by   a   secretary   of   the   embassy   or   legation,   consul    
general,   consul,   vice   consul,   or   consular   agent   or   by   any   officer   in   the   foreign   Two   years   after   the   divorce,   with   the   intention   of   marrying   another   Filipina,  
service  of  the  Philippines  stationed  in  the  foreign  country  in  which  the  record  is   Gerbert   went   to   the   Pasig   City   Civil   Registry   Office   and   registered   the   Canadian  
kept,  and  authenticated  by  the  seal  of  his  office.  (25a)   divorce   decree   on   his   and   Daisylyn’s   marriage   certificate.   Despite   the  
  registration,   an   official   of   the   NSO   informed   him   that   for   the   foreign   divorce  
11.  CORPUZ  V.  STO.  TOMAS   decree  to  be  enforceable,  it  must  first  be  judicially  recognized  by  a  competent  
G.R.  No.  186751  |  August  11,  2010   Philippine  court,  pursuant  to  NSO  Circular  No.  4.  
   
nd
DOCTRINES:   The  alien  spouse  can  claim  no  right  under  the  2  par.  of  Art.  26,  FC   Accordingly,   Gerbert   filed   a   petition   for   judicial   recognition   of   foreign   divorce  
as   the   substantive   right   it   establishes   is   in   favor   of   the   Filipino   spouse.   No   court   and/or  declaration  of  marriage  as  dissolved  with  the  RTC.  Daisylyn  offered  no  
in   this   jurisdiction   can   make   a   similar   declaration   for   the   alien   spouse   (other   opposition  and,  in  fact,  alleged  her  desire  to  file  a  similar  case  herself  but  was  
than   that   already   established   by   the   decree),   whose   status   and   legal   capacity   prevented   by   financial   and   personal   circumstances.   However,   the   RTC   denied  
are  generally  governed  by  his  national  law.   the  petition,  concluding  that  Gerbert  was  not  the  proper  party  to  institute  the  
  action   for   judicial   recognition   of   the   foreign   divorce   decree   as   he   is   a  
HOWEVER,   the   foreign   divorce   decree   itself,   after   its   authenticity   and   naturalized  Canadian  citizen.  Only  the  Filipino  spouse  can  avail  of  the  remedy,  
nd
conformity  with  the  alien’s  national  law  have  been  duly  proven  according  to  our   under  the  2  par.  of  Art.  26  of  the  FC.  
rules   of   evidence,   serves   as   a   presumptive   evidence   of   right   in   favor   of   the   alien    
spouse,  pursuant  to  Sec.  48,  Rule  39  of  the  ROC,  which  provides  for  the  effect  of   ISSUE:  
foreign   judgments.  Direct   involvement   or   being   the   subject   of   the   foreign   Whether  the  2nd  par.  of  Art.  26  of  the  FC  extends  to  aliens  the  right  to  petition  
judgment  is  sufficient  to  clothe  a  party  with  the  requisite  interest  to  institute  an   a  court  of  Philippine  jurisdiction  for  the  recognition  of  a  foreign  divorce  decree.  
action  before  our  courts  for  the  recognition  of  the  foreign  judgment.    
  HELD:  
Av:   This   is   just   a   mere   technicality.   Alien   spouse  cannot  invoke  Art.  26,  FC  to  file   No  but  this  fact  does  not  necessarily  strip  Gerbert  of  legal  interest  to  petition  
a   petition   for   judicial   recognition   of   foreign   divorce.   However,   he   may   invoke   the  RTC  for  the  recognition  of  his  foreign  divorce  decree.  

 
CONFLICT  OF  LAWS                                                                                    AV  DE  TORRES   16  
ATTY.  ARIS  L.  GULAPA                                            AY  2015-­‐2016  
  evidence   of   right   in   favor   of   Gerbert,   pursuant   to   Sec.   48,   Rule   39   of   the   ROC,  
nd
The   alien   spouse   can   claim   no   right   under   the   2   par.   of   Art.   26,   FC   as   the   which  provides  for  the  effect  of  foreign  judgments.    
substantive   right   it   establishes   is   in   favor   of   the   Filipino   spouse.   The   FC    
recognizes  only  two  types  of  defective  marriages–void  and   voidable  marriages.   Direct  involvement  or  being  the  subject  of  the  foreign  judgment  is  sufficient  
In   both   cases,   the   basis   for   the   judicial   declaration   of   absolute   nullity   or   to  clothe  a  party  with  the  requisite  interest  to  institute  an  action  before  our  
annulment   of   the   marriage   exists   before   or   at   the   time   of   the   marriage.   courts  for  the  recognition  of  the  foreign  judgment.  In  a  divorce  situation,  we  
Divorce,   on   the   other   hand,   contemplates   the   dissolution   of   the   lawful   union   have   declared,   no   less,   that   the   divorce   obtained   by   an   alien   abroad   may   be  
for  cause  arising  after  the  marriage.  Our  family  laws  do  not  recognize  absolute   recognized  in  the  Philippines,  provided  the  divorce  is  valid  according  to  his  or  
divorce  between  Filipino  citizens.   her   national   law.   The   recognition   may   be   made   in   an   action   instituted  
  specifically   for   the   purpose   or   in   another   action   where   a   party   invokes   the  
Recognizing   the   reality   that   divorce   is   a   possibility   in   marriages   between   a   foreign  decree  as  an  integral  aspect  of  his  claim  or  defense.  
nd
Filipino   and   an   alien,  the   2   par.   of   Art.   26   was   enacted  "to   avoid   the   absurd    
situation   where   the   Filipino   spouse   remains   married   to   the   alien   spouse   who,   The   records   show   that   Gerbert   attached   to   his   petition   a   copy   of   the   divorce  
after   obtaining   a   divorce,   is   no   longer   married   to   the   Filipino   spouse."  The   decree,  as  well  as  the  required  certificates  proving  its  authenticity,  but  failed  to  
legislative  intent  is  for  the  benefit  of  the  Filipino  spouse,  by  clarifying  his  or   include  a  copy  of  the  Canadian  law  on  divorce.  Under  this  situation,  the  court  
her   marital   status,   settling   the   doubts   created   by   the   divorce   decree.   may,  at  this  point,  simply  dismiss  the  petition  for  insufficiency  of  supporting  
Essentially,   the   par.   provided   the   Filipino   spouse   a   substantive   right   to   have   his   evidence   as   provided   in   Sec.   24,   Rule   132   of   the   ROC,   unless   it   deems   it   more  
or  her  marriage  to  the  alien  spouse  considered  as  dissolved,  capacitating  him  or   appropriate   to   remand   the   case   to   the   RTC   to   determine   whether   the   divorce  
her  to  remarry.  Art.  17,  NCC  provides  that  the  policy  against  absolute  divorces   decree  is  consistent  with  the  Canadian  divorce  law.  
cannot   be   subverted   by   judgments   promulgated   in   a   foreign   country.   The    
nd
inclusion  of  the  2  par.  in  Art.  26,  FC  provides  the  direct  exception  to  this  rule   The   Court   deemed   it   more   appropriate   to   take   the   latter   course   of   action,  
and   serves   as   basis   for   recognizing   the   dissolution   of   the   marriage   between   given   that   interests   under   Art.   26   will   be   served   and   the   Daisylyn’s   obvious  
the  Filipino  spouse  and  his  or  her  alien  spouse.   conformity  with  the  petition.  
   
Additionally,   an   action   based   on   the   second   paragraph   of   Article   26   of   the   N.B.:   The   Court   also   ruled   that   the   law   and   the   submission   of   the   decree   by  
Family   Code   is   not   limited   to   the   recognition   of   the   foreign   divorce   decree.   If   themselves  do  not  ipso  facto  authorize  a  divorce  decree’s  registration.  The  law  
the   court   finds   that   the   decree   capacitated   the   alien   spouse   to   remarry,   the   should  be  read  in  relation  with  the  requirement  of  a  judicial  recognition  of  the  
courts   can   declare   that   the   Filipino   spouse   is   likewise   capacitated   to   contract   foreign  judgment  before  it  can  be  given  res  judicata  effect.    
another  marriage.   No   court   in   this   jurisdiction,   however,   can   make   a   similar      
declaration   for   the   alien   spouse   (other   than   that   already   established   by   the   Rule   132,   Sec.   24.   Proof   of   official   record.  —  The   record   of   public   documents  
decree),   whose   status   and   legal   capacity   are   generally   governed   by   his   referred   to   in   paragraph   (a)   of   Section   19,   when   admissible   for   any   purpose,  
national  law.   may  be  evidenced  by  an  official  publication  thereof  or  by  a  copy  attested  by  the  
  officer   having   the   legal   custody   of   the   record,   or   by   his   deputy,   and  
However,   this   conclusion   is   not   sufficient   basis   to   dismiss   Gerbert’s   petition   accompanied,  if  the  record  is  not  kept  in  the  Philippines,  with  a  certificate  that  
nd
before  the  RTC.  In  other  words,  the  unavailability  of  the  2  par.  of  Art.  26  to   such  officer  has  the  custody.  If  the  office  in  which  the  record  is  kept  is  in  foreign  
aliens  does  not  necessarily  strip  Gerbert  of  legal  interest  to  petition  the  RTC   country,   the   certificate   may   be   made   by   a   secretary   of   the   embassy   or   legation,  
for  the  recognition  of  his  foreign  divorce  decree.  The  foreign  divorce  decree   consul   general,   consul,   vice   consul,   or   consular   agent   or   by   any   officer   in   the  
itself,  after  its  authenticity  and  conformity  with  the  alien’s  national  law  have   foreign   service   of   the   Philippines   stationed   in   the   foreign   country   in   which   the  
been   duly   proven   according   to   our   rules   of   evidence,   serves   as   a   presumptive   record  is  kept,  and  authenticated  by  the  seal  of  his  office.  (25a)  
 
CONFLICT  OF  LAWS                                                                                    AV  DE  TORRES   17  
ATTY.  ARIS  L.  GULAPA                                            AY  2015-­‐2016  
  RUST,   essentially   reiterating   the   allegations   in   the   labor   case.   The   complaint  
12.  RAYTHEON  INTERNATIONAL  V.  ROUZIE   also   averred   that   BMSI   and   RUST   as   well   as   Raytheon   had   combined   and  
G.R.  No.  162894  |  February  26,  2008   functioned  as  one  company.  
   
DOCTRINES:  Jurisdiction  and  choice  of  law  are  two  distinct  concepts.  Jurisdiction   In  its  Answer,  Raytheon   referred  to  the  NLRC  decision  which  disclosed  that  per  
considers  whether  it  is  fair  to  cause  a  defendant  to  travel  to  this  state;  choice  of   the   written   agreement   between   Rouzie   and   BMSI   and   RUST,   denominated   as  
law   asks   the   further   question   whether   the   application   of   a   substantive   law   "Special   Sales   Representative   Agreement,"   the   rights   and   obligations   of   the  
which  will  determine  the  merits  of  the  case  is  fair  to  both  parties.     parties   shall   be   governed   by   the   laws   of   the   State   of   Connecticut.   Raytheon  
  filed   an   Omnibus   Motion   for   the   dismissal   of   the   complaint   on   grounds   of  
While   it   is   within   the   discretion   of   the   trial   court   to   abstain   from   assuming   failure   to   state   a   cause   of   action   and   forum   non   conveniens   and   prayed   for  
jurisdiction   on   the   ground   of   forum   non   conveniens,   it   should   do   so   only   after   damages.  The  RTC  denied  Raytheon’s  omnibus  motion  stating  that  the  factual  
vital  facts  are  established,  to  determine  whether  special  circumstances  require   allegations  in  the  complaint,  assuming  the  same  to  be  admitted,  were  sufficient  
the  court’s  desistance.   for   the   trial   court   to   render   a   valid   judgment   thereon.   It   also   ruled   that   the  
  principle   of   forum   non   conveniens   was   inapplicable   because   the   trial   court  
There   are   3   consecutive   phases   involved   in   judicial   resolution   of   conflict-­‐of-­‐laws   could   enforce   judgment   on   Raytheon,   it   being   a   foreign   corporation   licensed   to  
problems,  namely:  jurisdiction,  choice  of  law,  and  recognition  and  enforcement   do  business  in  the  Philippines.  On  appeal,  the  CA  deferred  to  the  discretion  of  
of   judgments.   Thus,   in   the   instances   where   the   Court   held   that   the   local   judicial   the  trial  court  when  the  latter  decided  not  to  desist  from  assuming  jurisdiction  
machinery   was   adequate   to   resolve   controversies   with   a   foreign   element,   the   on  the  ground  of  the  inapplicability  of  the  principle  of  forum  non  conveniens.  
following   requisites   had   to   be   proved:   (1)   that   the   Philippine   Court   is   one   to    
which  the  parties  may  conveniently  resort;  (2)  that  the  Philippine  Court  is  in  a   ISSUE:  
position  to  make  an  intelligent  decision  as  to  the  law  and  the  facts;  and  (3)  that   Whether   the   CA   erred   in   refusing   to   dismiss   the   complaint   on   the   ground   of  
the  Philippine  Court  has  or  is  likely  to  have  the  power  to  enforce  its  decision.   forum  non  conveniens  
   
FACTS:   HELD:  
In  1990,  Brand  Marine  Services,  Inc.  (BMSI),  a  corporation  duly  organized  and   No.   Raytheon   mainly   asserts   that   the   written   contract   between   Rouzie   and  
existing  under  the  laws  of  the  State  of  Connecticut,  USA,  and  Stockton  Rouzie,   BMSI  included  a  valid  choice  of  law  clause—the  contract  shall  be  governed  by  
Jr.,  an  American  citizen,  entered  into  a  contract  whereby  BMSI  hired  Rouzie  as   the   laws   of   the   State   of   Connecticut.   It   also   mentions   the   presence   of   foreign  
its   representative   to   negotiate   the   sale   of   services   in   several   government   elements   in   the   dispute–the   parties   and   witnesses   involved   are   American  
projects   in   the   Philippines   for   an   agreed   remuneration   of   10%   of   the   gross   corporations  and  citizens  and  the  evidence  to  be  presented  is  located  outside  
receipts.  Rouzie  secured  a  service  contract  with  the  Republic  of  the  Philippines   the  Philippines–that  renders  our  local  courts  inconvenient  forums.  
on   behalf   of   BMSI   for   the   dredging   of   rivers   affected   by   the   Mt.   Pinatubo    
eruption  and  mudflows.   Recently   in   Hasegawa   v.   Kitamura,   the   Court   outlined   3   consecutive   phases  
  involved   in   judicial   resolution   of   conflict-­‐of-­‐laws   problems,   namely:  
In   1994,   Rouzie   filed   before   the   NLRC   a   suit   against   BMSI   and   Rust   jurisdiction,   choice   of   law,   and   recognition   and   enforcement   of   judgments.  
International,   Inc.   (RUST),   Gilbert   and   Browning   for   alleged   nonpayment   of   Thus,  in  the  instances  where  the  Court  held  that  the  local  judicial  machinery  
commissions,   illegal   termination   and   breach   of   employment   contract.   The   was  adequate  to  resolve  controversies  with  a  foreign  element,  the  following  
complaint  was  dismissed  on  the  ground  of  lack  of  jurisdiction.  In  1999,  Rouzie,   requisites  had  to  be  proved:  (1)  that  the  Philippine  Court  is  one  to  which  the  
then   a   resident   of   La   Union,   instituted   an   action   for   damages   before   the   RTC   of   parties  may  conveniently  resort;  (2)  that  the  Philippine  Court  is  in  a  position  
Bauang,   La   Union   against   Raytheon   International,   Inc.   as   well   as   BMSI   and   to   make   an   intelligent   decision   as   to   the   law   and   the   facts;   and   (3)   that   the  
Philippine  Court  has  or  is  likely  to  have  the  power  to  enforce  its  decision.  
 
CONFLICT  OF  LAWS                                                                                    AV  DE  TORRES   18  
ATTY.  ARIS  L.  GULAPA                                            AY  2015-­‐2016  
  foreign  elements.  In  the  same  manner,  the  Court  defers  to  the  sound  discretion  
On   the   matter   of   jurisdiction   over   a   conflicts-­‐of-­‐laws  problem  where   the   case   of  the  lower  courts  because  their  findings  are  binding  on  this  Court.  
is   filed   in   a   Philippine   court   and   where   the   court   has   jurisdiction   over   the    
subject  matter,  the  parties  and  the  res,  it  may  or  can  proceed  to  try  the  case    
even  if  the  rules  of  conflict-­‐of-­‐laws  or  the  convenience  of  the  parties  point  to  
MINIMUM  CONTACT  
a   foreign   forum.   This   is   an   exercise   of   sovereign   prerogative   of   the   country    
where  the  case  is  filed.    
  13.  HSBC  V.  SHERMAN  
Jurisdiction   over   the   nature   and   subject   matter   of   an   action   is   conferred   by   G.R.  No.  72494  |  August  11,  1989  
the  Constitution  and  the  law  and  by  the  material  allegations  in  the  complaint,    
irrespective  of  whether  or  not  the  plaintiff  is  entitled  to  recover  all  or  some  of   DOCTRINES:   A   State   does   not   have   jurisdiction   in   the   absence   of   some  
the   claims   or   reliefs   sought   therein.  This   case   is   an   action   for   damages   arising   reasonable  basis  for  exercising  it,  whether  the  proceedings  are  in  rem,  quasi  in  
from  an  alleged  breach  of  contract.  Undoubtedly,  the  nature  of  the  action  and   rem,  or  in  personam.  To  be  reasonable,  the  jurisdiction  must  be  based  on  some  
the  amount  of  damages  prayed  are  within  the  jurisdiction  of  the  RTC.   minimum   contacts   that   will   not   offend   traditional   notions   of   fair   play   and  
  substantial  justice.  
As   regards   jurisdiction   over   the   parties,   the   trial   court   acquired   jurisdiction    
over   herein   Rouzie   upon   the   filing   of   the   complaint.   On   the   other   hand,   The  defense  of  private  respondents  that  the  complaint  should  have  been  filed  in  
jurisdiction   over   the   person   of   Raytheon   was   acquired   by   its   voluntary   Singapore   is   based   merely   on   technicality.   They   did   not   even   claim,   much   less  
appearance  in  court.   prove,   that   the   filing   of   the   action   here   will   cause   them   any   unnecessary  
  trouble,  damage,  or  expense.  On  the  other  hand,  there  is  no  showing  that  HSBC  
That   the   subject   contract   included   a   stipulation   that   the   same   shall   be   filed  the  action  here  just  to  harass  private  respondents.  
governed   by   the   laws   of   the   State   of   Connecticut   does   not   suggest   that   the    
Philippine   courts,   or   any   other   foreign   tribunal   for   that   matter,   are   precluded   Av:  Since  jurisdiction  asks  whether  it  is  fair  to  cause  defendant  to  travel  to  this  
from   hearing   the   civil   action.   Jurisdiction   and   choice   of   law   are   two   distinct   state,   defendants   must   allege   and   prove   that   the   filing   of   the   action   in   the  
concepts.   Jurisdiction   considers   whether   it   is   fair   to   cause   a   defendant   to   forum  will  cause  unnecessary  trouble,  damage,  or  expense  on  his  part  and  that  
travel   to   this   state;   choice   of   law   asks   the   further   question   whether   the   it  was  filed  to  harass  him.  
application  of  a  substantive  law  which  will  determine  the  merits  of  the  case  is    
fair   to   both   parties.  The   choice   of   law   stipulation   will   become   relevant   only   FACTS:  
when  the  substantive  issues  of  the  instant  case  develop,  that  is,  after  hearing   A   complaint   for   collection   of   a   sum   of   money   was   filed   by   Hongkong   and  
on  the  merits  proceeds  before  the  trial  court.   Shanghai  Banking  Corporation  (HSBC)  against  private  respondents  Jack  Robert  
  Sherman  and  Deodato  Reloj,  before  the  RTC  QC.  
Moreover,   the   propriety   of   dismissing   a   case   based   on   the   principle   of  forum    
non   conveniens  requires   a   factual   determination;   hence,   it   is   more   properly   In   1981,   Eastern   Book   Supply   Service   PTE,   Ltd.   (company),   a   company  
considered  as  a  matter  of  defense.  While  it  is  within  the  discretion  of  the  trial   incorporated   in   Singapore   applied   with,   and   was   granted   by,   the   Singapore  
court   to   abstain   from   assuming   jurisdiction   on   this   ground,   it   should   do   so   branch   of   HSBC   an   overdraft   facility.   As   a   security   for   the   repayment   of   sums  
only   after   vital   facts   are   established,   to   determine   whether   special   advanced   through   the   aforesaid   overdraft   facility,   private   respondents   and   a  
circumstances  require  the  court’s  desistance.   certain   Robin   de   Clive   Lowe,   all   of   whom   were   directors   of   the   company,  
  executed  a  Joint  and  Several  Guarantee  in  favor  of  HSBC  whereby  the  former  
Finding   no   grave   abuse   of   discretion   on   the   trial   court,   the   CA   respected   its   agreed   to   pay,   jointly   and   severally,   on   demand   all   sums   owed   by   the  
conclusion  that  it  can  assume  jurisdiction  over  the  dispute  notwithstanding  its  
 
CONFLICT  OF  LAWS                                                                                    AV  DE  TORRES   19  
ATTY.  ARIS  L.  GULAPA                                            AY  2015-­‐2016  
COMPANY   to   petitioner   BANK   under   the   aforestated   overdraft   facility.   The    
Joint   and   Several   Guarantee   provides   that:   “This   guarantee   and   all   rights,   The   instant   case   presents   a   very   odd   situation.   In   the   ordinary   habits   of   life,  
obligations  and  liabilities  arising  hereunder  shall  be  construed  and  determined   anyone   would   be   disinclined   to   litigate   before   a   foreign   tribunal,   with   more  
under   and   may   be   enforced   in   accordance   with   the   laws   of   the   Republic   of   reason   as   a   defendant.   However,   in   this   case,   private   respondents   are  
Singapore.   We   hereby   agree   that   the   Courts   of   Singapore   shall   have   jurisdiction   Philippine   residents   (a   fact   which   was   not   disputed   by   them)   who   would   rather  
over   all   disputes   arising   under   this   guarantee.”   The   company   failed   to   pay   its   face  a  complaint  against  them  before  a  foreign  court  and  in  the  process  incur  
obligation.   Thus,   HSBC   demanded   payment   but   for   failure   to   pay,   HSBC   filed   considerable   expenses,   not   to   mention   inconvenience,   than   to   have   a  
the  above-­‐mentioned  complaint.   Philippine  court  try  and  resolve  the  case.  The   defense   of   private   respondents  
  that   the   complaint   should   have   been   filed   in   Singapore   is   based   merely   on  
Private  respondents  filed  a  MTD  on  the  ground  of  lack  of  jurisdiction  over  the   technicality.  They   did   not   even   claim,   much   less   prove,  that  the  filing  of  the  
subject   matter   and   over   their   persons.   The   trial   court   denied   the   MTD,   ruling   action  here  will  cause  them  any  unnecessary  trouble,  damage,  or  expense.  On  
that  there  is  nothing  in  the  Guarantee  which  says  that  the  courts  of  Singapore   the   other   hand,   there   is   no   showing   that   HSBC   filed   the   action   here   just   to  
shall   have   jurisdiction   to   the   exclusion   of   the   courts   of   other   countries   or   harass  private  respondents.  
nations   and   that   jurisdiction   over   the   persons   of   defendants   is   acquired   by    
service  of  summons  and  copy  of  the  complaint  on  them.  There  has  been  a  valid   Further,   the   parties   did   not   thereby   stipulate   that   only   the   courts   of  
service  of  summons  on  both  defendants  and  in  fact  the  same  is  admitted  when   Singapore,   to   the   exclusion   of   all   the   rest,   has   jurisdiction.   Neither   did   the  
said   defendants   filed   a   'Motion   for   Extension   of   Time   to   File   Responsive   clause   in   question   operate   to   divest   Philippine   courts   of   jurisdiction.   In  
Pleading.’   Private   respondents   then   filed   before   CA   a   petition   for   prohibition   International   Law,   jurisdiction   is   often   defined   as   the   right   of   a   State   to  
with  preliminary  injunction  and/or  prayer  for  a  restraining  order.  CA  rendered  a   exercise   authority   over   persons   and   things   within   its   boundaries   subject   to  
decision   granting   the   injunction   stating   that:   “The   loan   was   obtained   by   a   certain   exceptions.   Thus,   a   State   does   not   assume   jurisdiction   over   travelling  
company   incorporated   in  Singapore.   The   loan   was   granted   by   the   sovereigns,   ambassadors   and   diplomatic   representatives   of   other   States,   and  
Singapore  Branch  of  HSBC.  The  Joint  and  Several  Guarantee  was  also  concluded   foreign  military  units  stationed  in  or  marching  through  State  territory  with  the  
in  Singapore.   The   loan   was   in   Singaporean   dollars   and   the   repayment   thereof   permission   of   the   latter's   authorities.   This   authority,   which   finds   its   source   in  
also   in   the   same   currency.   The   transaction,   to   say   the   least,   took   place   in   the   concept   of   sovereignty,   is   exclusive   within   and   throughout   the   domain   of  
Singporean   setting   in   which   the   law   of   that   country   is   the   measure   by   which   the  State.  A  State  is  competent  to  take  hold  of  any  judicial  matter  it  sees  fit  by  
that  relationship  of  the  parties  will  be  governed.”   making   its   courts   and   agencies   assume   jurisdiction   over   all   kinds   of   cases  
  brought  before  them.  
ISSUE:    
Whether  Philippine  courts  have  jurisdiction  over  the  suit.   While   in   the   main,   the   motion   to   dismiss   fails   to   categorically   use   with  
  exactitude   the   words   'improper   venue'   it   can   be   perceived   from   the   general  
HELD:   thrust   and   context   of   the   motion   that   what   is   meant   is   improper   venue,   The  
Yes.  While  it  is  true  that  "the  transaction  took  place  in  Singaporean  setting"  and   use  of  the  word  'jurisdiction'  was  merely  an  attempt  to  copy-­‐cat  the  same  word  
that   the   Joint   and   Several   Guarantee   contains   a   choice-­‐of-­‐forum   clause,   the   employed   in   the   guarantee   agreement   but   conveys   the   concept   of   venue.   At  
very   essence   of   due   process   dictates   that   the   stipulation   be   liberally   any  rate,  this  issue  is  now  of  no  moment  because  the  Court  holds  that  venue  
construed.   One   basic   principle   underlies   all   rules   of   jurisdiction   in   International   here  was  properly  laid  for  the  same  reasons  discussed  above.  
Law:   A   State   does   not   have   jurisdiction   in   the   absence   of   some   reasonable    
basis  for  exercising  it,  whether  the  proceedings  are  in  rem  quasi  in  rem  or  in   The   respondent   Court   likewise   ruled   that:   “In   a   conflict   problem,   a   court   will  
personam.   To   be   reasonable,   the   jurisdiction   must   be   based   on   some   simply   refuse   to   entertain   the   case   if   it   is   not   authorized   by   law   to   exercise  
minimum   contacts   that   will   not   offend   traditional   notions   of   fair   play   and   jurisdiction.   And   even   if   it   is   so   authorized,   it   may   still   refuse   to   entertain   the  
substantial  justice.   case  by  applying  the  principle  of  forum  non  conveniens.”  However,  whether  a  
 
CONFLICT  OF  LAWS                                                                                    AV  DE  TORRES   20  
ATTY.  ARIS  L.  GULAPA                                            AY  2015-­‐2016  
suit  should  be  entertained  or  dismissed  on  the  basis  of  the  principle  of  forum   relationships   of   the   ship   and   of   its   master   or   owner   as   such.   It   also   covers  
non   conveniens   depends   largely   upon   the   facts   of   the   particular   case   and   is   contractual  relationships  particularly  contracts  of  affreightment.  
addressed   to   the   sound   discretion   of   the   trial   court.   Thus,   the   CA   should   not    
have  relied  on  such  principle.   What  is  important  here  is  the  place  where  the  over-­‐all  harm  or  the  totality  of  
  the  alleged  injury  to  the  person,  reputation,  social  standing  and  human  rights  
Although   the   Joint   and   Several   Guarantee   prepared   by   HSBC   is   a   contract   of   of   complainant,   had   lodged,   according   to   Morada.   All   told,   it   is   not   without  
adhesion   and   that   consequently,   it   cannot   be   permitted   to   take   a   stand   basis  to  identify  the  Philippines  as  the  situs  of  the  alleged  tort.  
contrary   to   the   stipulations   of   the   contract,   substantial   bases   exist   for   HSBC’s    
choice  of  forum,  as  discussed  earlier.   ALG:   Center   of   gravity   approach—the   law   of   the   state   which   has   the   most  
  important  connection  or  is  most  intimately  connected  with  the  outcome  of  the  
14.  SAUDI  ARABIAN  AIRLINES  V.  CA   particular  litigation  must  be  applied.  (PIA  v.  CA))  
G.R.  No.  122191  |  October  8,  1998    
  In   applying   the   “State   of   the   most   significant   relationship”   rule,   the   following  
DOCTRINES:   A  factual  situation  that  cuts  across  territorial  lines  and  is  affected   contacts  are  to  be  taken  into  account  and  evaluated  according  to  their  relative  
by  the  diverse  laws  of  two  or  more  states  is  said  to  contain  a  “foreign  element.”   importance  with  respect  to  the  particular  issue:  (a)  the  place  where  the  injury  
  occurred;  (b)  the  place  where  the  conduct  causing  the  injury  occurred;  (c)  the  
These  “test  factors”  or  “points  of  contact”  or   “connecting  factors”  could  be  any   domicile,   residence,   nationality,   place   of   incorporation   and   place   of   business   of  
of  the  following:  (SAFE  IN  SF)   the   parties;   and   (d)   the   place   where   the   relationship,   if   any,   between   the  
(1)  the  nationality  of  a  person,  his  domicile,  his  residence,  his  place  of  sojourn,   parties  is  centered.  
or  his  origin;    
(2)  the  seat  of  a  legal  or  juridical  person,  such  as  a  corporation;   CHOICE  OF  FORUM  
(3)  the  situs  of  a  thing,  that  is,  the  place  where  a  thing  is,  or  is  deemed  to  be   GR:  The  plaintiff’s  choice  of  forum  should  rarely  be  disturbed.  
situated.  In  particular,  the  lex  situs  is  decisive  when  real  rights  are  involved;   EXC  #3:  Unless  the  balance  is  strongly  in  favor  of  the  defendant.  Plaintiff  may  
(4)   the   place   where   an   act   has   been   done,   the   locus   actus,   such   as   the   place   not,   by   choice   of   an   inconvenient   forum,   ‘vex,’   ‘harass,’   or   ‘oppress’   the  
where  a  contract  has  been  made,  a  marriage  celebrated,  a  will  signed  or  a  tort   defendant,  e.g.  by  inflicting  upon  him  needless  expense  or  disturbance.    
committed.  The  lex  loci  actus  is  particularly  important  in  contracts  and  torts;    
(5)   the   place   where   an   act   is   intended   to   come   into   effect,   e.g.,   the   place   of   FACTS:  
performance  of  contractual  duties,  or  the  place  where  a  power  of  attorney  is  to   In   1988,   SAUDIA   hired   Milagros   Morada   as   a   Flight   Attendant   for   its   airlines  
be  exercised;   based   in   Jeddah,   Saudi   Arabia.   In   1990,   while   on   a   lay-­‐over   in   Jakarta,  
(6)   the   intention   of   the   contracting   parties   as   to   the   law   that   should   govern   Indonesia,   Morada   went   to   a   disco   dance   with   fellow   crew   members   Thamer  
their  agreement,  the  lex  loci  intentionis;   Al-­‐Gazzawi   and   Allah   Al-­‐Gazzawi,   both   Saudi   nationals.   Because   it   was   almost  
(7)   the   place   where   judicial   or   administrative   proceedings   are   instituted   or   morning   when   they   returned   to   their   hotels,   they   agreed   to   have   breakfast  
done.  The  lex  fori—the  law  of  the  forum—is  particularly  important  because,  as   together  at  the  room  of  Thamer.  When  they   were  in  the  room,  Allah  left  and  
we  have  seen  earlier,  matters  of  ‘procedure’  not  going  to  the  substance  of  the   shortly   after,   Thamer   attempted   to   rape   her.   Fortunately,   a   roomboy   and  
claim   involved   are   governed   by   it;   and   because   the   lex   fori   applies   whenever   several  security  personnel  heard  her  cries  for  help  and  rescued  her.  Later,  the  
the   content   of   the   otherwise   applicable   foreign   law   is   excluded   from   Indonesian  police  came  and  arrested  Thamer  and  Allah  Al-­‐Gazzawi,  the  latter  as  
application   in   a   given   case   for   the   reason   that   it   falls   under   one   of   the   an  accomplice.  
exceptions  to  the  applications  of  foreign  law;  and    
(8)   the   flag   of   a   ship,   which   in   many   cases   is   decisive   of   practically   all   legal   When   plaintiff   returned   to   Jeddah   a   few   days   later,   SAUDIA   officials  

 
CONFLICT  OF  LAWS                                                                                    AV  DE  TORRES   21  
ATTY.  ARIS  L.  GULAPA                                            AY  2015-­‐2016  
interrogated   her   about   the   Jakarta   incident.   They   then   requested   her   to   go   She  was  denied  any  assistance  by  SAUDIA  thus,  she  then  asked  the  Philippine  
back   to   Jakarta   to   help   arrange   the   release   of   Thamer   and   Allah.   Morada   did   Embassy  in  Jeddah  to  help  her  while  her  case  is  on  appeal.  Meanwhile,  to  pay  
not   cooperate   in   the   negotiation   with   the   Indonesian   police   because   she   was   for   her   upkeep,   she   worked   on   the   domestic   flight   of   SAUDIA,   while   Thamer  
afraid  that  she  might  be  tricked  into  something  she  did  not  want  because  of  her   and   Allah   continued   to   serve   in   the   international   flights.   Because   she   was  
inability   to   understand   the   local   dialect.   She   also   declined   to   sign   a   blank   paper   wrongfully  convicted,  the  Prince  of  Makkah  dismissed  the  case  against  her  and  
and   a   document   written   in   the   local   dialect.   However,   she   learned   that,   allowed  her  to  leave  Saudi  Arabia.  Shortly  before  her  return  to  Manila,  she  was  
through   the   intercession   of   the   Saudi   Arabian   government,   the   Indonesian   terminated   from   the   service   by   SAUDIA,   without   her   being   informed   of   the  
authorities   agreed   to   deport   Thamer   and   Allah   after   2   weeks   of   detention.   cause.  
Eventually,  they  were  again  put  in  service  by  defendant  SAUDIA.  In  September    
1990,  SAUDIA  transferred  Morada  to  Manila.   In  November  1993,  Morada  filed  a  Complaint  for  damages  against  SAUDIA,  and  
  Khaled   Al-­‐Balawi,   its   country   manager   in   the   trial   court.   SAUDIA   filed   an  
In  1992,  Morada’s  superiors  requested  her  to  see  Mr.  Ali  Meniewy,  Chief  Legal   Omnibus   MTD   which   raised   the   following   grounds,   to   wit:   (1)   that   the  
Officer  of  SAUDIA,  in  Jeddah,  Saudi  Arabia.  When  she  saw  him,  he  brought  her   Complaint   states   no   cause   of   action   against   SAUDIA;   (2)   that   defendant   Al-­‐
to   the   police   station   where   the   police   took   her   passport   and   questioned   her   Balawi  is  not  a  real  party  in  interest;  (3)  that  the  claim  or  demand  set  forth  in  
about  the  Jakarta  incident.  Not  until  she  agreed  to  do  so  did  the  police  return   the   Complaint   has   been   waived,   abandoned   or   otherwise   extinguished;   and   (4)  
her  passport  and  allowed  her  to  catch  the  afternoon  flight  out  of  Jeddah.   that  the  trial  court  has  no  jurisdiction  to  try  the  case  on  the  basis  of  Art.  21  of  
  the   NCC,   since   the   proper   law   applicable   is   the   law   of   the   Kingdom   of   Saudi  
One   year   and   a   half   later,   in   Riyadh,   Saudi   Arabia,   a   few   minutes   before   the   Arabia.  The  trial  court  denied  the  MTD.  
departure  of  her  flight  to  Manila,  Morada  was  not  allowed  to  board  the  plane    
and  instead  ordered  to  take  a  later  flight  to  Jeddah  to  see  Mr.  Miniewy  again.   Consequently,   SAUDIA   filed   its   Petition   for   Certiorari   and   Prohibition   with  
When   she   did,   she   was   brought   her   to   a   Saudi   court   where   she   was   asked   to   Prayer   for   Issuance   of   Writ   of   Preliminary   Injunction   and/or   TRO   with   the   CA,  
sign   a   document   written   in   Arabic.   They   told   her   that   this   was   necessary   to   which   ruled   that   the   Philippines   is   an   appropriate   forum   considering   that   the  
close   the   case   against   Thamer   and   Allah.   As   it   turned   out,   plaintiff   signed   a   Amended  Complaint’s  basis  for  recovery  of  damages  is  Art.  21,  and  thus,  clearly  
notice  to  her  to  appear  before  the  court  on  June  27,  1993.  She  then  returned  to   within  the  jurisdiction  of  respondent  Court.  
Manila.    
  ISSUE:  
Shortly   afterwards,   SAUDIA   summoned   plaintiff   to   report   to   Jeddah   once   again   1.  Whether  the  case  is  a  conflict  of  laws.  
where  a  Saudi  judge  interrogated  her  through  an  interpreter  about  the  Jakarta   2.  Whether  the  RTC  of  Quezon  City  has  jurisdiction.  
incident.   After   one   hour   of   interrogation,   when   she   was   about   to   return   to   3.  Whether  the  Philippine  Law  should  govern.  
Manila,   a   SAUDIA   officer   told   her   that   the   airline   had   forbidden   her   to   take    
flight  and  took  away  her  passport  and  told  her  to  remain  in  Jeddah,  at  the  crew   HELD:  
quarters,   until   further   orders.   After   a   few   days,   a   SAUDIA   legal   officer   again   1.  Yes.  A   factual   situation   that   cuts   across   territorial   lines   and   is   affected   by  
escorted  plaintiff  to  the  same  court  where  the  judge,  to  her  astonishment  and   the  diverse  laws  of  two  or  more  states  is  said  to  contain  a  “foreign  element.”    
shock,   rendered   a   decision   sentencing   her   to   5   months   imprisonment   and   to    
286   lashes.   Only   then   did   she   realize   that   the   Saudi   court   had   tried   her,   The   forms   in   which   this   foreign   element   may   appear   are   many.   The   foreign  
together  with  Thamer  and  Allah,  for  what  happened  in  Jakarta.  The  court  found   element  may  simply  consist  in  the  fact  that  one  of  the  parties  to  a  contract  is  an  
Morada  guilty  of  (1)  adultery;  (2)  going  to  a  disco,  dancing  and  listening  to  the   alien   or   has   a   foreign   domicile,   or   that   a   contract   between   nationals   of   one  
music   in   violation   of   Islamic   laws;   and   (3)   socializing   with   the   male   crew,   in   State  involves  properties  situated  in  another  State.  In  other  cases,  the  foreign  
contravention  of  Islamic  tradition.”   element  may  assume  a  complex  form.  In  the  instant  case,  the  foreign  element  
  consisted   in   the   fact   that   Morada   is   a   resident   Philippine   national,   and   that  
 
CONFLICT  OF  LAWS                                                                                    AV  DE  TORRES   22  
ATTY.  ARIS  L.  GULAPA                                            AY  2015-­‐2016  
SAUDIA  is  a  resident  foreign  corporation.  Also,  by  virtue  of  the  employment  of   “(1)   the   nationality   of   a   person,   his   domicile,   his   residence,   his   place   of  
Morada  with  the  SAUDIA  as  a  flight  stewardess,  events  did  transpire  during  her   sojourn,  or  his  origin;  
many   occasions   of   travel   across   national   borders,   particularly   from   Manila,   (2)  the  seat  of  a  legal  or  juridical  person,  such  as  a  corporation;  
Philippines   to   Jeddah,   Saudi   Arabia,   and   vice   versa,   that   caused   a   “conflicts”   (3)  the  situs  of  a  thing,  that  is,  the  place  where  a  thing  is,  or  is  deemed  to  be  
situation  to  arise.  Thus,  Morada’s  assertion  that  the  case  is  purely  domestic  is   situated.  In  particular,  the  lex  situs  is  decisive  when  real  rights  are  involved;  
imprecise.   A   conflicts   problem   presents   itself   here,   and   the   question   of   (4)   the   place   where   an   act   has   been   done,   the   locus   actus,   such   as   the   place  
jurisdiction  confronts  the  court  a  quo.   where  a  contract  has  been  made,  a  marriage  celebrated,  a  will  signed  or  a  
  tort  committed.  The  lex  loci  actus  is  particularly  important  in  contracts  and  
2)   Yes.   Based   on   the   allegations   in   the   Amended   Complaint,   read   in   the   light   of   torts;  
the  Rules  of  Court  on  jurisdiction,  the  Court  finds  that  the  RTC  of  Quezon  City   (5)  the  place  where  an  act  is  intended  to  come  into  effect,  e.g.,  the  place  of  
possesses   jurisdiction   over   the   subject   matter   of   the   suit.   Its   authority   to   try   performance  of  contractual  duties,  or  the  place  where  a  power  of  attorney  
and   hear   the   case   is   provided   for   under   Sec.   1   of   R.A.   No.   7691:   “RTCs   shall   is  to  be  exercised;  
exercise  exclusive  jurisdiction:  (8)  In  all  other  cases  in  which  demand,  exclusive   (6)  the  intention  of  the  contracting  parties  as  to  the  law  that  should  govern  
of  interest,  damages  of  whatever  kind,  attorney's  fees,  litigation  expenses,  and   their  agreement,  the  lex  loci  intentionis;  
costs  or  the  value  of  the  property  in  controversy  exceeds  One  hundred  thousand   (7)   the   place   where   judicial   or   administrative   proceedings   are   instituted   or  
pesos   (P100,000.00)   or,   in   such   other   cases   in   Metro   Manila,   where   the   done.   The   lex   fori—the   law   of   the   forum—is   particularly   important   because,  
demand,   exclusive   of   the   above-­‐mentioned   items   exceeds   Two   hundred   as  we  have  seen  earlier,  matters  of  ‘procedure’  not  going  to  the  substance  
Thousand  pesos  (P200,000.00).”   of   the   claim   involved   are   governed   by   it;   and   because   the   lex   fori   applies  
  whenever   the   content   of   the   otherwise   applicable   foreign   law   is   excluded  
Pragmatic  considerations,  including  the  convenience  of  the  parties,  also  weigh   from  application  in  a  given  case  for  the  reason  that  it  falls  under  one  of  the  
heavily   in   favor   of   the   RTC   of   QC   assuming   jurisdiction.   Paramount   is   the   exceptions  to  the  applications  of  foreign  law;  and  
private   interest   of   the   litigant.   Enforceability   of   a   judgment   if   one   is   obtained   is   (8)  the  flag  of  a  ship,  which  in  many  cases  is  decisive  of  practically  all  legal  
quite   obvious.   Relative   advantages   and   obstacles   to   a   fair   trial   are   equally   relationships   of   the   ship   and   of   its   master   or   owner   as   such.   It   also   covers  
important.   Plaintiff   may   not,   by   choice   of   an   inconvenient   forum,   ‘vex,’   contractual  relationships  particularly  contracts  of  affreightment.”  
‘harass,’   or   ‘oppress’   the   defendant,   e.g.   by   inflicting   upon   him   needless    
expense   or   disturbance.   But   unless   the   balance   is   strongly   in   favor   of   the   Considering  that  the  complaint  in  the  court  a  quo  is  one  involving  torts,  the  
defendant,  the  plaintiff’s  choice  of  forum  should  rarely  be  disturbed.   “connecting  factor”  or  “point  of  contact”  could  be  the  place  or  places  where  
  the   tortious   conduct   or   lex   loci   actus   occurred.   And   applying   the   torts  
Weighing   the   relative   claims   of   the   parties,   the   court   a   quo   found   it   best   to   principle   in   a   conflicts   case,   we   find   that   the   Philippines   could   be   said   as   a  
hear  the  case  in  the  Philippines.  Had  it  refused  to  take  cognizance  of  the  case,  it   situs   of   the   tort   (the   place   where   the   alleged   tortious   conduct   took   place).  
would   be   forcing   Morada   to   seek   remedial   action   elsewhere,   i.e.   in   the   This   is   because   it   is   in   the   Philippines   where   SAUDIA   allegedly   deceived  
Kingdom   of   Saudi   Arabia   where   she   no   longer   maintains   substantial   Morada,   a   Filipina   residing   and   working   here.   According   to   her,   she   had  
connections.   That   would   have   caused   a   fundamental   unfairness   to   her.   honestly   believed   that   SAUDIA   would,   in   the   exercise   of   its   rights   and   in   the  
Moreover,  by  hearing  the  case  in  the  Philippines  no  unnecessary  difficulties  and   performance  of  its  duties,  “act  with  justice,  give  her  due  and  observe  honesty  
inconvenience  have  been  shown  by  either  of  the  parties.   and   good   faith.”   Instead,   SAUDIA   failed   to   protect   her.   That   certain   acts   or  
  parts  of  the  injury  allegedly  occurred  in  another  country  is  of  no  moment.  What  
3)   Yes.   Note   that   one   or   more   circumstances   may   be   present   to   serve   as   the   is  important  here  is  the  place  where  the  over-­‐all  harm  or  the  totality  of  the  
possible  test  for  the  determination  of  the  applicable  law.  These  “test   factors”   alleged  injury  to  the  person,  reputation,  social  standing  and  human  rights  of  
or  “points  of  contact”  or  “connecting  factors”  could  be  any  of  the  following:   complainant,  had  lodged,  according  to  Morada.  All  told,  it  is  not  without  basis  
  to  identify  the  Philippines  as  the  situs  of  the  alleged  tort.  
 
CONFLICT  OF  LAWS                                                                                    AV  DE  TORRES   23  
ATTY.  ARIS  L.  GULAPA                                            AY  2015-­‐2016  
   
Moreover,  with  the  widespread  criticism  of  the  traditional  rule  of  lex  loci  delicti   Lex   celebrationis,   lex   contractus,   and   state   of   the   most   significant   relationship  
commissi,   modern   theories   and   rules   on   tort   liability   have   been   advanced   to   make   reference   to   the   law   applicable   to   a   dispute.   They   are   rules   proper   for   the  
offer  fresh  judicial  approaches  to  arrive  at  just  results.  In  keeping  abreast  with   second  phase  in  judicial  resolution  of  conflicts  problem—the  choice  of  law.  
the   modern   theories   on   tort   liability,   we   find   here   an   occasion   to   apply   the    
“State   of   the   most   significant   relationship”   rule,  which  in  our  view  should  be   FACTS:  
appropriate  to  apply  now,  given  the  factual  context  of  this  case.   In   determining   Petitioner   Nippon   Engineering   Consultants   Co.,   Ltd.,   a   Japanese   consultancy  
the  State  which  has  the  most  significant  relationship,  the  following  contacts   firm  providing  technical  and  management  support  in  the  infrastructure  projects  
are   to   be   taken   into   account   and   evaluated   according   to   their   relative   of   foreign   governments,   entered   into   an   Independent   Contractor   Agreement  
importance  with  respect  to  the  particular  issue:  (a)  the  place  where  the  injury   (ICA)   with   Minoru   Kitamura,   a   Japanese   national   permanently   residing   in   the  
occurred;   (b)   the   place   where   the   conduct   causing   the   injury   occurred;   (c)   the   Philippines.  The  agreement  provides  that  Kitamura  was  to  extend  professional  
domicile,  residence,  nationality,  place  of  incorporation  and  place  of  business   services   to   Nippon   for   a   year.   Nippon   then   assigned   Kitamura   to   work   as   the  
of  the  parties;  and  (d)  the  place  where  the  relationship,  if  any,  between  the   project   manager   of   the   Southern   Tagalog   Access   Road   (STAR)   Project   in   the  
parties  is  centered.   Philippines,   following   the   company's   consultancy   contract   with   the   Philippine  
  Government.  When  the  STAR  Project  was  near  completion,  the  DPWH  engaged  
As   already   discussed,   there   is   basis   for   the   claim   that   overall   injury   occurred   the  consultancy  services  of  Nippon,  this  time  for  the  detailed  engineering  and  
and  lodged  in  the  Philippines.  Thus,  the  “relationship”  between  the  parties  was   construction   supervision   of   the   Bongabon-­‐Baler   Road   Improvement   (BBRI)  
centered   here,   although   it   should   be   stressed   that   this   suit   is   not   based   on   Project.  
mere  labor  law  violations.  From  the  record,  the  claim  that  the  Philippines  has    
the   most   significant   contact   with   the   matter   in   this   dispute,   raised   by   private   Petitioner   Kazuhiro   Hasegawa,   Nippon's   general   manager   for   its   International  
respondent   as   plaintiff   below   against   SAUDIA,   in   our   view,   has   been   properly   Division,  then  informed  Kiramura  that  the  company  had  no  more  intention  of  
established.  Indubitably,  the  Philippines  is  the  state  intimately  concerned  with   automatically  renewing  his  ICA.  His  services  would  be  engaged  by  the   company  
the   ultimate   outcome   of   the   case   below,   not   just   for   the   benefit   of   all   the   only  up  to  the  substantial  completion  of  the  STAR  Project.  
litigants,  but  also  for  the  vindication  of  the  country’s  system  of  law  and  justice    
in  a  transnational  setting.   Threatened   with   impending   unemployment,   Kitamura,   through   his   lawyer,  
  requested  a  negotiation  conference  and  demanded  that  he  be  assigned  to  the  
15.  HASEGAWA  V.  KITAMURA   BBRI  project.  Nippon  insisted  that  Kitamura’s  contract  was  for  a  fixed  term  that  
G.R.  No.  149177  |  November  23,  2007   had   already   expired,   and   refused   to   negotiate   for   the   renewal   of   the   ICA.  
  Kitamura   then   filed   for   specific   performance   and   damages   with   the   RTC   of   Lipa  
DOCTRINE:   In   the   judicial   resolution   of   conflicts   problems,   three   consecutive   City.  Nippon  filed  a  MTD,  contending  that  the  ICA  had  been  perfected  in  Japan  
phases   are   involved:   jurisdiction,   choice   of   law,   and   recognition   and   and  executed  by  and  between  Japanese  nationals.  Thus,  petitioners  posit  that  
enforcement   of   judgments.   Corresponding   to   these   phases   are   the   following   local  courts  have  no  substantial  relationship  to  the  parties  following  the  state  
questions:  (1)  Where  can  or  should  litigation  be  initiated?  (2)  Which  law  will  the   of   the   most   significant   relationship   rule   in   PRIL.   The   claim   for   improper   pre-­‐
court  apply?  and  (3)  Where  can  the  resulting  judgment  be  enforced?   termination   of   Kitamaru’s   ICA   could   only   be   heard   and   ventilated   in   the   proper  
  courts   of   Japan   following   the   principles   of   lex   loci   celebrationis   and   lex  
To   succeed   in   its   motion   for   the   dismissal   of   an   action   for   lack   of   jurisdiction   contractus.    
over  the  subject  matter  of  the  claim,  the  movant  must  show  that  the  court  or    
tribunal  cannot  act  on  the  matter  submitted  to  it  because  no  law  grants  it  the   RTC   denied   the   motion   to   dismiss.   The   CA   ruled   hat   the   principle   of   lex   loci  
power  to  adjudicate  the  claims.   celebrationis   was   not   applicable   to   the   case,   because   nowhere   in   the   pleadings  

 
CONFLICT  OF  LAWS                                                                                    AV  DE  TORRES   24  
ATTY.  ARIS  L.  GULAPA                                            AY  2015-­‐2016  
was   the   validity   of   the   written   agreement   put   in   issue.   It   held   that   the   RTC   was   properly   vested   by   law   with   jurisdiction   to   hear   the   subject   controversy   for   a  
correct   in   applying   the   principle   of   lex   loci   solutionis   (law   of   the   place   where   civil  case  for  specific  performance  and  damages  is  one  not  capable  of  pecuniary  
performance  of  the  contract  was  due).   estimation   and   is   properly   cognizable   by   the   RTC   of   Lipa   City.   What   they   rather  
  raise  as  grounds  to  question  subject  matter  jurisdiction  are  the  principles  of  lex  
ISSUE:   loci   celebrationis   and   lex   contractus,   and   the   “state   of   the   most   significant  
Whether   the   subject   matter   jurisdiction   of   Philippine   courts   in   civil   cases   for   relationship  rule.  The  Court  finds  the  invocation  of  these  grounds  unsound.  
specific   performance   and   damages   involving   contracts   executed   outside   the    
country   by   foreign   nationals   may   be   assailed   on   the   principles   of   lex   loci   Lex  loci  celebrationis  relates  to  the  “law  of  the  place  of  the  ceremony”  or  the  
celebrationis,   lex   contractus,   the   “state   of   the   most   significant   relationship   law  of  the  place  where  a  contract  is  made.  The  doctrine  of  lex   contractus   or   lex  
rule,”  or  forum  non  conveniens.   loci  contractus  means  the  “law  of  the  place  where  a  contract  is  executed  or  to  
  be  performed.”  It  controls  the  nature,  construction,  and  validity  of  the  contract  
HELD:   and   it   may   pertain   to   the   law   voluntarily   agreed   upon   by   the   parties   or   the   law  
No.  In  the  judicial  resolution  of  conflicts  problems,  three  consecutive  phases   intended  by  them  either  expressly  or  implicitly.   Under   the   “state   of   the   most  
are  involved:  jurisdiction,  choice  of  law,  and  recognition  and  enforcement  of   significant   relationship   rule,”   to   ascertain   what   state   law   to   apply   to   a  
judgments.   dispute,   the   court   should   determine   which   state   has   the   most   substantial  
  connection  to  the  occurrence  and  the  parties.  In  a  case  involving  a  contract,  
Analytically,   jurisdiction   and   choice   of   law   are   two   distinct   concepts.   The   the  court  should  consider  where  the  contract  was  made,  was  negotiated,  was  
power   to   exercise   jurisdiction   does   not   automatically   give   a   state   to   be   performed,   and   the   domicile,   place   of   business,   or   place   of  
constitutional   authority   to   apply   forum   law.  While   jurisdiction   and   the   choice   incorporation   of   the   parties.   This   rule   takes   into   account   several   contacts   and  
of   the   lex   fori   will   often   coincide,   the   “minimum   contacts”   for   one   do   not   evaluates   them   according   to   their   relative   importance   with   respect   to   the  
always   provide   the   necessary   “significant   contacts”   for   the   other.   The   particular  issue  to  be  resolved.  
question   of   whether   the   law   of   a   state   can   be   applied   to   a   transaction   is    
different   from   the   question   of   whether   the   courts   of   that   state   have   Since   these   3   principles   in   conflict   of   laws   make   reference   to   the   law  
jurisdiction  to  enter  a  judgment.   applicable   to   a   dispute,   they   are   rules   proper   for   the   2nd   phase,   the   choice   of  
  law.   They   determine   which   state's   law   is   to   be   applied   in   resolving   the  
In  this  case,  only  the  first  phase  is  at  issue—jurisdiction.  Jurisdiction,  however,   substantive   issues   of   a   conflicts   problem.   Necessarily,   as   the   only   issue   in   this  
has  various  aspects.  In  assailing  the  trial  court's  jurisdiction  herein,  petitioners   case   is   that   of   jurisdiction,   choice-­‐of-­‐law   rules   are   not   only   inapplicable   but  
are  actually  referring  to  subject  matter  jurisdiction.   also  not  yet  called  for.  
   
Jurisdiction  over  the  subject  matter  in  a  judicial  proceeding  is  conferred  by  the   Further,   Nippon’s   premature   invocation   of   choice-­‐of-­‐law   rules   is   exposed   by  
sovereign  authority,  which  establishes  and  organizes  the  court.  It  is  given  only   the   fact   that   they   have   not   yet   pointed   out   any   conflict   between   the   laws   of  
by   law   and   in   the   manner   prescribed   by   law.   It   is   further   determined   by   the   Japan   and   ours.   Before   determining   which   law   should   apply,   1st   there   should  
allegations  of  the  complaint  irrespective  of  whether  the  plaintiff  is  entitled  to   exist   a   conflict   of   laws   situation   requiring   the   application   of   the   conflict   of   laws  
all   or   some   of   the   claims   asserted   therein.   To   succeed   in   its   motion   for   the   rules.  Also,  when  the  law  of  a  foreign  country  is  invoked  to  provide  the  proper  
dismissal   of   an   action   for   lack   of   jurisdiction   over   the   subject   matter   of   the   rules  for  the  solution  of  a  case,  the  existence  of  such  law  must  be  pleaded  and  
claim,   the   movant   must   show   that   the   court   or   tribunal   cannot   act   on   the   proved.  
matter  submitted  to  it  because  no  law  grants  it  the  power  to  adjudicate  the    
claims.   Neither  can  the  other  ground  raised,  forum  non  conveniens,  be  used  to  deprive  
  the  RTC  of  its  jurisdiction.  First,  it  is  not  a  proper  basis  for  a  motion  to  dismiss  
In  the  instant  case,  petitioners,  in  their  MTD,  do  not  claim  that  the  RTC  is  not   because  Sec.  1,  Rule  16  of  the  Rules  of  Court  does  not  include  it  as  a  ground.  
 
CONFLICT  OF  LAWS                                                                                    AV  DE  TORRES   25  
ATTY.  ARIS  L.  GULAPA                                            AY  2015-­‐2016  
Second,  whether  a  suit  should  be  entertained  or  dismissed  on  the  basis  of  the   required   to   be   made   by   employers   to   a   state   unemployment   compensation  
said   doctrine   depends   largely   upon   the   facts   of   the   particular   case   and   is   fund.  
addressed  to  the  sound  discretion  of  the  RTC.  In   this   case,   the   RTC   decided   to    
assume   jurisdiction.   Third,   the   propriety   of   dismissing   a   case   based   on   this   Appellant   is   a   Delaware   corporation,   having   its   principal   place   of   business   in   St.  
principle   requires   a   factual   determination;   hence,   this   conflicts   principle   is   Louis,   Missouri,   and   is   engaged   in   the   manufacture   and   sale   of   shoes   and   other  
more  properly  considered  a  matter  of  defense.   footwear.   Its   merchandise   is   distributed   interstate   through   several   sales   units  
  or  branches  located  outside  the  State  of  Washington.  Appellant  has  no  office  in  
  Washington,   and   makes   no   contracts   either   for   sale   or   purchase   of  
merchandise  there.  
JURISDICTION  OVER  THE  PERSON  
   
  During   the   years   1937   to   1940,   appellant   employed   11-­‐13   salesmen   under  
16.  INTERNATIONAL  SHOE  CO  V.  WASHINGTON   direct   supervision   and   control   of   sales   managers   located   in   St.   Louis.   These  
326  U.S.  310  (1945)   salesmen  resided  in  Washington;  their  principal  activities  were  confined  to  that  
  state,  and  they  were  compensated  by  commissions  based  upon  the  amount  of  
DOCTRINE:   GR:  Defendant  must  be  present  within  the  territorial  jurisdiction  to   their  sales.  
render  a  judgment  in  personam  against  him.    
  For  the  said  years,  the  State  of  Washington  issued  notice  of  assessment  which  
EXC:   Due   process   requires   that   if   the   defendant   is   not   present   within   the   was   personally   served   upon   the   sales   solicitor   employed   by   it   in   the   State   of  
territory   of   the   forum,   he   has   certain   minimum   contacts   with   it   such   that   the   Washington,   and   a   copy   of   the   notice   was   mailed   by   registered   mail   to  
maintenance   of   the   suit   does   not   offend   ‘traditional   notions   of   fair   play   and   appellant  at  its  address  in  St.  Louis,  Missouri.  
substantial  justice.’    
  Appellant  appeared  specially  before  the  office  of  unemployment,  and  moved  to  
The   test   is   the   extent   that   a   corporation   exercises   the   privilege   of   conducting   set   aside   the   order   and   notice   of   assessment   on   the   ground   that   the   service  
activities  within  a  state,  it  enjoys  the  benefits  and  protection  of  the  laws  of  that   upon   appellant's   salesman   was   not   proper   service   upon   appellant;   that  
state  and  subjects  it  to  its  jurisdiction.   appellant   was   not   a   corporation   of   the   State   of   Washington,   and   was   not   doing  
  business   within   the   state;   that   it   had   no   agent   within   the   state   upon   whom  
The   activities   must   be   neither   irregular   nor   casual,   rather,   systematic   and   service  could  be  made.  
continuous   throughout   the   years   in   question.   They   resulted   in   a   large   volume   of    
interstate   business,   in   the   course   of   which   appellant   received   the   benefits   and   The  appeal  tribunal,  ruled  that  appellee  Commissioner  was  entitled  to  recover  
protection  of  the  laws  of  the  state,  including  the  right  to  resort  to  the  courts  for   the  unpaid  contributions.  That  action  was  affirmed  by  the  Commissioner;  both  
the  enforcement  of  its  rights.  The  obligation  which  is  here  sued  upon  arose  out   the  Superior  Court  and  the  Supreme  Court  affirmed.  
of  those  very  activities.    
  ISSUE:  
FACTS:   Is   service   of   process   upon   Defendant’s   agent   sufficient   notice   when   the  
International   Shoe   Co.   (Appellant)   assails   the   constitutionality   of   the   corporation’s   activities   result   in   a   large   volume   of   interstate   business   so   that  
Washington   unemployment   compensation   statute   as   it   is   applied   to   them   for   the  corporation  receives  the  protection  of  the  laws  of  the  state  and  the  suit  is  
infringing   the   due   process   clause   of   the   Fourteenth   Amendment   and   the   related  to  the  activities,  which  make  the  corporation  present?  
commerce  clause.  The  statutes  in  question  set  up  a  comprehensive  scheme  of    
unemployment  compensation,  the  costs  of  which  are  defrayed  by  contributions   HELD:  
Yes.   SC  held  that  in  view  of  26  U.S.C.  §1606(a),  providing  that  no  person  shall  
 
CONFLICT  OF  LAWS                                                                                    AV  DE  TORRES   26  
ATTY.  ARIS  L.  GULAPA                                            AY  2015-­‐2016  
be   relieved   from   compliance   with   a   state   law   requiring   payments   to   an   that   the   corporation   be   amenable   to   suits   unrelated   to   that   activity,   there  
unemployment   fund   on   the   ground   that   he   is   engaged   in   interstate   commerce.   have   been   instances   in   which   the   continuous   corporate   operations   within   a  
The   fact   that   the   corporation   is   engaged   in   interstate   commerce   does   not   state   were   thought   so   substantial   and   of   such   a   nature   as   to   justify   suit  
relieve   it   from   liability   for   payments   to   the   state   unemployment   against  it  on  causes  of  action  arising  from  dealings  entirely  distinct  from  those  
compensation   fund.   The   activities   in   behalf   of   the   corporation   render   it   activities.  
amenable  to  suit  in  courts  of  the  State  to  recover  payments  due  to  the  state    
unemployment  compensation  fund.   To   the   extent   that   a   corporation   exercises   the   privilege   of   conducting  
  activities   within   a   state,   it   enjoys   the   benefits   and   protection   of   the   laws   of  
Historically,   the   jurisdiction   of   courts   to   render   judgment   in   personam   is   that  state  and  subjects  it  to  its  jurisdiction.  The  exercise  of  that  privilege  may  
grounded   on   their   de   facto   power   over   the   defendant's   person.   Hence,   his   give   rise   to   obligations,   and,   so   far   as   those   obligations   arise   out   of   or   are  
presence   within   the   territorial   jurisdiction   of   a   court   was   prerequisite   to   its   connected  with  the  activities  within  the  state,  a  procedure  which  requires  the  
rendition   of   a   judgment   personally   binding   him.   Now   that   the   capias   ad   corporation   to   respond   to   a   suit   brought   to   enforce   them   can,   in   most  
respondendum  has  given  way  to  personal  service  of  summons  or  other  form   instances,  hardly  be  said  to  be  undue.    
of   notice,   due   process   requires   only   that,   in   order   to   subject   a   defendant   to   a    
judgment   in   personam,   if   he   be   not   present   within   the   territory   of   the   forum,   Applying  these  standards,  the  activities  carried  on  in  behalf  of  appellant  in  the  
he  have  certain  minimum  contacts  with  it  such  that  the  maintenance  of  the   State  of  Washington  were  neither  irregular  nor  casual.  They  were  systematic  
suit  does  not  offend  "traditional  notions  of  fair  play  and  substantial  justice.”   and   continuous   throughout   the   years   in   question.   They   resulted   in   a   large  
  volume  of  interstate  business,  in  the  course  of  which  appellant  received  the  
Since  the  corporate  personality  is  a  fiction,  it  is  clear  that,  unlike  an  individual,   benefits  and  protection  of  the  laws  of  the  state,  including  the  right  to  resort  
its   "presence"   without,   as   well   as   within,   the   state   of   its   origin   can   be   to   the   courts   for   the   enforcement   of   its   rights.  The   obligation   which   is   here  
manifested  only  by  activities  carried  in  its  behalf  by  those  who  are  authorized   sued  upon  arose  out  of  those  very  activities.  It  is  evident  that  these  operations  
to   act   for   it.   Demands   of   due   process   may   be   met   by   such   contacts   of   the   establish   sufficient   contacts   or   ties   with   the   state   of   the   forum   to   make   it  
corporation  with  the  state  of  the  forum  as  make  it  reasonable,  to  require  the   reasonable   and   just,   according   to   our   traditional   conception   of   fair   play   and  
corporation  to  defend  the  particular  suit  which  is  brought  there.   substantial   justice,   to   permit   the   state   to   enforce   the   obligations   which  
  appellant  has  incurred  there.  Hence  we  cannot  say  that  the  maintenance  of  the  
"Presence"   in   the   state   in   this   sense   has   never   been   doubted   when   the   present   suit   in   the   State   of   Washington   involves   an   unreasonable   or   undue  
activities   of   the   corporation   there   have   not   only   been   continuous   and   procedure.  
systematic,  but  also  give  rise  to  the  liabilities  sued  on,  even  though  no  consent    
to  be  sued  or  authorization  to  an  agent  to  accept  service  of  process  has  been   17.  PERKINS  V.  BENGUET  CONSOLIDATED  MINING  
given.  Conversely  it  has  been  generally  recognized  that  the  casual  presence  of   342  U.S.  437,  72  S.  Ct.  413  96  (1952)  
the  corporate  agent  or  even  his  conduct  of  single  or  isolated  items  of  activities    
in   a   state   in   the   corporation's   behalf   are   not   enough   to   subject   it   to   suit   on   DOCTRINE:  Thus  he  carried  on  in  Ohio  a  continuous  and  systematic  supervision  
causes   of   action   unconnected   with   the   activities   there.   To   require   the   of   the   necessarily   limited   wartime   activities   of   the   company.   He   there  
corporation   in   such   circumstances   to   defend   the   suit   away   from   its   home   or   discharged   his   duties   as   president   and   general   manager,   both   during   the  
other   jurisdiction   where   it   carries   on   more   substantial   activities   has   been   occupation   of   the   company's   properties   by   the   Japanese   and   immediately  
thought   to   lay   too   great   and   unreasonable   a   burden   on   the   corporation   to   thereafter.  While  no  mining  properties  in  Ohio  were  owned  or  operated  by  the  
comport  with  due  process.   company,   many   of   its   wartime   activities   were   directed   from   Ohio   and   were  
  being  given  the  personal  attention  of  its  president  in  that  State  at  the  time  he  
While   it   has   been   held,   in   cases   on   which   appellant   relies,   that   continuous   was  served  with  summons.  
activity   of   some   sorts   within   a   state   is   not   enough   to   support   the   demand  
 
CONFLICT  OF  LAWS                                                                                    AV  DE  TORRES   27  
ATTY.  ARIS  L.  GULAPA                                            AY  2015-­‐2016  
  appropriate   to   accepting   service   or   receiving   notice   on   its   behalf,   we  
Av:   Although   the   actual   business   (i.e.   mining)   is   in   the   Philippines,   the   president   recognize   that   there   is   no   unfairness   in   subjecting   that   corporation   to   the  
exercised  continuous  and  systematic  supervision  over  the  whole  business  during   jurisdiction   of   the   courts   of   that   state   through   such   service   of   process   upon  
the   wartime   in   Ohio.   The   corporation   may   be   sued   in   matters   related   to   such   that   representative.   This   has   been   squarely   held   to   be   so   in   a   proceeding   in  
supervision.   personam  against  such  a  corporation,  at  least  in  relation  to  a  cause  of  action  
  arising  out  of  the  corporation's  activities  within  the  state  of  the  forum.  
FACTS:    
Idonah  Slade  Perkins,  a  nonresident  of  Ohio,  filed  2  actions  in  personam  in  the   The   amount   and   kind   of   activities   which   must   be   carried   on   by   the   foreign  
Court   of   Common   Pleas   of   Clermont   County,   Ohio,   against   the   several   corporation  in  the  state  of  the  forum  so  as  to  make  it  reasonable  and  just  to  
respondents   one   of   which   is   the   Benguet   Consolidated   Mining   Company.   It   is   subject  the  corporation  to  the  jurisdiction  of  that  state  are  to  be  determined  
styled  as  a  "sociedad  anonima"  under  the  laws  of  the  Philippine  Islands,  where   in   each   case.   The   corporate   activities   of   a   foreign   corporation   which,   under  
it  owns  and  has  operated  profitable  gold  and  silver  mines.  In  one  action  Perkins   state   statute,   make   it   necessary   for   it   to   secure   a   license   and   to   designate   a  
seeks   approximately   $68,400   in   dividends   claimed   to   be   due   her   as   a   statutory  agent  upon  whom  process  may  be  served  provide  a  helpful  but  not  a  
stockholder.  In  the  other  she  claims  $2,500,000  damages  largely  because  of  the   conclusive  test.  On  the  other  hand,  if  the  same  corporation  carries  on,  in  that  
company's  failure  to  issue  to  her  certificates  for  120,000  shares  of  its  stock.   state,   other   continuous   and   systematic   corporate   activities   as   it   did   here—
  consisting   of   directors'   meetings,   business   correspondence,   banking,   stock  
In  each  case  the  trial  court  sustained  a  motion  to  quash  the  service  of  summons   transfers,   payment   of   salaries,   purchasing   of   machinery,   etc.—those   activities  
on  the  mining  company.  The  CA  of  Ohio  affirmed  that  decision  as  did  the  SC  of   are   enough   to   make   it   fair   and   reasonable   to   subject   that   corporation   to  
Ohio.   The   SC   of   Ohio   held   that   the   Due   Process   Clause   of   the   Fourteenth   proceedings   in   personam   in   that   state,   at   least   insofar   as   the   proceedings   in  
Amendment   prohibits   the   Ohio   courts   from   exercising   jurisdiction   over   the   personam  seek  to  enforce  causes  of  action  relating  to  those  very  activities  or  to  
respondent  corporation.  The  present  case  is  a  consolidation  of  the  cases  filed   other  activities  of  the  corporation  within  the  state.  
against  the  respondents.    
  The   court   finds   no   requirement   of   federal   due   process   that   either   prohibits  
ISSUE:   Ohio  from  opening  its  courts  to  the  cause  of  action  here  presented  or  compels  
Whether  as  a  matter  of  federal  due  process,  the  business  done  in  Ohio  by  the   Ohio  to  do  so.  The  company's  mining  properties  were  in  the  Philippine  Islands.  
respondent   mining   company   was   sufficiently   substantial   and   of   such   a   nature   Its   operations   there   were   completely   halted   during   the   occupation   of   the  
as  to  permit  Ohio  to  entertain  a  cause  of  action  against  a  foreign  corporation,   Islands   by   the   Japanese.   During   that   interim   the   president,   who   was   also   the  
where   the   cause   of   action   arose   from   activities   entirely   distinct   from   its   general   manager   and   principal   stockholder   of   the   company,   returned   to   his  
activities  in  Ohio.   home   in   Clermont   County,   Ohio.   There   he   maintained   an   office   in   which   he  
  conducted  his  personal  affairs  and  did  many  things  on  behalf  of  the  company.  
HELD:   He   kept   there   office   files   of   the   company.   He   carried   on   there   correspondence  
Yes.   The   Due   Process   Clause   of   the   Fourteenth   Amendment   to   the   Constitution   relating   to   the   business   of   the   company   and   to   its   employees.   He   drew   and  
of   the   U.S.   leaves   Ohio   free   to   take   or   decline   jurisdiction   over   the   corporation.   distributed  there  salary  checks  on  behalf  of  the  company,  both  in  his  own  favor  
Provisions   for   making   foreign   corporations   subject   to   service   in   the   state   is   a   as   president   and   in   favor   of   two   company   secretaries   who   worked   there   with  
matter  of  legislative  discretion,  and  a  failure  to  provide  for  such  service  is  not  a   him.   He   used   and   maintained   in   Clermont   County,   Ohio,   two   active   bank  
denial  of  due  process.   accounts   carrying   substantial   balances   of   company   funds.   From   that   office   he  
  supervised   policies   dealing   with   the   rehabilitation   of   the   corporation's  
Today   if   an   authorized   representative   of   a   foreign   corporation   be   physically   properties   in   the   Philippines   and   he   dispatched   funds   to   cover   purchases   of  
present   in   the   state   of   the   forum   and   be   there   engaged   in   activities   machinery  for  such  rehabilitation.  Thus   he   carried   on   in   Ohio   a   continuous   and  
systematic   supervision   of   the   necessarily   limited   wartime   activities   of   the  
 
CONFLICT  OF  LAWS                                                                                    AV  DE  TORRES   28  
ATTY.  ARIS  L.  GULAPA                                            AY  2015-­‐2016  
company.   He   there   discharged   his   duties   as   president   and   general   manager,    
both  during  the  occupation  of  the  company's  properties  by  the  Japanese  and   The   spouses   Robinsons   filed   a   products   liability   action   in   the   District   Court   of  
immediately   thereafter.   While   no   mining   properties   in   Ohio   were   owned   or   Creek   County,   OK,   claiming   that   their   injuries   resulted   from   the   defective  
operated  by  the  company,  many  of  its  wartime  activities  were  directed  from   design   and   placement   of   Audi’s   gas   tank   and   fuel   system.   They   joined   as  
Ohio   and   were   being   given   the   personal   attention   of   its   president   in   that   defendants   the   automobile’s   manufacturer,   Audi   NSU   Auto   Union   AG;   its  
State   at   the   time   he   was   served   with   summons.   Consideration   of   the   importer,   Volkswagen   of   America,   Inc.;   its   regional   distributor,   petitioner  
circumstances   which,   under   the   law   of   Ohio,   ultimately   will   determine   World-­‐Wide   Volkswagen   Corp;   and   its   retail   dealer,   Seaway.   Seaway   and  
whether   the   courts   of   that   State   will   choose   to   take   jurisdiction   over   the   World-­‐Wide  entered  special  appearances,  claiming  the  Oklahoma’s  exercise  of  
corporation  is  reserved  for  the  courts  of  that  State.   jurisdiction   over   them   offended   the   limitations   on   the   State’s   jurisdiction  
th  
  imposed  by  the  Due  Process  Clause  (14 Amendment).  
The  court  concludes  that,  under  the  circumstances  above  recited,  it  would  not    
violate   federal   due   process   for   Ohio   either   to   take   or   decline   jurisdiction   of   the   World-­‐Wide   is   incorporated   and   has   its   business   office   in   NY.   It   distributes   to  
corporation  in  this  proceeding.  The  judgment  of  the  SC  of  Ohio  is  vacated  and   retailers   in   New   York,   New   Jersey   and   Connecticut.   Seaway   is   incorporated   and  
the  cause  is  remanded  to  that  court  for  further  proceedings.   has   its   business   office   in   NY.   Seaway   and   World-­‐Wide   are   fully   independent  
  with  each  other  and  with  regard  to  VW  and  Audi,  contractual  only.  The  spouses  
18.  WORLD-­‐WIDE  VOLKSWAGEN  CORP  V.  WOODSON   Robinsons   adduced   no   evidence   to   show   that   either   World-­‐Wide   nor   Seaway  
444  U.S.  286  |  1980   does  business  in  OK,  or  sells  or  ships  to  that  state,  or  has  an  agent  to  receive  
  process,   or   purchases   advertisements   in   any   media   to   reach   OK.   Robinson’s  
DOCTRINE:   The   Due   Process   Clause   does   not   contemplate   that   a   state   may   counsel   conceded   that   World-­‐Wide   and   Seaway   never   entered   OK,   with   the  
make   binding   a   judgment   in   personam   against   an   individual   or   corporate   exception  of  the  vehicle  owned  by  Robinsons.  
defendant   with   which   the   state   has   no   contacts,   ties,   or   relations.   One,   isolated    
occurrence  cannot  be  the  basis  of  jurisdiction.   The  District  Court  rejected  the  constitutional  claim.  World-­‐Wide’s  MR  was  also  
  denied.  World-­‐Wide  sought  a  Writ  of  Prohibition  from  the  SC  of  Oklahoma  to  
The  foreseeability  that  is  critical  to  due  process  is  not  in  the  mere  likelihood  that   restrain  the  District  Court  Judge,  which  was  also  denied   holding  that  personal  
a   product   will   find   its   way   into   the   forum   State,   but   it   is   that   the   defendant’s   jurisdiction   over   World-­‐Wide   was   authorized   by   Oklahoma’s   “long-­‐arm”  
conduct   and   connection   with   the   forum   State   are   such   that   he   should   statute.  The   SC   of   Oklahoma   ruled   that   considering   that   the   product   sold   is,   by  
reasonably  anticipate  being  haled  into  court  there.  (such  as  when  the  products   its  very  design,  so  mobile  that  World-­‐Wide  can  foresee  its  possible  use  in  OK.  
are  being  purchased  by  consumers  in  the  forum  State.)   This  is  especially  true  to  the  distributor  who  has  the  exclusive  right  to  distribute  
  in  NY,  NJ  and  Connecticut.  The  SC  of  OK  found  it  reasonable  to  infer  that  World-­‐
Av:   Foreseeability   does   not   refer   to   whether   it   is   foreseeable   that   the   product   Wide  derive  substantial  income  from  automobiles  which  from  time  to  time  are  
will  find  its  way  into  the  forum  State  but  whether  it  is  foreseeable  to  be  haled   used  in  OK.  
into  court  in  the  forum  State.    
  ISSUE:  
th
FACTS:   Whether,  consistently  with  the  Due  Process  Clause  of  the  14  Amendment,  an  
Harry   and   Kay   Robinson   purchased   a   new   Audi   automobile   from   Seaway   Oklahoma   court   may   exercise   in   personam   jurisdiction   over   a   nonresident  
Volkswagen  in  Massena,  NY  in  1976.  In  1977,  the  Robinson  family  moved  from   automobile   retailer   and   its   wholesale   distributor   in   a   products   liability   action,  
NY   to   Arizona.   Passing   through   Oklahama   en   route   to   Arizona,   their   car   was   hit   when   the   defendants'   only   connection   with   Oklahoma   is   the   fact   that   an  
in  the  rear  by  a  truck,  which  caused  a  fire  and  severely  burned  Kay  and  their  2   automobile   sold   in   New   York   to   New   York   residents   became   involved   in   an  
children.   accident  in  Oklahoma?  
 
 
CONFLICT  OF  LAWS                                                                                    AV  DE  TORRES   29  
ATTY.  ARIS  L.  GULAPA                                            AY  2015-­‐2016  
  power  to  render  a  valid  judgment.  
HELD:    
No.  The  District  Court  may  not  exercise  in  personam  jurisdiction  over  World-­‐ The   US   Supreme   Court   finds   that   there   is   a   total   absence   of   those   affiliating  
Wide   because   of   the   lack   of   contacts,   ties   or   relations   with   the   State   of   circumstances   that   are   a   necessary   predicate   to   any   exercise   of   state   court  
Oklahoma.  Judgment  of  the  SC  of  OK  is  reversed.   jurisdiction.  World-­‐Wide  carry  no  activity  whatsoever  in  Oklahoma.  They  close  
  no   sales   and   perform   no   services   there.   They   avail   themselves   of   none   of   the  
A   State   Court   may   exercise   personal   jurisdiction   over   a   nonresident   privileges   and   benefits   of   Oklahoma   law.   They   solicit   no   business   there   either  
defendant   only   as   long   as   there   exists   “minimum   contacts”   between   the   through  salespersons  or  through  advertising  reasonably   calculated  to  reach  the  
defendant   and   the   forum   State.   The   concept   of   “minimum   contacts”   (1)   State.   Nor   does   the   record   show   that   they   regularly   sell   cars   at   wholesale   or  
protects   the   defendant   against   the   burdens   of   litigating   in   a   distant   or   retail   to   Oklahoma   customers   or   residents,   or   that   they   indirectly,   through  
inconvenient  forum,  and  (2)  it  acts  to  ensure  that  States,  through  their  courts,   others,   serve   or   seek   to   serve   the   Oklahoma   market.  In   short,   the   Robinsons  
do   not   reach   out   beyond   the   limits   imposed   on   them   by   their   status   as   seek   to   base   jurisdiction   on   one,   isolated   occurrence—the   fortuitous  
coequal   sovereigns   in   a   federal   system.  The   protection   against   inconvenient   circumstance   that   a   single   Audi   automobile,   sold   in   New   York   to   New   York  
litigation  is  typically  described  in  terms  of  “reasonableness”  or  “fairness.”  The   residents,  happened  to  suffer  an  accident  while  passing  through  Oklahoma.  
defendant’s  contacts  with  the  forum  State  must  be  such  that  maintenance   of    
the   suit   “does   not   offend   traditional   notions   of   fair   play   and   substantial   Even  if  it  is  foreseeable  that  the  automobile  is  mobile  by  its  very  design,  it  is  
justice.”     never   been   a   sufficient   benchmark   for   personal   jurisdiction   under   the   Due  
  Process   Clause.   Nonetheless,   foreseeability   is   not   wholly   irrelevant.   The  
This  limitation  on  state  jurisdiction  by  the  Due  Process  Clause  has  been  relaxed,   foreseeability  that  is  critical  to  due  process  is  not  in  the  mere  likelihood  that  a  
due  to  the  fundamental  transformation  in  the  American  economy.  There  is  an   product   will   find   its   way   into   the   forum   State,   but   it   is   that   the   defendant’s  
increase   in   commercial   transactions   touching   2   or   more   states.   Also,   the   conduct   and   connection   with   the   forum   State   are   such   that   he   should  
modern   transportation   and   communication   have   made   it   much   less   reasonably  anticipate  being  haled  into  court  there.  The  Due  Process  Clause,  by  
burdensome  for  a  party  sued  to  defend  himself  in  a  State  where  he  engages  in   ensuring   the   orderly   administration   of   the   laws,   gives   a   degree   of   predictability  
economic   activity.   Nonetheless,  the  proposition  that  state  lines  are  irrelevant   to  the  legal  system  that  allows  potential  defendants  to  structure  their  primary  
for   jurisidictional   purposes   is   not   accepted   by   the   US   SC.   Despite   the   conduct  with  some  minimum  assurance  as  to  where  that  conduct  will  and  will  
Commerce  Clause  providing  for  the  Nation  as  a  common  market,  a  “free  trade   not  render  them  liable  to  suit.  
unit,”  the  Framers  also  intended  the   States   retain   many   essential   attributes   of    
sovereignty,  including  the  sovereign  power  to  try  causes  in  their  courts.  The   The  forum  State  does  not  exceed  its  powers  under  the  Due  Process  Clause  if  it  
sovereignty  of  each  Sate,  in  turn,  implied  a  limitation  on  the  sovereignty  of  all   asserts  personal  jurisdiction  over  a  corporation  that  delivers  its  products  into  
its   sister   States–a   limitation   express   or   implicit   in   both   the   original   scheme   of   the  stream  of  commerce  with  the  expectation  that  they  will  be  purchased  by  
th  
the  Constitution  and  the  14 Amendment.     consumers   in   the   forum   State.   But   there   is   no   such   or   similar   basis   for  
  Oklahoma  jurisdiction  over  World-­‐Wide  or  Seaway  in  this  case.  Seaway's  sales  
The  Due  Process  Clause  does  not  contemplate  that  a  state  may  make  binding   are  made  in  Massena,  N.  Y.  World-­‐Wide's  market,  although  substantially  larger,  
a   judgment   in   personam   against   an   individual   or   corporate   defendant   with   is   limited   to   dealers   in   New   York,   New   Jersey,   and   Connecticut.   There   is   no  
which   the   state   has   no   contacts,   ties,   or   relations.   Even  if  the  defendant  would   evidence  of  record  that  any  automobiles  distributed  by  World-­‐Wide  are  sold  to  
suffer   minimal   or   no   inconvenience   from   being   forced   to   litigate   before   retail  customers  outside  this  tristate  area.  It  is  foreseeable  that  the  purchasers  
tribunals   of   another   State;   even   if   the   forum   State   has   a   strong   interest   in   of  automobiles  sold  by  World-­‐Wide  and  Seaway  may  take  them  to  Oklahoma.  
applying   its   law   to   the   controversy;   even   if   the   forum   State   is   the   most   But   the   mere   "unilateral   activity   of   those   who   claim   some   relationship   with   a  
convenient   location   for   litigation,   the   Due   Process   Clause,   acting   as   an   nonresident   defendant   cannot   satisfy   the   requirement   of   contact   with   the  
instrument  of  interstate  federalism,  may  sometimes  act  to  divest  the  State  of  its   forum  State."  
 
CONFLICT  OF  LAWS                                                                                    AV  DE  TORRES   30  
ATTY.  ARIS  L.  GULAPA                                            AY  2015-­‐2016  
  Whether   Jones,   a   California   resident,   can   bring   suit   against   a   Florida  
19.  CALDER  V.  JONES   corporation’s  employees  (Florida  residents)  in  a  California  court  for  a  claim  of  
465  US  783  |  1984   libel  by  asserting  that  California  has  personal  jurisdiction  over  Florida  residents  
   
DOCTRINE:   Under   the   circumstances,   petitioners   must   “reasonably   anticipate   HELD:  
being   haled   into   court   there”   to   answer   for   the   truth   of   the   statements   made   in   Yes.   California   is   the   focal   point   of   both   the   story   and   the   harm   suffered.  
their   article.   Petitioners   knew   that   the   brunt   of   that   injury   would   be   felt   by   Therefore,   jurisdiction   over   Calder,   South,   and   the   company   is   proper   in  
respondent   in   the   state   in   which   she   lives/works   and   in   which   the   National   California  based  on  the  “effects”  of  their  Florida  conduct  in  California.  
Enquirer  has  its  largest  circulation.  An  individual  injured  in  California  need  not    
go   to   Florida   to   seek   redress   from   persons   who,   though   remaining   in   Florida,   California’s   “long-­‐arm”   statute   permits   an   assertion   of   jurisdiction   over   a  
knowingly  cause  the  injury  in  California;  or  through  their  intentional  conduct  in   nonresident   person   whenever   permitted   by   the   state   and   federal  
Florida  calculated  to  cause  injury  to  Jones  in  California.   Constitutions.  Therefore,  turning  to  the  Constitution,  the  Due  Process  Clause  of  
th
  the   14   Amendment   permits   personal   jurisdiction   over   a   person   in   any   state  
FACTS:   with   which   the   person   has   “certain   minimum   contacts...   such   that   the  
Shirley   Jones,   a   resident   of   California,   brought   this   suit   against   the   National   maintenance   of   the   suit   does   not   offend   ‘traditional   notions   of   fair   play   and  
Enquirer,   Inc.,   its   local   distributing   company,   president/editor   Calder,   and   substantial  justice."  
reporter  South  for  libel  and  invasion  of  privacy.  The  article  at  issue  questioned    
the   professionalism   of   Jones   stating   that   the   she   drank   so   heavily   preventing   This   follows   that   South   and   Calder   wrote/edited   an   article   that   they   knew  
her  from  fulfilling  her  professional  obligations.   would   have   a   potentially   devastating   impact   upon   Jones.   Furthermore,   they  
  knew   that   the   brunt   of   that   injury   would   be   felt   by   Jones   in   the   state   in   which  
South,   a   reporter   employed   by   the   Enquirer,   a   resident   of   Florida,   wrote   the   she  lives/works  and  in  which  the  National  Enquirer  has  its  largest  circulation.  
first   draft   of   the   challenged   article,   his   byline   appeared   on   it.   Most   of   his   Under   the   circumstances,   Calder,   South   and   the   company   must   “reasonably  
research   for   the   article   was   done   in   Florida   with   the   aid   of   sources   located   in   anticipate   being   haled   into   court   there”   to   answer   for   the   truth   of   the  
California.   Aside   from   his   frequent   trips   and   phone   calls,   South   has   no   other   statements   made   in   their   article.   Finally,   an   individual   injured   in   California  
relevant  contacts  with  California.   need  not  go  to  Florida  to  seek  redress  from  persons  who,  though  remaining  in  
  Florida,  knowingly  cause  the  injury  in  California.  
Calder,   also   a   Florida   resident   and   president/editor   of   the   Enquirer,   has   only    
been  to  California  twice;  once  on  a  pleasure  trip  and  the  other  to  testify  in  an   Thus,  jurisdiction  over  National  Enquirer  et  al.  in  California  is  proper  because  of  
unrelated   trial   and   has   no   other   relevant   contacts   with   California.   He   their   intentional   conduct   in   Florida   calculated   to   cause   injury   to   Jones   in  
reviewed/approved  the  initial  evaluation  of  the  subject  of  the  article  &  edited  it   California.  
in  its  final  form.    
  20.  PHILSEC  INVESTMENT  V.  CA  
Originally,  the  suit  was  filed  in  California  SC,  but  was  dismissed  on  the  grounds   G.R.  No.  103493  |  June  19,  1997  
that   First   Amendment   concerns   weighed   against   an   assertion   of   jurisdiction    
otherwise   proper   under   the   Due   Process   Clause.   The   California   CA   reversed,   FACTS:  
stating  that  a  valid  basis  for  jurisdiction  did  exist  on  the  theory  that  petitioners   Ventura   Ducat   obtained   separate   loans   from   Ayala   International   Finance  
intended  to,  and  did,  cause  tortuous  injury  to  Jones  in  California.   Limited  and  Philsec  Investment  Corp.  in  the  sum  of  $  2.5  M  secured  by  shares  
  of  stock  owned  by  Ducat.  In  order  to  facilitate  the  payment  of  the  loans,  1488,  
ISSUE:   Inc.,   through   its   president   Daic,   assumed   Ducat’s   obligation   by   executing   a  
Warranty  Deed  with  Vendor’s  lien  by  which  it  sold  to  Athona  Holdings  a  parcel  
 
CONFLICT  OF  LAWS                                                                                    AV  DE  TORRES   31  
ATTY.  ARIS  L.  GULAPA                                            AY  2015-­‐2016  
of   land   in   Harris   County,   Texas   while   Philsec   and   Ayala   extended   a   loan   to   actions  in  rem,  a  foreign  judgment  merely  constitutes  prima  facie  evidence  of  
Athona   to   finance   payment   of   the   purchase   price.   Thus,   Ducat   was   released   the   justness   of   the   claim   of   a   party   and,   as   such,   is   subject   to   proof   to   the  
from  the  obligation.  As  Athona  failed  to  pay  the  balance  of  the  purchase  price,   contrary.  
the   entire   debt   became   due   and   demandable.   1488   sued   Philsec,   Ayala   and    
Athona  in  the  US  for  payment  of  the  balance  plus  damages.  While  the  civil  case   In  the  case  at  bar,  it  cannot  be  said  that  petitioners  were  given  the  opportunity  
was   pending,   Philsec   filed   a   complaint   and   Writ   of   Preliminary   Attachment   to   challenge   the   judgment   of   the   U.S.   court   as   basis   for   declaring   it   res   judicata  
against  Ducat,  et  al.  in  the  RTC  of  Makati  alleging  that  Ducat  committed  fraud   or  conclusive  of  the  rights  of  private  respondents.  The  proceedings  in  the  trial  
by  selling  the  property  at  a  price  400%  more  than  its  true  value.  RTC  issued  a   court  were  summary.  Neither  the  trial  court  nor  the  appellate  court  was  even  
writ  of  preliminary  attachment  against  the  real  and  personal  property  of  Ducat.   furnished  copies  of  the  pleadings  in  the  US  Court  or  apprised  of  the  evidence  
Ducat  filed  a  MTD  on  the  ground  of  litis  pendentia  and  forum  non  conveniens   presented   thereat,   to   assure   a   proper   determination   of   whether   the   issues  
and  lack  of  cause  of  action  for  Philsec  as  Philsec  is  not  a  party  to  the  sale.  1488,   then  being  litigated  in  the  US  court  were  exactly  the  issues  raised  in  this  case  
Inc.   also   filed   a   MTD   contending   that   the   action   being   in   personam,   such  that  the  judgment  that  might  be  rendered  would  constitute  res  judicata.  
extraterritorial  service  of  summons  by  publication  was  ineffectual  and  did  not   The  trial  court  arbitrarily  dismissed  the  case  even  after  finding  that  Ducat  was  
vest   the   court   with   jurisdiction   over   it,   which   is   a   non-­‐resident   foreign   not  a  party  in  the  U.S.  case.  
corporation.    
  2.  No.  Forum  non  conveniens  is  not  applicable  base  on  three  grounds:  first,  it  is  
The   trial   court   granted   Ducat’s   motion   even   as   it   noted   that   Ducat   was   not   a   not  a  ground  for  a  motion  to  dismiss  under  Rule  16.  Second,  while  it  is  within  
party  in  the  US  case.  The  Motion  to  Dismiss  of  1488,  Inc.  was  also  granted  on   the   discretion   of   the   trial   court   to   abstain   from   assuming   jurisdiction   on   this  
the  ground  of  litis  pendentia  considering  that  the  “main  factual  element”  of  the   ground,   it   should   do   so   only   after   "vital   facts   are   established,   to   determine  
cause   of   action   is   the   validity   of   the   sale   of   real   property   in   the   US.   The   trial   whether  special  circumstances"  require  the  court's  desistance,  and;  third,   the  
court  also  held  that  it  was  w/o  jurisdiction  over  the  case  as  1488,  Inc.  is  a  non-­‐ extraterritorial   service   of   summons   is   valid.   In   this   case,   the   trial   court  
resident   and   the   action   involved   is   not   in   rem   or   quasi   in   rem.   The   abstained   from   taking   jurisdiction   solely   on   the   basis   of   the   pleadings   filed   by  
extraterritorial  service  of  summons  was  therefore  ineffective.   private   respondents   in   connection   with   the   motion   to   dismiss.   It   failed   to  
  consider   that   PHILSEC   is   a   domestic   corporation   and   Ducat   is   a   Filipino,   and  
ISSUE:   that   it   was   the   extinguishment   of   the   latter's   debt   which   was   the   object   of  
1.  Whether  the  dismissal  of  the  case,  on  the  ground  that  the  judgment  in  the   the   transaction   under   litigation.   Rule   14,   §17   [now   §15]   on   extraterritorial  
US  court  bars  the  civil  case,  was  proper     service   provides   that   service   of   summons   on   a   non-­‐resident   defendant   may  
2.  Whether  the  principle  of  forum  non  conveniens  is  applicable   be   effected   out   of   the   Philippines   by   leave   of   Court   where,   among   others,  
  "the  property  of  the  defendant  has  been  attached  within  the  Philippines."  It  
HELD:   is  not  disputed  that  the  properties,  real  and  personal,  of  the  Ducat  and  Daic  
1.  No.  The  case  was  arbitrarily  dismissed.  While  this  court  has  given  the  effect   had   been   attached   prior   to   service   of   summons   under   the   Order   of   the   trial  
of   res   judicata   to   foreign   judgments   in   several   cases,   it   was   after   the   parties   court.  
opposed  to  the  judgment  had  been  given  ample  opportunity  to  repel  them  on    
grounds  allowed  under  Rule  39,  §50  [now  §48]  of  the  Rules  of  Court,  to  wit:   N.B.:   Section   15.   Extraterritorial   service.   —   When   the   defendant   does   not  
"want   of   jurisdiction,   want   of   notice   to   the   party,   collusion,   fraud,   or   clear   reside  and  is  not  found  in  the  Philippines,  and  the  action  affects  the   personal  
mistake   of   law   or   fact."   It   is   not   necessary   for   this   purpose   to   initiate   a   status  of  the  plaintiff  or  relates  to,  or  the  subject  of  which  is,  property  within  
separate  action  or  proceeding  for  enforcement  of  the  foreign  judgment.  What   the  Philippines,  in  which  the  defendant  has  or  claims  a  lien  or  interest,  actual  or  
is  essential  is  that  there  is  opportunity  to  challenge  the  foreign  judgment,  in   contingent,   or   in   which   the   relief   demanded   consists,   wholly   or   in   part,   in  
order  for  the  court  to  properly  determine  its  efficacy.  This  is  because  in  this   excluding   the   defendant   from   any   interest   therein,   or   the   property   of   the  
jurisdiction,   with   respect   to   actions   in   personam,   as   distinguished   from  
 
CONFLICT  OF  LAWS                                                                                    AV  DE  TORRES   32  
ATTY.  ARIS  L.  GULAPA                                            AY  2015-­‐2016  
defendant   has   been   attached   within   the   Philippines,   service   may,   by   leave   of    
court,  be  effected  out  of  the  Philippines  by  personal  service  as  under  section  6;   About   46   days   later,   Asuncion   filed   a   petition   for   relief   from   said   order  
or   by   publication   in   a   newspaper   of   general   circulation   in   such   places   and   for   declaring  him  in  default  and  from  said  judgment,  upon  the  ground  of  mistake  
such   time   as   the   court   may   order,   in   which   case   a   copy   of   the   summons   and   and   excusable   negligence.   Annexed   to   said   petition   were   Asuncion’s   affidavit  
order   of   the   court   shall   be   sent   by   registered   mail   to   the   last   known   address   of   and  his  verified  answer.  In  the  affidavit,  he  stated  that,  on  Sept.  26,  1955,  at  34  
the   defendant,   or   in   any   other   manner   the   court   may   deem   sufficient.   Any   Pitimine  Street,  San  Francisco  del  Monte  Quezon  City,  which  is  his  residence,  he  
order   granting   such   leave   shall   specify   a   reasonable   time,   which   shall   not   be   received  notice  of  a  registered  letter  at  the  Post  Office  in  San  Jose,  Nueva  Ecija,  
less   than   sixty   (60)   days   after   notice,   within   which   the   defendant   must   answer.   his   old   family   residence;   that   he   proceeded   immediately   to   the   latter  
(17a)   municipality  to  claim  said  letter,  which  he  received  on  Sept.  28;  that  the  letter  
  contained  copy  of  the  assailed  order  and  judgment,  much  to  his  surprise,  for  he  
21.  PANTALEON  V.  ASUNCION   had  not  been  summoned  or  notified  of  the  hearing  of  this  case.  Had  copy  of  the  
G.R.  No.  L-­‐13141  |  May  22,  1959   summons   and   of   the   order   for   its   publication   been   sent   to   him   by   mail,   as  
  provided   in   Rule   7,   §21,   of   the   ROC   said   summons   and   order   would   have  
DOCTRINE:   It  is  a  well-­‐settled  principle  of  Constitutional  Law  that,  in  an  action   reached   him.   His   failure   to   appear   before   the   court   is   excusable   it   being   due   to  
strictly   in   personam,   like   the   one   at   bar,   personal   service   of   summons,   within   the  mistake  of  the  authorities  concerned  in  not  complying  with  the  provisions  
the   forum,   is   essential   to   the   acquisition   of   jurisdiction   over   the   person   of   the   of  said  section.  
defendant,   who   does   not   voluntarily   submit   himself   to   the   authority   of   the    
court.   In   other   words,   summons   by   publication   cannot   –   consistently   with   the   ISSUE:  
due  process  clause  in  the  Bill  of  Rights  –  confer  upon  the  court  jurisdiction  over   Whether   the   summons   by   publication   had   been   made   in   conformity   with   the  
said  defendant.  (Superseded  by  #22  Santos  v.  PNOC)   ROC  
   
FACTS:   HELD:  
Vicenta   Pantaleon,   instituted   in   the   CFI   of   Nueva   Ecija,   an   action   to   recover   No.   Asuncion   maintains   that   copy   of   the   summons   and   of   the   order   for   the  
from   Asuncion,   the   sum   of   P2,000,   with   interest   thereon,   in   addition   to   publication   thereof   were   not   deposited   “in   the   post   office,   postage   prepaid,  
attorney’s   fees.   The   summons   originally   issued   was   returned   by   the   sheriff   of   directed   to   the   defendant   by   ordinary   mail   to   his   last   known   address”,   in  
Nueva   Ecija   unserved,   with   the   statement   that,   according   to   reliable   violation   of   Rule   7,   §21,   of   the   ROC,   which   reads:   If  the  service  has  been  made  
information,  Asuncion  was  residing  in  B-­‐24  Tala  Estate,  Caloocan,  Rizal.  An  alias   by   publication,   service   may   be   proved   by   the   affidavit   of   the   printer,   his  
summons   was   issued,   therefore,   for   service   in   the   place   last   mentioned.   foreman  or  principal  clerk,  or  of  the  editor,  business  or  advertising  manager,  
However,  the  provincial  sheriff  of  Rizal  returned  it  unserved,  with  information   to   which   affidavit   a   copy   of   the   publication   shall   be   attached,   and   by   an  
that   Asuncion   had   left   the   Tala   Estate   and   that   diligent   efforts   to   locate   him   affidavit   showing   the   deposit   of   a   copy   of   the   summons   and   order   for  
proved  to  no  avail.   publication   in   the   post   office,   postage   prepaid,   directed   to   the   defendant   by  
  ordinary  mail  to  his  last  known  address.  Pantaleon  alleges,  however,  that  the  
On   Pantaleon’s   motion,   the   court   ordered   that   Asuncion   be   summoned   by   provision   applicable   to   the   case   at   bar   is   not   this   §21,   but   §16,   of   Rule   7,   which  
publication,   and   the   summons   was   published   in   the   “Examiner”,   said   to   be   a   provides:  Whenever  the  defendant  is  designated  as  an  unknown  owner,  or  the  
newspaper   of   general   circulation   in   Nueva   Ecija.   Having   failed   to   appear   or   like,   or   whenever   the   address   of   a   defendant   is   unknown   and   cannot   be  
answer  the  complaint  within  the  period  stated  in  the  summons,  Asuncion  was   ascertained   by   diligent   inquiry,   service   may,   by   leave   of   court,   be   effect   upon  
declared   in   default.   Subsequently,   after   a   hearing   held   in   the   absence   of   the   him   by   publication   in   such   places   and   for   such   times   as   the   court   may   order.  
Asuncion   and   without   notice   to   him,   the   court   rendered   judgment   for   the   Further,  she  also  alleges  that  the  requirement,  in  said  §21,  of  an  affidavit  refers  
Pantaleon  for  the  sum  of  P2,300,  with  interest  thereon  at  the  legal  rate.   to  the  extraterritorial  service  of  summons.  
 
 
CONFLICT  OF  LAWS                                                                                    AV  DE  TORRES   33  
ATTY.  ARIS  L.  GULAPA                                            AY  2015-­‐2016  
Said   §21,   however,   is   unqualified.   It   prescribes   the   “proof   of   service   by   22.  SANTOS  V.  PNOC  
publication”,   regardless   of   whether   the   defendant   is   a   resident   of   the   G.R.  No.  170943  |  September  23,  2008  
Philippines  or  not.  §16  must  be  read  in  relation  to  §21,  which  complements  it.    
Then,  too,  we  conceive  of  no  reason,  and  Pantaleon  has  suggested  none,  why   DOCTRINE:   The  in   rem/in   personam  distinction   with   regard   to   substituted  
copy  of  the  summons  and  of  the  order  for  its  publication  should  be  mailed  to   service  was  significant  under  the  old  rule.  This  has  been  changed.  The  present  
non-­‐resident  defendants,  but  not  to  resident  defendants.  We  cannot  even  say   rule   expressly   states   that   it   applies   "[i]n   any   action  where   the   defendant   is  
that  Asuncion,  who,  according  to  the  return  of  the  Sheriff  of  Nueva  Ecija,  was   designated   as   an   unknown   owner,   or   the  like,   or   whenever   his   whereabouts   are  
reportedly   residing   in   Rizal–where   he,   in   fact   (San   Francisco   del   Monte   and   unknown  and  cannot  be  ascertained  by  diligent  inquiry..."  Thus,  it  now  applies  
Quezon   City   used   to   be   part   of   Rizal),   was   residing–could   reasonably   be   to  any  action,  whether  in  personam,  in  rem  or  quasi  in  rem.  
expected  to  read  the  summons  published  in  a  newspaper  said  to  be  a  general    
circulation  in  Nueva  Ecija.   The  rules,  however,  do  not  require  that  the  affidavit  of  complementary  service  
  be   executed   by   the   clerk   of   court.   While   the   trial   court   ordinarily   does   the  
Considering  that  strict  compliance  with  the  terms  of  the  statute  is  necessary   mailing   of   copies   of   its   orders   and   processes,   the   duty   to   make   the  
to   confer   jurisdiction   through   service   by   publication,   the   conclusion   is   complementary  service  by  registered  mail  is  imposed  on  the  party  who  resorts  
inescapable   that   the   lower   court   had   no   authority   whatsoever   to   issue   the   to  service  by  publication.  
order  declaring  the  defendant  in  default  and  to  render  the  decision  thus,  both    
are  null  and  void  ad  initio.   Av:  Service  by  publication  grants  jurisdiction  over  the  person  to  the  court  even  
  in  in  personam  proceedings.  
Apart   from   the   foregoing,   it   is   a   well-­‐settled   principle   of   Constitutional   Law    
that,   in   an   action   strictly   in   personam,   like   the   one   at   bar,   personal   service   of   FACTS:  
summons,  within  the  forum,  is  essential  to  the  acquisition  of  jurisdiction  over   PNOC   Exploration   Corp.   filed   a   complaint   for   a   sum   of   money   for   the   unpaid  
the  person  of  the  defendant,  who  does  not  voluntarily  submit  himself  to  the   balance  of  a  car  loan  against  Pedro  Santos  Jr.  in  the  RTC  of  Pasig  City.  Personal  
authority   of   the   court.   In   other   words,   summons   by   publication   cannot   –   service   of   summons   to   Santos   failed   because   he   could   not   be   located   in   his   last  
consistently  with  the  due  process  clause  in  the  Bill  of  Rights  –  confer  upon  the   known   address   despite   earnest   efforts   to   do   so.   Subsequently,   on   PNOC’s  
court   jurisdiction   over   said   defendant.  Due   process   of   law   requires   personal   motion,   the   trial   court   allowed   service   of   summons   by   publication.   PNOC  
service  to  support  a  personal  judgment,  and,  when  the  proceeding  is  strictly   caused   the   publication   of   the   summons   in   Remate,   a   newspaper   of   general  
in  personam  brought  to  determine  the  personal  rights  and  obligations  of  the   circulation   in   the   Philippines.   Thereafter,   PNOC   submitted   the   affidavit   of  
parties,   personal   service   within   the   state   or   a   voluntary   appearance   in   the   publication  of  the  advertising  manager  of  Remate  and  an  affidavit  of  service  of  
case   is   essential   to   the   acquisition   of   jurisdiction   so   as   to   constitute   PNOC’s   employee  to   the   effect   that   he   sent   a   copy   of   the   summons   by  
compliance  with  the  constitutional  requirement  of  due  process.   registered   mail   to   Santos’   last   known   address.   When   Santos   failed   to   file   his  
  answer  within  the  prescribed  period,  PNOC  moved  that  the  case  be  set  for  the  
Lastly,   from   the   viewpoint   of   substantial   justice   and   equity,   we   are   of   the   reception   of   its   evidence  ex   parte.   The   trial   court   granted   the   motion   in   an  
opinion   that   Asuncion’s   petition   for   relief   should   have   been   granted.   It   was   order.   Thereafter,   ex   parte   presentation   proceeded   and   the   case   was  
filed  well  within  the  periods  provided  in  the  ROC.  The  order  and  judgment  are   submitted  for  decision.    
set   aside   and   annulled.   The   case   is   remanded   to   the   lower   court   for   further    
proceedings.   However,   Santos   filed   an   "Omnibus   Motion   for   Reconsideration   and   to   Admit  
  Attached   Answer"   seeking   reconsideration   of   the   order   granting   reception   of  
evidence  ex  parte  and  alleging  that  the  affidavit  of  service  submitted  by  PNOC  
failed  to  comply  with  §19,  Rule  14  of  the  Rules  of  Court  as  it  was  not  executed  

 
CONFLICT  OF  LAWS                                                                                    AV  DE  TORRES   34  
ATTY.  ARIS  L.  GULAPA                                            AY  2015-­‐2016  
by  the  clerk  of  court.  He  also  claimed  that  he  was  denied  due  process  as  he  was   are   unknown   and   cannot   be   ascertained   by   diligent   inquiry."   Thus,   it   now  
not   notified   of   the   assailed   order.   Trial   court   denied   the   MR   holding   that   the   applies  to  any  action,  whether  in  personam,  in  rem  or  quasi  in  rem.  
rules  did  not  require  the  affidavit  of  complementary  service  by  registered  mail    
to   be   executed   by   the   clerk   of   court   and   that   the   assailed   order   was   actually   Regarding   the   matter   of   the   affidavit   of   service,   the   relevant   portion   of  
mailed  to  Santos  at  his  last  known  address.   §19,  Rule   14   simply   speaks   of   the   following:   “…an   affidavit   showing   the   deposit  
  of  a  copy  of  the  summons  and  order  for  publication  in  the  post  office,  postage  
ISSUE:   prepaid,   directed   to   the   defendant   by   registered   mail   to   his   last   known  
Whether  there  was  lack  of  jurisdiction  over  his  person  due  to  improper  service   address.”   The   rules,   however,   do   not   require   that   the   affidavit   of  
of   summons,   failure   of   the   trial   court   to   furnish   him   with   copies   of   its   orders   complementary   service   be   executed   by   the   clerk   of   court.   While   the   trial  
and   processes   and   preference   for   technicality   rather   than   justice   and   equity   (In   court   ordinarily   does   the   mailing   of   copies   of   its   orders   and   processes,   the  
particular,  he  claims  that  the  rule  on  service  by  publication  under  §14,  Rule  14   duty  to  make  the  complementary  service  by  registered  mail  is  imposed  on  the  
of   the   ROC   applies   only   to   actions  in   rem,   not   actions  in   personam  and   that   the   party  who  resorts  to  service  by  publication.  
affidavit  of  service  of  a  copy  of  the  summons  should  have  been  prepared  by  the    
clerk  of  court,  not  PNOC’s  messenger.)   Moreover,  even  assuming  that  the  service  of  summons  was  defective,  the  trial  
  court   acquired   jurisdiction   over   the   person   of   Santos   by   his   own   voluntary  
HELD:   appearance   in   the   action  against   him.   §   20,   Rule   14   states:   “The   defendant’s  
No.   voluntary   appearance   in   the   action   shall   be   equivalent   to   service   of  
  summons.”  
PROPRIETY  OF  SERVICE  BY  PUBLICATION    
§14,  Rule  14  provides:  “Service  upon  defendant  whose  identity  or  whereabouts   ENTITLEMENT  TO  NOTICE  OF  PROCEEDINGS  
are   unknown.   –  In   any   action  where   the   defendant   is   designated   as   an   §3  and  4,  Rule  9  states:  “If   the   defending   party   fails   to   answer   within   the   time  
unknown  owner,  or  the  like,  or  whenever   his   whereabouts   are   unknown   and   allowed   therefor,   the   court   shall,   upon   motion   of   the   claiming   party   with  
cannot  be  ascertained  by  diligent  inquiry,  service  may,  by  leave  of  court,  be   notice  to  the  defending  party,  and  proof  of  such  failure,  declare  the  defending  
effected   upon   him   by   publication   in   a   newspaper   of   general   circulation  and  in   party   in   default.  x  x  x  Sec.  4.  Effect  of  order  of  default.  –  A  party  in  default  shall  
such  places  and  for  such  times  as  the  court  may  order.”   be   entitled   to   notice   of   subsequent   proceedings  but   not   to   take   part   in   the  
  trial.”  
Since   Santos   could   not   be   personally   served   with   summons   despite   diligent    
efforts  to  locate  his  whereabouts,  PNOC  sought  and  was  granted  leave  of  court   If  the  defendant  fails  to  file  his  answer  on  time,  he  may  be  declared  in  default  
to   effect   service   of   summons   upon   him   by   publication   in   a   newspaper   of   upon  motion  of  the  plaintiff  with  notice  to  the  said  defendant.  In  case  he  is  
general   circulation.   Thus,   Santos   was   properly   served   with   summons   by   declared  in  default,  the  court  shall  proceed  to  render  judgment  granting  the  
publication.   plaintiff   such   relief   as   his   pleading   may   warrant,   unless   the   court   in   its  
  discretion   requires   the   plaintiff   to   submit   evidence.   The   defaulting   defendant  
Santos   claims   that   substituted   service   may   be   availed   of   only   in   an   action  in   may   not   take   part   in   the   trial   but   shall   be   entitled   to   notice   of   subsequent  
rem.   He   is   wrong.   The  in   rem/in   personam  distinction   was   significant   under   proceedings.   In  this  case,  Pantaleon  moved  only  for  the  ex  parte  presentation  
the   old   rule   because   it   was   silent   as   to   the   kind   of   action   to   which   the   rule   of   evidence,   not   for   the   declaration   of   Santos   in   default.  However,   the   order  
was   applicable.  Because   of   this   silence,   the   Court   limited   the   application   of   did  not  limit  itself  to  permitting  respondent  to  present  its  evidence  ex  parte  but  
the   old   rule   to  in   rem  actions   only.   This   has   been   changed.   The   present   rule   in  effect  issued  an  order  of  default.  But  the  trial  court  could  not  validly  do  that  
expressly   states   that   it   applies   "[i]n   any   action  where   the   defendant   is   as  an  order  of  default  can  be  made  only  upon  motion  of  the  claiming  party.  If  a  
designated  as  an  unknown  owner,  or  the  like,  or  whenever  his  whereabouts   party  declared  in  default  is  entitled  to  notice  of  subsequent  proceedings,  all  
the  more  should  a  party  who  has  not  been  declared  in  default  be  entitled  to  
 
CONFLICT  OF  LAWS                                                                                    AV  DE  TORRES   35  
ATTY.  ARIS  L.  GULAPA                                            AY  2015-­‐2016  
such   notice.   But   what   happens   if   the   residence   or   whereabouts   of   the   present  case  an  order  was  made  directing  the  clerk  to  mail  the  required  copy  to  
defending  party  is  not  known  or  he  cannot  be  located?  In  such  a  case,  there  is   the   defendant   at   Amoy   China.   No   evidence   appeared   of   record   showing   that  
obviously   no   way   notice   can   be   sent   to   him   and   the   notice   requirement   such  notice  had  in  fact  been  mailed  by  the  clerk;  but  publication  was  regularly  
cannot   apply   to   him.   The   law   does   not   require   that   the   impossible   be   made   in   a   periodical   as   the   law   requires.  The   court   held   that   the   making   of   the  
done.  Nemo   tenetur   ad   impossibile.   The   law   obliges   no   one   to   perform   an   order  by  the  court  constituted  a  compliance  with  the  law,  in  so  far  as  necessary  
 
impossibility. Laws   and   rules   must   be   interpreted   in   a   way   that   they   are   in   to  constitute  due  process  of  law,  and  that  if  the  clerk  failed  to  send  the  notice,  
accordance   with   logic,   common   sense,   reason   and   practicality.   Be   that   as   it   his  dereliction  in  the  performance  of  his  duty  was  in  irregularity  which  did  not  
may,   a   copy   of   the   order   was   nonetheless   still   mailed   to   Santos   at   his   last   constitute  an  infringement  of  the  provision  of  the  Philippine  Bill  declaring  that  
known  address.   no  person  shall  have  deprived  of  property  without  due  process  of  law.  
   
  A   defendant   who   seeks   to   vacate   a   judgment   in   a   foreclosure   proceeding   on  
the   ground   of   irregularity   in   the   sending   of   notice   by   post,   or   failure   to   send  
JURISDICTION  OVER  THE  RES  
  such   notice   pursuant   to   an   order   of   the   court,   must   show   that   as   result   of  
  which   irregularity   he   suffered   some   prejudice   of   which   the   law   can   take  
23.  EL  BANCO  ESPANOL-­‐FILIPINO  V.  PALANCA   account.  
G.R.  No.  L-­‐11390  |  March  26,  1918    
  N.B.:   Service   by   publication   is   still   necessary   in   quasi   in   rem   proceedings  
DOCTRINES:   Where   the   defendant   in   a   mortgage   foreclosure   lives   out   of   the   because  of  due  process.  
Islands  and  refuses  to  appear  otherwise  submit  himself  to  the  authority  of  the    
court,   the   jurisdiction   of   the   latter   is   limited   to   the   mortgaged   property,   with   FACTS:  
respect   to   which   the   jurisdiction   of   the   court   is   based   upon   the   fact   that   the   Engracio   Palanca   Tanquinyeng   mortgaged   his   lands   in   Manila   as   security   for  
property  is  located  within  the  district  and  that  the  court,  under  the  provisions   debt   he   owed   to   El   Banco   Español-­‐Filipino.   After   the   execution   of   the  
of   law   applicable   in   such   cases,   is   vested   with   the   power   to   subject   the   mortgage,   Tanquinyeng   returned   to   China   (where   he’s   a   native)   and   eventually  
property   to   the   obligation   created   by   the   mortgage.   In   such   case   personal   died   there.   Because   the   Tanquinyeng   was   a   nonresident   at   the   time   of   the  
jurisdiction  over  the  nonresident  defendant  is  nonessential  and  in  fact  cannot   institution   of   the   present   action,   it   was   necessary   for   the   bank   to   give   notice   to  
be  acquired.   Tanquinyeng  by  publication  in  a  newspaper  of  the  city  of  Manila.  The  Court  also  
  ordered  that  a  copy  of  the  summons  and  complaint  be  sent  to  Tanquinyeng  at  
The  failure  of  the  clerk  to  send  notice  by  mail  to  the  nonresident  defendant  in  a   his  last  place  of  residence  at  the  city  of  Amoy,  in  China.  This  order  was  made  
foreclosure  proceeding,  as  required  by  an  order  of  the  court,  does  not  defeat   pursuant   to   the   following   provision   contained   in   §399   of   the   Code   of   Civil  
the  jurisdiction  of  the  court  over  the  mortgaged  property.   Procedure:   “In   case   of   publication,   where   the   residence   of   a   nonresident   or  
  absent  defendant  is  known,  the  judge  must  direct  a  copy  of  the  summons  and  
In   an   action   to   foreclose   a   mortgage   against   a   nonresident   defendant   who   fails   complaint   to   be   forthwith   deposited   by   the   clerk   in   the   post-­‐office,   postage  
to  submit  himself  to  the  jurisdiction  of  the  court,  no  adjudication  can  be  made   prepaid,  directed  to  the  person  to  be  served,  at  his  place  of  residence.”  
which   involves   a   determination   of   a   personal   liability   of   either   party   arising   out    
of  the  contract  of  mortgage.   It   does   not   appear   that   the   clerk   fulfilled   the   order   of   sending   the   letter   to  
  China.   There   was   only   an   affidavit   signed   by   Bernardo   Chan,   employee   of   the  
In   a   foreclosure   proceeding   against   a   nonresident   defendant,   the   court   is   lawyers   of   the   bank,   showing   that   he   deposited   in   the   Manila   post-­‐office   a  
required   to   make   an   order   for   the   clerk   to   mail   a   copy   of   the   summons   and   registered  letter,  addressed  to  Tanquinyeng,  at  Manila,  containing  copies  of  the  
complaint   to   the   defendant   at   his   last   place   of   residence   if   known.   In   the   complaint,   the   bank's   affidavit,   the   summons,   and   the   order   of   the   court  

 
CONFLICT  OF  LAWS                                                                                    AV  DE  TORRES   36  
ATTY.  ARIS  L.  GULAPA                                            AY  2015-­‐2016  
directing  publication.   court,  as  in  all  cases  of  foreclosure,  to  ascertain  the  amount  due  and  to  make  
  an   order   requiring   the   defendant   to   pay   the   money   into   court.   This   step   is   a  
CFI   ruled   in   favor   of   the   bank.   In   case   of   the   failure   of   Tanquinyeng   to   satisfy   necessary   precursor   of   the   order   of   sale.   In   the   present   case   the   judgment  
the  judgment  within  such  period,  the  mortgage  property  located  in  the  city  of   which   was   entered   contains   the   following   words:   “Because   it   is   declared   that  
Manila   should   be   exposed   to   public   sale.   The   payment   contemplated   in   said   the  said  defendant  Engracio  Palanca  Tanquinyeng  y  Limquingco,  is  indebted  in  
order   was   never   made;   so   the   court   ordered   the   sale   of   the   property.   The   the  amount  of  P249,355.32,  plus  the  interest,  to  the  'Banco  Espanol-­‐Filipino'…  
property  was  bought  by  the  bank  for  P110,200.  7  years  after  the  confirmation   therefore  said  appellant  is  ordered  to  deliver  the  above  amount  etc.,  etc.”  
of   the   sale,   Vicente   Palanca,   as   administrator   of   the   estate   of   Tanquinyeng,    
requested  the  court  to  set  aside  the  order  of  default  alleging  that  the  judgment   This   is   not   the   language   of   a   personal   judgment.   Instead   it   is   clearly   intended  
rendered   was   void   because   the   court   never   acquired   jurisdiction   over   merely   as   a   compliance   with   the   requirement   that   the   amount   due   shall   be  
Tanquinyeng  or  the  subject  matter  of  the  action.   ascertained  and  that  the  evidence  of  this  it  may  be  observed  that  according  to  
  the   Code   of   Civil   Procedure   a   personal   judgment   against   the   debtor   for   the  
ISSUES:   deficiency  is  not  to  be  rendered  until  after  the  property  has  been  sold  and  the  
1.  Whether  the  court  acquired  necessary  jurisdiction   proceeds  applied  to  the  mortgage  debt.  
2.  Whether  there  was  denial  of  due  process  of  law    
  Whatever  may  be  the  effect  in  other  respects  of  the  failure  of  the  clerk  of  the  
HELD:   CFI  to  mail  the  proper  papers  to  the  defendant  in  Amoy,  China,  such  irregularity  
1.  Yes.  Jurisdiction  over  the  property  which  is  the  subject  of  the  litigation  may   could  not  impair  or  defeat  the  jurisdiction  of  the  court.  
result  either  from  (1)  a  seizure  of  the  property  under  legal  process,  whereby  it    
is   brought   into   the   actual   custody   of   the   law,   or   it   may   result   from   (2)   the   In  the  latter  case  the  property,  though  at  all  times  within  the  potential  power  
institution  of  legal  proceedings  wherein,  under  special  provisions  of  law,  the   of  the  court,  may  never  be  taken  into  actual  custody  at  all.  An  illustration  of  the  
power  of  the  court  over  the  property  is  recognized  and  made  effective.   jurisdiction   acquired   by   actual   seizure   is   found   in   attachment   proceedings,  
  where   the   property   is   seized   at   the   beginning   of   the   action,   or   some  
The   jurisdiction   of   the   court   over   the   property,   considered   as   the   exclusive   subsequent   stage   of   its   progress,   and   held   to   abide   the   final   event   of   the  
object   of   such   action,   is   evidently   based   upon   the   following   conditions   and   litigation.  An  illustration  of  what  we  term  potential  jurisdiction  over  the  res,  is  
considerations,  namely:  (1)  that  the  property  is  located  within  the   district;   (2)   found   in   the   proceeding  to   register  the   title   of   land   under   our   system   for   the  
that   the   purpose   of   the   litigation   is   to   subject   the   property   by   sale   to   an   registration  of  land.  Here  the  court,  without  taking  actual  physical  control  over  
obligation   fixed   upon   it   by   the   mortgage;   and   (3)   that   the   court   at   a   proper   the   property   assumes,   at   the   instance   of   some   person   claiming   to   be   owner,   to  
stage   of   the   proceedings   takes   the   property   into   custody,   if   necessary,   and   exercise   a   jurisdiction   in   rem   over   the   property   and   to   adjudicate   the   title   in  
expose  it  to  sale  for  the  purpose  of  satisfying  the  mortgage  debt.   favor  of  the  petitioner  against  all  the  world.  
   
The   Court   formulated   the   following   proposition   regarding   foreclosure   In  the  terminology  of  American  law  the  action  to  foreclose  a  mortgage  is  said  to  
proceeding   against   the   property   of   a   nonresident   mortgagor   who   fails   to   be  a  proceeding  quasi  in  rem,  by  which  is  expressed  the  idea  that  while  it  is  not  
come  in  and  submit  himself  personally  to  the  jurisdiction  of  the  court:  (I)  That   strictly   speaking   an   action   in   rem   yet   it   partakes   of   that   nature   and   is  
the  jurisdiction  of  the  court  is  derived  from  the  power  which  it  possesses  over   substantially  such.  The  expression  "action  in  rem"  is,  in  its  narrow  application,  
the  property;  (II)  that  jurisdiction  over  the  person  is  not  acquired  and  is  non-­‐ used  only  with  reference  to  certain  proceedings  in  courts  of  admiralty  wherein  
essential;  (III)  that  the  relief  granted  by  the  court  must  be  limited  to  such  as   the   property   alone   is   treated   as   responsible   for   the   claim   or   obligation   upon  
can  be  enforced  against  the  property  itself.   which   the   proceedings   are   based.   The   action   quasi   rem   differs   from   the   true  
  action  in  rem  in  the  circumstance  that  in  the  former  an  individual  is  named  as  
In  a  foreclosure  proceeding  against  a  nonresident  owner  it  is  necessary  for  the   defendant,  and  the  purpose  of  the  proceeding  is  to  subject  his  interest  therein  
 
CONFLICT  OF  LAWS                                                                                    AV  DE  TORRES   37  
ATTY.  ARIS  L.  GULAPA                                            AY  2015-­‐2016  
to   the   obligation   or   lien   burdening   the   property.   All   proceedings   having   for   process  of  law.  
their   sole   object   the   sale   or   other   disposition   of   the   property   of   the   defendant,    
whether  by  attachment,  foreclosure,  or  other  form  of  remedy,  are  in  a  general   24.  PERKINS  V.  DIZON  
way  thus  designated.  The  judgment  entered  in  these  proceedings  is  conclusive   G.R.  No.  46631  |  November  16,  1939  
only  between  the  parties.    
xxx   DOCTRINES:   A   quasi   in   rem   fixes   and   settles   the   title   to   the   property   in  
  controversy  and  to  that  extent  partakes  of  the  nature  of  the  judgment  in  rem.  
It   is   true   that   in   proceedings   of   this   character,   if   the   defendant   for   whom    
publication   is   made   appears,   the   action   becomes   as   to   him   a   personal   action   The   situs   of   the   shares   is   in   the   jurisdiction   where   the   corporation   is   created,  
and   is   conducted   as   such.   This,   however,   does   not   affect   the   proposition   that   whether  the  certificate  evidencing  the  ownership  of  those  shares  are  within  or  
where   the   defendant   fails   to   appear   the   action   is   quasi   in   rem;   and   it   should   without  that  jurisdiction.  
therefore   be   considered   with   reference   to   the   principles   governing   actions   in    
rem.   Service   of   the   summons   by   publication   ordered   by   virtue   of   an   action   quasi   in  
  rem  against  the  non-­‐resident  defendant  is  valid.  
2)  No.  The  failure  of  the  clerk  to  mail  the  notice  is  not  such  an  irregularity,  as    
amounts   to   a   denial   of   due   process   of   law;   and   hence   that   irregularity,   if   The  action  being  quasi  in  rem  and  notice  having  been  made  by  publication,  the  
proved,   would   not   avoid   the   judgment   in   this   case.   Notice   was   given   by   relief  that  may  be  granted  by  the  Philippine  court  must  be  confined  to  the  res,  it  
publication  in  a  newspaper  and  this  is  the  only  form  of  notice  which  the  law   having  no  jurisdiction  to  render  a  personal  judgment  against  the  non-­‐resident.  
unconditionally   requires.   The   provision   of   our   law   relative   to   the   mailing   of    
notice  does  not  absolutely  require  the  mailing  of  notice  unconditionally  and   FACTS:  
in   every   event,   but   only   in   the   case   where   the   defendant's   residence   is   Eugene  Arthur  Perkins,  instituted  an  action  in  the  CFI  of  Manila  against  Benguet  
known.   In   the   light   of   all   these   facts,   it   is   evident   that   actual   notice   to   the   Consolidated   Mining   Company   for   payment   of   dividends   on   52,874   shares   of  
defendant   in   cases   of   this   kind   is   not,   under   the   law,   to   be   considered   stock   registered   in   his   name   being   withheld   by   the   company   and,   for   the  
absolutely  necessary.   recognition   of   his   right   to   the   control   and   disposal   of   said   shares   against   the  
  claims  of  2  other  persons:  George  Engelhard  and  Idonah  Perkins  (both  are  non  
The  publication  was  regularly  made  in  a  periodical  as  the  law  requires  and  the   residents,   but   the   case   did   not   indicate   the   country/ies   they   are   residing   at).  
court  ordered  the  clerk  to  mail  the  notice.  Such  a  compliance  with  the  law,  in   The  Benguet  Co.  motioned  the  court  that  such  parties  be  required  to  interplead  
so  far  as  necessary  to  constitute  due  process  of  law,  and  that  if  the  clerk  failed   and  settle  the  rights  among  themselves.  
to   send   the   notice,   his   dereliction   in   the   performance   of   his   duty   was   in    
irregularity   which   did   not   constitute   an   infringement   of   the   provision   of   the   CFI   Manila   ordered   Eugene   Perkins   to   include   in   his   complaint   as   party  
Philippine   Bill   declaring   that   no   person   shall   have   deprived   of   property   without   defendants  George  and  Idonah.  Pursuant  to  the  order  of  the  CFI,  summons  by  
due  process  of  law.   publication   were   served   upon   George   and   Idonah.   George   filed   an   answer.  
  Idonah,   through   counsel,   filed   her   pleading   entitled   "objection   to   venue,  
J.Malcolm   (Dissent):   No   man   shall   be   condemned   in   his   person   or   property   motion   to   quash,   and   demurrer   to   jurisdiction"   wherein   she   challenged   the  
without  notice  and  an  opportunity  of  being  heard  in  his  defense.  Protection  of   jurisdiction  of  the  lower  court  over  her  person.  
the   parties   demands   a   strict   and   an   exact   compliance   with   this   constitutional    
provision   in   our   organic   law   and   of   the   statutory   provisions   in   amplification.   Idonah   contends   that   the   proceeding   instituted   against   her   is   one   of  
Literally   hundreds   of   precedents   could   be   cited   in   support   of   these   axiomatic   interpleading  and  is  thus,  an  action  in  personam  on  the  basis  of  §120  of  Code  of  
principles.  Where  as  in  the  instant  case  the  defendant  received  no  notice  and   Civil  Procedure  which  provides  that  whenever  conflicting  claims  are  or  may  be  
had   no   opportunity   to   be   heard,   certainly   we   cannot   say   that   there   is   due  
 
CONFLICT  OF  LAWS                                                                                    AV  DE  TORRES   38  
ATTY.  ARIS  L.  GULAPA                                            AY  2015-­‐2016  
made   upon   a   person   for   or   relating   to   personal   property,   or   the   performance   of    
an  obligation  or  any  portion  thereof,  so  that  he  may  be  made  subject  to  several   Had   not   the   complaint   been   amended,   and   the   CFI   issued   an   order   under  
actions   by   different   persons,   such   person   may   bring   an   action   against   the   calling   the   conflicting   claimants   to   interplead   with   one   another,   such   order  
conflicting  claimants,  disclaiming  personal  interest  in  the  controversy,  and  the   could  not  perhaps  have  validly  been  served  by  publication  or  otherwise,  upon  
court  may  order  them  to  interplead  with  one  another  and  litigate  their  several   the  non-­‐resident  Idonah,  for  then  the  proceeding  would  be  a  personal  action.  
claims   among   themselves,   there   upon   proceed   to   determine   their   several    
claims.   Second,  no  money  judgment  or  other  relief  in  personam  is  prayed  for  against  
  Idonah.  The  only  relief  sought  therein  is  that  she  be  declared  to  be  without  any  
ISSUE:   interest  in  the  shares.  
Whether,   through   the   summons   by   publication,   the   CFI   has   acquired    
jurisdiction   over   the   person   of   Idonah   as   a   non-­‐resident   defendant,   or,   Third,  on  the  claim  of  Idonah  Perkins  that  an  interpleader  is  in  personam,  the  
notwithstanding   the   want   of   such   jurisdiction,   whether   said   court   may   validly   court  has  not  issued  an  order  compelling  the  conflicting  claimants  to  interplead  
try  the  case   with   one   another   and   litigate   their   several   claims   among   themselves,   but  
  instead   ordered   the   plaintiff   to   amend   his   complaint   including   the   other   two  
HELD:   claimants   as   defendants.   Thus,   the   publication   of   the   summons   was   ordered  
Yes,   the   CFI   has   acquired   jurisdiction.   §398   of   Code   of   Civil   Procedure   [now   not   in   virtue   of   an   interpleading,   but   upon   the   filing   of   the   amended   complaint  
Rule   14,   §15]   provides   that   when   a   non-­‐resident   defendant   is   sued   in   the   wherein  an  action  quasi  in  rem  is  alleged.  
Philippine   courts   and   it   appears,   by   the   complaint   or   by   affidavits,   that   the    
action  relates  to  real  or  personal  property  within  the  Philippines  in  which  said   25.  TRAVELERS  HEALTH  ASSN  V.  VIRGINIA  
defendant  has  or  claims  a  lien  or  interest,  actual  or  contingent,  or  in  which  the   339  U.S.  643  |  1950  
relief  demanded  consists,  wholly  or  in  part,  in  excluding  such  person  from  any    
interest  therein,  service  of  summons  maybe  made  by  publication.   DOCTRINES:  GR:  A  Minnesota  association  obtained  members  in  Montana  by  the  
  same   mail   solicitation   process   used   by   Travelers   to   get   Virginia   members.   The  
First,   the   action   brought   by   Eugene   Perkins   is   quasi   in   rem,   for   while   the   Court   held   that   since   the   contracts   were   "executed   and   to   be   performed"   in  
judgment   that   may   be   rendered   therein   is   not   strictly   a   judgment   in   rem,   "it   Minnesota,   the   Association   was   not   "doing   business"   in   Montana   and   therefore  
fixes   and   settles   the   title   to   the   property   in   controversy   and   to   that   extent   could  not  be  sued  in  Montana  courts  unless  "consent"  to  Montana  suits  could  
partakes   of   the   nature   of   the   judgment   in   rem."   The   amended   complaint   be  implied.  
against  Idonah  seeks  to  exclude  her  from  any  interest  in  a  property  located  in    
the   Philippines.   That   property   consists   in   certain   shares   of   stocks   of   Benguet   EXC:   But   where   business   activities   reach   out   beyond   one   state   and   create  
Consolidated,   a   sociedad   anonima,   organized   in   the   Philippines   under   the   continuing   relationships   and   obligations   with   citizens   of   another   state,   courts  
provisions  of  the  Spanish  Code  of  Commerce,  with  its  principal  office  in  the  City   need   not   resort   to   a   fictional   'consent'   in   order   to   sustain   the   jurisdiction   of  
of   Manila   and   which   conducts   its   mining   activities   therein.   The   situs   of   the   regulatory  agencies  in  the  latter  state.  
shares   is   in   the   jurisdiction   where   the   corporation   is   created,   whether   the    
certificate   evidencing   the   ownership   of   those   shares   are   within   or   without   Due  process  requires  only  that  in  order  to  subject  a  defendant  to  a  judgment  in  
that   jurisdiction.   Thus,  there  is  no  question  as  to  the  adequacy  of  publication   personam,  if  he  be  not  present  within  the  territory  of  the  forum,  he  have  certain  
made   nor   as   to   the   mailing   of   the   order   of   publication   to   the   petitioner's   last   minimum  contacts  with  it  such  that  the  maintenance  of  the  suit  does  not  offend  
known  place  of  residence  in  the  US.  But,   of   course,   the   action   being   quasi   in   'traditional  notions  of  fair  play  and  substantial  justice.’  
rem   and   notice   having   been   made   by   publication,   the   relief   that   may   be    
granted   by   the   Philippine   court   must   be   confined   to   the   res,   it   having   no   There   is,   of   course,   one   method   by   which   claimants   could   recover   from  
jurisdiction  to  render  a  personal  judgment  against  the  non-­‐resident.  
 
CONFLICT  OF  LAWS                                                                                    AV  DE  TORRES   39  
ATTY.  ARIS  L.  GULAPA                                            AY  2015-­‐2016  
appellants   in   Virginia   courts   without   the   aid   of   substituted   service   of   process:   The   Commission   rejected   Travelers'   objection   to   jurisdiction   and   their   motion  
certificate   holders   in   Virginia   could   all   be   garnished   to   the   extent   of   their   to   quash   service.   Travelers   and   Pratt   were   ordered   to   cease   and   desist   from  
obligations  to  the  Association.   further   solicitations   or   sales   of   certificates   to   Virginia   residents   unless   and   until  
  it   obtained   authority   in   accordance   with   the   “Blue   Sky   Law.”   The   Virginia   CA  
FACTS:   affirmed  this  order.  
The   Virginia   “Blue   Sky   Law”   was   enacted   to   protect   its   citizens   from   unfairness,    
imposition  and  fraud  in  the  sales  of  certificates  of  insurance  and  other  forms  of   ISSUE:  
securities.  The  law  requires  those  selling  or  offering  such  securities  to  obtain  a   Whether  the  Commonwealth  of  Virginia  has  jurisdiction  over  Travelers  
permit   from   the   State   Corporation   Commission.   Applicants   for   permits   must    
provide   detailed   information   concerning   their   solvency,   and   must   agree   that   HELD:  
suits   can   be   filed   against   them   in   Virginia   by   service   of   process   on   the   Secretary   Yes.   Travelers’   contention   is   that   this   is   in   violation   of   the   due   process   clause  
of   the   Commonwealth.   §6   provides   that   after   notice   and   a   hearing   on   the   and  that  all  their  activities  take  place  in  Nebraska,  consequently  Virginia  has  no  
merits,   the   State   Corporation   Commission   is   authorized   to   issue   a   cease   and   power   to   reach   them   in   cease   and   desist   proceedings   to   enforce   any   part   of   its  
desist   order,   restraining   violations   of   the   Act.   It   also   provides   for   service   by   regulatory   law.   The   court   held   that   the   state   has   power   to   issue   a   cease   and  
registered   mail   where   other   types   of   service   are   unavailable   because   the   desist   order   enforcing   at   least   that   regulatory   provision   requiring   Travelers   to  
offering   is   by   advertisement   and/or   solicitation   through   periodicals,   mail,   accept   service   of   process   by   Virginia   claimants   on   the   Sec.   of   the  
telephone,   telegraph,   radio,   or   other   means   of   communication   from   beyond   Commonwealth.  
the  limits  of  the  State.    
  In   Minnesota   Commercial   Men's   Ass'n   v.   Benn,   a   Minnesota   association  
Travelers   Health   Association   was   incorporated   in   Nebraska   as   a   nonprofit   obtained   members   in   Montana   by   the   same   mail   solicitation   process   used   by  
membership   association.   It   conducts   a   mail-­‐order   health   insurance   business   Travelers  to  get  Virginia  members.  The  Court  held  that  since  the  contracts  were  
from   its   office   in   Omaha,   Nebraska.   New   members   pay   an   initiation   fee   and   "executed  and  to  be  performed"  in  Minnesota,  the  Association  was  not  "doing  
obligate   themselves   to   pay   periodic   assessments   at   the   said   office.   The   funds   business"   in   Montana   and   therefore   could   not   be   sued   in   Montana   courts  
collected  are  used  for  operating  expenses  and  sick  benefits  to  members.  New   unless   "consent"   to   Montana   suits   could   be   implied.   But   where   business  
members  are  obtained  through  the  unpaid  activities  of  those  already  members,   activities   reach   out   beyond   one   state   and   create   continuing   relationships   and  
who  are  encouraged  to  recommend  the  Association  to  friends  and  submit  their   obligations  with  citizens  of  another  state,  courts  need  not  resort  to  a  fictional  
names  to  the  home  office.  Pratt  then  mails  solicitations  to  these  prospects.  He   'consent'   in   order   to   sustain   the   jurisdiction   of   regulatory   agencies   in   the  
encloses   blank   applications   which,   if   signed   and   returned   to   the   home   office   latter  state.  
with   the   required   fee,   usually   result   in   election   of   applicants   as   members.    
Certificates   are   then   mailed.   Travelers   has   solicited   Virginia   members   in   this   A   state   has   a   legitimate   interest   in   all   insurance   policies   protecting   its   residents  
manner  and  had  approximately  800  Virginia  members.   against   risks,   an   interest   which   the   state   can   protect   even   though   the   state  
  action   may   have   repercussions   beyond   state   lines.   The   court   rejected   the  
Cease   and   desist   proceedings   §6   were   instituted   by   the   State   Corporation   contention   that   a   state's   power   to   regulate   must   be   determined   by   a  
Commission   against   Travelers   and   R.   E.   Pratt,   as   treasurer   of   the   Association   conceptualistic   discussion   of   theories   of   the   place   of   contracting   or   of  
and   in   his   personal   capacity.   Having   received   notice   by   registered   mail   only,   performance.   Instead,   great   weight   was   given   to   the   consequences   of   the  
they  appeared  'specially'  for  the  purpose  of  objecting  to  the  alleged  jurisdiction   contractual  obligations  in  the  state  where  the  insured  resided  and  the  'degree  
of  the  Commonwealth  of  Virginia  and  of  its  State  Corporation  Commission,  and   of   interest'   that   state   had   in   seeing   that   those   obligations   were   faithfully  
of  moving  to  set  aside  and  quash  service  of  summons.   carried   out.   Due   process   requires   only   that   in   order   to   subject   a   defendant   to  
  a   judgment   in   personam,   if   he   be   not   present   within   the   territory   of   the  
forum,  he  have  certain  minimum  contacts  with  it  such  that  the  maintenance  
 
CONFLICT  OF  LAWS                                                                                    AV  DE  TORRES   40  
ATTY.  ARIS  L.  GULAPA                                            AY  2015-­‐2016  
of   the   suit   does   not   offend   'traditional   notions   of   fair   play   and   substantial   Due  Process  Clause.  
justice.’    
   
In   the   case   at   bar,   the   contacts   and   ties   of   Travelers   with   Virginia   residents,  
ACT  OF  STATE  DOCTRINE  
together   with   that   state's   interest   in   faithful   observance   of   the   certificate    
obligations,   justify   subjecting   Travelers   and   Pratt   to   cease   and   desist    
proceedings  under  §6.  Travelers  did  not  engage  in  mere  isolated  or  short-­‐lived   26.  FRENCH  V.  BANCO  NATIONAL  DE  CUBA  
transactions.  Its  insurance  certificates,  systematically  and  widely  delivered  in   23  N.Y.2d  46  |  1968  
Virginia   following   solicitation,   create   continuing   obligations   between    
Travelers  and  each  of  the  many  certificate  holders  in  the  state.  Travelers  and   DOCTRINE:   Courts   will   not   inquire   into   the   validity   of   the   acts   of   a   foreign  
Pratt  have  caused  claims  for  losses  to  be  investigated  and  the  Virginia  courts   government   done   within   its   own   territory.   Every   sovereign   state   is   bound   to  
were  available  to  them  in  seeking  to  enforce  obligations  created  by  the  group   respect  the  independence  of  every  other  sovereign  state,  and  the  courts  of  one  
of  certificates.   country  will  not  sit  in  judgment  of  the  acts  of  the  government  of  another  done  
  within  its  own  territory.  
For  these  reasons,  Virginia  has  power  to  subject  Travelers  to  the  jurisdiction  of    
its   Corporation   Commission,   and   its   cease   and   desist   provisions   designed   to   The  plaintiff  adduced  evidence  to  the  effect  that  the  Decision  did  not  conform  to  
accomplish   this   purpose   can   not   be   attacked   merely   because   they   affect   Cuba's   fundamental   law   and   that   it   had   not   been   published   in   the   "Official  
business  activities  which  are  carried  on  outside  the  state.   Gazette."  But  that  was  insufficient,  as  matter  of  law,  to  establish  that  the  action  
  dishonoring   and   repudiating   the   certificates   was   not   an   act   of   state.   It   was  
Moreover,   if   Virginia   is   without   power   to   require   this   Association   to   accept   incumbent   on   the   plaintiff   to   prove   that   the   Cuban   authorities   themselves  
service  of  process  on  the  Secretary  of  the  Commonwealth,  the  only  forum  for   would  deem  Decision  No.  346  invalid  and  would  disregard  it.  
injured  certificate  holders  might  be  Nebraska.  Health  benefit  claims  are  seldom    
so   large   that   Virginia   policyholders   could   afford   the   expense   and   trouble   of   a   FACTS:  
Nebraska  law  suit.  In  addition,  suits  on  alleged  losses  can  be  more  conveniently   Case   is   grounded   in   a   proclamation   by   Fidel   Castro's   government   that   foreign  
tried   in   Virginia   where   witnesses   would   most   likely   live   and   where   claims   for   investors  can  only  receive  their  return  on  investments  in  the  Cuban  Peso.  The  
losses   would   presumably   be   investigated.   Such   factors   have   been   given   great   investor   here   involved   was   the   plaintiff's   assignor,   Alexander   Ritter,   an  
weight  in  applying  the  doctrine  of  forum  non  conveniens.   American  citizen,  now  living  in  Florida,  who  resided  in  Cuba  at  the  time  of  the  
  events  from  which  this  lawsuit  arises.  Ritter  invested  $350,000  in  a  cuban  farm.  
There   is,   of   course,   one   method   by   which   claimants   could   recover   from   At   the   time   the   Cuban   government   allowed   foreign   investors   to   convert   their  
appellants   in   Virginia   courts   without   the   aid   of   substituted   service   of   process:   proceeds   from   their   enterprises   into   foreign   currency   and   exempted   such  
certificate   holders   in   Virginia   could   all   be   garnished   to   the   extent   of   their   proceeds  from  tax  on  the  importation  of  money.  Certificates  of  tax  exemption  
obligations   to   the   Association.   While   such   an   indirect   procedure   would   amounting  to  $150,000  were  acquired  by  the  investor.  
undeniably   be   more   troublesome   to   claimants   than   the   plan   adopted   by   the   The  certificates  stated  that:  
state   in   its   "Blue   Sky   Law,"   it   would   clearly   be   even   more   harassing   to   the    
Association   and   its   Virginia   members.   Metaphysical   concepts   of   "implied   ALEXANDER   S.   RITTER   or   a   member   Bank   of   the   System,   as   endorsee  
consent"  and  "presence"  in  a  state  should  not  be  solidified  into  a  constitutional   hereof,   will   receive   from   Banco   Nacional   de   Cuba   [defendant   herein]  
barrier   against   Virginia's   simple,   direct   and   fair   plan   for   service   of   process   on   against  delivery  to  said  Bank  of  $  ____  Cuban  Pesos  and  surrender  of  this  
the   Secretary   of   the   Commonwealth.   Virginia's   subjection   of   this   Association   to   Certificate,   a   check   on   New   York   for   an   equal   amount   of   United   States  
the   jurisdiction   of   that   State's   Corporation   Commission   in   a   proceeding   is   Dollars,  exempt  from  the  Tax  on  Exportation  of  Money.  
consistent   with   "fair   play   and   substantial   justice,"   and   is   not   offensive   to   the  
 
CONFLICT  OF  LAWS                                                                                    AV  DE  TORRES   41  
ATTY.  ARIS  L.  GULAPA                                            AY  2015-­‐2016  
  establish,  as  matter  of  law,  that  the  breach  of  contract,  of  which  the  plaintiff  
This   certificate   was   signed   by   defendant   bank   and   the   government   agency   complains,  resulted  from,  and,  indeed,  itself  constitutes,  an  act  of  state.  
responsible   for   issuing   the   certificates   (Cuban   Government's   Currency      
Stabilization   Fund).   Later   that   government   agency   issued   Decision   No.   346,   On   this   analysis,   there   is   no   issue   of   burden   of   proof.   Rather,   the   question   is,  
which   suspended   the   processing   of   tax   exemption   certificates   "for   the   time   what   need   be   proved.   The   defendant   introduced   evidence   showing   that  
being   until   reorganization   of   the   system   of   exemptions."   When   Ritter   tendered   Decision   No.   346   had   been   issued   by   the   Currency   Stabilization   Fund,   that   it  
his  certificates  for  redemption,  together  with  the  appropriate  number  of  pesos,   was  adopted  as  a  measure  to  control  currency  and  foreign  exchange  and  that  
payment  in  American  dollars  was  refused  under  the  mandate  of  the  Decision.   defendant  bank  had  regarded  the  Decision  as  binding  upon  it  and  as  prohibiting  
Hence,  Ritter  filed  a  case  in  the  SC,  New  York  County.  The  act  of  state  doctrine   performance   of   the   agreement   in   the   tax   exemption   certificates.   The   plaintiff  
was   raised   as   a   defense   because   the   decision   of   the   Cuban   Government's   adduced   evidence   to   the   effect   that   the   Decision   did   not   conform   to   Cuba's  
Currency  Stabilization  Fund  was  not  published  in  the  official  gazette.   fundamental   law   and   that   it   had   not   been   published   in   the   "Official   Gazette."  
  But   that   was   insufficient,   as   matter   of   law,   to   establish   that   the   action  
ISSUE:   dishonoring   and   repudiating   the   certificates   was   not   an   act   of   state.   It   was  
Whether  the  act  of  state  doctrine  may  be  raised  by  Banco   incumbent   on   the   plaintiff   to   prove   that   the   Cuban   authorities   themselves  
  would   deem   Decision   No.   346   invalid   and   would   disregard   it.   This   she   was  
HELD:   obviously  unable  to  do.  
Yes.  The  economic  measure  was  one  taken  by  the  Cuban  Government  and  with    
which  the  Bank  complied  with  is  an  act  of  state.  Courts  will  not  inquire  into  the   The   act   of   state   doctrine   is   a   sign   of   respect   between   countries   and   between  
validity   of   the   acts   of   a   foreign   government   done   within   its   own   territory.   the  judiciary  and  the  political  branch  that  the  handles  foreign  affairs.  
Every   sovereign   state   is   bound   to   respect   the   independence   of   every   other    
sovereign  state,  and  the  courts  of  one  country  will  not  sit  in  judgment  of  the   27.   IN   RE:   PHILIPPINE   NATIONAL   BANK   V.   UNITED   STATES   DISTRICT   COURT  
acts   of   the   government   of   another   done   within   its   own   territory.  Courts  will   FOR  THE  DISTRICT  OF  HAWAII  
not  examine  a  foreign  law  to  determine  whether  it  was  adopted  in  conformity   No.  04-­‐71843  |  February  4,  2005  
with  the  internal  procedures  and  requirements  of  the  enacting  state.  So  long  as    
the   act   is   the   act   of   the   foreign   sovereign,   it   matters   not   how   grossly   the   DOCTRINE:   Every   sovereign   state   is   bound   to   respect   the   independence   of   every  
sovereign   has   transgressed   its   own   laws.   If   no   institution   of   legal   authority   other  sovereign  state,  and  the  courts  of  one  country  will  not  sit  in  judgment  on  
would   refuse   to   effectuate   the   decree,   its   formal   status   (because   it   was   not   the  acts  of  the  government  of  another,  done  within  its  own  territory.  Redress  of  
published   in   the   Official   Gazette   in   Cuba)   is   irrelevant.   It   has   not   been   seriously   grievances   by   reason   of   such   acts   must   be   obtained   through   the   means   open   to  
contended   that   the   judicial   institutions   of   Cuba   would   declare   the   decree   be  availed  of  by  sovereign  powers  as  between  themselves.  
invalid.  Nor  does  the  plaintiff  make  any  such  claim.    
  FACTS:  
Consequently,  there  is  no  basis  whatever  for  the  plaintiff's  contention  that  the   There  are  2  parties  in  this  case  which  sought  the  estate  of  Marcos.  One  is  the  
action   dishonoring   and   repudiating   the   certificates   held   by   Ritter   was   not   an   Class  Plaintiffs  and  the  other  is  the  Republic  of  the  Philippines  (RP).  
"act  of  state."  Regardless  of  whether  or  not  Decision  No.  346  was  published  in    
the   Official   Gazette   or   otherwise   complied   with   internal   Cuban   standards   of   The  U.S.  District  Court  (USDC)  in  Hawaii  rendered  judgment  in  favor  of  the  class  
regularity,   it   was   issued   by   the   Currency   Stabilization   Fund,   an   official   of  plaintiffs  against  the  Marcos  estate  for  HR  violations  by  the  Marcos  Regime.  
instrumentality  of  the  Cuban  Government.  Moreover,   in   compliance   with   that   The   judgment   included   an   injunction   restraining   the   estate   from   transferring  
Decision—or   even   if   only   in   purported   compliance—Banco   Nacional,   also   an   any   estate’s   assets.   On   the   other   hand,   the   RP   sought   to   forfeit   the   Marcos  
agency  of  the  Cuban  Government,  refused  and  continues  to  refuse  to  exchange   estate’s   assets   on   the   ground   that   they   were   stolen   by   Marcos   from   the   Phil.  
pesos   for   dollars   as   the   certificates   had   required.   These   undisputed   facts  
 
CONFLICT  OF  LAWS                                                                                    AV  DE  TORRES   42  
ATTY.  ARIS  L.  GULAPA                                            AY  2015-­‐2016  
Gov’t  and  its  people.   means  open  to  be  availed  of  by  sovereign  powers  as  between  themselves.  
   
There   was   an   earlier   case   (Credit   Suisse   Case)   wherein   the   Swiss   Asset   of   The   DC’s   orders  in  issue  violated  this  principle.  In  order  to  obtain  assets  from  
Marcos  estate  had  been  frozen  by  the  Swiss  Gov’t  at  the  request  of  the  RP.  The   the  PNB,  or  to  hold  the  Bank  in  contempt  for  the  transfer  of  those  assets  to  the  
Class  plaintiffs  obtained  injunction  from  USDC  of  Hawaii  to  hold  the  assets  for   RP,   the   District   court   necessarily   (and   expressly)   held   invalid   the   forfeiture  
the   benefit   of   the   class   Plaintiffs.   The   US   CA   issued   a   writ   of   mandamus   and   judgment  of  the  Philippine  SC.  
held   that   the   injunction   violated   the   act   of   state   doctrine,   which   preclude    
American  courts  from  declaring  “invalid”  a  foreign  sovereign’s  official  act,  that   N.B.:   Other   issues   raised   by   the   Class   Plaintiffs:   Whether   the   act   of   state  
is,  the  freeze  order  of  the  Swiss  gov’t.   doctrine   does   not   apply   to   judicial   decisions.   Although   the   act   of   state   doctrine  
  is  normally  inapplicable  to  court  judgments  arising  from  private  litigation,  there  
Thereafter,  the  Swiss  government  released  the  funds  frozen  in  Switzerland  for   is   no   inflexible   rule   preventing   a   judgment   sought   by   a   foreign   government  
transfer  to  the  PNB  in  escrow  pending  a  determination  of  proper  disposal  by  a   from  qualifying  as  an  act  of  state.  
competent  court  in  the  Philippines.  The  PNB  deposited  the  funds  in  Singapore.    
The  Philippine  SC  subsequently  held  that  the  assets  were  forfeited  to  the  RP.   Whether   the   act   of   state   doctrine   was   inapplicable   because   the   judgment   of  
  the   Philippine   SC   did   not   concern   matters   within   its   own   territory.   The   US   CA  
The  USDC  of  Hawaii  then  made  a  ruling  that  the  Philippine  SC  had  violated  “due   held  that,  “generally,  the  act  of  state  doctrine  applies  to  official  acts  of  foreign  
process   by   any   standard”   and   the   latters   judgment   was   entitled   to   no   sovereigns  “performed  within  their  own  territory.”  (Credit  Suisse  Case).  The  act  
deference.  It  ordered  reinstatement  of  an  earlier  settlement  agreement  in  the   of  the  Philippine  SC  was  not  wholly  external,  however.  Its  judgment,  which  the  
District  Court  wherein  the  RP  refused  to  approve  and  consent  to  it.   district   court   declared   invalid,   was   issued   in   the   Philippines   and   much   of   its  
  force   upon   the   PNB   arose   from   the   fact   that   the   Bank   is   a   Philippine   corp.  
The  DC  then  issued  an  “Order  to  Show  Cause”  against  the  PNB,  which  was  not  a   Because   the   RP’s   “interest   in   the   enforcement   of   its   laws   does   not   end   at   its  
party  to  the  litigation  in  the  DC,  requiring  the  Bank  to  show  why  it  should  not   borders,”  the  fact  that  the  escrow  funds  were  deposited  in  Singapore  does  not  
be   held   in   contempt   for   violating   the   court’s   injunction   against   transfer   of   preclude  the  application  of  the  act  of  state  doctrine.  
assets  by  the  estate.  The  PNB  then  filed  the  present  petition  for  mandamus  in    
the  U.S.  9th  Circuit  CA,  seeking  to  restrain  the  DC  from  enforcing  its  “Order  to   28.  REPUBLIC  V.  MARCOS  
Show  Cause”  and  from  pursuing  discovery  against  the  Bank  officer.   862  F.2d  1355  |  December  1,  1988  
   
The  Bank  asserted  that  it  had  transferred  nearly  all  of  the  funds  in  issue  to  the   DOCTRINE:  As  a  practical  tool  for  keeping  the  judicial  branch  out  of  the  conduct  
RP  pursuant  to  the  judgment  of  the  Philippine  SC.  It  contended  that  the  entire   of  foreign  affairs,  the  classification  of  "act  of  state"  is  not  a  promise  to  the  ruler  
proceeding   against   it   for   its   transfer   of   funds   to   the   RP   violated   the   “act   of   of  any  foreign  country  that  his  conduct,  if  challenged  by  his  own  country  after  
state”  doctrine.   his  fall,  may  not  become  the  subject  of  scrutiny  in  our  courts.  No  estoppel  exists  
  insulating  a  deposed  dictator  from  accounting.  No  guarantee  has  been  granted  
ISSUE:   that   immunity   may   be   acquired   by   an   ex-­‐chief   magistrate   invoking   the   magic  
Whether  USDC  of  Hawaii  violated  the  act  of  State  doctrine?   words  "act  of  state"  to  cover  his  or  her  past  performance.  
   
HELD:   The   act   of   state   doctrine   is   supple,   flexible,   ad   hoc.   The   doctrine   is   meant   to  
Yes.   Every   sovereign   state   is   bound   to   respect   the   independence   of   every   facilitate  the  foreign  relations  of  the  United  States,  not  to  furnish  the  equivalent  
other  sovereign  state,  and  the  courts  of  one  country  will  not  sit  in  judgment   of  sovereign  immunity  to  a  deposed  leader.  
of   the   acts   of   the   government   of   another,   done   within   its   own   territory.    
Redress   of   grievances   by   reason   of   such   acts   must   be   obtained   through   the  
 
CONFLICT  OF  LAWS                                                                                    AV  DE  TORRES   43  
ATTY.  ARIS  L.  GULAPA                                            AY  2015-­‐2016  
*N.B.:   What   is   the   difference   between   AOS   and   sovereign   immunity?   AOS   =/=   from   making   pronouncements   on   matters   over   which   it   has   no   power;  
sovereign  immunity   maximally   interpreted,   the   classification   prevents   the   embarrassment   of   a  
AOS  =  facilitates  foreign  relations  and  is  a  sign  of  respect  to  other  sovereigns.   court  offending  a  foreign  government  that  is  "extant  at  the  time  of  suit."  
Sovereign   immunity   merely   pertains   to   insulating   a   deposed   dictator   from    
accountability.   As   a   practical   tool   for   keeping   the   judicial   branch   out   of   the   conduct   of  
  foreign   affairs,   the   classification   of   "act   of   state"   is   not   a   promise   to   the   ruler  
FACTS:   of  any  foreign  country  that  his  conduct,  if  challenged  by  his  own  country  after  
The  Republic  of  the  Philippines  brought  a  civil  suit  against  its  former  president,   his   fall,   may   not   become   the   subject   of   scrutiny   in   our   courts.   No   estoppel  
Ferdinand   Marcos,   and   his   wife   Imelda,   asserting   claims   under   the   Racketeer   exists  insulating  a  deposed  dictator  from  accounting.  No  guarantee  has  been  
Influenced  and  Corrupt  Organizations  Act  (RICO),  and  other  applicable  law.   granted  that  immunity  may  be  acquired  by   an   ex-­‐  chief  magistrate  invoking  the  
  magic  words  "act  of  state"  to  cover  his  or  her  past  performance.  
The   Republic   alleges   the   following:   that   the   Marcoses   engaged   in   mail   fraud,    
wire   fraud,   and   the   transportation   of   stolen   property   in   the   foreign   or   The   classification   might   be   used   to   prevent   judicial   challenge   in   our   courts   to  
interstate   commerce   of   the   United   States.   The   acts   were   repeated,   forming   a   many  deeds  of  a  dictator  in  power,  at  least  when  it  is  apparent  that  sustaining  
pattern   of   predicate   acts.   The   Marcoses   and   the   other   defendants   arranged   for   such   challenge   would   bring   our   country   into   a   hostile   confrontation   with   the  
the   investment   in   real   estate   in   Beverly   Hills,   California   of   $4   million   dictator.  Once  deposed,  the  dictator  will  find  it  difficult  to  deploy  the  defense  
fraudulently  obtained  by  the  Marcoses.  They  arranged  for  the  creation  of  two   successfully.   The   "balance   of   considerations"   is   shifted.   A   fortiori,   when   a  
bank   accounts   in   the   name   of   Imelda   Marcos   at   Lloyds   Bank   of   California   ruler's   former   domain   has   turned   against   him   and   seeks   the   recovery   of   what  
totaling   over   $800,000   also   fraudulently   obtained   by   the   Marcoses;   and   that   it  claims  he  has  stolen,  the  classification  has  little  or  no  applicability.  The  act  
the  Marcoses  transported  into  Hawaii  money,  jewels,  and  other  property  worth   of   state   doctrine   is   supple,   flexible,   ad   hoc.   The   doctrine   is   meant   to   facilitate  
over  $7  million  also  fraudulently  obtained  by  them.   the   foreign   relations   of   the   US,   not   to   furnish   the   equivalent   of   sovereign  
  immunity  to  a  deposed  leader.  
Before   determining   whether   issuance   of   an   injunction   was   appropriate   we    
consider   two   defenses   which,   if   accepted,   would   block   trial   of   the   case:   the   In  the  instant  case  the  Marcoses  offered  no  evidence  whatsoever  to  support  
Marcoses  maintain,  first,  that  their  acts  are  insulated  because  they  were  acts  of   the  classification  of  their  acts  as  acts  of  state.  The  burden  of  proving  acts  of  
state  not  reviewable  by  our  courts;  and  second,  that  any  adjudication  of  these   state   rested   upon   them.   They   did   not   even   undertake   the   proof.   The   act   of  
acts   would   involve   the   investigation   of   political   questions   beyond   our   courts'   state   doctrine,   the   Executive   declares,   has   "no   bearing"   on   this   case   as   it  
competence.   stands.   As   the   doctrine   is   a   pragmatic   one,   we   cannot   exclude   the   possibility  
  that,   at   some   later   point   in   the   development   of   this   litigation,   the   Marcoses  
ISSUE:   might   produce   evidence   that   would   warrant   its   application.   On   the   present  
Whether  the  acts  of  state  doctrine  applies  as  a  defense  in  this  case   record,  the  defense  does  not  apply.  
 
HELD:  
No.   The   classification   of   certain   acts   as   "acts   of   state"   with   the   consequence  
that   their   validity   will   be   treated   as   beyond   judicial   review   is   a   pragmatic  
device,   not   required   by   the   nature   of   sovereign   authority   and   inconsistently  
applied  in  international  law.  The  purpose  of  the  device  is  to  keep  the  judiciary  
from  embroiling  the  courts  and  the  country  in  the  affairs  of  the  foreign  nation  
whose   acts   are   challenged.   Minimally   viewed,   the   classification   keeps   a   court  

 
CONFLICT  OF  LAWS                                                                                    AV  DE  TORRES   44  
ATTY.  ARIS  L.  GULAPA                                            AY  2015-­‐2016  
 
ASSUMPTION  OF  JURISDICTION  V.  FORUM  NON-­‐CONVENIENS  
  Gilbert   brought   this   action   based   on   tort   in   New   York,   but   he   resides   at  
  Lynchburg,   Virginia.   Gulf   Oil   is   a   corporation   organized   under   the   laws   of  
29.  GULF  OIL  CORPORATION  V.  GILBERT   Pennsylvania,  qualified  to  do  business  in  both  Virginia  and  New  York,  and  it  has  
330  U.S.  501  |  1947   designated  officials  in  each  state  as  agents  to  receive  service  of  process.  
   
DOCTRINE:   A   federal   district   court   has   the   power   to   dismiss   an   action   at   law   Gulf   Oil,   invoking   the   doctrine   of   forum   non   conveniens,   claimed   that   the  
pursuant   to   the   doctrine   of   forum   non   conveniens—at   least   where   its   appropriate  place  for  trial  is  Virginia,  not  New  York  where  Gilbert  lives  and  the  
jurisdiction   is   based   on   diversity   of   citizenship   and   the   state   courts   have   such   former  does  business,  where  all  events  in  litigation  took  place,  where  most  of  
power.   the  witnesses  reside,  and  where  both  state  and  federal  courts  are  available  to  
  Gilbert  and  are  able  to  obtain  jurisdiction  of  the  Gulf  Oil.  
The   principle   of   forum   non   conveniens   means   simply   that   a   court   may   resist    
imposition  upon  its  jurisdiction  even  when  jurisdiction  is  authorized  by  the  letter   The   US   District   Court   dismissed   the   case   stating   that   the   case   is   best   left   to  
of  a  general  venue  statute.   Virginia   courts.   On   appeal,   the   Circuit   CA   disagreed.   Hence,   this   case   is   here   on  
  certiorari.  
GR:  The  plaintiff  has  a  choice  of  courts  to  pursue  his  remedy.      
  ISSUE:  
EXC:   The   plaintiff   may   not,   by   choosing   an   inconvenient   forum,   harass   a   1.   Whether   the   US   District   Court   has   the   inherent   power   to   dismiss   a   suit  
defendant   by   inflicting   upon   him   trouble   not   necessary   to   his   own   right   to   pursuant  to  the  doctrine  of  forum  non  conveniens  
pursue  his  remedy.   2.  Whether  the  power  was  abused  in  this  case  
   
Important   considerations   in   the   application   of   the   doctrine   of  forum   non   HELD:  
conveniens,  from   the   standpoint   of   litigants,   are:   relative   ease   of   access   to   1.   Yes.   A   federal   district   court   has   the   power   to   dismiss   an   action   at   law  
sources  of  proof,  availability  of  compulsory  process  for  attendance  of  unwilling   pursuant   to   the   doctrine   of   forum   non   conveniens—at   least   where   its  
witnesses,  cost  of  obtaining  attendance  of  willing  witnesses,  possibility  of  view   jurisdiction  is  based  on  diversity  of  citizenship  and  the  state  courts  have  such  
of   the   premises   if   that   be   appropriate,   and   all   other   practical   problems   that   power.  The  principle  of  forum  non  conveniens  means  simply  that  a  court  may  
make  trial  of  a  case  easy,  expeditious,  and  inexpensive.   resist  imposition  upon  its  jurisdiction  even  when  jurisdiction  is  authorized  by  
  the  letter  of  a  general  venue  statute.  These  are  statutes  drawn  with  generality,  
Considerations   of   public   interest   in   applying   the   doctrine   include   the   giving  a  plaintiff  a  choice  of  courts  to  pursue  his  remedy.  However,  the  plaintiff  
undesirability  of  piling  up  litigation  in  congested  centers,  the  burden  of  jury  duty   may   not,   by   choosing   an   inconvenient   forum,   harass   a   defendant   by   inflicting  
on  people  of  a  community  having  no  relation  to  the  litigation,  the  local  interest   upon   him   trouble   not   necessary   to   his   own   right   to   pursue   his   remedy.   But,   if  
in   having   localized   controversies   decided   at   home,   and   the   unnecessary   the   choice   of   forum   is   strongly   in   favor   of   the   defendant,   the   choice   should  
injection  of  problems  in  conflict  of  laws.   rarely  be  disturbed.  
   
FACTS:   Here,   the   venue   statues   of   US   permit   Gilbert   to   commence   his   action   in   New  
Gilbert   operates   a   public   warehouse.   He   alleges   that   Gulf   Oil   Corp.   carelessly   York.   But   that   does   not   settle   the   question   whether   he   must   do   so.   The  
handled  a  delivery  of  gasoline  to  his  warehouse  tanks  and  pumps,  causing  fire   doctrine   of   forum   non   conveniens   can   never   apply   if   there   is   absence   of  
and   an   explosion,   which   consumed   the   warehouse   building,   merchandise   and   jurisdiction   or   mistake   of   venue.   Courts   of   equity   and   of   law   occasionally  
fixtures  within.     decline,   in   the   interest   of   justice,   to   exercise   jurisdiction   where   the   suit   is  
between  aliens  or  nonresidents,  or  where,  the  litigation  can  more  appropriately  
 
CONFLICT  OF  LAWS                                                                                    AV  DE  TORRES   45  
ATTY.  ARIS  L.  GULAPA                                            AY  2015-­‐2016  
be  conducted  in  a  foreign  tribunal.   reversed.  Petition  granted.  
   
An   interest   to   be   considered,   and   the   one   likely   to   be   most   pressed,   is   the   30.   K.K.   SHELL   SEKIYU   OSAKA   HATSUBAISHO   AND   FU   HING   OIL   CO,   LTD   V.  
private   interest   of   the   litigant.   Important   considerations   are   the   relative   ease   COURT  OF  APPEALS  
of   access   to   sources   of   proof;   availability   of   compulsory   process   for   G.R.  Nos.  90306-­‐07  |  July  30,  1990  
attendance   of   unwilling,   and   the   cost   of   obtaining   attendance   of   willing,    
witnesses;  possibility  of  view  of  premises,  if  view  would  be  appropriate  to  the   FACTS:  
action,   and   all   other   practical   problems   that   make   trial   of   a   case   easy,   Kumagai  Kaiun  Kaisha,  Ltd.,  a   corporation   formed   and   existing   under   the   law   of  
expeditious,   and   inexpensive.   There   may   also   be   questions   as   to   the   Japan,  filed  a  complaint  for  the  collection  of  a  sum  of  money  with  preliminary  
enforceability   of   a   judgment   if   one   is   obtained.  The  court  will  weigh  relative   attachment   before   RTC   Manila   against   Atlantic   Venus   Co.,   a   corporation  
advantages   and   obstacles   to   fair   trial.   The   court   is   not   required   to   always   registered   in   Panama,   the   vessel   MV   Estella   and   Crestamonte   Shipping  
respect  the  forum  chosen  by  the  plaintiff.  What  is  clear  in  a  long  list  of  cases,  is   Corporation,  a  Philippine  corporation.  Atlantic  is  the  owner  of  MV  Estella.  
that   when   the   defendant   consents   to   be   sued,   it   extends   only   to   give   the   court    
jurisdiction  over  the  person.   The   complaint   alleged   that   Crestamonte,   as   bareboat   charter   and   operator   of  
  the   MV   Estella,   appointed   N.S.   Shipping   Corporation   (NSS),   a   Japanese  
2.   No.   Turning   to   the   question   whether   this   is   one   of   those   rather   rare   cases   corporation,  as  its  general  agent  in  Japan.  The  appointment  was  formalized  in  
where  the  doctrine  should  be  applied,  we  look  to  the  interests  of  the  litigants.   an   Agency   Agreement,   which   stated:   “12.0   —   That   this   Agreement   shall   be  
Gilbert   is   not   a   resident   of   New   York,   nor   did   any   event   connected   with   the   governed   by   the   Laws   of   Japan.   Any   matters,   disputes,   and/or   differences  
case   take   place   there,   nor   does   any   witness   with   the   possible   exception   of   arising  between  the  parties  hereto  concerned  regarding  this  agreement  shall  be  
experts  live  there.  No  one  connected  with  that  side  of  the  case  save  counsel  for   subject  exclusively  to  the  jurisdiction  of  the  District  Courts  of  Japan.”  
the   plaintiff   resides   there.   His   affidavits   and   arguments   are   devoted   to    
controverting   claims   as   to   Gulf   Oil’s   inconvenience,   rather   than   to   showing   NSS   in   turn   appointed   Kumagai   as   its   local   agent   in   Osaka,   Japan.   Kumagai  
that   the   present   forum   serves   any   convenience   for   himself,   with   one   supplied   the   MV   Estella   with   supplies   and   services   but   despite   repeated  
exception—that   this   case   involves   a   claim   for   an   amount   close   to   $400,000   demands  Crestamonte  failed  to  pay  the  amounts  due.  NSS  and  Keihin  Narasaki  
which  may  stagger  the  imagination  of  a  local  jury  unaccustomed  to  dealing  with   Corporation  (Keihin)  filed  complaints  in  intervention.  
such   amounts.   This   is   a   strange   and   unproven   premise,   an   assumption   we   do    
not  easily  make.   Fu   Hing   Oil   Co.,   Ltd.   (Fu   Hing),   a   corporation   organized   in   HK   and   not   doing  
  business   in   the   Phil.,   filed   a   motion   for   leave   to   intervene   with   an   attached  
Gulf  Oil  points  out  that  not  only  Gilbert  but  every  person  who  ever  participated   complaint-­‐in-­‐intervention,   alleging   that   Fu   Hing   supplied   marine   diesel   oil   to  
in   the   acts   charged   to   be   negligent   resides   in   or   near   Virginia.   The   Complaint   the   MV   Estella   and   incurred   barge   expenses   for   the   total   sum   of   152,412.56  
itself   alleges   that   the   Gulf   Oil   violated   Virginia   (Lynchburg)   ordinances.   The   USD   but   such   has   remained   unpaid   despite   demand   and   that   the   claim  
source   of   all   proofs   in   either   side   is   in   Virginia,   which   is   some   400   miles   from   constitutes   a   maritime   lien.   The   issuance   of   a   writ   of   attachment   was   also  
New   York.   It   does   not   do   well   to   fix   the   place   of   trial   at   an   inconvenient   forum.   prayed  for.  
The   course   of   adjudication   in   New   York   federal   court   might   be   beset   with    
conflict  of  laws  problems  all  avoided  if  the  case  is  litigated  in  Virginia,  where  it   K.K.  Shell  Sekiyu  Osaka  Hatsubaisho,  a  corporation  organized  in  Japan  and  not  
arose.   doing   business   in   the   Phil.,   likewise   filed   a   motion   to   intervene   with   an  
  attached  complaint  in  intervention  alleging  that  upon  request  of  NSS,  KK  Shell  
Hence,   the   District   Court   did   not   exceed   its   powers   or   the   bounds   of   its   provided   and   supplied   maritime   diesel   oil   to   the   MV   Estella   at   the   ports   of  
discretion  in  dismissing  Gilbert’s  complaint  and  remitting  him  to  the  courts  of   Tokyo  and  Mutsure  in  Japan  and  that  despite  previous  demands,  Crestamonte  
his   own   community.   The   judgment   of   the   Circuit   Court   of   Appeals   is   thus   has   failed   to   pay   the   amounts   of   16,996.96   USD   and   ¥1M   and   that   KK   Shell's  
 
CONFLICT  OF  LAWS                                                                                    AV  DE  TORRES   46  
ATTY.  ARIS  L.  GULAPA                                            AY  2015-­‐2016  
claim   constitutes   a   maritime   lien   on   the   MV   Estella.   The   complaint   in    
intervention  sought  the  issuance  of  a  writ  of  preliminary  attachment.  The  trial   In   the   same   vein,   as   the   choice-­‐of-­‐forum   clause   in   the   agreement   (paragraph  
court  allowed  the  intervention  of  Fu  Hing  and  KK  Shell.   12.0)   has   not   been   conclusively   shown   to   be   binding   upon   K.K.   Shell,   additional  
  evidence   would   also   still   have   to   be   presented   to   establish   this   defense,   K.K.  
Writs   of   preliminary   attachment   were   issued   and   upon   the   posting   of   the   Shell   cannot   therefore,   as   of   yet,   be   barred   from   instituting   an   action   in   the  
counter-­‐bonds,   writs   of   attachment   were   discharged.   Atlantic   and   MV   Estella   Philippines.  
moved  to  dismiss  the  complaints  in  intervention  and  Atlantic  filed  a  petition  in    
the   CA   against   the   lower   court   judge,   Kumagai   NSS   and   Keihin   seeking   the   2.   No.   Atlantic   and   MV   Estella   are   invoking   the   doctrine   of   forum   non  
annulment   of   the   orders   of   the   trial   court.   The   CA   annulled   such   orders   and   conveniens   to   be   a   valid   ground   for   the   dismissal   of   KK   Shell's   complaint   in  
directed  the  RTC  to  cease  and  desist  from  proceeding  with  the  case.   intervention.   KK   Shell,   in   turn,   argued   by   invoking   his   right   as   maritime  
  lienholder  under  PD  No.  1521,  the  Ship  Mortgage  Decree  of  1978.  
CA:   Fu   Hing   and   KK   Shell   were   not   suppliers   but   sub-­‐agents   of   NSS   therefore    
they   were   bound   by   the   Agency   Agreement   between   Crestamonte   and   NSS.   Section   21.   Maritime   Lien   for   Necessaries;   persons   entitled   to   such   lien  
The  LC  should  have  disallowed  the  motions  for  intervention.   Any   person   furnishing   repairs,   supplies,   towage,   use   of   dry   dock   or  
  marine   railway,   or   other   necessaries   to   any   vessel,   whether   foreign   or  
ISSUES:   domestic,   upon   the   order   of   the   owner   of   such   vessel,   or   of   a   person  
1.   Whether   Fu   Hing   and   KK   Shell   should   be   allowed   to   intervene   (since   if   Fu   authorized  by  the  owner,  shall  have  a  maritime  lien  on  the  vessel,  which  
Hing   and   KK   Shell   are   sub-­‐agents   of   NSS,   they   should   not   be   allowed   to   may   be   enforced   by   suit   in   rem,   and   it   shall   be   necessary   to   allege   or  
intervene)   prove  that  credit  was  given  to  the  vessel.  
2.  Whether  the  doctrine  of  forum  non  conveniens  may  be  invoked    
  However,   in   order   to   invoke   this,   it   must   be   established   that   the   credit   was  
HELD:   extended   to   the   vessel   itself.   In   other   words,   considering   the   dearth   of  
1.  No.  No  express  reference  to  the  contracting  of  sub-­‐agents  or  the  applicability   evidence   due   to   the   fact   that   the   private   respondents   have   yet   to   file   their  
of  the  term  of  the  agreement,  particularly  the  choice-­‐of-­‐forum  clause,  to  sub-­‐ answer   in   the   proceeding   below   and   trial   on   the   merits   is   still   to   be   conducted,  
agents  is  made  in  the  text  of  the  agreement.  What  the  contract  clearly  states   whether   or   not   petitioner   are   indeed   maritime   lienholder   and   as   such   may  
are   NSS'   principal   duties,   i.e.,   that   it   shall   provide   for   the   necessary   services   enforce   the   lien   against   the   MV   Estella   are   matters   that   still   have   to   be  
required   for   the   husbanding   of   Crestamonte's   vessels   in   Japanese   ports   and   established.   Neither   is   the   court   ready   to   rule   on   the   private   respondents'  
shall   be   responsible   for   fixing   southbound   cargoes   with   revenues   sufficient   to   invocation  of  the  doctrine  of  forum  non  conveniens,  as  the  exact  nature  of  the  
cover   ordinary   expenses.   Also,   the   complaint   in   intervention   filed   by   KK   Shell   relationship  of  the  parties  is  still  to  be  established.  We  leave  this  matter  to  the  
merely  alleges  it  provided  and  supplied  MV  Estella  with  marine  diesel  oil  upon   LC  who  is  the  best  position  to  decide  such.  It  was  clearly  reversible  error  on   the  
request   of   NSS   who   was   acting   for   and   as   duly   appointed   agent   of   part  of  the  CA  to  annul  the  LC's  orders.  There  are  still  numerous  material  facts  
Crestamonte.  There  is  no  basis  for  the  CA  to  state  that  KK  Shell  admitted  in  its   to  be  established  in  order  to  arrive  at  a  conclusion  as  to  the  true  nature  of  the  
intervention  that  it  was  appointed  as  local  agent/sub-­‐agent  or  representatives   relationship  between  Crestamonte  and  KK  Shell  and  between  NSS  and  KK  Shell.  
by  NSS  by  virtue  of  said  Agency  Agreement.  The  CA  was  erroneously  referring   The   best   recourse  would  have  been  to  allow  the  trial  court  to  proceed  with  the  
to   another   case   involving   another   ship   in   another   court.   Thus,   additional   case   and   consider   whatever   defenses   may   be   raised   by   private   respondents  
evidence  must  be  given  to  establish  such  allegation.   after   they   have   filed   their   answer   and   evidence   to   support   their   conflicting  
  claims  has  been  presented.  
In   view   of   the   inconclusiveness   of   the   Agency   Agreement   and   the   pleadings    
filed  in  the  trial  court,  additional  evidence,  if  there  be  any,  would  still  have  to  
be  presented  to  establish  the  allegation  that  K.K.  Shell  is  a  sub-­‐agent  of  NSS.  
 
CONFLICT  OF  LAWS                                                                                    AV  DE  TORRES   47  
ATTY.  ARIS  L.  GULAPA                                            AY  2015-­‐2016  
31.  COMMUNICATIONS  MATERIAL  AND  DESIGN  V.  COURT  OF  APPEALS   FACTS:  
G.R.  No.  102223  |  August  22,  1996   Petitioners   CMDI   and   ASPAC   Multi-­‐trade   Inc.   are   both   domestic   corporations,  
  while   petitioner   Francisco   S.   Aguirre   is   their   President   and   majority  
DOCTRINE:   GR:   A   foreign   corporation   transacting   business   in   the   Philippines   stockholder.   Private   Respondent   ITEC   is   a   corporation   duly   organized   and  
without  a  license  shall  not  be  permitted  to  maintain  or  intervene  in  any  action,   existing   under   the   laws   of   the   State   of   Alabama,   USA,   and   is   a   foreign  
suit,  or  proceeding  in  any  court  or  administrative  agency  of  the  Philippines,  but   corporation  not  licensed  to  do  business  in  the  Philippines.    
it  may  be  sued  on  any  valid  cause  of  action  recognized  under  Philippine  laws.    
  ITEC   entered   into   a   contract   with   ASPAC   referred   to   as   “Representative  
EXC:   A   foreign   corporation   doing   business   in   the   Philippines   may   sue   in   Agreement”.   Pursuant   to   the   contract,   ITEC   engaged   ASPAC   as   its   “exclusive  
Philippine   Courts   although   not   authorized   to   do   business   here   against   a   representative”   in   the   Philippines   for   the   sale   of   ITEC’s   products,   in  
Philippine   citizen   or   entity   who   had   contracted   with   and   benefited   by   said   consideration   of   which,   ASPAC   was   paid   a   stipulated   commission.   The   said  
corporation.   One   who   has   dealt   with   a   corporation   of   foreign   origin   as   a   agreement  was  initially  for  a  term  of  24  months.  After  the  lapse  of  the  agreed  
corporate  entity  is  estopped  to  deny  its  corporate  existence  and  capacity  having   period,  the  agreement  was  renewed  for  another  24  months.  
ignored  or  taken  advantage  of  the  latter’s  incapacity.    
  ASPAC   and   ITEC   also   entered   into   a   “License   Agreement”   wherein   ASPAC   was  
N.B.:  Corporation  Code,  Se.  133:  Code  of  1987:   able  to  incorporate  and  use  the  name  “ITEC”  in  its  own  name  becoming  legally  
“[S]oliciting  orders,  purchases,  service  contracts,  opening  offices,  whether  called   and   publicly   known   as   ASPAC-­‐ITEC   (Philippines).   By   virtue   of   said   contracts,  
“liaison”   offices   or   branches;   appointing   representatives   or   distributors   who   are   ASPAC  sold  electronic  products,  exported  by  ITEC,  to  their  sole  customer  PLDT.  
domiciled  in  the  Philippines  or  who  in  any  calendar  year  stay  in  the  Philippines   One  year  into  the  second  term  of  the  parties’  Representative  Agreement,  ITEC  
for   a   period   or   periods   totaling   one   hundred   eighty   (180)   days   or   more;   decided   to   terminate   the   same,   because   ASPAC   allegedly   violated   its  
participating   in   the   management,   supervision   or   control   of   any   domestic   contractual  commitment  as  stipulated  in  their  agreements.  
business  firm,  entity  or  corporation  in  the  Philippines,  and  any  other  act  or  acts    
that   imply   a   continuity   or   commercial   dealings   or   arrangements   and   ITEC   filed   a   civil   case   against   the   petitioners   with   the   RTC   of   Makati   and  
2
contemplate   to   that   extent   the   performance   of   acts   or   works,   or   the   exercise   of   charged   them   and   another   Philippine   corporation,   DIGITAL,  of   using  
some   of   the   functions   normally   incident   to,   and   in   progressive   prosecution   of,   knowledge   and   information   of   ITEC’s   products   specifications   to   develop   their  
commercial  gain  or  of  the  purpose  and  object  of  the  business  organization.”  1.   own  line  of  equipment  and  product  support,  which  are  similar,  if  not  identical  
Soliciting,  2.  Opening  offices,  3.  Appointing  representatives  who  stay  in  the  Phil.   to   ITEC’s   own,   and   offering   them   to   ITEC’s   former   customer.   A   MTD   the  
For  180  days,  4.  Participating  in  the  management  of  domestic  business   complaint  was  filed  by  CMDI,  et  al.  on  the  following  grounds:  (1)  that  plaintiff  
  has  no  legal  capacity  to  sue  as  it  is  a  foreign  corporation  doing  business  in  the  
Omnibus  Investments  Code,  Sec.  1:     Philippines   without   the   required   BOI   authority   and   SEC   license,   and   (2)   that  
“(1)   A   foreign   firm   is   deemed   not   engaged   in   business   in   the   Philippines   if   it   plaintiff   is   simply   engaged   in   forum   shopping   which   justifies   the   application  
transacts   business   through   middlemen,   acting   in   their   own   names,   such   as   against   it   of   the   principle   of   “forum   non   conveniens.”   CMDI,   et   al.   argue   that  
indebtors,  commercial  bookers  or  commercial  merchants.   since  plaintiff  has  no  capacity  to  bring  suit  here,  the  Philippines  is  not  the  “most  
(2)   A   foreign   corporation   is   deemed   not   “doing   business”   if   its   representative   convenient  forum”  because  the  trial  court  is  devoid  of  any  power  to  enforce  its  
domiciled   in   the   Philippines   has   an   independent   status   in   that   it   transacts   orders   issued   or   decisions   rendered   in   a   case   that   could   not   have   been  
business  in  its  name  and  for  its  account.”   commenced  to  begin  with.  
   
RTC   of   Makati   issued   an   Order:   (1)   denying   the   MTD,   and   (2)   directing   the  
                                                                                                               
2
 Petitioner  Aguirre  is  also  the  President  of  DIGITAL.  
 
CONFLICT  OF  LAWS                                                                                    AV  DE  TORRES   48  
ATTY.  ARIS  L.  GULAPA                                            AY  2015-­‐2016  
issuance   of   a   writ   of   preliminary   injunction.   CA   affirmed   the   ruling   of   the   lower   whether   the   foreign   corporation   is   continuing   the   body   or   substance   of   the  
court.   business  or  enterprise  for  which  it  was  organized.  
   
ISSUES:   When   a   foreign   corporation   enters   into   a   single   or   isolated   transaction   or  
1.  Whether  ITEC  is  an  unlicensed  corporation  doing  business  in  the  Philippines?     occasional,   incidental,   or   casual   transactions,   which   do   not   come   within   the  
2.  If  it  is,  whether  this  fact  bars  it  from  invoking  the  injunctive  authority  of  our   meaning  of  the  law,  the  foreign  corporation  is  deemed  not  engaged  in  business  
courts?   in   the   Philippines.   Where   a   single   act   or   transaction,   however,   is   not   merely  
  incidental   or   casual   but   indicates   the   foreign   corporation’s   intention   to   do  
HELD:   other   business   in   the   Philippines,   said   single   act   or   transaction   constitutes  
1.  Yes.  Generally,  a  “foreign  corporation”  has  no  legal  existence  within  the  state   “doing”  or  “engaging  in”  or  “transacting”  business  in  the  Philippines.  
in   which   it   is   foreign.   This   proceeds   from   the   principle   that   juridical   existence   of    
a  foreign  corporation  is  confined  within  the  territory  of  the  state  under  whose   The   SC   is   persuaded   to   conclude   that   ITEC   had   been   “engaged   in”   or   “doing  
laws  it  was  incorporated  and  organized,  and  it  has  no  legal  status  beyond  such   business”  in  the  Philippines.  This  is  the  inevitable  result  after  a  scrutiny  of  the  
territory.  Before  a  foreign  corporation  can  transact  business  in  the  Philippines,   different   contracts   and   agreements   entered   into   by   ITEC   with   its   various  
it   must   first   obtain   a   license   to   transact   business   in   the   Philippines,   and   a   business  contacts  in  the  country.  Its  arrangements,  with  these  entities  indicate  
certificate  from  the  appropriate  government  agency.  If  it  transacts  business  in   convincingly   ITEC’s   purpose   to   bring   about   the   situation   among   its   customers  
the  Philippines  without  such  a  license,  it  shall  not  be  permitted  to  maintain  or   and  the  general  public  that  they  are  dealing  directly  with  ITEC,  and  that  ITEC  is  
intervene   in   any   action,   suit,   or   proceeding   in   any   court   or   administrative   actively  engaging  in  business  in  the  country.  
agency   of   the   Philippines,   but   it   may   be   sued   on   any   valid   cause   of   action    
3
recognized  under  Philippine  laws.   A  perusal  of  the  agreements  between  ASPAC  and  the   ITEC  shows  that  there  
  are   provisions   (ex.   “No   Competing   Product”   provision)   which   are   highly  
However,   the   SC   has   not   altogether   prohibited   a   foreign   corporation   not  
licensed  to  do  business  in  the  Philippines  from  suing  or  maintaining  an  action  in                                                                                                                                                                                                                                                          
Philippine   Courts.   What   it   seeks   to   prevent   is   a   foreign   corporation   doing   “[S]oliciting   orders,   purchases,   service   contracts,   opening   offices,   whether   called  
business   in   the   Philippines   without   a   license   from   gaining   access   to   Philippine   “liaison”  offices  or  branches;  appointing  representatives  or  distributors  who  are  
Courts.   domiciled   in   the   Philippines   or   who   in   any   calendar   year   stay   in   the   Philippines  
for   a   period   or   periods   totaling   one   hundred   eighty   (180)   days   or   more;  
 
participating   in   the   management,   supervision   or   control   of   any   domestic  
The  purpose  of  the  law  in  requiring  that  foreign  corporations  doing  business  in   business  firm,  entity  or  corporation  in  the  Philippines,  and  any  other  act  or  acts  
the  Philippines  be  licensed  to  do  so  and  that  they  appoint  an  agent  for  service   that  imply  a  continuity  or  commercial  dealings  or  arrangements  and  contemplate  
of   process   is   to   subject   the   foreign   corporation   doing   business   in   the   to  that  extent  the  performance  of  acts  or  works,  or  the  exercise  of  some  of  the  
Philippines   to   the   jurisdiction   of   its   courts.   The   object   is   not   to   prevent   the   functions   normally   incident   to,   and   in   progressive   prosecution   of,   commercial  
foreign   corporation   from   performing   single   acts,   but   to   prevent   it   from   gain  or  of  the  purpose  and  object  of  the  business  organization.”  
acquiring   a   domicile   for   the   purpose   of   business   without   taking   steps   Acts   that   do   not   constitute   “doing   business   in   the   Philippines”;   Section   1   (f)   (1)   and   1   (f)  
necessary  to  render  it  amenable  to  suit  in  the  local  courts.   (2)   of   the   Rules   and   Regulations   Implementing   the   Omnibus   Investments   Code   of  
  1987,  the  following:  
4 “(1)  A  foreign  firm  is  deemed  not  engaged  in  business  in  the  Philippines  if  it  transacts  
The   true   test   of   what   constitutes   “doing   business   in   the   Philippines”  is  
business   through   middlemen,   acting   in   their   own   names,   such   as   indebtors,  
                                                                                                                commercial  bookers  or  commercial  merchants.  
3
 Section  133  of  the  Corporation  Code.     (2)   A   foreign   corporation   is   deemed   not   “doing   business”   if   its   representative  
4
 Acts   that   constitute   “doing   business   in   the   Philippines”;   Article   44   of   the   Omnibus   domiciled   in   the   Philippines   has   an   independent   status   in   that   it   transacts  
Investments  Code  of  1987:   business  in  its  name  and  for  its  account.”  
 
CONFLICT  OF  LAWS                                                                                    AV  DE  TORRES   49  
ATTY.  ARIS  L.  GULAPA                                            AY  2015-­‐2016  
restrictive   in   nature,   such   as   to   reduce   ASPAC   to   a   mere   extension   or   Thus,  having  acquired  jurisdiction,  it  is  now  for  the  Philippine  Court,  based  on  
instrument  of  ITEC.   the  facts  of  the  case,  whether  to  give  due  course  to  the  suit  or  dismiss  it,  on  
  the   principle   of   forum   non   conveniens.   Hence,   the   Philippine   Court   may  
According   to   petitioner,   the   Philippine   Court   has   no   venue   to   apply   its   refuse   to   assume   jurisdiction   in   spite   of   its   having   acquired   jurisdiction.  
discretion  whether  to  give  cognizance  or  not  to  the  present  action,  because  it   Conversely,   the   court   may   assume   jurisdiction   over   the   case   if   it   chooses   to  
has   not   acquired   jurisdiction   over   the   person   of   the   plaintiff   in   the   case,   the   do  so;  provided,  that  the  following  requisites  are  met:  1)  That  the  Philippine  
latter  allegedly  having  no  personality  to  sue  before  Philippine  Courts.       Court   is   one   to   which   the   parties   may   conveniently   resort   to;   2)   That   the  
  Philippine  Court  is  in  a  position  to  make  an  intelligent  decision  as  to  the  law  
2.  No,  Petitioner  ASPAC  is  however,  barred  by  estoppel.  Petitioner’s  insistence   and  the  facts;  and,  3)  That  the  Philippine  Court  has  or  is  likely  to  have  power  
on  the  dismissal  of  this  action  due  to  the  application,  or  non-­‐application,  of  the   to  enforce  its  decision.  
private  international  law  rule  of  forum  non  conveniens  defies  well-­‐settled  rules    
of  fair  play.     The   aforesaid   requirements   having   been   met,   and   in   view   of   the   court’s  
  disposition   to   give   due   course   to   the   questioned   action,   the   matter   of   the  
Firstly,  the  court  has  already  acquired  jurisdiction  over  the  plaintiff  in  the  suit,   present   forum   not   being   the   “most   convenient”   as   a   ground   for   the   suit’s  
 
by   virtue   of   his   filing   the   original   complaint.    Secondly,   notwithstanding   such   dismissal,  deserves  scant  consideration.
finding   that   ITEC   is   doing   business   in   the   country,   petitioners   are   nonetheless    
estopped  from  raising  this  fact  to  bar  ITEC  from  instituting  this  injunction  case   32.  FIRST  PHILIPPINE  NATIONAL  BANK  V.  CA  
against  it.  A  foreign  corporation  doing  business  in  the  Philippines  may  sue  in   G.R.  No.  115849  |  January  24,  1996  
Philippine   Courts   although   not   authorized   to   do   business   here   against   a    
Philippine   citizen   or   entity,   who   had   contracted   with   and   benefited   by   said   N.B.:  2  REQUISITES  WHEN  THE  COURT  WILL  REFUSE  TO  ASSUME  JURISDICTION:  
corporation.   One   who   has   dealt   with   a   corporation   of   foreign   origin   as   a   1.  When  it  is  in  inconvenient  forum  
corporate  entity  is  estopped  to  deny  its  corporate  existence  and  capacity.   2.  There  is  another  available  convenient  forum  
   
The  rule  is  deeply  rooted  in  the  time-­‐honored  axiom  of  Commodum  ex  injuria   FACTS:  
sua   non   habere   debet   -­‐   no   person   ought   to   derive   any   advantage   of   his   own   Producers  Bank  (now  called  First  Philippine  International  Bank),  which  has  been  
wrong.  This  is  as  it  should  be  for  as  mandated  by  law,  “every  person  must  in  the   under  conservatorship  since  1984,  is  the  owner  of  6  parcels  of  land.  The  Bank  
exercise   of   his   rights   and   in   the   performance   of   his   duties,   act   with   justice,   give   had   an   agreement   with   Demetrio   Demetria   and   Jose   Janolo   for   the   two   to  
everyone  his  due,  and  observe  honesty  and  good  faith.”   purchase   the   parcels   of   land   for   P5.5   million.   The   said   agreement   was   made   by  
  Demetria  and  Janolo  with  the  Bank’s  manager,  Mercurio  Rivera.  Later  however,  
The   doctrine   of   lack   of   capacity   to   sue   based   on   the   failure   to   acquire   a   local   the  Bank,  through  its  conservator,  Leonida  Encarnacion,  sought  the  repudiation  
license   is   based   on   considerations   of   sound   public   policy.   The   license   of  the  agreement  as  it  alleged  that  Rivera  was  not  authorized  to  enter  into  such  
requirement  was  imposed  to  subject  the  foreign  corporation  doing  business  in   an   agreement,   hence   there   was   no   valid   contract   of   sale.   Subsequently,  
the  Philippines  to  the  jurisdiction  of  its  courts.   Demetria  and  Janolo  sued  Producers  Bank.  The  RTC  ruled  in  favor  of  Demetria  
  et  al.  The  Bank  filed  an  appeal  with  the  CA.  
By   entering   into   the   “Representative   Agreement”   with   ITEC,   Petitioner   is    
charged   with   knowledge   that   ITEC   was   not   licensed   to   engage   in   business   Meanwhile,   Henry   Co,   who   holds   80%   shares   of   stocks   with   the   said   Bank,   filed  
activities   in   the   country,   and   is   thus   estopped   from   raising   in   defense   such   a  motion  for  intervention  with  the  trial  court.  The  trial  court  denied  the  motion  
incapacity   of   ITEC,   having   chosen   to   ignore   or   even   presumptively   take   since   the   trial   has   been   concluded   already   and   the   case   is   now   pending   appeal.  
advantage  of  the  same.   Subsequently,   Co,   assisted   by   ACCRA   law   office,   filed   a   separate   civil   case  
 
 
CONFLICT  OF  LAWS                                                                                    AV  DE  TORRES   50  
ATTY.  ARIS  L.  GULAPA                                            AY  2015-­‐2016  
against  Demetria  and  Janolo  seeking  to  have  the  purported  contract  of  sale  be   “In   either   of   these   situations   (choice   of   venue   or   choice   of   remedy),   the   litigant  
declared   unenforceable   against   the   Bank.   Demetria   et   al   argued   that   the   actually   shops   for   a   forum   of   his   action.   This   was   the   original   concept   of   the  
second  case  constitutes  forum  shopping.   term  forum  shopping.  
   
ISSUES:   “Eventually,  however,  instead  of  actually  making  a  choice  of  the  forum  of  their  
Whether  there  is  forum  shopping   actions,  litigants,  through  the  encouragement  of  their  lawyers,  file  their  actions  
  in   all   available   courts,   or   invoke   all   relevant   remedies   simultaneously.   This  
HELD:   practice   had   not   only   resulted   to   (sic)   conflicting   adjudications   among   different  
Yes.   To   begin   with,   forum-­‐shopping   originated   as   a   concept   in   private   courts  and  consequent  confusion  enimical  (sic)  to  an  orderly  administration  of  
international  law,  where  non-­‐resident  litigants  are  given  the  option  to  choose   justice.   It   had   created   extreme   inconvenience   to   some   of   the   parties   to   the  
the   forum   or   place   wherein   to   bring   their   suit   for   various   reasons   or   excuses,   action.  
including   to   secure   procedural   advantages,   to   annoy   and   harass   the   defendant,    
to  avoid  overcrowded  dockets,  or  to  select  a  more  friendly  venue.  To  combat   “Thus,   ‘forum-­‐shopping’   had   acquired   a   different   concept   -­‐   which   is   unethical  
these  less  than  honorable  excuses,  the  principle  of  forum  non  conveniens  was   professional   legal   practice.   And   this   necessitated   or   had   given   rise   to   the  
developed  whereby  a  court,  in  conflicts   of   law   cases,   may   refuse   impositions   formulation   of   rules   and   canons   discouraging   or   altogether   prohibiting   the  
on   its   jurisdiction   where   it   is   not   the   most   “convenient”   or   available   forum   practice.”  
and  the  parties  are  not  precluded  from  seeking  remedies  elsewhere.    
  What  therefore  originally  started  both  in  conflicts  of  laws  and  in  our  domestic  
In   this   light,   Black’s   Law   Dictionary   says   that   forum-­‐shopping   “occurs   when   a   law  as  a  legitimate  device  for  solving  problems  has  been  abused  and  misused  
party   attempts   to   have   his   action   tried   in   a   particular   court   or   jurisdiction   to  assure  scheming  litigants  of  dubious  reliefs.  
where   he   feels   he   will   receive   the   most   favorable   judgment   or   verdict.”   Hence,    
according   to   Words   and   Phrases,   “a   litigant   is   open   to   the   charge   of   ‘forum   33.  MANILA  HOTEL  CORP  V.  NLRC  
shopping’   whenever   he   chooses   a   forum   with   slight   connection   to   factual   G.R.  No.  120077  |  October  13,  2000  
circumstances   surrounding   his   suit,   and   litigants   should   be   encouraged   to    
attempt   to   settle   their   differences   without   imposing   undue   expense   and   DOCTRINE:   Under   the   rule   of  forum   non   conveniens,   a   Philippine   court   or  
vexatious  situations  on  the  courts.”   agency  may  assume  jurisdiction  over  the  case  if  it  chooses  to  do  so  provided:  (1)  
  that  the  Philippine  court  is  one  to  which  the  parties  may  conveniently  resort  to  
In   the   Philippines,   forum-­‐shopping   has   acquired   a   connotation   encompassing   (Are   the   parties   residents   or   nationals   of   the   state   of   the   court?);   (2)   that   the  
not   only   a   choice   of   venues,   as   it   was   originally   understood   in   conflicts   of   laws,   Philippine  court  is  in  a  position  to  make  an  intelligent  decision  as  to  the  law  and  
but   also   to   a   choice   of   remedies.   As   to   the   first   (choice   of   venues),   the   Rules   of   the   facts   (Is   the   state   of   the   court   where   the   contract   is   executed   and  
Court,  for  example,  allow  a  plaintiff  to  commence  personal  actions  “where  the   performed?);  and  (3)  that  the  Philippine  court  has  or  is  likely  to  have  power  to  
defendant   or   any   of   the   defendants   resides   or   may   be   found,   or   where   the   enforce   its   decision   (Does   the   court   have   jurisdiction   over   both   parties?).   The  
plaintiff  or  any  of  the  plaintiffs  resides,  at  the  election  of  the  plaintiff”  (Rule  4,   conditions  are  unavailing  in  the  case  at  bar.  
Sec.  2  [b]).  As  to  remedies,  aggrieved  parties,  for  example,  are  given  a  choice  of    
pursuing  civil  liabilities  independently  of  the  criminal,  arising  from  the  same  set   FACTS:  
of  facts.  A  passenger  of  a  public  utility  vehicle  involved  in  a  vehicular  accident   Marcelo   Santos   was   an   overseas   worker,   a   printer   at   the   Mazoon   Printing  
may  sue  on  culpa  contractual,  culpa  aquiliana  or  culpa  criminal  -­‐  each  remedy   Press,   Sultanate   of   Oman   when   he   was   directly   hired   by   the   Palace   Hotel,  
being   available   independently   of   the   others   -­‐   although   he   cannot   recover   more   Beijing  by  its  GM  Gerhard  Shmidt  as  he  was  recommended  by  Nestor  Buenio,  
than  once.   his   friend.   By   virtue   of   a   "management   agreement,"  Manila   Hotel   Intl.  
 
 
CONFLICT  OF  LAWS                                                                                    AV  DE  TORRES   51  
ATTY.  ARIS  L.  GULAPA                                            AY  2015-­‐2016  
Company,   Ltd   (MHICL)   trained   the   personnel   and   staff   of   the   Palace   Hotel   at   then   employed.   He   was   hired   without   the   intervention   of   the   POEA   or   any  
Beijing,   China.   Santos   resigned   from   Mazoon   and   thereafter   signed   an   authorized  recruitment  agency  of  the  government.  
employment   contract   mailed   to   him   by   Mr.   Henk.   He   went   home   to   the    
Philippines   then,   proceeded   to   Beijing.   The   contract   stated   it   would   be   for   a   Under   the   rule   of  forum   non   conveniens,   a   Philippine   court   or   agency   may  
period  of  2  years.   assume  jurisdiction  over  the  case  if  it  chooses  to  do  so  provided:  (1)  that  the  
  Philippine   court   is   one   to   which   the   parties   may   conveniently   resort   to;   (2)  
After  a  short  vacation  in  the  Phil  &  barely  a  year  into  the  contract,  Santos  was   that  the  Philippine  court  is  in  a  position  to  make  an  intelligent  decision  as  to  
terminated   from   his   job   due   to   retrenchment,   and   repatriated   to   the   Phil.   the   law   and   the   facts;   and   (3)   that   the   Philippine   court   has   or   is   likely   to   have  
Santos,   through   his   lawyer,   demanded   full   compensation   pursuant   to   the   power  to  enforce  its  decision.  The  conditions  are  unavailing  in  the  case  at  bar.  
employment   agreement,   which   Shmidt   denied.   Santos   then   filed   a   complaint    
with   the   NLRC   against   MHC,   MHICL,   the   Palace   Hotel   &   Shmidt   for   illegal   Not   Convenient—From   the   time   of   recruitment,   to   employment   to   dismissal  
dismissal.   occurred  outside  the  Philippines.  The  inconvenience  is  compounded  by  the  fact  
  that   the   proper   defendants,   the   Palace   Hotel   and   MHICL   are   not   nationals   of  
The  Labor  Arbiter  granted  payment  of  damages  to  Santos.  petitioners  appealed   the  Philippines.  Neither  are  they  "doing  business  in  the  Philippines."  Likewise,  
to   the   NLRC,   arguing   that   the   POEA,   not   the   NLRC   had   jurisdiction   over   the   the   main   witnesses,   Mr.   Shmidt   and   Mr.   Henk   are   non-­‐residents   of   the  
case.   NLRC   thus,   declared   the   arbiter’s   decision   as   null   and   void   for   want   of   Philippines.  
jurisdiction.  Santos  filed  a  MR  arguing  that  the  case  was  not  cognizable  by  the    
POEA  as  he  was  not  an  "overseas  contract  worker."  NLRC  granted  the  motion   No  power  to  determine  applicable   law—Neither  can  an  intelligent  decision  be  
and  reversed  itself,  finding  Santos  illegally  dismissed  &  recommended  that  he   made  as  to  the  law  governing  the  employment  contract  as  such  was  perfected  
be  paid  actual  damages  equivalent  to  his  salaries  for  the  unexpired  portion  of   in   foreign   soil.   This   calls   to   fore   the   application   of   the   principle   of   lex   loci  
his  contract.   contractus   (the   law   of   the   place   where   the   contract   was   made).   The  
  employment  contract  was  not  perfected  in  the  Philippines.  Santos  signified  his  
 
Hence,   this   petition   for   certiorari to   annul   the   orders   of   the   NLRC   for   having   acceptance  by  writing  a  letter  while  he  was  in  Oman.  This  letter  was  sent  to  the  
been   issued   without   or   with   excess   jurisdiction   and   with   grave   abuse   of   Palace  Hotel  in  the  People's  Republic  of  China.  
discretion.    
  No   power   to   determine   the   facts—Neither   can   the   NLRC   determine   the   facts  
ISSUE:   surrounding  the  alleged  illegal  dismissal  as  all  acts  complained  of  took  place  in  
1.  Whether  the  NLRC  is  a  convenient  forum   Beijing,   People's   Republic   of   China.   The   NLRC   was   not   in   a   position   to  
2.  Whether  Manila  Hotel  Corp.  is  liable  to  Santos   determine   whether   the   Tiannamen   Square   incident   truly   adversely   affected  
  operations  of  the  Palace  Hotel  as  to  justify  respondent  Santos'  retrenchment.  
HELD:    
1.   No,   the   NLRC   was   a   seriously   inconvenient   forum.   The   main   aspects   of   the   Principle  of  effectiveness,  no  power  to  execute  decision.  —  Even  assuming  that  
case   transpired   in   two   foreign   jurisdictions   and   the   case   involves   purely   foreign   a   proper   decision   could   be   reached   by   the   NLRC,   such   would   not   have   any  
elements.   The   only   link   that   the   Philippines   has   with   the   case   is   that   binding   effect   against   the   employer,   the   Palace   Hotel.   The   Palace   Hotel   is   a  
respondent  Santos  is  a  Filipino  citizen.  The  Palace  Hotel  and  MHICL  are  foreign   corporation   incorporated   under   the   laws   of   China   and   was   not   even   served  
corporations.  Not  all  cases  involving  our  citizens  can  be  tried  here.   with  summons.  Jurisdiction  over  its  person  was  not  acquired.  
   
The   employment   contract—Santos   was   hired   directly   by   the   Palace   Hotel,   a   This  is  not  to  say  that  Philippine  courts  and  agencies  have  no  power  to  solve  
foreign   employer,   through   correspondence   sent   to   Oman,   where   Santos   was   controversies  involving  foreign  employers.  Neither  are  we  saying  that  we  do  
not  have  power  over  an  employment  contract  executed  in  a  foreign  country.  
 
CONFLICT  OF  LAWS                                                                                    AV  DE  TORRES   52  
ATTY.  ARIS  L.  GULAPA                                            AY  2015-­‐2016  
If  Santos  were  an  "overseas  contract  worker,"  a  Philippine  forum,  specifically   34.  PACIFIC  CONSULTANTS  V.  SCHONFELD  
 
the   POEA,   not   the   NLRC,   would   protect   him. He   is   not   an   "overseas   contract   G.R.  No.  166920  |  February  19,  2007  
worker"  a  fact  which  he  admits  with  conviction.    
  N.B.:  Requisite  #  1  –  Plaintiff  is  a  resident  of  the  Philippines.  PPI  (employer)  is  a  
Even   assuming   that   the   NLRC   was   the   proper   forum,   even   on   the   merits,   the   Philippine  Corporation  
NLRC's  decision  cannot  be  sustained.    
  Requisite  #  2  –  The  employer  was  a  Philippine  Corporation.  Plaintiff  was  issued  
2.   Granted.   Piercing   the   veil   of   corporate   fiction–fact   that   MHC   is   an   an   Alien   Employment   Permit   by   DOLE   (as   required   by   the   Omnibus   Rules  
incorporator   &   owns   50%   of   the   capital   stock   of   MHICL   is   not   enough   to   pierce   Implementing  the  Labor  Code).  Thus,  the  applicable  law  is  the  Labor  Code.  As  
the  veil.  Even  if  we  assume:  NLRC  had  jurisdiction  over  the  case  &  MHICL  was   claimed   by   respondent,   he   had   an   employment   contract   with   petitioner   PPI;  
liable   for   Santos’   retrenchment,   still   MHC,   as   a   separate   &   distinct   juridical   otherwise,  petitioner  PPI  would  not  have  filed  an  application  for  a  Permit  with  
entity,  cannot  be  held  liable.  Piercing  the  veil  is  an  equitable  remedy.  When  the   the   DOLE.   Petitioners   are   thus   estopped   from   alleging   that   the   PCIJ,   not  
notion   of   legal   entity   is   used   to   defeat   public   convenience,   justify   wrong,   petitioner  PPI,  had  been  the  employer  of  respondent  all  along.  
protect  fraud,  or  defend  crime,  the  law  will  regard  the  corp.  as  an  association    
of   persons.   It   is   done   only   when   the   corp   is   a   mere   alter   ego   or   business   Requisite  #  3  –  The  NLRC/Court  had  jurisdiction  over  the  parties.  
conduit  of  a  person  or  another  corp.    
  FACTS:  
Clear   &   convincing   evidence   is   needed   to   pierce   the   veil   of   corporate   fiction.   Schonfeld   is   a   Canadian   citizen.   Pacicon   Philippines,   Inc.   (PPI)   is   a   corporation  
There  is  no  such  evidence  to  show  that  MHICL  &  MHC  are  1  &  the  same  entity.   duly   established   and    incorporated   in   accordance   with   the   laws   of   the  
  Philippines   and   is   a   subsidiary   of    Pacific   Consultants   International   of   Japan  
Test  to  enable  piercing  of  the  veil,  except  in  express  agency,  estoppel  or  direct   (PCIJ).    
tort:   a)   Control,   not   mere   majority   or   complete   domination;   b)   Such   control    
must  have  been  used  by  the  defendant  to  commit  fraud  or  wrong,  etc.;  c)  The   In   1997,   PCIJ   decided   to   engage   in   consultancy   services   for   water   and  
aforesaid   control   &   breach   of   duty   must   approximately   cause   the   injury   or    sanitation   in   the   Philippines.   Schonfeld   was   employed   by   PCIJ   as   PPI   Sector  
unjust  loss  complained  of.   Manager   in   its   Water   and   Sanitation   Department.   His   salary   was   to   be   paid  
  partly  by  PPI  and  PCIJ.    
Fact  that  the  Palace  Hotel  is  a  member  of  the  Manila  Hotel  Group  is  not  enough    
to  pierce  the  corporate  veil  –  there  is  no  evidence  to  show  that  they  are  1  &  the   In   January   1998,   Henrichsen   sent   a   letter   of   employment   to   Schonfeld   in  
same  entity.   Canada.  Schonfeld  made  some  revisions,  signed  the  contract,  and  sent  a  copy  
  to   Henrichsen.   The   arbitration   clause   of   the   contract   provides   that   “any  
Contrary   to   what   Santos   claims   that   MHICL   signed   his   employment   contract,   question   of   interpretation,   understanding   or   fulfillment   of   the   conditions   of  
MHICL   Vice-­‐President   signed   as   a   mere   witness   under   the   word   ‘noted’.   employment,   as   well   as   any   question   arising   between   the   Employee   and   the  
Furthermore,  there  is  no  EER  between  Santos  &  MHICL.   Company  which  is  in  consequence  of  or  connected  with  his  employment  with  
  the  Company  and  cannot  be  settled  amicably,  is  to  be  finally  settled,  binding  to  
both   parties   through   written   submissions,   by   the   Court   of   Arbitration   in  
London.”    
 
Schonfeld  received  his  compensation  from  PPI  and  was  also  reimbursed  for  the  
expenses   he   incurred   in   connection   with   his   work   as   sector   manager.   He  

 
CONFLICT  OF  LAWS                                                                                    AV  DE  TORRES   53  
ATTY.  ARIS  L.  GULAPA                                            AY  2015-­‐2016  
reported   for   work   in   Manila   except   for   occasional   assignments   abroad,   and   them.  
received   instructions   from   Henrichsen.   On   May   5,   1999,   Schonfeld   received   a    
letter   from   Henrichsen   informing   him   that   his   employment   had   been   In   the   instant   case,   no   restrictive   words   like   "only,"   "solely,"   "exclusively   in  
terminated   effective   Aug.   4,   1999.   However,   on   July   24,   1999,   Henrichsen,   this   court,"   "in   no   other   court   save   —,"   "particularly,"   "nowhere   else  
requested  him  to  stay  put  in  his  job  after  Aug.  5,  1999.  Schonfeld  continued  his   but/except   —,"   or   words   of   equal   import   were   stated   in   the   contract.   It  
work  with  PPI  until  Oct.  1,  1999.     cannot  be  said  that  the  court  of  arbitration  in  London  is  an  exclusive  venue  to  
  bring  forth  any  complaint  arising  out  of  the  employment  contract.  
Thus,   Schonfeld   filed   with   PPI   several   money   claims,   including   unpaid   salary,    
leave   pay,   air   fare   from   Manila   to   Canada,   and   cost   of   shipment   of   goods   to   Petitioners  contend  that  respondent  Schonfeld  should  have  filed  his  complaint  
Canada.   PPI   partially   settled   some   of   his   claims   (US$5,635.99),   but   refused   to   in  his  place  of  permanent  residence,  or  where  the  PCIJ  holds  its  principal  office,  
pay  the  rest.  In  2000,  he  filed  a  Complaint  for  Illegal  Dismissal  against  PPI  and   at  the  place  where  the  contract  of  employment  was  signed,  in  London  as  stated  
Henrichsen  with  the  Labor  Arbiter.     in  their  contract.  By  enumerating  possible  venues  where  Schonfeld  could  have  
  filed   his   complaint,   petitioners   themselves   admitted   that   the   provision   on  
Labor   Arbiter:   The   employment   contract   between   Schonfeld   and   PCIJ   was   venue  in  the  employment  contract  is  merely  permissive.  
controlling.   Since   the   parties   had   agreed   that   any   differences   regarding    
employer-­‐employee  relationship  should  be  submitted  to  the  jurisdiction  of  the   As   for   petitioners’   insistence   on   the   application   of   the   principle   of   forum   non  
court   of   arbitration   in   London,   only   the   court   of   arbitration   in   London   has   conveniens,   the   same   must   be   rejected.   The   bare   fact   that   Schonfeld   is   a  
jurisdiction.   NLRC   affirmed.   However,   on   appeal,   the   CA   held   that   the   parties   Canadian  citizen  and  was  a  repatriate  does  not  warrant  the  application  of  the  
were  not  precluded  from  bringing  a  case  related  thereto  in  other  venues.  The   principle  for  the  following  reasons:  
venue  is  not  exclusive,  since  there  is  no  stipulation  that  the  complaint  cannot    
be  filed  in  any  other  forum  other  than  in  the  Philippines.   (1)  The  Labor  Code  of  the  Philippines  does  NOT  include  forum  non  conveniens  
  as   a   ground   for   the   dismissal   of   the   complaint.  (2)  The   propriety   of   dismissing  
ISSUE:   a   case   based   on   this   principle   requires   a   factual   determination;   hence,   it   is  
Whether   the   labor   arbiter   has   jurisdiction   over   Schonfeld’s   claim   despite   the   properly   considered   as   defense.   (3)   In   Bank   of   America,   NT&SA,   Bank   of  
undisputed   fact   that   Schonfeld,   a   foreign   national,   was   hired   abroad   by   a   America   International,   Ltd.   v.   Court   of   Appeals,   the   Court   held   that  
foreign   corporation,   executed   his   employment   contract   abroad,   and   had   the  Philippine  Court  may  assume  jurisdiction  over  the  case  if  it  chooses  to  do  
agreed  that  any  dispute  between  them  “shall  finally  be  settled  by  the  court  of   so   if   the   following   requisites   are   met:   (1)   the  Philippine  Court  is  one  to  which  
arbitration  in  London”   the  parties  may  conveniently  resort  to;  (2)  the  Philippine  Court  is  in  a  position  
  to  make  an  intelligent  decision  as  to  the  law  and  the  facts;  and,  
HELD:    
Yes.   The   settled   rule   on   stipulations   regarding   venue,   as   held   in   the   vintage    
case   of   Philippine   Banking   Corporation   v.   Tensuan,   is   that   while   they   are  
WHICH/WHAT  LAW  THE  ASSUMING  COURT  WILL  APPLY  
considered   valid   and   enforceable,   venue   stipulations   in   a   contract   do   NOT,   as    
a  rule,  supersede  the  general  rule  set  forth  in  Rule  4  of  the  Revised  Rules  of    
Court   in   the   absence   of   qualifying   or   restrictive   words.   They   should   be   35.  AZNAR  V.  CHRISTENSEN-­‐GARCIA  
considered   merely   as   an   agreement   on   additional   forum,   not   as   limiting   G.R.  No.  L-­‐16749  |  January  31,  1963  
venue  to  the  specified  place.  They  are  not  exclusive  but  rather  permissive.  If    
the   intention   of   the   parties   were   to   restrict   venue,   there   must   be   FACTS:  
accompanying   language   clearly   and   categorically   expressing   their   purpose   Edward  Christensen,  born  in  New  York,  migrated  to  California  where  he  resided  
and  design  that  actions  between  them  be  litigated  only  at  the  place  named  by   and   consequently   was   considered   citizen   thereof.   He   came   to   the   Philippines  
 
CONFLICT  OF  LAWS                                                                                    AV  DE  TORRES   54  
ATTY.  ARIS  L.  GULAPA                                            AY  2015-­‐2016  
where  he  became  a  domiciliary  until  the  time  of  his  death.  However,  during  the    
entire   period   of   his   residence   in   this   country,   he   had   always   considered   himself   However,   intestate   and   testamentary   successions,   both   with   respect   to  
a  citizen  of  California.   the  order  of  succession  and  to  the  amount  of  successional  rights  and  to  
  the   intrinsic   validity   of   testamentary   provisions,   shall   be   regulated   by   the  
In   his   will,   Edward   instituted   his   daughter   Maria   Lucy   Christensen   as   his   only   national   law   of   the   person   whose   succession   is   under   consideration,  
heir,  but  left  a  legacy  of  P3600  in  favor  of  Helen  Christensen  Garcia  who,  in  his   whatever   may   be   the   nature   of   the   property   and   regardless   of   the  
will  was  described  as  "not  in  any  way  related  to"  him  but  in  a  decision  rendered   country  where  said  property  may  be  found.  
by   the   SC   in   another   case   had   been   declared   as   an   acknowledged   natural    
daughter  of  his.   The  laws  of  California  have  prescribed  two  sets  of  laws  for  its  citizens,  one  for  
  residents   therein   and   another   for   those   domiciled   in   other   jurisdictions.   Art.  
Helen   alleged   that   the   will   deprives   her   of   her   legitime   as   an   acknowledged   946   of   the   California   Civil   Code   is   its   conflict   of   laws   rule,   while   the   rule  
natural  child.  She  claims  that  under  Art.  16  of  the  Civil  Code,  the  California  law   applied  in  Kaufman,  is  its  internal  law.  If  the  law  on  succession  and  the  conflict  
should   be   applied,   and   in   accordance   therewith   and   following   the   doctrine   of   of  laws  rules  of  California  are  to  be  enforced  jointly,  each  in  its  own  intended  
the  renvoi,   the   question   of   the   validity   of   the   testamentary   provision   in   and   appropriate   sphere,   the   principle   cited   in   Kaufman   should   apply   to  
question  should  be  referred  back  to  the  law  of  the  decedent's  domicile,  which   citizens  living  in  the  State,  but  Art.  946  should  apply  to  such  of  its  citizens  as  
is   the   Philippines.   The   question   of   the   validity   of   the   testamentary   provision   are  not  domiciled  in  California  but  in  other  jurisdictions.  
should  thus  be  referred  back  to  the  law  of  the  decedent’s  domicile,  which  is  the    
Philippines.   She   invokes   the   provisions   of   Art.   946   of   the   Civil   Code   of   The   national   law   mentioned   in   Art.   16   is   the   law   on   conflict   of   laws   in   the  
California,  which  is  as  follows:  “If  there  is  no  law  to  the  contrary,  in  the  place   California  Civil  Code,  i.e.,  Art.  946,  which  authorizes  the  reference  or  return  of  
where   personal   property   is   situated,   it   is   deemed   to   follow   the   person   of   its   the   question   to   the   law   of   the   testator's   domicile.   The   conflict   of   laws   rule   in  
owner,  and  is  governed  by  the  law  of  his  domicile.”  Accordingly,  her  share  must   California,   Art.   946,   Civil   Code,   precisely   refers   back   the   case,   when   a   decedent  
be   increased   in   view   of   successional   rights   of   illegitimate   children   under   is  not  domiciled  in  California,  to  the  law  of  his  domicile,  which  is  the  Philippines  
Philippine  laws.   in  the  case  at  bar.  
   
On   the   other   hand,   the   executor   and   Lucy   argue   that   the   national   law   of   the   The   Philippine   court   therefore   must   apply   its   own   law   as   directed   in   the  
deceased  must  apply,  and  thus  the  courts  must  apply  internal  law  of  California   conflict   of   laws   rule   of   the   state   of   the   decedent.   Wherefore,   the   decision  
on   the   matter.   Under   California   law,   there   are   no   compulsory   heirs   and   appealed   from   is   hereby   reversed   and   the   case   returned   to   the   lower   court   with  
consequently   a   testator   may   dispose   of   his   property   by   will   in   the   form   and   instructions   that   the   partition   be   made   as   the   Philippine   law   on   succession  
manner  he  desires  (Kaufman  Case).   provides.  
   
ISSUE:   36.  BELLIS  V.  BELLIS  
Whether  Philippine  law  should  ultimately  be  applied   G.R.  No.  L-­‐23678  |  June  6,  1967  
   
HELD:   FACTS:  
Yes.  Edward  was  a  US  Citizen  and  domiciled  in  the  Philippines  at  the  time  of  his   Amos  Bellis  was  a  citizen  and  resident  of  Texas  at  the  time  of  his  death.         He  
death.   The   law   that   governs   the   validity   of   his   testamentary   dispositions   is   had  5  legitimate  children  with  his  first  wife,  Mary  Mallen,  whom  he  divorced.  
defined  in  Art.  16,  CC,  which  is  as  follows:   He  had  3  legitimate  daughters  with  his  second  wife,  Violet,  who  survived  him,  
  and   another   3   illegitimate   children   with   another   woman.   Before   he   died,   he  
ART.  16.  Real  property  as  well  as  personal  property  is  subject  to  the  law   executed   2   wills—the   first   one   disposing   of   his   Texas   properties,   the   other  
of  the  country  where  it  is  situated.   disposing   his   Philippine   properties.  In   his   will,   which   he   executed   in   the  
 
CONFLICT  OF  LAWS                                                                                    AV  DE  TORRES   55  
ATTY.  ARIS  L.  GULAPA                                            AY  2015-­‐2016  
Philippines,   he   directed   that   after   all   taxes,   obligations,   and   expenses   of   country  wherein  said  property  may  be  found.  
administration  are  paid  for,  his  distributable  estate  should  be  divided,  in  trust,    
in  the  following  order  and  manner:  a)  $240,000.00  to  his  first  wife  Mary  Mallen   As   a   counter-­‐argument,   appellants   invoked   Art.   17,   Par.   3,   of   the   Civil   Code,  
b)  $120,000.00  to  his  three  illegitimate  children  Amos  Bellis,  Jr.,  Maria  Cristina   stating—  
Bellis,  Miriam  Palma  Bellis,  or  $40,000.00  each,  and  c)  After  foregoing  the  two    
items  have  been  satisfied,  the  remainder  shall  go  to  his  seven  surviving  children   Prohibitive   laws   concerning   persons,   their   acts   or   property,   and   those  
by  his  first  and  second  wives.   which  have  for  their  object  public  order,  public  policy  and  good  customs  
  shall   not   be   rendered   ineffective   by   laws   or   judgments   promulgated,   or  
Maria  Cristina  Bellis  and  Miriam  Palma  Bellis,  filed  their  respective  oppositions   by  determinations  or  conventions  agreed  upon  in  a  foreign  country.  
to   the   project   of   partition   on   the   ground   that   they   were   deprived   of   their    
legitimes   as   illegitimate   children   and,   therefore,   compulsory   heirs   of   the   However,   it   is   evident   that   whatever   public   policy   or   good   customs   may   be  
deceased.   involved  in  our  System  of  legitimes  (that  is  the  public  policy  that  a  compulsory  
  heir  cannot  be  deprived  of  his  legitime),  Congress  has  not  intended  to  extend  
The  lower  court  issued  an  order  overruling  the  oppositions  and  approving  the   the  same  to  the  succession  of  foreign  nationals.  For  it  has  specifically  chosen  
executor’s   final   account,   report   and   administration,   and   project   of   partition.   to   leave,  inter   alia,   the  amount  of   successional   rights,   to   the   decedent's  
Relying   upon   Art.   16   of   the   Civil   Code,   it   applied   the   national   law   of   the   national  law.  Specific  provisions  must  prevail  over  general  ones.  A  provision  in  
decedent,  which  in  this  case  did  not  provide  for  legitimes.   a   foreigner's   will   to   the   effect   that   his   properties   shall   be   distributed   in  
  accordance  with  Philippine  law  and  not  with  his  national  law,  is  illegal  and  void,  
ISSUE:   for   his   national   law   cannot   be   ignored   in   regard   to   those   matters   that   Article  
Whether  such  illegitimate  children  of  Bellis  be  entitled  to  successional  rights   10—now  Article  16—of  the  Civil  Code  states  said  national  law  should  govern.  
   
HELD:   In   this   regard,   the   parties   do   not   submit   the   case   on,   nor   even   discuss,   the  
No.   The   said   illegitimate   children   are   not   entitled   to   their   legitimes.   Under   doctrine  of  renvoi,  applied  by  this  Court  in  Aznar  v.  Christensen.  Said  doctrine  is  
Texas   law,   there   are   no   legitimes.   Even   if   the   other   will   was   executed   in   the   usually   pertinent   where   the   decedent   is   a   national   of   one   country,   and   a  
Philippines,  his  national  law,  still,  will  govern  the  properties  for  succession  even   domicile   of   another.   In   the   present   case,   it   is   not   disputed   that   the   decedent  
 
if  it  is  stated  in  his  testate  that  it  shall  be  governed  by  the  Philippine  law.   was  both  a  national  of  Texas  and  a  domicile  thereof  at  the  time  of  his  death. So  
  that   even   assuming   Texas   has   a   conflict   of   law   rule   providing   that   the  
Art.16,  Par.  2  renders  applicable  the  national  law  of  the  decedent,  in  intestate   domiciliary   system   (law   of   the   domicile)   should   govern,   the   same   would   not  
and   testamentary   successions,   with   regard   to   four   items:   (a)   the   order   of   result   in   a   reference   back   (renvoi)   to   Philippine   law,   but   would   still   refer   to  
succession,   (b)   the   amount   of   successional   rights,   (c)   the   intrinsic   validity   of   Texas   law.   Nonetheless,   if   Texas   has   a   conflicts   rule   adopting   the   situs   theory  
provisions  of  will,  and  (d)  the  capacity  to  succeed.   (lex   rei   sitae)   calling   for   the   application   of   the   law   of   the   place   where   the  
  properties  are  situated,  renvoi  would  arise,  since  the  properties  here  involved  
ART.16  Real  property  as  well  as  personal  property  is  subject  to  the  law  of   are  found  in  the  Philippines.  In  the  absence,  however,  of  proof  as  to  the  conflict  
the  country  to  where  it  is  situated.   of  law  rule  of  Texas,  it  should  not  be  presumed  different  from  ours.  Appellants'  
  position  is  therefore  not  rested  on  the  doctrine  of  renvoi.  As  stated,  they  never  
However,   intestate   and   testamentary   successions,   both   with   respect   to   invoked  nor  even  mentioned  it  in  their  arguments.  Rather,  they  argue  that  their  
the  order  of  successions  and  to  the  amount  of  successional  rights  and  to   case  falls  under  the  circumstances  mentioned  in  the  third  paragraph  of  Article  
the   intrinsic   validity   of   testamentary   provisions,   shall   be   regulated   by   the   17  in  relation  to  Article  16  of  the  Civil  Code.  
national   law   of   the   person   whose   succession   is   under   consideration,    
whatever   may   be   the   nature   of   the   property   and   regardless   of   the  
 
CONFLICT  OF  LAWS                                                                                    AV  DE  TORRES   56  
ATTY.  ARIS  L.  GULAPA                                            AY  2015-­‐2016  
37.  CADALIN  V.  POEA  ADMINISTRATOR   projects  in  several  countries  in  the  Middle  East  as  well  as  in  Southeast  Asia,  in  
G.R.  No.  L-­‐104776  |  December  5,  1994   Indonesia   and   Malaysia.   The   case   arose   when   their   overseas   employment  
  contracts   were   terminated   even   before   their   expiration.   Under   Bahrain   law,  
DOCTRINE:  GR:  A  foreign  procedural  law  will  not  be  applied  in  the  forum.   where   some   of   the   complainants   were   deployed,   the   prescriptive   period   for  
  claims  arising  out  of  a  contract  of  employment  is  one  year.  
EXC:  Law  on  prescription  of  actions  is  sui  generis  and  may  be  viewed  procedural    
or  substantive  depending  on  its  characterization.   ISSUE:    
  1.   Whether   it   is   the   Bahrain   law   on   prescription   of   action   based   on   the   Amiri  
EXC   TO   THE   EXC:   The   characterization   is   irrelevant   when   the   country   of   the   Decree  No.  23  of  1976  (1  year)  or  a  Philippine  law  on  prescription  that  shall  be  
forum  has  a  "borrowing  statute,"  the  country  of  the  forum  will  apply  the  foreign   the  governing  law.  
statute  of  limitations.   2.  Whether  the  prescriptive  period  governing  the  filing  of  the  claims  is  3  years,  
  as  provided  by  the  Labor  Code  or  10  years,  as  provided  by  the  NCC.  
EXC  TO  THE  EXC  TO  THE  EXC:  The  court  of  the  forum  will  not  enforce  any  foreign   3.  Whether  it  is  the  overseas-­‐employment  contracts,  which  became  the  law  of  
claim  obnoxious  to  the  forum's  public  policy.   the  parties,  or  the  Amiri  Decree,  which  is  more  favorable  and  beneficial  to  the  
  workers,  that  shall  be  enforced.  
FACTS:    
On   June   6,   1984,   Bienvenido   M.   Cadalin,   Rolando   M.   Amul   and   Donato   B.   HELD:  
Evangelista,   in   their   own   behalf   and   on   behalf   of   728   other   overseas   contract   1.   Philippine   law.   Art.   156   of   the   Amiri   Decree   No.   23   of   1976   provides:   A   claim  
workers   (OCWs)   instituted   a   class   suit   by   filing   an   "Amended   Complaint"   with   arising  out  of  a  contract  of  employment  shall  not  be  actionable  after  the  lapse  
the   Philippine   Overseas   Employment   Administration   (POEA)   for   money   claims   of  one  year  from  the  date  of  the  expiry  of  the  contract.  
arising  from  their  recruitment  by  AIBC  and  employment  by  BRII.    
  As  a  general  rule,  a  foreign  procedural  law  will  not  be  applied  in  the  forum.  
BRII   is   a   foreign   corporation   with   headquarters   in   Houston,   Texas,   and   is   Procedural  matters,  such  as  service  of  process,  joinder  of  actions,  period  and  
engaged   in   construction;   while   AIBC   is   a   domestic   corporation   licensed   as   a   requisites   for   appeal,   and   so   forth,   are   governed   by   the   laws   of   the   forum.  
service  contractor  to  recruit,  mobilize  and  deploy  Filipino  workers  for  overseas   This  is  true  even  if  the  action  is  based  upon  a  foreign  substantive  law.  
employment  on  behalf  of  its  foreign  principals.    
  However,  a  law  on  prescription  of  actions  is  sui  generis  in  Conflict  of  Laws  in  
The   amended   complaint   principally   sought   the   payment   of   the   unexpired   the   sense   that   it   may   be   viewed   either   as   procedural   or   substantive,  
portion  of  the  employment  contracts,  which  was  terminated  prematurely,  and   depending  on  the  characterization  given  such  a  law.  
secondarily,   the   payment   of   the   interest   of   the   earnings   of   the   Travel   and    
Reserved   Fund,   interest   on   all   the   unpaid   benefits;   area   wage   and   salary   Thus  in  Bournias  v.  Atlantic  Maritime  Company,  the  American  court  applied  the  
differential  pay;  fringe  benefits;  refund  of  SSS  and  premium  not  remitted  to  the   statute  of  limitations  of  New  York,  instead  of  the  Panamanian  law,  after  finding  
SSS;  refund  of  withholding  tax  not  remitted  to  the  BIR;  penalties  for  committing   that   there   was   no   showing   that   the   Panamanian   law   on   prescription   was  
prohibited   practices;   as   well   as   the   suspension   of   the   license   of   AIBC   and   the   intended  to  be  substantive.  Being  considered  merely  a  procedural  law  even  in  
accreditation  of  BRII.   Panama,  it  has  to  give  way  to  the  law  of  the  forum  on  prescription  of  actions.  
   
EASIER   FACTS:   Cadalin   et   al.   are   overseas   contract   workers   recruited   by   However,   the   characterization   of   a   statute   into   a   procedural   or   substantive  
respondent-­‐appellant   AIBC   for   its   accredited   foreign   principal,   Brown   &   Root,   law   becomes   irrelevant   when   the   country   of   the   forum   has   a   "borrowing  
on   various   dates   from   1975   to   1983.   As   such,   they   were   all   deployed   at   various   statute."  Said  statute  has  the  practical  effect  of  treating  the  foreign  statute  of  
limitation  as  one  of  substance.  A  "borrowing  statute"  directs  the  state  of  the  
 
CONFLICT  OF  LAWS                                                                                    AV  DE  TORRES   57  
ATTY.  ARIS  L.  GULAPA                                            AY  2015-­‐2016  
forum  to  apply  the  foreign  statute  of  limitations  to  the  pending  claims  based   The   claims   in   the   cases   at   bench   all   arose   from   the   employer-­‐employee  
on   a   foreign   law.   While   there   are   several   kinds   of   "borrowing   statutes,"   one   relations,  which  is  broader  in  scope  than  claims  arising  from  a  specific  law  or  
form  provides  that  an  action  barred  by  the  laws  of  the  place  where  it  accrued,   from   the   collective   bargaining   agreement.   The   contention   of   the   POEA  
will  not  be  enforced  in  the  forum  even  though  the  local  statute  has  not  run   Admin,   that   the   3-­‐year   prescriptive   period   under   Art.   291   of   the   LC   applies  
against  it.  Sec.  48  of  our  Code  of  Civil  Procedure  is  of  this  kind,  which  provides:   only  to  money  claims  specifically  recoverable  under  said  Code,  does  not  find  
If  by  the  laws  of  the  state  or  country  where  the  cause  of  action  arose,  the  action   support  in  the  plain  language  of  the  provision.    
is  barred,  it  is  also  barred  in  the  Philippines  Islands.    
  3.   The   overseas-­‐employment   contracts   should   be   read   as   adopting   the  
Sec.  48  has  not  been  repealed  or  amended  by  the  NCC.  Art.  2270  of  said  Code   provisions   of   the   Amiri   Decree   No.   23   of   1976   as   part   and   parcel   thereof.   NLRC  
repealed  only  those  provisions  of  the  Code  of  Civil  Procedures  as  to  which  were   applied   the   Amiri   Decree   No.   23   of   1976,   which   provides   for   greater   benefits  
inconsistent  with  it.  There  is  no  provision  in  the  NCC,  which  is  inconsistent  with   than  those  stipulated  in  the  overseas-­‐employment  contracts  of  the  claimants.  It  
or  contradictory  to  Section  48  of  the  Code  of  Civil  Procedure  (Paras,  Philippine   was  of  the  belief  that  "where  the  laws  of  the  host  country  are  more  favorable  
Conflict  of  Laws  104  [7th  ed.]).   and  beneficial  to  the  workers,  then  the  laws  of  the  host  country  shall  form  part  
  of  the  overseas  employment  contract."  It  quoted  with  approval  the  observation  
However,   in   the   light   of   the   1987   Constitution,   Section   48   cannot   be   enforced   of   the   POEA   Administrator   that   "...in   labor   proceedings,   all   doubts   in   the  
ex  proprio  vigore  (by  its  own  force)  insofar  as  it  ordains  the  application  in  this   implementation   of   the   provisions   of   the   Labor   Code   and   its   implementing  
jurisdiction  of  Sec.  156  of  the  Amiri  Decree  No.  23  of  1976.  The  courts  of  the   regulations  shall  be  resolved  in  favor  of  labor."  
forum   will   not   enforce   any   foreign   claim   obnoxious   to   the   forum's   public    
policy.  To  enforce  the  one-­‐year  prescriptive  period  of  the  Amiri  Decree  No.  23   AIBC   and   BRII   claimed   that   NLRC   acted   capriciously   and   whimsically   when   it  
of  1976  as  regards  the  claims  in  question  would  contravene  the  public  policy  on   refused  to  enforce  the  overseas-­‐employment  contracts,  which  became  the  law  
the   protection   to   labor.   The   1987   Constitution   emphasized   that:   The   state   shall   of  the  parties.  They  contend  that  the  principle  that  a  law  is  deemed  to  be  a  part  
promote  social  justice  in  all  phases  of  national  development  (Sec.  10).  The  state   of  a  contract  applies  only  to  provisions  of  Philippine  law  in  relation  to  contracts  
affirms   labor   as   a   primary   social   economic   force.   It   shall   protect   the   rights   of   executed  in  the  Philippines.  
workers   and   promote   their   welfare   (Sec.   18).   The   State   shall   afford   full    
protection   to   labor,   local   and   overseas,   organized   and   unorganized,   and   The  overseas-­‐employment  contracts,  which  were  prepared  by  AIBC  and  BRII  
promote  full  employment  and  equality  of  employment  opportunities  for  all  (Art.   themselves,  provided  that  the  laws  of  the  host  country  became  applicable  to  
XIII,  Sec.  3).   said   contracts   if   they   offer   terms   and   conditions   more   favorable   that   those  
  stipulated  therein.  It  was  stipulated  in  said  contracts  that:  
2.   3   years.   The   claimants   are   of   the   view   that   the   applicable   provision   is   Art.    
1144,   NCC,   which   provides:   The   following   actions   must   be   brought   within   ten   “...the   benefits   provided   to   Employee   hereunder   are   substituted   for  
years   from   the   time   the   right   of   action   accrues:   (1)   Upon   a   written   contract;   (2)   and   in   lieu   of   all   other   benefits   provided   by   any   applicable  
Upon  an  obligation  created  by  law;  (3)  Upon  a  judgment.   law,  provided   of   course,   that   total   remuneration   and   benefits   do   not  
  fall   below   that   of   the   host   country   regulation   or   custom,   it   being  
NLRC,   on   the   other   hand,   believes   that   the   applicable   provision   is   Art.   291   of   understood  that  should  applicable  laws  establish  that  fringe  benefits,  or  
the  LC,  which  in  pertinent  part  provides:   Money  claims-­‐all  money  claims  arising   other   such   benefits   additional   to   the   compensation   herein   agreed  
from   employer-­‐employee   relations   accruing   during   the   effectivity   of   this   Code   cannot   be   waived,   Employee   agrees   that   such   compensation   will   be  
shall   be   filed   within   three   years   from   the   time   the   cause   of   action   accrued,   adjusted   downward   so   that   the   total   compensation   hereunder,   plus  
otherwise  they  shall  be  forever  barred.   the   non-­‐waivable   benefits   shall   be   equivalent   to   the   compensation  
  herein  agreed.”  
 
 
CONFLICT  OF  LAWS                                                                                    AV  DE  TORRES   58  
ATTY.  ARIS  L.  GULAPA                                            AY  2015-­‐2016  
Any   ambiguity   in   the   overseas-­‐employment   contracts   should   be   interpreted    
against  AIBC  and  BRII,  the  parties  that  drafted  it.  The  interpretation  of  obscure   FACTS:  
words   or   stipulations   in   a   contract   shall   not   favor   the   party   who   caused   the   Petitioner   Esther   Kiobel,   representing   a   group   of   individuals   from   the   Ogoni  
obscurity  (Art.  1377,  NCC).   region   in   Nigeria,   filed   a   class   action   lawsuit   under   the   Alien   Tort   Statute  
  (“ATS”)   against   Respondents,   the   Royal   Dutch   Petroleum   Co.,   Shell   Transport  
The  parties  to  a  contract  may  select  the  law  by  which  it  is  to  be  governed.   In   and   Trading   Company   PLC,   and   Shell   Petroleum   Development   Company   of  
such  a  case,  the  foreign  law  is  adopted  as  a  "system"  to  regulate  the  relations   Nigeria,   LTD   (“Royal   Dutch”),   which   allegedly   aided   and   abetted   the   Nigerian  
of  the  parties,  including  questions  of  their  capacity  to  enter  into  the  contract,   Government   in   committing   violations   of   the   law   of   nations   in   Nigeria.   Royal  
the  formalities  to  be  observed  by  them,  matters  of  performance,  and  so  forth.   Dutch   allegedly   aided   the   Nigerian   government   in   committing   various   acts   of  
  violence  against  protestors  of  the  oil  exploration  projects  in  the  Ogoni  region.      
Instead   of   adopting   the   entire   mass   of   the   foreign   law,   the   parties   may   just    
agree   that   specific   provisions   of   a   foreign   statute   shall   be   deemed   Petitioners   claim   that   they   have   standing   to   sue   under   the   ATS.   The   ATS  
incorporated  into  their  contract  "as  a  set  of  terms."  By  such  reference  to  the   provides   that   "[t]he   district   courts   shall   have   original   jurisdiction   of   any   civil  
provisions  of  the  foreign  law,  the  contract  does  not  become  a  foreign  contract   action  by  an  alien  for  a  tort  only,  committed  in  violation  of  the  law  of  nations  or  
to  be  governed  by  the  foreign  law.  The  said  law  does  not  operate  as  a  statute   a   treaty   of   the   United   States."   In   other   words,   it   grants   jurisdiction   to   some  
but  as  a  set  of  contractual  terms  deemed  written  in  the  contract     federal   courts   for   certain   violations   of   international   law.   Further,   because   the  
  history,  text,  and  purpose  of  the  statute  support  the  application  of  the  ATS  to  
A  basic  policy  of  contract  is  to  protect  the  expectation  of  the  parties.  Such  party   actions   in   foreign   countries.   Petitioner   also   contends   that   previous   court  
expectation   is   protected   by   giving   effect   to   the   parties'   own   choice   of   the   decisions   interpreted   the   ATS   to   extend   beyond   U.S.   territory.   In   response,  
applicable  law.  The  choice  of  law  must,  however,  bear  some  relationship  to  the   Royal   Dutch   argues   that   the   ATS   is   not   an   exception   to   the   presumption   that  
parties   or   their   transaction.   There   is   no   question   that   the  contracts   sought   to   U.S.   law   does   not   apply   extraterritorially,   and   should   not   be   applicable   to  
be   enforced   by   claimants   have   a   direct   connection   with   the   Bahrain   law   actions   outside   of   the   U.S.   The   District   Court   dismissed   several   of   petitioners'  
because  the  services  were  rendered  in  that  country.   claims,   but   on   interlocutory   appeal,   the   Second   Circuit   dismissed   the   entire  
  complaint,   reasoning   that   the   law   of   nations   does   not   recognize   corporate  
38.  KIOBEL  V.  ROYAL  DUTCH   liability.   This   Court   granted   certiorari,   and   ordered   supplemental   briefing   on  
No.  10-­‐1491  |  April  17,  2013   whether   and   under   what   circumstances   courts   may   recognize   a   cause   of   action  
  under  the  ATS,  for  violations  of  the  law  of  nations  occurring  within  the  territory  
DOCTRINE:   GR:   When   a   statute   gives   no   clear   indication   of   an   extraterritorial   of  a  sovereign  other  than  the  United  States.  
application,  it  has  none.  There  is  a  presumption  against  extraterritoriality,  that    
is   a   local   law   does   not   apply   extraterritorially.   The   presumption   "serves   to   ISSUE:  
protect   against   unintended   clashes   between   our   laws   and   those   of   other   Whether   an   American   federal   court   can   hear   a   claim   under   the   Alien   Tort  
nations  which  could  result  in  international  discord."   statute,  when  that  claim  arose  out  of  conduct  in  a  foreign  country  
   
EXC:  When  a  statute  gives  a  clear  indication  of  an  extraterritorial  application.     HELD:  
  No.  The  presumption  against  extraterritoriality  applies  to  claims  under  the  ATS,  
Nothing   in   the   ATS's   text   evinces   a   clear   indication   of   extraterritorial   reach.   and  nothing  in  the  statute  rebuts  that  presumption.  
Violations   of   the   law   of   nations   affecting   aliens   can   occur   either   within   or    
outside   the   United   States.   And   generic   terms,   like   "any"   in   the   phrase   "any   civil   Passed   as   part   of   the   Judiciary   Act   of   1789,   the   ATS   is   a   jurisdictional   statute  
action,"  do  not  rebut  the  presumption  against  extraterritoriality.   that  creates  no  causes  of  action.  It  permits  federal  courts  to  "recognize  private  
claims   [for   a   modest   number   of   international   law   violations]   under   federal  
 
CONFLICT  OF  LAWS                                                                                    AV  DE  TORRES   59  
ATTY.  ARIS  L.  GULAPA                                            AY  2015-­‐2016  
common   law.”   In   contending   that   a   claim   under   the   ATS   does   not   reach   jurisdiction   of   the   United   States   or   any   other   country,   applying   U.   S.   law   to  
conduct   occurring   in   a   foreign   sovereign's   territory,   respondents   rely   on   the   pirates   does   not   typically   impose   the   sovereign   will   of   the   US   onto   conduct  
presumption  against  extraterritorial  application,  which  provides  that  “[w]hen  a   occurring  within  the  territorial  jurisdiction  of  another  sovereign,  and  therefore  
statute  gives  no  clear  indication  of  an  extraterritorial  application,  it  has  none,"   carries   less   direct   foreign   policy   consequences.   A   1795   opinion   of   Attorney  
The   presumption   "serves   to   protect   against   unintended   clashes   between   our   General   William   Bradford   regarding   the   conduct   of   U.   S.   citizens   on   both   the  
laws  and  those  of  other  nations  which  could  result  in  international  discord."  It   high   seas   and   a   foreign   shore   is   at   best   ambiguous   about   the   ATS's  
is   typically   applied   to   discern   whether   an   Act   of   Congress   regulating   conduct   extraterritorial  application;  it  does  not  suffice  to  counter  the  weighty  concerns  
applies  abroad,  see,  e.g.,  but  its  underlying  principles  similarly  constrain  courts   underlying   the   presumption   against   extraterritoriality.   Finally,   there   is   no  
when   considering   causes   of   action   that   may   be   brought   under   the   ATS.   Indeed,   indication   that   the   ATS   was   passed   to   make   the   United   States   a   uniquely  
the  danger  of  unwarranted  judicial  interference  in  the  conduct  of  foreign  policy   hospitable  forum  for  the  enforcement  of  international  norms.  
is  magnified  in  this  context,  where  the  question  is  not  what  Congress  has  done    
but   what   courts   may   do.   These   foreign   policy   concerns   are   not   diminished   by    
the  fact  that  Sosa  limited  federal  courts  to  recognizing  causes  of  action  only  for  
V.  FOREIGN  LAW  
alleged  violations  of  international  law  norms  that  are  "`specific,  universal,  and    
obligatory."  
ASCERTAINMENT  AND  PROOF  OF  FOREIGN  LAW;  
 
The   presumption   is   not   rebutted   by   the   text,   history,   or   purposes  of   the   ATS.   DOCTRINE  OF  PROCESSUAL  PRESUMPTION  
 
Nothing   in   the   ATS's   text   evinces   a   clear   indication   of   extraterritorial   reach.    
Violations   of   the   law   of   nations   affecting   aliens   can   occur   either   within   or   39.  SY  JOC  LIENG  V.  PETRONILA  ENCARNACION,  GREGORIO  SY  QUIA  
outside   the   United   States.   And   generic   terms,   like   "any"   in   the   phrase   "any   G.R.  No.  4718  |  March  19,  1910  
civil   action,"   do   not   rebut   the   presumption   against   extraterritoriality.    
Petitioners   also   rely   on   the   common-­‐law   "transitory   torts"   doctrine,   but   that   N.B.:  The  Chinese  Law  was  not  1.  alleged  and  2.  proven.  
doctrine   is   inapposite   here;   as   the   Court   has   explained,   "the   only   justification    
for   allowing   a   party   to   recover   when   the   cause   of   action   arose   in   another   The  evidence  were  not  sufficient  because:  
civilized  jurisdiction  is  a  well-­‐founded  belief  that  it  was  a  cause  of  action  in  that   (1)  The  evidence  of  the  Chinese  law  were  books  or  pamphlets  written  in  Chinese  
place."  The  question  under  Sosa  is  not  whether  a  federal  court  has  jurisdiction   characters,  which  they  claim  contain  a  compilation  of  the  laws  of  China,  being  
to   entertain   a   cause   of   action   provided   by   foreign   or   even   international   law.   useless  and  of  no  value.  
The  question  is  instead  whether  the  court  has  authority  to  recognize  a  cause  of   (2)  It  may  be  that  contain,  as  plaintiff  claim,  the  laws  of  China,  but  we  have  no  
action  under  U.  S.  law  to  enforce  a  norm  of  international  law.  That  question  is   Spanish   translation   of   them,   they   being   in   the   Chinese   language,   and   written  
not  answered  by  the  mere  fact  that  the  ATS  mentions  torts.   with   characters   which   are   absolutely   unknown   to   this   court   and   to   the  
  defendants.  
The   historical   background   against   which   the   ATS   was   enacted   also   does   not   (3)   There   is   no   evidence   that   these   four   books   or   pamphlets   were   printed   by  
overcome   the   presumption.   When   the   ATS   was   passed,   "three   principal   authority  of  the  Chinese  Government  or  
offenses   against   the   law   of   nations"   had   been   identified   by   Blackstone:   (4)   that   they   have   been   duly   authenticated   by   the   certificate   of   competent  
violation   of   safe   conducts,   infringement   of   the   rights   of   ambassadors,   and   authorities  or  
piracy.     Prominent   contemporary   examples   of  the   first   two   offenses   —   (5)   that   they   are   properly   sealed   with   the   seal   of   the   nation   to   which   they  
immediately   before   and   after   passage   of   the   ATS   —   provide   no   support   for   the   belong.   For   this   reason   the   said   books   or   pamphlets   can   not,   under   any  
proposition   that   Congress   expected   causes   of   action   to   be   brought   under   the   circumstances,  be  considered  as  documentary  proof  of  the  laws  of  China.  
statute  for  violations  of  the  law  of  nations  occurring  abroad.  And  although  the    
offense   of   piracy   normally   occurs   on   the   high   seas,   beyond   the   territorial  
 
CONFLICT  OF  LAWS                                                                                    AV  DE  TORRES   60  
ATTY.  ARIS  L.  GULAPA                                            AY  2015-­‐2016  
The   oral   testimonies   were   also   not   sufficient   to   establish   the   foreign   law   ISSUES:  
because:   Should   the   estate   of   Vicente   Romero   Sy   Quia,   deceased,   be   distributed   in  
(1)  they  were  unable  to  say  positively  at  least  that  the  book  contains  an  exact   accordance  with  Chinese  laws?  
copy  of  the  original.   N.B.:  Summary  of  rulings  are  provided  at  the  end  in  case  Gulapa  asks  for  them  
st
(2)  The  1  witness  he  had  never  made  a  regular  study  of  the  laws  of  his  country,   during  recitation  
simply  consulting  the  same  in  connection  with  his  official  reports,  admitted  that    
he   had   never   read   or   seen   the   original   copy   of   this   alleged   compilation,   the   HELD:  
books  not  being  duly  certified,  adding  that  he  could  not  say  whether  the  book   No.   The   plaintiffs   in   this   case   have   invoked   certain   provisions   of   the   Chinese  
was  an  exact  copy  of  the  original.   laws  as  one  of  the  grounds  of  the  action  by  them  instituted  and  now  contend  
nd
(3)   The   2   witness,   the   interpreter,   as   to   the   written   and   unwritten   laws   of   that  the  estate  of  Vicente  Romero  Sy  Quia,  deceased,  should  be  distributed  in  
China,  does  not  show,  as  required  by  the  Code  of  Civil  Procedure,  that  he  knew   accordance   with   the   laws   of   that   country.   Even   disregarding   the   fact   that   the  
such  laws  or  that  he  was  acquainted  with  the  nature  of  the  laws  alleged  to  be   plaintiffs   should   have,   but   have   not,   alleged   in   their   complaint,   as   one   of   the  
contained  in  the  said  books.  He  merely  confined  himself  to  expressing  his  own   facts   constituting   their   cause   of   action,   the   existence   of   a   law   passed   and  
opinion  with  reference  to  the  two  classes  of  laws.   promulgated   in   China,   the   existence   of   which   law,   being   foreign,   should   have  
  been   alleged   in   the   complaint,   the   fact   remains   that   there   is   absolutely   no  
SUMMARY  OF  FACTS:   evidence   in   the   record   as   to   the   existence   of   the   Chinese   laws   referred   to   by  
1847  –  married  in  china   plaintiffs   in   their   subsequent   pleadings,   the   evidence   of   this   character  
1853  –  married  in  the  Philippines  to  a  Filipina   introduced   by   them   consisting   of   books   or   pamphlets   written   in   Chinese  
1894  –  died   characters  and  marked  "Exhibits  AH,  AI,  AJ,  and  AK,’  which  they  claim  contain  a  
  compilation  of  the  laws  of  China,  being  useless  and  of  no  value.    
Plaintiffs  (all  chinese  citizens)  claim  that  they  are  the  heirs  of  Vicente  Romero    
Sy   Quia,   being   married   to   Yap   Puan   Niu   in   or   about   the   year   1847   in   Amoy,    
China;   thereby,   they   are   entitled   to   the   inheritance   left   by   Vicente.   On   the   It   may   be   that   contain,   as   plaintiff   claim,   the   laws   of   China,   but   we   have   no  
other   hand,   defendants   (Filipinos)   are   as   found   by   the   lower   courts,   the   Spanish   translation   of   them,   they   being   in   the   Chinese   language,   and   written  
legitimate  heirs  of  Vicente  as  he  was  married  to  Petronila  Encarnacion  (a  native   with   characters   which   are   absolutely   unknown   to   this   court   and   to   the  
of  Vigan,  Ilocos  Sur)  on  June  9,  1853  as  shown  in  their  certificate  of  marriage.   defendants.   Further,   the   plaintiffs   have   not   prescribed   by   section   292   of   the  
Plaintiffs   allege   that   they   are   the   rightful   heirs   and   that   the   subsequent   Code   of   Civil   Procedure,   and,   finally,   there   is   no   evidence   that   these   four   books  
marriage   of   Vicente   was   void.   Defendants   maintained   that   the   claim   of   the   or   pamphlets   were   printed   by   authority   of   the   Chinese   Government   or   that  
plaintiffs   regarding   the   alleged   first   marriage   of   Vicente   were   fraudulent.   The   they  have  been  duly  authenticated  by  the  certificate  of  competent  authorities  
plaintiffs  presented  witnesses  and  got  their  respective  depositions  in  China  to   or   that   they   are   properly   sealed   with   the   seal   of   the   nation   to   which   they  
prove  that  the  first  marriage  took  place.  The  lower  courts  found  that  there  is  no   belong.   For   this   reason   the   said   books   or   pamphlets   can   not,   under   any  
marriage   to   Yap   contrary   to   the   allegations   of   the   plaintiffs.   The   decision   is   circumstances,  be  considered  as  documentary  proof  of  the  laws  of  China.  (The  
based   on   the   improbability   of   the   allegations   of   the   plaintiffs   that   Vicente   lived   Court  cited  sections  3004,  3015  and  3026  of  the  Code  of  Civil  Procedure)    
in   China   for   around   four   years   after   such   marriage.   There   being   no   valid    
marriage  certificate  presented  and  the  testimonies  of  the  witnesses  presented   The  jurisprudence  of  American  and  Spanish  tribunals  is  uniform  on  this  subject:  
contradict  each  other.  In  the  plaintiffs'  subsequent  pleadings,  they  prayed  that   "Whenever   a   foreign   law   is   invoked   in   our   tribunals,   its   existence   must   be  
the   decision   be   reversed   and   that   the   properties   of   Vicente   be   distributed   to   satisfactory  established  as  any  other  fact.”    
them  according  to  the  laws  of  China.    
  If   the   pamphlets   or   books,   written   in   Chinese   characters,   do   not   satisfactory  
establish   the   existence   of   certain   Chinese   laws   invoked   by   the   plaintiffs,   not  
 
CONFLICT  OF  LAWS                                                                                    AV  DE  TORRES   61  
ATTY.  ARIS  L.  GULAPA                                            AY  2015-­‐2016  
only   because   such   pamphlets   or   books   lack   the   aforesaid   formalities   and   Article  816.  The  will  of  an  alien  who  is  abroad  produces  effect  in  the  Philippines  
requisites,  but  further  because  there  is  no  evidence  as  to  the  nature  of  the  laws   if   made   with   the   formalities   prescribed   by   the   law   of   the   place   in   which   he  
contained  in  those  books  or  pamphlets  and  the  subjects  with  which  they  deal;   resides,   or   according   to   the   formalities   observed   in   his   country,   or   in   conformity  
on  the  other  hand,  the  two  witnesses  whose  testimony  was  introduced  for  the   with  those  which  this  Code  prescribes.  
purpose   of   establishing   the   authenticity   of   the   laws   which,   according   to   the    
plaintiffs,  are  contained  in  the  said  books,  were  unable  to  say  positively  at  least   Summary  of  Facts:  
that  the  book  marked  Exhibit  AH  contains  an  exact  copy  of  the  original.  And  the   Emil   Johnson   died   in   the   City   of   Manila;   however   he   was   a   naturalized  
Chinese  consul  of  this  city,  Sy  Int  Chu,  after  stating  that  he  had  never  made  a   American  citizen.  He  left  a  will  through  which  he  disposed  an  estate  valued  at  
regular   study   of   the   laws   of   his   country,   simply   consulting   the   same   in   231,800php.   However,   the   will   was   signed   by   two   witnesses   instead   of   the  
connection   with   his   official   reports,   admitted   that   he   had   never   read   or   seen   three  required  by  Sec.  618  of  the  Code  of  Civil  Procedure.  However,  a  petition  
the  original  copy  of  this  alleged  compilation,  the  books  not  being  duly  certified,   was  made  stating  that  the  will  was  made  in  conformity  with  US  Law,  thus  valid  
adding   that   he   could   not   say   whether   the   book   marked.   "Exhibit   AH"   was   an   in  the  Philippines  as  provided  for  in  Section  636  in  the  Code  of  Civil  Procedure.  
exact  copy  of  the  original.     The   will   was   later   probated   and   declared   legal,   however   the   testator's  
  daughter,  Ebba  Ingeborg  entered  an  appearance,  claiming  that  as  a  legitimate  
The  testimony  of  the  witness  Ly  Ung  Bing,  the  interpreter,  as  to  the  written  and   child   she   cannot   be   deprived   of   the   legitime   which   she   is   entitled   to   as  
unwritten   laws   of   China,   does   not   show,   as   required   by   the   Code   of   Civil   provided  by  Philippine  law.  She  moved  to  annul  the  decree  of  probate  and  put  
Procedure,  that  he  knew  such  laws  or  that  he  was  acquainted  with  the  nature   the  estate  into  intestate  administration  in  order  for  her  to  claim  the  estate  as  
of   the   laws   alleged   to   be   contained   in   the   said   books.   He   merely   confined   the  sole  legitimate  heir  of  her  father.  
himself  to  expressing  his  own  opinion  with  reference  to  the  two  classes  of  laws.    
He,   not   being   an   expert   on   the   subject   throughly   conversant  with  the  laws  of   FACTS:  
China,   his   testimony,   considering   the   manner   in   which   laws   of   China,   his   Emil   H.   Johnson,   a   native   of   Sweden   and   a   naturalized   citizen   of   the   United  
testimony,   considering   the   manner   in   which   he   testified,   can   not   even   be   States,   died   in   the   city   of   Manila,   leaving   a   will   by   which   he   disposed   of   an  
accepted   as   a   partial   evidence   that   the   said   four   books   really   contain   the   estate,   the   value   of   which   was   P231,800.   This   document   is   an   holographic  
written  and  unwritten  laws  of  China.     instrument,   being   written   in   the   testator's   own   handwriting,   and   is   signed   by  
  himself  and  two  witnesses  only,  instead  of  three  witnesses  required  by  section  
40.   IN   THE   MATTER   OF   THE   ESTATE   OF   EMIL   H.   JOHNSON.   EBBA   INGEBORG   618   of   the   Code   of   Civil   Procedure.   This   will,   therefore,   was   not   executed   in  
JOHNSON   conformity  with  the  provisions  of  law  generally  applicable  to  wills  executed  by  
G.R.  No.  L-­‐12767  |  November  16,  1918   inhabitants   of   these   Islands,   and   hence   could   not   have   been   proved   under  
  section  618.  
ALG:   At   the   time   of   the   decision   of   this   case,   Phil.   courts   can   take   judicial   notice    
of   US   Federal   Laws   because   it   was   then   a   colony.   Now,   the   Philippine   courts   However,  a  petition  was  presented  in  the  Court  of  First  Instance  of  the  city  of  
cannot  take  judicial  notice  of  any  US  Law  except  under  certain  exceptions.   Manila   for   the   probate   of   this   will,   on   the   ground   that   Johnson   was   at   the   time  
  of  his  death  a  citizen  of  the  State  of  Illinois,  United  States  of  America;  that  the  
N.B.:  The  following  articles  govern  foreign  wills  now:   will   was   duly   executed   in   accordance   with   the   laws   of   that   State;   and   hence  
  could   properly   be   probated   here   pursuant   to   section   636   of   the   Code   of   Civil  
Article   17.   The   forms   and   solemnities   of   contracts,   wills,   and   other   public   Procedure.  This  section  reads  as  follows:  
instruments   shall   be   governed   by   the   laws   of   the   country   in   which   they   are    
executed.   Will  made  here  by  alien.  -­‐  A  will  made  within  the  Philippine  Islands  by  
  a   citizen   or   subject   of   another   state   or   country,   which   is   executed   in  

 
CONFLICT  OF  LAWS                                                                                    AV  DE  TORRES   62  
ATTY.  ARIS  L.  GULAPA                                            AY  2015-­‐2016  
accordance  with  the  law  of  the  state  or  country  of  which  he  is  a  citizen   About  three  months  after   the   will   had   been   probated,   the   attorneys   for   Ebba  
or  subject,  and  which  might  be  proved  and  allowed  by  the  law  of  his   Ingeborg   Johnson   entered   an   appearance   in   her   behalf   and   noted   an   exception  
own   state   or   country,   may   be   proved,   allowed,   and   recorded   in   the   to   the   other   admitting   the   will   to   probate.   The   purpose   of   the   proceeding   on  
Philippine   Islands,   and   shall   have   the   same   effect   as   if   executed   behalf   of   the   petitioner   is   to   annul   the   decree   of   probate   and   put   the   estate  
according  to  the  laws  of  these  Islands.   into  intestate  administration,  thus  preparing  the  way  for  the  establishment  of  
  the  claim  of  the  petitioner  as  the  sole  legitimate  heir  of  her  father.  
The  hearing  on  said  application  was  set  and  three  weeks  publication  of  notice    
was   ordered   in   the   "Manila   Daily   Bulletin."   Witnesses   were   examined   relative   ISSUES:  
to  the  execution  of  the  will.  Thereafter  the  document  was  declared  to  be  legal   Whether  the  will  was  executed  in  conformity  with  the  statutes  of  the  State  of  
and   was   admitted   to   probate.   Victor   Johnson   was   appointed   sole   Illinois    
administrator.    
  HELD:  
By  virtue  of  the  will,  the  testator  bequeath  his  property  to  his  brother  Victor,  to   No?   It   does   not   affirmatively   appear   from   the   transaction   of   the   testimony  
his   father   and   mother   in   Sweden,   to   his   daughter   Ebba   Ingeborg,   to   his   wife,   adduced  in  the  trial  court  that  any  witness  was  examined  with  reference  to  the  
Alejandra  Ibañez,  if  she  remains  single  and  to  Simeona  Ibañez,  spinster,  if  she   law  of  Illinois  on  the  subject  of  the  execution  of  will.  The  trial  judge  no  doubt  
remains   single.   The   rest   of   the   property   is   left   to   the   testator's   five   children   -­‐   was  satisfied  that  the  will  was  properly  executed  by  examining  section  1874  of  
Mercedes,  Encarnacion,  Victor,  Eleonor  and  Alberto.     the   Revised   Statutes   of   Illinois   and   he   may   have   assumed   that   he   could   take  
  judicial   notice   of   the   laws   of   Illinois   under   section   275   of   the   Code   of   Civil  
The  biographical  facts  relative  to  the  deceased  necessary  to  an  understanding   Procedure.   If   so,   he   was   in   our   opinion   mistaken.   that   section   authorizes   the  
of  the  case  are  these:  Emil  H.  Johnson  was  born  in  Sweden,  May  25,  1877,  from   courts   here   to   take   judicial   notice,   among   other   things,   of   the   acts   of   the  
which  country  he  emigrated  to  the  United  States  and  lived  in  Chicago,  Illinois,   legislative  department  of  the  United  States.  These  words  clearly  have  reference  
from   1893   to   1898.   On   May   9,   1898,   at   Chicago,   he   was   married   to   Rosalie   to   Acts   of   the   Congress   of   the   United   States;   and   we   would   hesitate   to   hold  
Ackeson,  and  immediately  thereafter  embarked  for  the  Philippine  Islands  as  a   that  our  courts  can,  under  this  provision,  take  judicial  notice  of  the  multifarious  
soldier   in   the   Army   of   the   United   States.   As   a   result   of   relations   between   laws   of   the   various   American   States.   Nor   do   we   think   that   any   such   authority  
Johnson  and  Rosalie  Ackeson  a  daughter,  named  Ebba  Ingeborg,  was  born.   can   be   derived   from   the   broader   language,   used   in   the   same   action,   where   it   is  
  said   that   our   courts   may   take   judicial   notice   of   matters   of   public   knowledge  
After   Johnson   was   discharged   as   a   soldier   from   the   service   of   the   United   States   "similar"  to  those  therein  enumerated.  The  proper  rule  we  think  is  to  require  
he  continued  to  live  in  the  Philippine  Islands,  and  on  November  20,  1902,  the   proof   of   the   statutes   of   the   States   of   the   American   Union   whenever   their  
wife,  Rosalie  Johnson,  was  granted  a  decree  of  divorce  from  him  in  the  Circuit   provisions   are   determinative   of   the   issues   in   any   action   litigated   in   the  
Court   of   Cook   County,   Illinois,   on   the   ground   of   desertion.   A   little   later   Johnson   Philippine  courts.  
appeared   in   the   United   States   on   a   visit   and   on   January   10,   1903,   procured   a    
certificate   of   naturalization   at   Chicago.   After   a   short   trip   to   Sweden,   the   Nevertheless,   even   supposing   that   the   trial   court   may   have   erred   in   taking  
deceased  returned  to  Manila,  where  he  prospered  in  business  and  continued  to   judicial  notice  of  the  law  of  Illinois  on  the  point  in  question,  such  error  is  not  
live  until  his  death.   now  available  to  the  petitioner,  first,  because  the  petition  does  not  state  any  
  fact   from   which   it   would   appear   that   the   law   of   Illinois   is   different   from   what  
In   this   city   he   appears   to   have   entered   into   marital   relations   with   Alejandra   the   court   found,   and,   secondly,   because   the   assignment   of   error   and  
Ibañez,  by  whom  he  had  three  children:  Mercedes,  Encarnacion  and  Victor.  The   argument   for   the   appellant   in   this   court   raises   no   question   based   on   such  
other  two  children  (Eleonor  and  Alberto)  mentioned  in  the  will  were  borne  to   supposed  error.  
the  deceased  by  Simeona  Ibañez.    
 
 
CONFLICT  OF  LAWS                                                                                    AV  DE  TORRES   63  
ATTY.  ARIS  L.  GULAPA                                            AY  2015-­‐2016  
The   petitioner,   it   is   true,   states   in   general   terms   that   the   will   in   question   is   41.   IN   THE   MATTER   ESTATE   OF   EDWARD   RANDOLPH   HIX,   DECEASED.    
invalid   and   inadequate   to   pass   real   and   personal   property   in   the   State   of   A.W.  FLUEMER  V.  ANNIE  COUSHING  HIX  
Illinois,   but   this   is   merely   a   conclusion   of   law.   The   affidavits   by   which   the   G.R.  No.  L-­‐3263  |  March  17,  1930  
petition  is  accompanied  contain  no  reference  to  the  subject,  and  we  are  cited    
to  no  authority  in  the  appellant's  brief  which  might  tent  to  raise  a  doubt  as  to   DOCTRINES:   The   submission   of   the   aforementioned   copy   of   the   law   was   far  
the   correctness   of   the   conclusion   of   the   trial   court.   It   is   very   clear,   therefore,   from  a  compliance  with  the  law.  The  laws  of  a  foreign  jurisdiction  do  not  prove  
that  this  point  cannot  be  urged  as  of  serious  moment.   themselves   in   our   courts.   The   courts   of   the   Philippine   Islands   are   not   authorized  
  to  take  judicial  notice  of  the  laws  of  the  various  States  of  the  American  Union.  
But  it  is  insisted  in  the  brief  for  the  appellant  that  the  will  in  question  was  not   Such  laws  must  be  proved  as  facts.  
properly  admissible  to  probate  because  it  contains  provisions  which  cannot  be    
given   effect   consistently   with   the   laws   of   the   Philippine   Islands;   and   it   is   There   was   no   showing   that   the   book   from   which   an   extract   was   taken   was  
suggested  that  as  the  petitioner  is  a  legitimate  heir  of  the  testator  she  cannot   printed   or   published   under   the   authority   of   the   State   of   West   Virginia,   as  
be   deprived   of   the   legitime   to   which   she   is   entitled   under   the   law   governing   provided  in  section  300  of  the  Code  of  Civil  Procedure.  Nor  was  the  extract  from  
testamentary  successions  in  these  Islands.  Upon  this  point  it  is  sufficient  to  say   the  law  attested  by  the  certificate  of  the  officer  having  charge  of  the  original,  
that  the  probate  of  the  will  does  not  affect  the  intrinsic  validity  of  its  provisions,   under   the   seal   of   the   State  of   West   Virginia,   as   provided   in   section   301   of   the  
the  decree  of  probate  being  conclusive  only  as  regards  the  due  execution  of  the   Code   of   Civil   Procedure.   No   evidence   was   introduced   to   show   that   the   extract  
will.   from   the   laws   of   West   Virginia   was   in   force   at   the   time   the   alleged   will   was  
  executed.    
If,   therefore,   upon   the   distribution   of   this   estate,   it   should   appear   that   any    
legacy  given  by  the  will  or  other  disposition  made  therein  is  contrary  to  the  law   FACTS:  
applicable  in  such  case,  the  will  must  necessarily  yield  upon  that  point  and  the   The  special  administrator  of  the  estate  of  Edward  Randolph  Hix  appeals  from  a  
law   must   prevail.   Nevertheless,   it   should   not   be   forgotten   that   the   intrinsic   decision   of   Judge   of   First   Instance   Tuason   denying   the   probate   of   the   alleged  
validity  of  the  provisions  of  this  will  must  be  determined  by  the  law  of  Illinois   last  will  and  testament  of  the  deceased.  
and   not,   as   the   appellant   apparently   assumes.   It   is   settled   that   in   legal   and    
testamentary  successions,  with  regard  to  the  order  of  succession,  as  well  as  to   It   is   the   theory   of   the   petitioner   that   the   alleged   will   was   executed   in   Elkins,  
the   amount   of   the   successional   rights   and   to   the   intrinsic   validity   of   their   West   Virginia,   on   November   3,   1925,   by   Hix   who   had   his   residence   in   that  
provisions,   shall   be   regulated   by   the   laws   of   the   nation   of   the   person   whose   jurisdiction,  and  that  the  laws  of  West  Virginia  govern.  To  this  end,  there  was  
succession  is  in  question,  whatever  may  be  the  nature  of  the  property  and  the   submitted  a  copy  of  section  3868  of  Acts  1882,  c.  84  as  found  in  West  Virginia  
country  where  it  may  be  situate.     Code,  Annotated,  by  Hogg,  Charles  E.,  vol.  2,  1914,  p.  1690,  and  as  certified  to  
  by  the  Director  of  the  National  Library.  
Given   the   matters   discussed   hereinabove,   the   trial   court   committed   no   error   in    
denying  the  relief  sought.     ISSUE:  
  Whether   or   not   the   will   should   be   allowed   probate   in   the   Philippines   despite  
the  absence  of  proof  showing  compliance  with  the  laws  of  West  Virginia  for  the  
execution  of  wills?  
 
HELD:  
No.   The   submission   of   the   aforementioned   copy   of   the   law   was   far   from   a  
compliance   with   the   law.   The   laws   of   a   foreign   jurisdiction   do   not   prove  

 
CONFLICT  OF  LAWS                                                                                    AV  DE  TORRES   64  
ATTY.  ARIS  L.  GULAPA                                            AY  2015-­‐2016  
themselves   in   our   courts.   The   courts   of   the   Philippine   Islands   are   not   the  presentation  of  other  competent  evidence  to  prove  the  existence  of  foreign  
authorized   to   take   judicial   notice   of   the   laws   of   the   various   States   of   the   law   (i.e.   as   long   as   the   court   is   “satisfied”   with   the   evidence   presented).   Hence,  
American  Union.  Such  laws  must  be  proved  as  facts.  (In  re  Estate  of  Johnson   the   law   may   be   proved   by   expert   testimony   (i.e.   practicing   lawyer)  
[1918],   39   Phil.,   156.)   Here   the   requirements   of   the   law   were   not   met.  There   accompanied   by   a   copy   of   the   particular   law   being   cited.   On   the   other   hand,  
was  no  showing  that  the  book  from  which  an  extract  was  taken  was  printed   the   court   was   not   satisfied   with   the   evidence   presented   in   the   Hix   case   (i.e.  
or  published  under  the  authority  of  the  State  of  West  Virginia,  as  provided  in   testimony   of   the   special   administrator   and   West   Virginia   Code,   Annotated,   and  
section  300  of  the  Code  of  Civil  Procedure.  Nor  was  the  extract  from  the  law   as  certified  to  by  the  Director  of  the  National  Library)    
attested  by  the  certificate  of  the  officer  having  charge  of  the  original,  under    
the  seal  of  the  State  of  West  Virginia,  as  provided  in  section  301  of  the  Code   FACTS:  
of   Civil   Procedure.  No   evidence   was   introduced   to   show   that   the   extract   from   This  case  involves  the  liability  of  A.H.  Muzzal,  a  former  resident  of  the  State  of  
the   laws   of   West   Virginia   was   in   force   at   the   time   the   alleged   will   was   California,  now  residing  in  the  Philippines,  for  obligations  contracted  by  Meyer-­‐
executed.     Muzzal   Company,   a   California   corporation,   on   5   Nov   1928   and   22   Dec   1928  
  with  Willamette  Iron  &  Steel  Works.  Stanley  Hermann,  a  CPA/auditor  hired  by  
It   was   also   necessary   for   the   petitioner   to   prove   that   the   testator   had   his   Meyer-­‐   Muzzal   Company   on   said   dates,   testified   that   A.H.   Muzzal   subscribed  
domicile  in  West  Virginia  and  not  in  the  Philippine  Islands.   The   only   evidence   and  owned  1,433  out  of  the  5,000  Meyer-­‐Muzzal  Company  shares  (par  value  of  
introduced  to  establish  this  fact  consisted  of  the  recitals  in  the  alleged  will  and   $10   each)   at   the   time   the   obligations   were   contracted.   Willamette   seeks   to  
the   testimony   of   the   petitioner.   Also   in   beginning   administration   proceedings   recover  under  Sec.  322  of  the  Civil  Code  of  California,  which  provides:  
originally   in   the   Philippine   Islands,   the   petitioner   violated   his   own   theory   by    
attempting  to  have  the  principal  administration  in  the  Philippine  Islands.     SEC.   322.   Each   stockholder   of   a   corporation   is   individually   and  
  personally   liable   for   such   proportion   of   all   its   debts   and   liabilities  
42.  WILLIAMETTE  v.  MUZZAL   contracted   or   incurred   during   the   time   he   was   a   stockholder   as   the  
G.R.  No.  L-­‐42538  |  May  21,  1935   amount   of   stock   or   shares   owned   by   him   bears   to   the   whole   of   the  
  subscribed   capital   stock   or   shares   of   the   corporation.   Any   creditor   of  
DOCTRINE:   GR:   Foreign   law   must   be   proven   in   accordance   with   the   procedure   the   corporation   may   institute   joint   or   several   actions   against   any   of   its  
under  the  Rules  of  Court.   stockholders,   for   the   proportion   of   his   claim   payable   by   each,   and   in  
  such   action   the   court   must   (1)   ascertain   the   proportion   of   the   claim   or  
EXC   #1:   A   reading   of   sections   300   and   301   of   our   Code   of   Civil   Procedure   will   debt   for   which   each   defendant   is   liable,   and   (2)   a   several   judgment  
convince   one   that   these   sections   do   not   exclude   the   presentation   of   other   must   be   rendered   against   each,   in   conformity   therewith.   If   any  
competent  evidence  to  prove  the  existence  of  a  foreign  law.   stockholder  pays  his  proportion  of  any  debt  due  from  the  corporation,  
  incurred   while   he   was   such   stockholder,   he   is   relieved   from   any  
The   foreign   law   is   a   matter   of   fact   ...   You   ask   the   witness   what   the   law   is;   he   further   personal   liability   for   such   debt,   and   if   an   action   has   been  
may  from  his  recollection,  or  on  producing  and  referring  to  books,  say  what  it  is.   brought  against  him  upon  such  debt,  it  must  be  dismissed,  as  to  him,  
  upon   his   paying   the   costs,   or   such   proportion   thereof   as   may   be  
N.B.:   Other   than   the   lawyer’s   testimony,   an   annotated   Civil   Code   of   California   properly   chargeable   against   him.   The   liability   of   each   stockholder   is  
was  also  presented  as  evidence.  The  law  was  deemed  proved.   determined   by   the   amount   of   stock   or   shares   owned   by   him   at   the  
  time  the  debt  or  liability  was  incurred;  and  such  liability  is  not  released  
Kyna’s   notes:   Comparison   between   Hix   and   Muzzal   case   –   Although   it   is   by  any  subsequent  transfer  of  stock.  
desirable  that  that  foreign  law  be  proved  in  accordance  with  Rule  132  Section    
24,  the  Supreme  Court  held  in  Muzzal  that  the  foregoing  rule  does  not  exclude   The   CFI   of   Zamboanga   rendered   judgment   in   favor   of   Willamette;   hence   this  

 
CONFLICT  OF  LAWS                                                                                    AV  DE  TORRES   65  
ATTY.  ARIS  L.  GULAPA                                            AY  2015-­‐2016  
appeal  by  A.H.  Muzzal.   Company  inc.  And  as  part  of  his  testimony,  a  full  quotation  of  the  cited  section  
  was  offered  in  evidence.  This  was  held  to  be  sufficient  proof.  
ISSUES:    
Whether  the  lower  court  erred  in  finding  that  plaintiff  has  proven  the  existence   FACTS:  
of  the  foreign  law  involved  in  this  action     Walter  G.  Stevenson,  born  in  the  Philippines  of  British  parents  and  married  in  
  Manila  to  Beatrice  Mauricia  Stevenson,  another  British  subject,  died  in  1951  in  
HELD:   San   Francisco,   California,   USA,   where   he   and   his   wife   established   their  
No.   Mr.   Arthur   W.   Bolton,   an   attorney-­‐at-­‐law   of   San   Francisco,   California,   since   permanent   residence   since   1945.   In   his   probated   will   (executed   in   SF,   probated  
the  year  1918,  under  oath,  quoted  verbatim  section  322  of  the  California  Civil   in   Superior   Court   of   California),   Stevenson   instituted   his   wife   Beatrice   as   his  
Code  and  stated  that  said  section  was  in  force  at  the  time  the  obligations  of  the   sole  heiress  to  several  real  and  personal  properties  (2  parcels  of  land  in  Baguio,  
defendant   to   the   plaintiff   were   incurred,   i.e.,   on   November   5,   1928   and   shares  of  stock,  credit,  and  cash)  acquired  by  the  spouses  while  residing  in  the  
December   22,   1928.   This   evidence   sufficiently   established   the   fact   that   the   Philippines.  
section   in   question   was   the   law   of   the   State   of   California   on   the   above   dates.   A    
reading  of  sections  300  and  301  of  our  Code  of  Civil  Procedure  will  convince   In  1951,  ancillary  administration  proceedings  were  instituted  in  CFI  Manila  for  
one  that  these  sections  do  not  exclude  the  presentation  of  other  competent   the   settlement   of   the   estate   in   the   Philippines.   The   will   was   admitted   to  
evidence  to  prove  the  existence  of  a  foreign  law.   probate  and  Ian  Murray  Statt  was  appointed  ancillary  administrator.  Statt  filed  
  a  preliminary  estate  and  inheritance  tax  return  with  the  reservation  of  having  
"The   foreign   law   is   a   matter   of   fact   ...   You   ask   the   witness   what   the   law   is;   he   the   properties   declared   finally   appraised   at   their   values   6   months   after   the  
may  from  his  recollection,  or  on  producing  and  referring  to  books,  say  what  it   death   of   Stevenson.   The   preliminary   return   was   made   in   order   to   secure   the  
is."  (Lord  Campbell  concurring  in  an  opinion  of  Lord  Chief  Justice  Denman  in  a   waiver   of   the   CIR   on   the   inheritance   tax   due   on   210k   shares   of   stock   in   the  
well  known  English  case  where  a  witness  was  called  upon  to  prove  the  Roman   Mindanao   Mother   Lode   Mines   Inc.   (MMLMI)   which   the   estate   desired   to  
laws   of   marriage   and   was   permitted   to   testify,   though   he   referred   to   a   book   dispose   in   the   US.   The   CIR   accepted   the   valuation   of   the   personal   properties  
containing   the   decrees   of   the   Council   of   Trent   as   controlling,   Jones   on   declared   in   the   return,   but   increased   the   appraisal   of   the   2   parcels   of   land   in  
Evidence,   Second   Edition,   Volume   4,   pages   3148-­‐3152.)   Aside   from   the   Baguio   City.   Ultimately,   the   estate   was   assessed   an   estate   tax   of   5.15k   and  
testimony   of   Attorney   Bolton   Ragland's   Annotated   Civil   Code   of   California   inheritance  tax  of  10.9k,  both  of  which  paid  by  the  estate  in  1952.  
was   presented   as   evidence.   This   book   contains   that   State's   Civil   Code   as    
adopted  March  21,  1872,  with  the  subsequent  official  statute  amendments  to   The  ancillary  administrator  filed  an  amended  estate  and  inheritance  tax  return  
and  including  the  year  1929.   in  pursuance  of  the  reservation  he  made  and  Sec.  91  of  the  NIRC.  The  amended  
  return  reduced  the  value  of  the  shares  of  stock  in  MMLMI  from  0.38  per  share  
43.  CIR  v  FISHER   to   0.20,   based   on   the   market   notation   of   the   stock   obtaining   at   the   SF  
G.R.  No.  L-­‐11622  |  January  28,  1961   California  Stock  Exchange  6  months  after  the  death  of  Stevenson.  Statt  likewise  
  made   claim   for   several   deductions   in   the   form   of   funeral   and   judicial   expenses,  
In  the  case  at  bar,  to  prove  the  pertinent  California  law,  Attorney  Allison  Gibbs,   real  estate  tax,  and  claims  against  the  estate.  
counsel  for  Fisher,  testified  that  as  an  active  member  of  the  California  Bar  since    
1931,   he   is   familiar   with   the   revenue   and   taxation   laws   of   the   State   of   Beatrice   assigned   all   her   rights   and   interests   in   the   estate   to   spouses   Douglas  
California.  When  asked  by  the  lower  court  to  state  the  pertinent  California  law   and  Bettina  Fisher.  
as   regards   exemption   of   intangible   personal   properties,   the   witness   cited    
nd
article  4,  section  13851  (a)  and  (b)  of  the  California  Internal  and  Revenue  Code   In  1953,  the  ancillary  administrator  filed  a  2  amended  return  containing  new  
as  published  in  Derring's  California  Code,  a  publication  of  the  Bancroft-­‐Whitney   claims   for   additional   exemption   and   deduction   to   wit:   (1)   deduction   in   the  

 
CONFLICT  OF  LAWS                                                                                    AV  DE  TORRES   66  
ATTY.  ARIS  L.  GULAPA                                            AY  2015-­‐2016  
amount  of  P4,000.00  from  the  gross  estate  of  the  decedent  as  provided  for  in   the  Tax  Court,  the  pertinent  English  law  that  allegedly  vests  in  the  decedent  
Section   861   (4)   of   the   U.S.   Federal   Internal   Revenue   Code   which   the   ancillary   husband   full   ownership   of   the   properties   acquired   during   the   marriage   has  
administrator   averred   was   allowable   by   way   of   the   reciprocity   granted   by   not  been  proven  by  the  CIR.  Except  for  a  mere  allegation  in  his  answer,  which  
Section  122  of  the  NIRC,  as  then  held  by  the  Board  of  Tax  Appeals  in  a  previous;   is   not   sufficient,   the   record   is   bereft   of   any   evidence   as   to   what   English   law  
and  (2)  exemption  from  the  imposition  of  estate  and  inheritance  taxes  on  the   says  on  the  matter.  In  the  absence  of  proof,  the  Court  is  justified,  therefore,  
shares  of  stock  in  the  MMLMI  also  pursuant  to  the  said  reciprocity  proviso.  In   in   indulging   in   what   Wharton   calls   "processual   presumption,"   in   presuming  
this   last   return,   the   estate   claimed   that   it   was   liable   only   for   the   amount   of   that  the  law  of  England  on  this  matter  is  the  same  as  our  law.  
P525.34  for  estate  tax  and  P238.06  for  inheritance  tax.  A  refund  of  15,259  was    
requested  by  the  estate  but  was  denied  by  the  CIR.   CIR   can   neither   make   use   of   Art.   16   of   the   NCC   (art.   10   of   the   old   code)   to  
  bolster   his   stand.   A   reading   of   Art.   10,   which   is   applicable   in   this   case,   shows  
The  Fishers,  as  assignees  of  Beatrice,  commenced  action  in  CFI  Manila  for  the   that  it  does  not  encompass  or  contemplate  to  govern  the  question  of  property  
recovery  of  the  said  amount.  The  case  was  forwarded  to  the  CTA,  which  set  the   relation   between   spouses.   Said   article   distinctly   speaks   of  amount   of  
allowable  exemptions  and  deductions  in  dispute.  Both  parties  appealed.   successional   rights  and   this   term,   in   speaks   in   our   opinion,   properly   refers   to  
  the   extent   or   amount   of   property   that   each   heir   is   legally   entitled   to   inherit  
ISSUE:   from   the   estate   available   for   distribution.   It   needs   to   be   pointed   out   that  
Whether   in   determining   the   taxable   net   estate   of   the   decedent,   ½   of   the   net   the  property  relation  of  spouses,  as  distinguished  from  their  successional  rights,  
estate   should   be   deducted   therefrom   as   the   share   of   the   surviving   spouse   in   is   governed   differently   by   the   specific   and   express   provisions   of   Title   VI,  
accordance  with  our  law  on  conjugal  partnership  and  in  relation  to  Section  89   Chapter  I  of  our  new  Civil  Code  (Title  III,  Chapter  I  of  the  old  Civil  Code.)  Thus,  
(c)  of  the  NIRC   the   lower   court   correctly   deducted   the   half   of   the   conjugal   property   in  
  determining  the  hereditary  estate  left  by  the  deceased  Stevenson.  
HELD:    
Yes.   Since   the   marriage   of   the   Stevensons   in   the   Philippines   took   place   in   1909,   However,   as   held   in   the   case   of   Willamette   Iron   and   Steel   Works   v.   Muzzal,   a  
the   applicable   law   is   Art.   1325   of   the   old   Civil   Code,   not   Art.   124   of   the   NCC,   as   reading  of  the  Code  of  Civil  Procedure  will  convince  one  that  these  sections  do  
posited   by   the   CIR.   It   is   true   that   both   articles   adhere   to   the   so-­‐called   not   exclude   the   presentation   of   other   competent   evidence   to   prove   the  
nationality   theory   of   determining   the   property   relation   of   spouses   where   one   existence   of   a   foreign   law.   In   that   case,   we   considered   the   testimony   of   an  
of   them   is   a   foreigner   and   they   have   made   no   prior   agreement   as   to   the   attorney-­‐at-­‐law   of   San   Francisco,   California   who   quoted   verbatim   a   section   of  
administration  disposition,  and  ownership  of  their  conjugal  properties.  In  such   California   Civil   Code   and   who   stated   that   the   same   was   in   force   at   the   time   the  
a   case,   the   national   law   of   the   husband   becomes   the   dominant   law   in   obligations  were  contracted,  as  sufficient  evidence  to  establish  the  existence  of  
determining   the   property   relation   of   the   spouses.   There   is,   however,   a   said   law.   In   the   case   at   bar,   to   prove   the   pertinent   California   law,   Attorney  
 
difference   between   the   two   articles   in   that   Article   124 of   the   new   Civil   Code   Allison   Gibbs,   counsel   for   Fisher,   testified   that   as   an   active   member   of   the  
expressly   provides   that   it   shall   be   applicable   regardless   of   whether   the   California  Bar  since  1931,  he  is  familiar  with  the  revenue  and  taxation  laws  of  
marriage  was  celebrated  in  the  Philippines  or  abroad  while  Article  1325  of  the   the   State   of   California.   When   asked   by   the   lower   court   to   state   the   pertinent  
old  Civil  Code  is  limited  to  marriages  contracted  in  a  foreign  land.   California   law   as   regards   exemption   of   intangible   personal   properties,   the  
  witness  cited  article  4,  section  13851  (a)  and  (b)  of  the  California  Internal  and  
It   must   be   noted,   however,   that   what   has   just   been   said   refers   to   mixed   Revenue   Code   as   published   in   Derring's   California   Code,   a   publication   of   the  
marriages  between  a  Filipino  citizen  and  a  foreigner.  In  the  instant  case,  both   Bancroft-­‐Whitney  Company  inc.  And  as  part  of  his  testimony,  a  full  quotation  of  
spouses  are  foreigners  who  married  in  the  Philippines.  If  we  adopt  the  view  of   the  cited  section  was  offered  in  evidence.  This  was  held  to  be  sufficient  proof.  
Manresa,   the   law   determinative   of   the   property   relation   of   the   Stevensons,    
married  in  1909,  would  be  the  English  law  even  if  the  marriage  was  celebrated  
in  the  Philippines,  both  of  them  being  foreigners.  But,  as  correctly  observed  by  
 
CONFLICT  OF  LAWS                                                                                    AV  DE  TORRES   67  
ATTY.  ARIS  L.  GULAPA                                            AY  2015-­‐2016  
44.  PARDO  v  REPUBLIC  OF  THE  PHILIPPINES   Guttierez,   a   respectable   citizen   who   has   known   the   applicant   for   27   years,  
GR  No.  L-­‐2248  |  January  23,  1950   granted   the   application,   considering   the   fact   that   applicant   arrived   in   the  
(In  the  matter  of  the  petition  of  Vicente  Rosal  Pardo  to  be  admitted  a  citizen   Philippines   when   he   was   only   10   years   old   and   has   lived   here   for   44   years  
of  the  Philippines)   continuously   except   for   a   few   months   of   visit   in   Spain.   His   mingling   and   dealing  
  by   reason   of   his   work   with   people   who   use   Tagalog   in   their   daily   intercourse,  
DOCTRINE:   EXC   #2:   Evidence   of   the   law   of   a   foreign   country   on   reciprocity   lends   credence   in   his   testimony   that   he   has   acquired   a   good   working  
regarding   the   acquisition   of   citizenship,   although   not   meeting   the   prescribed   knowledge   of   the   language.   He   also   owned   2   stores   on   the   Escolta   and   has  
rule  of  practice,  may  be  allowed  and  used  as  basis  for  favorable  action,  if,  in  the   been  a  foreman  and  warehouseman  at  Soriano  &  Co.  
light  of  all  the  circumstances,  the  Court  is  "satisfied  of  the  authenticity  of  the    
written  proof  offered."   2.   Yes.   The   applicant   introduced   a   certificate   signed   by   the   Consul   General   of  
  Spain  in  the  Philippines,  stating  that  in  accordance  with  articles  17  and  225  of  
"Applicability   of   the   Rules,"   provides   that   "These   rules   shall   not   apply   to   land   the   Spanish   Civil   Code,   among   other   Spanish   legislation,   Filipinos   are   eligible   to  
registration,   cadastral   and   election   cases,   naturalization   and   insolvency   Spanish   citizenship   in   Spain.   Article   17   provides   that   foreigners   who   have  
proceedings,  and  other  cases  not  herein  provided  for,  except  by  analogy  or  in  a   obtained  a  certificate  of  naturalization  and  those  who  have  not  obtained  such  
suppletory  character  and  whenever  practicable  and  convenience.”  [NICOLE]   certificate   but   have   acquired   domicile   in   any   town   of   the   Monarchy   are  
  Spaniards.   No   discrimination   being   made   in   these   provisions,   they   apply   to  
By   reason   of   this   provision,   literal   adherence   to   the   Rules   of   Court,   which   persons  of  any  nationality.  
include   rules   of   evidence,   is   not   obligatory   in   a   proceeding   like   that   under   the    
Philippine   law   is   judicial   in   character,   and   strict   compliance   with   the   process   As   the   Spanish   Civil   Code   has   been   and   still   is   "the   basic   code   in   force   of   the  
prescribed  by  statute,  if  there  were  one,  would  be  essential,  yet  when,  as  here,   Philippines,"  articles  17  et  seq.  thereof  may  be  regarded  as  matters  known  to  
no  specific  procedure  is  indicated  in  the  premises,  it  is  only  necessary  that  the   judges   of   the   Philippines   by   reason   of   their   judicial   functions   and   may   be  
merits   of   the   petition   be   passed   on   and   a   decision   reached   on   a   far   judicially   recognized   by   them   without   the   introduction   of   proof.   Moreover,   in   a  
consideration  of  the  evidence  on  satisfactory  proof.   number  of  decisions  mere  authentication  of  the  Chinese  Naturalization  Law  by  
  the   Chinese   Consulate   General   of   Manila   has   been   held   to   be   competent   proof  
N.B.:   A   copy   of   a   foreign   law   certified   only   by   the   local   consul   of   the   applicant's   of  that  law.  
country  was  considered  as  sufficient  proof.    
  MOTION   FOR   RECONSIDERATION:   In   a   number   of   decisions,   mere  
ISSUE:   authentication   of   the   Chinese   naturalization   law   by   the   Chinese   Consulate  
1.  Whether  Vicente  is  eligible  for  Philippine  citizenship   general  in  Manila  has  been  taken  as  competent  proof  of  that  law.  The  Solicitor  
2.   Whether   the   laws   of   Spain   grant   Filipinos   the   right   to   become   naturalized   General   takes   exception   to   this   passage   –     in   several   jurisprudence,   the  Court  
citizens  of  that  country   “did  not  rule  that  the  mere  authentication  of  the  Chinese  Naturalization  Law  by  
  the   Chinese   Consulate   General   of   manila   constitute   competent   proof   of   that  
FACTS  and  HELD:   law,   but   that   the   question   as   to   whether   or   not   the   copy   of   the   Chinese  
1.  Yes.  Vicente  Rosal  Pardo,  a  Spanish  citizen  born  in  Spain  in   1895  and  residing   Nationality   Law   presented   in   said   cases   were   properly   authenticated   and  
in  the  Philippines  since  1905,  where  he  married  a  Filipino  woman  and  where  he   admissible  in  evidence  to  prove  reciprocity,  as  required  in  section  4  (h)  of  the  
is   at   present   employed   in   Manila,   has   been   adjudged   by   the   CFI   of   Manila   Revised   Naturalization   Law,   has   become   academic   because   of   the   admission  
entitled   to   become   a   Filipino   citizen.   The   government   appealed   said   decision,   made   by   counsel   for   the   oppositor   (Republic   of   the   Philippines)   to   the   effect  
contending   that   Vicente   is   unable   to   speak   and   write   any   of   the   principal   that   in   another   case,   there   has   been   presented   a   copy   of   the   Naturalization  
Filipino   languages.   The   trial   judge,   strengthened   by   the   testimony   of   Lino   Laws  of  China  duly  authenticated  in  accordance  with  the  Rules  of  the  Court.”  
 
 
CONFLICT  OF  LAWS                                                                                    AV  DE  TORRES   68  
ATTY.  ARIS  L.  GULAPA                                            AY  2015-­‐2016  
The   SC   corrected   said   argument,   stating   that   in  Yap   vs.   Solicitor   General,   the   regarding  the  acquisition  of  citizenship,  although  not  meeting  the  prescribed  
document  admitted  purported  to  be  "a  copy  of  the  Chinese  law  of  citizenship,   rule  of  practice  by  section  41  of  Rule  123,  may  be  allowed  and  used  as  basis  
where   it   appears   that   Filipinos   can   acquire   Chinese   Citizenship   by   for   a   favorable   action   if,   in   the   light   of   all   circumstances,   the   court   is   satisfied  
naturalization."  There  was  nothing  in  that  decision  which  would  show  that  the   of  the  authenticity  of  the  written  proof  offered.  
certificate   or   authentication   was   made   by   a   Philippine   diplomatic   or   consular    
representative   in   China.   In  Jose   Leelin   vs.   Republic   of   the   Philippines,   we   said   45.  PCIB  v  ESCOLIN  
that  "in  previous  cases,  a  translation  of  the  Chinese  Naturalization  Law,  made   GR  Nos.  L-­‐27860  and  L-­‐27896  |  March  29,  1974  
and   certified   to   be   correct   by   the   Chinese   Consulate   General   in   Manila,   was    
admitted  and  considered  efficient  evidence  to  establish  that  the  laws  of  China   DOCTRINE:   Processual   Presumption—The   foreign   law,   whenever   applicable,  
permit   Filipinos   to   become   citizens   of   that   country."   In  Yee   Boo   Mann   vs.   should   be   proved   by   the   proponent   thereof,   otherwise,   such   law   shall   be  
Republic  of  the  Philippines,  the  petitioner  introduced  in  evidence  a  translation   presumed  to  be  exactly  the  same  as  the  law  of  the  forum.  
of  the  Chinese  Naturalization  Law,  certified  to  be  correct  by  the  Chinese  Consul    
General   in   Manila.   The   court   held   in   that   case   that   the   objection   to   the   Aznar   v.   Garcia   case   can't   be   used   to   show   what   Texas   law   may   contain,   as  
evidence   "is   of   no   moment,   since   this   court   has   already   accepted   it   as   fact   in   there's   a   time   difference   between   this   case   and   that   case,   thus   the   Texas   law  
previous   naturalization   cases   that   the   laws   of   China   permit   Filipinos   to   might  have  changed  in  between  the  rulings.  
naturalize  in  that  country."    
  FACTS:      
If   it   be   true,   as   the   Solicitor   General   notes,   that   in   the   Yap   case   the  ratio   The  Hodges  spouses,  both  Texas  citizens  and  domiciled  in  the  Philippines,  made  
decidendi  was  that  "there  has  been  presented  a  copy  of  the  Naturalization  Laws   in  their  individual  wills,  provisions  which  provide  that  upon  their  deaths,  their  
of   China   duly   authenticated   in   accordance   with   the   Rules   of   the   Court,"   then   whole  estate  would  be  inherited  by  the   surviving  spouse  and  that  spouse  could  
the   decision   recognized   as   a   fact   the   existence   of   a   law   of   China   under   which   manage  and  alienate  the  said  lands,  with  the  exception  of  the  Texas  property,  
Filipinos  may  be  naturalized.  Of   this   fact   the   court   properly   assumed   judicial   and   the   remainder   upon   the   death   of   the   surviving   spouse   would   redound   to  
knowledge  in  the  cases  that  came  up  before  it  soon  after.   the  brothers  and  sisters  of  the  deceased  surviving  spouse.  
   
We  realize  that  a  copy  of  a  foreign  law  certified  only  by  the  local  consul  of  the   Linnie   Jane   Hodges   died   first   in   Iloilo   leaving   a   will   executed   in   Texas   but  
applicant's   country   does   not   conform   to   the   requirement   concerning   the   probated   in   the   CFI   of   Iloilo   City   with   the   widower   Charles   Newton   Hodges  
certification  and  authentication  of  such  law  (sec.  41,  Rule  123).  But  the  case  at   appointed   as   Executor,   pursuant   to   the   provisions   of   the   will.   Charles   was  
bar   and   the   cases   cited   therein   as   precedents   are   not   governed   by   the   Rules   of   previously   appointed   Special   Administrator,   in   which   capacity   he   filed   an  
the   Court.   Rule   1342,   entitled   "Applicability   of   the   Rules,"   provides   that   “urgent   ex-­‐parte   motion   to   allow   or   authorize   himself   to   continue   the   business  
"These  rules  shall  not  apply  to  land  registration,  cadastral  and  election  cases,   in  which  he  was  engaged  and  to  perform  acts  which  he  had  been  doing  while  
naturalization   and   insolvency   proceedings,   and   other   cases   not   herein   deceased   was   living”   which   was   granted   by   the   court.   As   Executor   he   filed  
provided   for,   except   by   analogy   or   in   a   suppletory   character   and   whenever   another   motion   to   approve   all   sales,   conveyances,   leases,   mortgages   that   he  
practicable  and  convenience.  By  reason  of  this  provision,  literal  adherence  to   had  made  further  and  subsequent  transaction  which  he  may  do  in  accordance  
the   Rules   of   Court,   which   include   rules   of   evidence,   is   not   obligatory   in   a   with   the   last   wishes   of   his   wife”   which   was   also   approved   by   the   court.   In  
proceeding  like  that  under  the  Philippine  law  is  judicial  in  character,  and  strict   financial  statements  submitted  before  the  court,  he  made  statements  that  the  
compliance  with  the  process  prescribed  by  statute,  if  there  were  one,  would   estate   of   Linnie   is   1/2   of   the   conjugal   estate.   He   also   allegedly   renounced   his  
be   essential,   yet   when,   as   here,   no   specific   procedure   is   indicated   in   the   inheritance  in  a  tax  declaration  in  the  U.S.  
premises,  it  is  only  necessary  that  the  merits  of  the  petition  be  passed  on  and    
a   decision   reached   on   a   far   consideration   of   the   evidence   on   satisfactory  
proof.   Accordingly,   evidence   of   the   law   of   a   foreign   country   or   reciprocity  
 
CONFLICT  OF  LAWS                                                                                    AV  DE  TORRES   69  
ATTY.  ARIS  L.  GULAPA                                            AY  2015-­‐2016  
Subsequently,  Charles  also  died.  Magno  was  appointed  as  the  admistratrix  for   the  court,  such  as  when  they  are  well  and  generally  known  or  they  have  been  
the   estate   of   both   spouses   but   later   replaced   by   PCIB   as   to   Charles’   estate.   actually  ruled  upon  in  other  cases  before  it  and  none  of  the  parties  concerned  
Charles  before  his  death,  failed  to  make  accounting,  and  also  failed  to  acquire   to  not  claim  otherwise.  
final   adjudication   of   Linnie’s   estate.   Since   there   was   no   liquidation   of   Linnie’s    
estate,  the  brothers  and  sisters  of  Linnie  wanted  to  determine  the  extent  of  her   When,   with   respect   to   certain   aspects   of   the   foreign   laws   concerned,   the  
estate  that  they  could  inherit.   parties   in   a   given   case   do   not   have   any   controversy   or   are   more   or   less   in  
  agreement,   the   Court   may   take   it   for   granted   for   the   purposes   of   the  
ISSUE:   particular   case   before   it   that   the   said   laws   are   as   such   virtual   agreement  
Whether  Philippine  Law  or  Texas  Law  shall  govern  (regarding  Conflict  of  Laws)   indicates,  without  the  need  of  requiring  the  presentation  of  what  otherwise  
Remanded  (Texas  law  may  be  applied  but  must  be  proved)   would  be  competent  evidence  on  the  point.  PCIB’s  representations  in  regard  
  to   the   laws   of   Texas   virtually   constitute   admissions   of   fact   which   the   other  
HELD:   parties   and   the   court   are   being   made   to   rely   and   act   upon.   PCIB   is   “not  
It   is   necessary   that   the   Texas   law   be   ascertained.   Here   it   must   be   proven   permitted  to  contradict  them  or  subsequently  take  a  position  contradictory  to  
whether   a   renvoi   will   happen   or   whether   Texas   law   makes   the   testamentary   or  inconsistent  with  them.”  
provisions   valid.   In   line   with   Texas   law,   which   should   be   proven   is   the   law    
enforced  during  the  death  of  Linnie  and  not  in  any  other  time.   PCIB   can't   claim   that   the   estate   of   Linnie   is   not   entitled   to   at   least   1/4   of  
  conjugal  property,  they  having  argued  that  it  is  so.  
Art.  16  of  the  Civil  Code  provides  that  the  law  of  the  nationality  of  the  decedent    
applies.  But  if  we  apply  Texas  law,  personal  property  is  governed  by  the  law  of   The  SC  held  that  the  estate  of  Linnie  inherited  by  her  brothers  and  sisters  could  
domicile   of   the   decedent   and   real   property   is   governed   by   its   situs   (both   of   be   more   than   just   stated,   but   this   would   depend   on   (1)   whether   upon   the  
which  is  the  Philippines).  Furthermore  Texas  law  provides  no  legitime.  Thus,  the   proper   application   of   the   principle   of   renvoi   in   relation   to   Article   16   of   the   Civil  
renvoi   doctrine.   Philippine   law   provides   that   the   Surviving   Spouse,   being   the   Code   and   the   pertinent   laws   of   Texas,   it   will   appear   that   Hodges   had   no  
sole  heir,  gets  1/2  of  the  conjugal  property,  then  1/2  goes  to  the  estate  of  the   legitime   as   contended   by   Magno,   and   (2)   whether   or   not   it   can   be   held   that  
spouse.   If   1/2   of   the   estate   of   the   decedent   goes   to   the   surviving   spouse   which   Hodges   had   legally   and   effectively   renounced   his   inheritance   from   his   wife.  
is  the  sole  heir,  then  Charles  gets  1/4  of  the  whole  conjugal  property.  The  Court   Under  the  circumstances  presently  obtaining  and  in  the  state  of  the  record  of  
said   that   Texas   law   may   apply,   but   was   not   proven.   The   laws   of   a   foreign   these   cases,   as   of   now,   the   Court   is   not   in   a   position   to   make   a   final   ruling,  
jurisdiction  do  not  prove  themselves  in  our  courts,  The  courts  of  the  Philippines   whether   of   fact   or   of   law,   on   any   of   these   two   issues,   and   We,   therefore,  
Islands   are   not   authorized   to   take   judicial   notice   of   the   laws   of   the   various   reserve  said  issues  for  further  proceedings  and  resolution  in  the  first  instance  
States   of   the   American   Union.   Such   laws   must   be   proved   as   facts.   The   by   the   court   o   quo,   as   hereinabove   indicated.   We   reiterate,   however,   that  
proponent  should  show  the  foreign  law;  as  certified  by  person  holding/having   pending   such   further   proceedings,   as   matters   stand   at   this   stage,   Our  
custody   of   such   law,   with   a   certificate   that   such   officer   does   have   custody   over   considered  opinion  is  that  it  is  beyond  cavil  that  since,  under  the  terms  of  the  
said   law.   Aznar   v.   Garcia   case   can't   be   used   to   show   what   Texas   law   may   will  of  Mrs.  Hodges,  her  husband  could  not  have  anyway  legally  adjudicated  or  
contain,  as  there's  a  time  difference  between  this  case  and  that  case,  thus  the   caused   to   be   adjudicated   to   himself   her   whole   share   of   their   conjugal  
Texas  law  might  have  changed  in  between  the  rulings.   partnership,  albeit  he  could  have  disposed  any  part  thereof  during  his  lifetime,  
  the   resulting   estate   of   Mrs.   Hodges,   of   which   Magno   is   the   uncontested  
The  question  of  what  are  the  laws  of  Texas  governing  the  matters  in  issue  is,   administratrix,   cannot   be   less   than   one-­‐fourth   of   the   conjugal   partnership  
in  the  first  instance,  one  of  fact,  not  of  law.  Elementary  is  the  rule  that  foreign   properties,  as  of  the  time  of  her  death,  minus  what,  as  explained  earlier,  have  
laws   may   not   be   taken   judicial   notice   of   and   have   to   be   proven   like   any   other   been   gratuitously   disposed   of   therefrom,   by   Hodges   in   favor   of   third   persons  
fact  in  dispute  between  the  parties  in  any  proceeding,  with  the  rare  exception   since   then,   for   even   if   it   were   assumed   that,   as   contended   by   PCIB,   under  
in   instances   when   the   said   laws   are   already   within   the   actual   knowledge   of   Article  16  of  the  Civil  Code  and  applying  renvoi  the  laws  of  the  Philippines  are  
 
CONFLICT  OF  LAWS                                                                                    AV  DE  TORRES   70  
ATTY.  ARIS  L.  GULAPA                                            AY  2015-­‐2016  
the   ones   ultimately   applicable,   such   one-­‐fourth   share   would   be   her   free   record   that   overbooking   of   flights   is   a   common   and   accepted   practice   of  
disposable   portion,   taking   into   account   already   the   legitime   of   her   husband   airlines   in   the   US   and   is   specifically   allowed   under   the   Code   of   Federal  
under  the  Civil  Code.   Regulations   by   the   Civil   Aeronautics   Board,   no   fraud   nor   bad   faith   could   be  
  imputed  on  respondent  TWA.  TWA  was  remiss  in  not  informing  petitioners  that  
46.  ZALAMEA  VS.  COURT  OF  APPEALS  AND  TRANSWORLD  AIRLINES   the  flight  was  overbooked.  But  there  was  no  bad  faith  in  placing  the  petitioners  
G.R.  No.  104235  |  November  18,  1993   in  the  waitlist  along  with  48  passengers.  
   
DOCTRINE:   In  other  jurisprudence  mentioned,  the  CFR  was  pleaded  and  proved.   ISSUE:  
This  was  TWA’s  failure.  It  presented  only  a  deposition—the  one  who  presented   Whether   there   was   bad   faith   on   the   part   of   TWA,   considering   that   TWA  
the  statement  is  not  an  expert  of  the  CFR.  He  was  only  an  employee.   contends  that  overbooking  of  flights  is  a  common  and  accepted  practice  in  the  
  US  
GR:  Foreign  written  law  must  be  proven  by  official  publication  etc.    
EXC:  Foreign  written  law  may  be  proven  by  testimony.  (Muzzal)   HELD:  
EXC  TO  THE  EXC:  When  the  one  who  testified  is  not  an  expert  of  the  foreign  law.   Yes,   there   was   fraud   or   bad   faith.   The   US   law   or   regulation   allegedly  
(Zalamea)   authorizing   overbooking   has   never   been   proved.   Foreign   laws   do   not   prove  
  themselves   nor   can   the   courts   take   judicial   notice   of   them.   Like   any   other  
FACTS:   fact,   they   must   be   alleged   and   proved.  Written  law  may  be  evidenced  by  an  
Petitioner-­‐spouses   Zalamea   and   their   daughter   purchased   3   airline   tickets   from   official  publication  thereof  or  by  a  copy  attested  by  the  officer  having  the  legal  
the  Manila  agent  of  TransWorld  Airlines  for  a  flight  to  NY-­‐LA.  The  tickets  were   custody  of  the  record,  or  by  his  deputy,  and  accompanied  with  a  certificate  that  
at  a  discount  of  75%  and  the  daughter  was  a  full  fare.   such   officer   has   custody.   The   certificate   may   be   made   by   a   secretary   of   an  
  embassy   or   legation,   consul   general,   consul,   vice-­‐consul,   or   consular   agent   or  
While   in   NY,   they   received   a   notice   of   the   reconfirmation.   On   the   appointed   by  any  officer  in  the  foreign  service  of  the  Philippines  stationed  in  the  foreign  
date,   they   checked   in   at   10am   for   their   11am   flight   but   were   placed   on   the   country  in  which  the  record  is  kept,  and  authenticated  by  the  seal  of  his  office.  
waitlist.   The   daughter   appeared   as   No.   13   on   the   waitlist   while   the   two    
Zalameas  were  listed  as  No.  34,  showing  a  party  of  two.  Out  of  the  42  names,   TWA  relied  solely  on  the  statement  of  its  customer  service  agent,  Ms.  Lather,  in  
the  first  22  names  were  eventually  allowed  to  board,  including  the  father.  The   her   deposition.   Aside   from   such,   there   is   no   official   publication   of   said   code  
others  weren’t  able  to  fly.  As  it  were,  those  holding  full-­‐fare  tickets  were  given   presented  as  evidence.  Respondent  court’s  finding  that  overbooking  is  allowed  
first  priority.  The  father  later  discovered  that  he  was  holding  his  daughter’s  full-­‐ has  no  basis.  
fare   ticket.   Those   with   discounted   tickets   were   denied   boarding.   Even   in   the    
next  flight  to  LA,  the  mother  and  daughter  couldn’t  be  accommodated  because   Even   if   the   claimed   U.S.   Code   of   Federal   Regulations   exist,   the   same   isn’t  
it   was   fully   book.   They   were   constrained   to   book   in   another   flight   and   applicable  in  accordance  with  the  principle  of  lex  loci  contractus  which  require  
purchased  2  tickets  from  American  Airlines  at  $918.   that  the  law  of  the  place  where  the  airline  ticket  was  issued  should  be  applied  
  by   the   court   where   the   passengers   are   residents   and   nationals   of   the   forum  
In   the   Philippines,   petitioners   filed   an   action   for   damages   based   on   breach   of   and  the  ticket  is  issued  in  such  State  by  the  defendant  airline.  Since  the  tickets  
contract   of   air   carriage   before   the   RTC   Makati.   The   RTC   ordered   the   airline   were  sold  and  issued  in  the  Philippines,  the  applicable  law  would  be  Philippine  
company  to  pay  the  ticket  costs,  as  well  as  moral  damages  and  attorney’s  fees.   law.  
   
However,  the  CA  held  that  moral  damages  are  recoverable  only  where  there  is  
fraud   or   bad   faith   (in   a   breach   of   contract   of   carriage).   Since   it   is   a   matter   of  

 
CONFLICT  OF  LAWS                                                                                    AV  DE  TORRES   71  
ATTY.  ARIS  L.  GULAPA                                            AY  2015-­‐2016  
47.  MANUFACTURERS  HANOVER  TRUST  CO  V.  GUERRERO   defined  in  Sec.  19,  Rule  132:  “Public  documents  are:  (a)  the  written  official  acts,  
G.R.  No.  135804  |  February  19,  2003   or   records   of   the   official   acts   of   the   sovereign   authority,   official   bodies   and  
  tribunals,   an   public   officers,   whether   of   the   Philippines,   or   of   a   foreign  
DOCTRINE:  An  affidavit  cannot  prove  a  foreign  law.   country.”  Thus,  the  procedure  outlined  in  Sec.  24,  Rule  132  should  be  followed.  
   
Walden   affidavit   was   taken   abroad  ex   parte  and   the   affiant   never   testified   in   CA   likewise   rejected   the   Bank’s   argument   that   Sec.   2,   Rule   34   of   the   old   ROC  
open   court.  The   Walden   affidavit   cannot   be   considered   as   proof   of   New   York   allows   the   Bank   to   move   with   the   supporting   Walden   affidavit   for   partial  
law  on  damages  not  only  because  it  is  self-­‐serving  but  also  because  it  does  not   summary   judgment   in   its   favor.   CA   clarified   that   the   Walden   affidavit   is   not   the  
state   the   specific   New   York   law   on   damages.   The   Walden   affidavit   states   supporting   affidavit   referred   to   in   the   provision   that   would   prove   the   lack   of  
conclusions   from   the   affiant’s   personal   interpretation   and   opinion   of   the   facts   genuine   issue   between   the   parties.   CA   concluded   that   even   if   the   Walden  
of  the  case  vis-­‐a-­‐vis  the  alleged  laws  and  jurisprudence  without  citing  any  law  in   affidavit   is   used   for   purposes   of   summary   judgment,   the   Bank   must   still   comply  
particular.  the  Bank  attached  copies  of  some  of  the  U.S.  court  decisions  cited  in   with  the  procedure  prescribed  by  the  Rules  to  prove  the  foreign  law.  
the  Walden  affidavit,  these  copies  do  not  comply  with  Section  24  of  Rule  132  on    
proof  of  official  records  or  decisions  of  foreign  courts.   ISSUES:  
  Whether  the  Bank’s  affidavit  may  serve  as  proof  for  the  NY  law  
N.B.:  (1)  He  didn’t  testify  in  court.  (2)  The  affidavit  did  not  mention  any  specific    
law.     HELD:  
ALG  disagrees.  Tort  is  common  law.  There  really  is  no  written  law  on  tort.   No.   Under   Section   24   of   Rule   132,   the   record   of   public   documents   of   a  
  sovereign   authority   or   tribunal   may   be   proved   by   (1)   an   official   publication  
FACTS:   thereof   or   (2)   a   copy   attested   by   the   officer   having   the   legal   custody   thereof.  
In  1994,  respondent  Rafael  Ma.  Guerrero  filed  a  complaint  for  damages  against   Such  official  publication  or  copy  must  be  accompanied,  if  the  record  is  not  kept  
petitioner   Manufacturers   Hanover   Trust   Co.   and/or   Chemical   Bank   with   the   in   the   Philippines,   with   a   certificate   that   the   attesting   officer   has   the   legal  
RTC   of   Manila   for   payment   of   damages   allegedly   for   illegally   withheld   taxes   custody   thereof.   The   certificate   may   be   issued   by   any   of   the   authorized  
charged   against   interests   on   his   checking   account   with   the   Bank,   a   returned   Philippine   embassy   or   consular   officials   stationed   in   the   foreign   country   in  
check   worth   $18k   due   to   signature   verification   problems,   and   unauthorized   which   the   record   is   kept,   and   authenticated   by   the   seal   of   his   office.   The  
conversion   of   his   account.   The   Bank   in   it   Answer,   alleged   that   by   stipulation,   attestation   must   state,   in   substance,   that   the   copy   is   a   correct   copy   of   the  
Guerrero’s   account   is   governed   by   New   York   law   and   such   law   does   not   permit   original,  or  a  specific  part  thereof,  as  the  case  may  be,  and  must  be  under  the  
any  of  Guerrero’s  claims  except  actual  damages.  The  Bank  then  filed  a  Motion   official  seal  of  the  attesting  officer.  
for   Partial   Summary   Judgment   seeking   for   the   dismissal   of   Guerrero’s   claims    
except   for   actual   damages.   The   affidavit   of   Alyssa   Walden,   a   NY   attorney,   Certain   exceptions   to   this   rule   were   recognized   in  Asiavest   Limited   v.   Court   of  
supported   the   Bank’s   motion,   stating   that   Guerrero’s   NY   bank   account   Appeals.   In   that   case,   the   SC   considered   the   testimony   under   oath   of   an  
stipulated   that   the   governing   law   is   NY   law,   which   reiterates   what   the   Bank   attorney-­‐at-­‐law   of   San   Francisco,   California,   who   quoted   verbatim   a   section  
alleged.   The   Philippine   Consular   Office   in   NY   authenticated   the   Walden   of  California  Civil  Code  and  who  stated  that  the  same  was  in  force  at  the  time  
affidavit.   the   obligations   were   contracted,   as   sufficient   evidence   to   establish   the  
  existence  of  said  law.  Likewise,  in  several  naturalization  cases,  it  was  held  by  
RTC  denied  Bank’s  Motion  and  MR.  Bank  filed  a  petition  for  certiorari  with  the   the   Court   that   evidence   of   the   law   of   a   foreign   country   on   reciprocity  
CA   assailing   the   RTC   Orders.   The   CA   dismissed   petition.   It   held   that   the   Walden   regarding  the  acquisition  of  citizenship,  although  not  meeting  the  prescribed  
affidavit  does  not  serve  as  proof  of  the  NY  law  and  jurisprudence  relied  on  by   rule  of  practice,  may  be  allowed  and  used  as  basis  for  favorable  action,  if,  in  
the   Bank.   The   CA   considered   NY   law   and   jurisprudence   as   public   documents   the   light   of   all   the   circumstances,   the   Court   is   "satisfied   of   the   authenticity   of  
the  written  proof  offered."  
 
CONFLICT  OF  LAWS                                                                                    AV  DE  TORRES   72  
ATTY.  ARIS  L.  GULAPA                                            AY  2015-­‐2016  
  was   time-­‐chartered   by   SCI   to   Halla   Merchant   Co.,   a   South   Korean   company.  
In   the   case   at   bar,   the   Bank   cannot   rely   on   such   rulings   to   support   its   cause.   Halla   sub-­‐chartered   the   vessel   through   a   time   charter   to   Transmar   Shipping,  
These   cases   involved   attorneys   testifying   in   open   court   during   the   trial   in   the   Inc.  who  further  subchartered  to  vessel  to  Portserv  Limited.  Both  Transmar  and  
Philippines  and  quoting  the  particular  foreign  laws  sought  to  be  established.  On   Portserv   are   Canadian   corporations.   In   1995,   Portserv   requested   petitioner  
the   other   hand,   the   Walden   affidavit   was   taken   abroad  ex   parte  and   the   Crescent  Petroleum,  a  Canadian  corporation  engaged  in  selling  petroleum  and  
affiant   never   testified   in   open   court.  The   Walden   affidavit   cannot   be   oil  products  for  the  use  and  operation  of  oceangoing  vessels,  to  deliver  marine  
considered  as  proof  of  New  York  law  on  damages  not  only  because  it  is  self-­‐ fuel   oils   (bunker   fuels)   to   the   vessel,   which   Crescent   granted   through   an   advice  
serving   but   also   because   it   does   not   state   the   specific   New   York   law   on   via   facsimile.   As   security   for   the   payment   and   related   services,   Crescent  
damages.  The  Walden  affidavit  states  conclusions  from  the  affiant’s  personal   received   2   checks   of   $100k   and   $200k.   Thus,   Crescent   contracted   with   its  
interpretation   and   opinion   of   the   facts   of   the   case   vis-­‐a-­‐vis  the   alleged   laws   supplier,   Marine   Petrobulk,   another   Canadian   corporation,   for   the   physical  
and   jurisprudence   without   citing   any   law   in   particular.   The   citations   in   the   delivery  of  the  bunker  fuels  to  the  Vessel.  
Walden  affidavit  of  various  U.S.  court  decisions  do  not  constitute  proof  of  the    
official  records  or  decisions  of  the  U.S.  courts.  While  the  Bank  attached  copies   Marine   Petrobulk   delivered   the   bunker   fuels   inclusive   of   barging   and  
of   some   of   the   U.S.   court   decisions   cited   in   the   Walden   affidavit,   these   copies   demurrage  charges  at  the  port  of  Pioneer  Grain,  Vancouver,  Canada.  The  Chief  
do   not   comply   with   Section   24   of   Rule   132   on   proof   of   official   records   or   Engineer   Officer   of   the   Vessel   duly   acknowledged   and   received   the   delivery  
decisions  of  foreign  courts.   receipt.  Marine  Petrobulk  issued  an  invoice  to  Crescent,  which  issued  a  check  
  for  the  same  invoice  amount  in  favor  of  Marine  Petrobulk.  The  check  was  then  
Next,   the   Bank   makes   much   of   Guerrero’s   failure   to   submit   an   opposing   encashed.   Crescent   issued   a   revised   invoice   to   Portserv   and/or   charterers   of  
affidavit  to  the  Walden  affidavit.  The  Bank  still  had  the  burden  of  proving  New   the   vessel   with   instruction   to   remit   the   amount   on   or   before   December   1,  
York   law   and   jurisprudence   even   if   Guerrero   did   not   present   an   opposing   1995.   The   period   lapsed   and   several   demands   were   made   but   no   payment   was  
affidavit.   As   the   party   moving   for   summary   judgment,   the   Bank   has   the   burden   received.  The  checks  issued  to  Crescent  as  security  were  dishonored  as  well.  
of  clearly  demonstrating  the  absence  of  any  genuine  issue  of  fact  and  that  any    
doubt   as   to   the   existence   of   such   issue   is   resolved   against   the   movant.   Petition   While   the   Vessel   was   docked   at   the   port   of   Cebu   City,   Crescent   instituted  
denied.   before   the   RTC   of   Cebu   an   action   for   sum   of   money   with   TRO   and   writ   of  
  preliminary   attachment   against   respondents   Vessel,   SCI,   Portserv,   and  
48.   CRESCENT   PETROLEUM   v   M/V   “LOK   MAHESHWARI”,   THE   SHIPPING   Transmar.   The   attachment   was   granted.   Summonses   were   then   served   to  
CORPORATION  OF  INDIA   respondents   through   the   Master   of   the   Vessel.   A   letter   of   undertaking   issued  
G.R.  No.  155014  |  November  11,  2005   by  Pioneer  Insurance  and  Surety  Corporation  was  approved  by  the  trial  court  as  
  counter-­‐bond,  thus,  the  attachment  was  lifted.  Respondents  were  declared  in  
DOCTRINES:   ALG:   What   was   odd   about   this   case?   Crescent   claims   that   default.  RTC  ruled  in  favor  of  Crescent,  holding  defendants  solidarily  liable.  
Canadian   Law   applies   but   since   it   failed   to   plead   and   prove   Canadian   law,   it    
then  said  that  Philippine  law  applies.   Respondents,   on   appeal,   alleged   that   as   per   Part   II   of   the   Bunker   Fuel  
  Agreement   between   Crescent   and   Portserv,   NY   law   governs   the   construction,  
SC  denied  the  attachment  because  there  was  failure  to  state  cause  of  action—it   validity,  and  performance  of  the  contract.  They  also  submitted  certified  copies  
had  to  state  the  law  that  applies.   of   the   Commercial   Instruments   and   Maritime   Lien   Act   of   the   US,   some   US  
  cases,   and   some   Canadian   cases   to   support   their   defense.   CA   reversed,  
FACTS:   dismissing  the  action  primarily  on  the  ground  of  forum  non  conveniens.  
M/V   “Lok   Maheshwari”   is   an   oceangoing   vessel   of   Indian   registry,   owned   by    
Shipping   Corporation   of   India,   principally   owned   by   the   Government   of   India.   It   ISSUE:  
Whether  the  enforcement  of  a  maritime  lien  is  expressly  granted  by  law  
 
CONFLICT  OF  LAWS                                                                                    AV  DE  TORRES   73  
ATTY.  ARIS  L.  GULAPA                                            AY  2015-­‐2016  
  One.  "In  a  suit  to  establish  and  enforce  a  maritime  lien  for  supplies  furnished  to  
HELD:   a   vessel   in   a   foreign   port,   whether   such   lien   exists,   or  whether   the   court   has   or  
No.  Crescent  bases  its  claim  of  a  maritime  lien  on  Sections  21,  22  and  23  of  PD   will   exercise   jurisdiction,   depends   on   the  law   of   the   country   where   the   supplies  
No.  1521,  also  known  as  the  Ship  Mortgage  Decree  of  1978,  viz:   were  furnished,  which  must  be  pleaded  and  proved."  
   
Sec.   21.   Maritime   Lien   for   Necessaries;   persons   entitled   to   such   lien.   -­‐   Any   Two.   The   Lauritzen-­‐Romero-­‐Rhoditis   trilogy   of   cases,   which   replaced   such  
person  furnishing  repairs,  supplies,  towage,  use  of  dry  dock  or  maritime  railway,   single-­‐factor   methodologies   as   the   law   of   the   place   of   supply.   In  Lauritzen   v.  
or  other  necessaries,  to  any  vessel,  whether  foreign  or  domestic,  upon  the  order   Larsen,  a  Danish  seaman,  while  in  Havana  and  in  the  course  of  his  employment,  
of  the  owner  of  such  vessel,  or  of  a  person  authorized  by  the  owner,  shall  have  a   was   negligently   injured.   He   sued   the   shipowner   in   a   federal   district   court   in  
maritime  lien  on  the  vessel,  which  may  be  enforced  by  suit  in  rem,  and  it  shall   New   York   for   damages   under   the   Jones   Act.   In   holding   that   Danish   law   and   not  
be  necessary  to  allege  or  prove  that  credit  was  given  to  the  vessel.   the   Jones   Act   was   applicable,   the   Supreme   Court   adopted   a  multiple-­‐contact  
  test  to   determine,   in   the   absence   of   a   specific   Congressional   directive   as   to   the  
Sec.  22.  Persons  Authorized  to  Procure  Repairs,  Supplies  and  Necessaries.  -­‐  The   statute’s   reach,   which   jurisdiction’s   law   should   be   applied.   The   following  
following   persons   shall   be   presumed   to   have   authority   from   the   owner   to   factors   were   considered:  (1)   place   of   the   wrongful   act;   (2)   law   of   the   flag;   (3)  
procure  repairs,  supplies,  towage,  use  of  dry  dock  or  marine  railway,  and  other   allegiance  or  domicile  of  the  injured;  (4)  allegiance  of  the  defendant  shipowner;  
necessaries  for  the  vessel:  The  managing  owner,  ship’s  husband,  master  or  any   (5)   place   of   contract;   (6)   inaccessibility   of   foreign   forum;   and   (7)   law   of   the  
person   to   whom   the   management   of   the   vessel   at   the   port   of   supply   is   forum.  
entrusted.  No  person  tortuously  or  unlawfully  in  possession  or  charge  of  a  vessel    
shall  have  authority  to  bind  the  vessel.   In   Romero   v.   International   Terminal   Operating   Co,   SC   again   considered   a  
  foreign   seaman’s   personal   injury   claim   under   both   the   Jones   Act   and   the  
Sec.   23.   Notice   to   Person   Furnishing   Repairs,   Supplies   and   Necessaries.   -­‐   The   general   maritime   law.   The   Court   held   that   the   factors   first   announced   in   the  
officers   and   agents   of   a   vessel   specified   in   Section   22   of   this   Decree   shall   be   case   of   Lauritzen   were  applicable   not   only   to   personal   injury   claims   arising  
taken  to  include  such  officers  and  agents  when  appointed  by  a  charterer,  by  an   under  the  Jones  Act  but  to  all  matters  arising  under  maritime  law  in  general.  
owner  pro  hac  vice,  or  by  an  agreed  purchaser  in  possession  of  the  vessel;  but    
nothing   in   this   Decree   shall   be   construed   to   confer   a   lien   when   the   furnisher   Hellenic   Lines,   Ltd.   v.   Rhoditis   was   also   a   suit   under   the   Jones   Act   by   a   Greek  
knew,   or   by   exercise   of   reasonable   diligence   could   have   ascertained,   that   seaman  injured  aboard  a  ship  of  Greek  registry  while  in  American  waters.  The  
because   of   the   terms   of   a   charter   party,   agreement   for   sale   of   the   vessel,   or   for   U.S.   Supreme   Court   observed   that  of   the   seven   factors   listed   in   the   Lauritzen  
any  other  reason,  the  person  ordering  the  repairs,  supplies,  or  other  necessaries   test,  four  were  in  favor  of  the  shipowner  and  against  jurisdiction.  In  arriving  at  
was  without  authority  to  bind  the  vessel  therefor.   the   conclusion   that   the   Jones   Act   applies,   it   ruled   that   the   application   of   the  
  Lauritzen  test  is  not  a  mechanical  one.  It  stated  thus:  "[t]he  significance  of  one  
Petitioner   Crescent   submits   that   these   provisions   apply   to   both   domestic   and   or  more  factors  must  be  considered  in  light  of  the  national  interest  served  by  
foreign  vessels,  as  well  as  domestic  and  foreign  suppliers  of  necessaries.  The  SC   the   assertion   of   Jones   Act   jurisdiction.   Moreover,   the   list   of   seven   factors   in  
does   not   agree.   Mortgage   Decree   of   1978   was   enacted   "to   accelerate   the   Lauritzen   was   not   intended   to   be   exhaustive.   The   shipowner’s   base   of  
growth  and  development  of  the  shipping  industry"  and  "to  extend  the  benefits   operations   is   another   factor   of   importance   in   determining   whether   the   Jones  
accorded   to   overseas   shipping   under   PD   No.   214   to   domestic   shipping.”   It   is   Act  is  applicable;  and  there  well  may  be  others."  
patterned   closely   from   the   U.S.   Ship   Mortgage   Act   of   1920   and   the   Liberian    
Maritime  Law  relating  to  preferred  mortgage.  The  various  tests  used  in  the  U.S.   The  principles  enunciated  in  these  maritime  tort  cases  have  been  extended  to  
to  determine  whether  a  maritime  lien  exists  are  the  following:   cases  involving  unpaid  supplies  and  necessaries.  
   

 
CONFLICT  OF  LAWS                                                                                    AV  DE  TORRES   74  
ATTY.  ARIS  L.  GULAPA                                            AY  2015-­‐2016  
Three.  The  factors   provided   in   Restatement   (Second)   of   Conflicts   of   Law  have    
also   been   applied,   especially   in   resolving   cases   brought   under   the   Federal   But   under   which   law   should   petitioner   Crescent   prove   the   existence   of   its  
Maritime  Lien  Act.  Their  application  suggests  that  in  the  absence  of  an  effective   maritime  lien?  It  is  clear  that  Canada  has  the  most  significant  interest  in  this  
choice  of  law  by  the  parties,  the  forum  contacts  to  be  considered  include:  (a)   dispute.   The   injured   party   is   a   Canadian   corporation,   the   sub-­‐charterer   which  
the   place   of   contracting;   (b)   the   place   of   negotiation   of   the   contract;   (c)   the   placed  the  orders  for  the  supplies  is  also  Canadian,  the  entity  which  physically  
place   of   performance;   (d)   the   location   of   the   subject   matter   of   the   contract;   delivered   the   bunker   fuels   is   in   Canada,   the   place   of   contracting   and  
and   (e)   the   domicile,   residence,   nationality,   place   of   incorporation   and   place   of   negotiation  is  in  Canada,  and  the  supplies  were  delivered  in  Canada.  
business  of  the  parties.    
  The   arbitration   clause   contained   in   the   Bunker   Fuel   Agreement   which   states  
In  the  case  at  bar,  the  Court  cannot  sustain  petitioner  Crescent’s  insistence  on   that  New  York  law  governs  the  "construction,  validity  and  performance"  of  the  
the   application   of   P.D.   No.   1521   or   the   Ship   Mortgage   Decree   of   1978   and   contract   is   only   a   factor   that   may   be   considered   in   the   choice-­‐of-­‐law   analysis  
hold  that  a  maritime  lien  exists.   but  is  not  conclusive.  As  in  the  cases  of  Gulf  Trading  and  Swedish  Telecom,  the  
  lien   that   is   the   subject   matter   of   this   case   arose   by   operation   of   law   and   not   by  
First.  Out   of   the   seven   basic   factors   listed   in   the   case   of  Lauritzen,   Philippine   contract   because   the   shipowner   was   not   a   party   to   the   contract   under   which  
law  only  falls  under  one  –  the  law  of  the  forum.  All  other  elements  are  foreign  –   the  goods  were  supplied.  
Canada   is   the   place   of   the   wrongful   act,   of   the   allegiance   or   domicile   of   the    
injured   and   the   place   of   contract;   India   is   the   law   of   the   flag   and   the   allegiance   It   is   worthy   to   note   that   petitioner   Crescent   never   alleged   and   proved  
of  the  defendant  shipowner.  Balancing  these  basic  interests,  it  is  inconceivable   Canadian   law   as   basis   for   the   existence   of   a   maritime   lien.   To   the   end,   it  
that   the   Philippine   court   has   any   interest   in   the   case   that   outweighs   the   insisted   on   its   theory   that   Philippine   law   applies.   Petitioner   contends   that  
interests  of  Canada  or  India  for  that  matter.   even   if   foreign   law   applies,   since   the   same   was   not   properly   pleaded   and  
  proved,  such  foreign  law  must  be  presumed  to  be  the  same  as  Philippine  law  
Second.  P.D.   No.   1521   or   the   Ship   Mortgage   Decree   of   1978   is   inapplicable   pursuant   to   the   doctrine   of   processual   presumption.   Thus,   we   are   left   with  
following  the  factors  under  Restatement  (Second)  of  Conflict  of  Laws.  Like  the   two  choices:  (1)  dismiss  the  case  for  petitioner’s  failure  to  establish  a  cause  of  
Federal   Maritime   Lien   Act   of   the   U.S.,   P.D.   No.   1521   or   the   Ship   Mortgage   action  or  (2)  presume  that  Canadian  law  is  the  same  as  Philippine  law.  In  either  
Decree  of  1978  was  enacted  primarily  to  protect  Filipino  suppliers  and  was  not   case,  the  case  has  to  be  dismissed.  
intended  to  create  a  lien  from  a  contract  for  supplies  between  foreign  entities    
delivered  in  a  foreign  port.   It   is   well-­‐settled   that   a   party   whose   cause   of   action   or   defense   depends   upon  
  a  foreign  law  has  the  burden  of  proving  the  foreign  law.  Such  foreign  law  is  
Third.  Applying   P.D.   No.   1521   or   the   Ship   Mortgage   Decree   of   1978   and   rule   treated   as   a   question   of   fact   to   be   properly   pleaded   and   proved.   Petitioner  
that   a   maritime   lien   exists   would   not   promote   the   public   policy   behind   the   Crescent’s   insistence   on   enforcing   a   maritime   lien   before   our   courts  
enactment   of   the   law   to   develop   the   domestic   shipping   industry.   Opening   up   depended   on   the   existence   of   a   maritime   lien   under   the   proper   law.   By  
our  courts  to  foreign  suppliers  by  granting  them  a  maritime  lien  under  our  laws   erroneously  claiming  a  maritime  lien  under  Philippine  law  instead  of  proving  
even  if  they  are  not  entitled  to  a  maritime  lien  under  their  laws  will  encourage   that  a  maritime  lien  exists  under  Canadian  law,  petitioner  Crescent  failed  to  
forum  shopping.   establish  a  cause  of  action.  
   
Finally.  The   submission   of   petitioner   is   not   in   keeping   with   the   reasonable   Even  if  we  apply  the  doctrine  of  processual  presumption,  the  result  will  still  
expectation   of   the   parties   to   the   contract.   Indeed,   when   the   parties   entered   be  the  same.  Under  P.D.  No.  1521  or  the  Ship  Mortgage  Decree  of  1978,  the  
into  a  contract  for  supplies  in  Canada,  they  could  not  have  intended  the  laws  of   following  are  the  requisites  for  maritime  liens  on  necessaries  to  exist:  (1)  the  
a   remote   country   like   the   Philippines   to   determine   the   creation   of   a   lien   by   the   "necessaries"  must  have  been  furnished  to  and  for  the  benefit  of  the  vessel;  
mere  accident  of  the  Vessel’s  being  in  Philippine  territory.   (2)   the   "necessaries"   must   have   been   necessary   for   the   continuation   of   the  
 
CONFLICT  OF  LAWS                                                                                    AV  DE  TORRES   75  
ATTY.  ARIS  L.  GULAPA                                            AY  2015-­‐2016  
voyage  of  the  vessel;  (3)  the  credit  must  have  been  extended  to  the  vessel;  (4)    
there   must   be   necessity   for   the   extension   of   the   credit;   and   (5)   the   Finally.  The  necessaries  were  not  ordered  by  persons  authorized  to  contract  in  
necessaries   must   be   ordered   by   persons   authorized   to   contract   on   behalf   of   behalf  of  the  vessel  as  provided  under  Section  22  of  P.D.  No.  1521  or  the  Ship  
the  vessel.  These  do  not  avail  in  the  instant  case.   Mortgage   Decree   of   1978   -­‐   the   managing   owner,   the   ship’s   husband,   master   or  
  any  person  with  whom  the  management  of  the  vessel  at  the  port  of  supply  is  
First.  It   was   not   established   that   benefit   was   extended   to   the   vessel.   While   this   entrusted.   Clearly,   Portserv,   a   sub-­‐charterer   under   a   time   charter,   is   not  
is  presumed  when  the  master  of  the  ship  is  the  one  who  placed  the  order,  it  is   someone  to  whom  the  management  of  the  vessel  has  been  entrusted.  A  time  
not   disputed   that   in   this   case   it   was   the   sub-­‐charterer   Portserv   which   placed   charter  is  a  contract  for  the  use  of  a  vessel  for  a  specified  period  of  time  or  for  
the  orders  to  petitioner  Crescent.  Hence,  the  presumption  does  not  arise  and  it   the  duration  of  one  or  more  specified  voyages  wherein  the  owner  of  the  time-­‐
is   incumbent   upon   petitioner   Crescent   to   prove   that   benefit   was   extended   to   chartered   vessel   retains   possession   and   control   through   the   master   and   crew  
the  vessel.  Petitioner  did  not.   who   remain   his   employees.   Not   enjoying   the   presumption   of   authority,  
  petitioner   Crescent   should   have   proved   that   Portserv   was   authorized   by   the  
Second.   Petitioner   Crescent   did   not   show   any   proof   that   the   marine   products   shipowner  to  contract  for  supplies.  Petitioner  failed.  
were  necessary  for  the  continuation  of  the  vessel.    
  49.  EDI-­‐STAFF  BUILDERS  INTERNATIONAL  INC.  V.  NLRC  
Third.   It   was   not   established   that   credit   was   extended   to   the   vessel.   It   is   G.R.  No.  145587  |  October  26,  2007  
presumed   that   "in   the   absence   of   fraud   or   collusion,   where   advances   are    
made  to   a   captain  in   a   foreign   port,   upon   his   request,   to   pay   for   necessary   DOCTRINE:   In   international   law,   the   party   who   wants   to   have   a   foreign   law  
repairs   or   supplies   to   enable   his   vessel   to   prosecute   her   voyage,   or   to   pay   applied   to   a   dispute   or   case   has   the   burden   of   proving   the   foreign   law.   The  
harbor   dues,   or   for   pilotage,   towage   and   like   services   rendered   to   the   vessel,   foreign  law  is  treated  as  a  question  of  fact  to  be  properly  pleaded  and  proved  as  
that   they   are   made   upon   the   credit   of   the   vessel   as   well   as   upon   that   of   her   the   judge   or   labor   arbiter   cannot   take   judicial   notice   of   a   foreign   law.   He   is  
owners."   In  this  case,  it  was  the  sub-­‐charterer  Portserv  which  requested  for  the   presumed  to  know  only  domestic  or  forum  law.  
delivery   of   the   bunker   fuels.   The   issuance   of   two   checks   amounting   to    
US$300,000  in  favor  of  petitioner  Crescent  prior  to  the  delivery  of  the  bunkers   GR:  Stipulation  that  Saudi  law  would  apply  to  an  agreement  is  valid.  
as   security   for   the   payment   of   the   obligation   weakens   petitioner   Crescent’s   BUT   SEE:   Stipulations   cannot   render   illusory   Phil.   Laws   and   regulations   on   labor  
contention  that  credit  was  extended  to  the  Vessel.   because  it  is  imbued  with  public  interest  (see  Pakistan  v  Ople)  
   
We   also   note   that   when   copies   of   the   charter   parties   were   submitted   by   FACTS:  
respondents  in  the  Court  of  Appeals,  the  time  charters  between  respondent  SCI   Petitioner   EDI-­‐Staff   builders   International,   Inc.   is   a   corporation   engaged   in  
and   Halla   and   between   Halla   and   Transmar   were   shown   to   contain   a   clause   recruitment   and   placement   of   OFWs.   Expertise   Search   International   (ESI)   is  
which  states  that  "the  Charterers  shall  provide  and  pay  for  all  the  fuel  except  as   another   recruitment   agency,   which   collaborated   with   EDI   to   process   the  
otherwise   agreed."   This   militates   against   petitioner   Crescent’s   position   that   documentation   and   deployment   of   private   respondent   Eleazar   Gran   to   Saudi  
Portserv  is  authorized  by  the  shipowner  to  contract  for  supplies  upon  the  credit   Arabia.   Gran   was   recruited   by   EDI,   and   deployed   by   ESI   to   work   for   Omar  
of  the  vessel.   Ahmed  Ali  Bin  Bechr  Est.  (OAB),  in  Riyadh,  KSA.  
   
Fourth.  There  was  no  proof  of  necessity  of  credit.  A  necessity  of  credit  will  be   It  appears  that  OAB  asked  EDI  for  curricula  vitae  of  qualified  applicants  for  the  
presumed   where   it   appears   that   the   repairs   and   supplies   were   necessary   for   position   of   "Computer   Specialist."   In   a   facsimile   transmission,   OAB   informed  
the   ship   and   that   they   were   ordered   by   the   master.   This   presumption   does   not   EDI   that   it   selected   Gran   for   the   position   of   "Computer   Specialist”   and   that   if  
arise   in   this   case   since   the   fuels   were   not   ordered   by   the   master   and   there   was   the  latter  agrees  to  the  terms  and  conditions  of  employment,  which  included:  
no  proof  of  necessity  for  the  supplies.  
 
CONFLICT  OF  LAWS                                                                                    AV  DE  TORRES   76  
ATTY.  ARIS  L.  GULAPA                                            AY  2015-­‐2016  
(1)   a   monthly   salary   of   SR   (Saudi   Riyal)   2,250.00   (USD   600.00),   EDI   may   arrange   the   OFW,   the   local   recruiter/agent,   and   the   foreign   employer/principal   are  
for   Gran's   immediate   dispatch;   and   that   (2)   Saudi   Labor   Laws   will   govern   governed   by   the   employment   contract.   A   contract   freely   entered   into   is  
matters   not   provided   for   in   the   contract   (e.g.   specific   causes   for   termination,   considered   law   between   the   parties;   and   hence,   should   be   respected.   In  
termination  procedures,  etc.);  among  others.   formulating   the   contract,   the   parties   may   establish   such   stipulations,   clauses,  
  terms   and   conditions   as   they   may   deem   convenient,   provided   they   are   not  
After   accepting   OAB's   offer   of   employment,   Gran   signed   an   employment   contrary  to  law,  morals,  good  customs,  public  order,  or  public  policy.  
contract   that   granted   him   a   monthly   salary   of   USD   850.00   for   a   period   of   2    
years.  Upon  arrival  in  Riyadh,  however,  Gran  questioned  the  discrepancy  in  his   In   the   present   case,   Saudi   Labor   Laws,   being   the   law   intended   by   the   parties  
monthly   salary—his   employment   contract   stated   USD   850.00,   while   his   (lex   loci   intentiones)   to   apply   to   the   contract,   these   should   govern   all   matters  
Philippine   Overseas   Employment   Agency   (POEA)   Information   Sheet   indicated   relating  to  the  termination  of  the  employment  of  Gran.  
USD  600.00  only.  However,  through  the  assistance  of  the  EDI  office  in  Riyadh,    
OAB  agreed  to  pay  Gran  USD  850.00.   In  international  law,  the  party  who  wants  to  have  a  foreign  law  applied  to  a  
  dispute  or  case  has  the  burden  of  proving  the  foreign  law.  The  foreign  law  is  
After  Gran  had  been  working  for  about  five  months  for  OAB,  his  employment   treated  as  a  question  of  fact  to  be  properly  pleaded  and  proved  as  the  judge  
was   terminated   on   the   following   grounds:   1.   Non-­‐compliance   to   contract   or  labor  arbiter  cannot  take  judicial  notice  of  a  foreign  law.  He  is  presumed  to  
requirements   by   the   recruitment   agency   primarily   on   salary   and   contract   know  only  domestic  or  forum  law.  
duration.  2.   Non-­‐compliance   to   pre-­‐qualification   requirements   by   the    
recruitment   agency.   3.   Insubordination   or   disobedience   to   Top   Management   Unfortunately   for   EDI,   it   did   not   prove   the   pertinent   Saudi   laws   on   the  
Order  and/or  instructions.   matter;   thus,   the   International   Law   doctrine   of   presumed-­‐identity   approach  
  or   processual   presumption   comes   into   play.   Where   a   foreign   law   is   not  
Gran  received  from  OAB  the  total  amount  of  SR  2,948.00  representing  his  final   pleaded   or,   even   if   pleaded,   is   not   proved,   the   presumption   is   that   foreign  
pay,  and  on  the  same  day,  he  executed  a  Declaration  releasing  OAB  from  any   law  is  the  same  as  ours.  Thus,  we  apply  Philippine  labor  laws  in  determining  
financial   obligation   or   otherwise,   towards   him.   However,   after   his   arrival   in   the   the  issues  presented  before  us.  
Philippines,  Gran  instituted  a  complaint  against  ESI/EDI,  OAB,  Country  Bankers    
Insurance   Corporation,   and   Western   Guaranty   Corporation   with   the   NLRC   In   illegal   dismissal   cases,   it   has   been   established   by   Philippine   law   and  
Quezon  City  for  underpayment  of  wages/salaries  and  illegal  dismissal.   jurisprudence  that  the  employer  should  prove  that  the  dismissal  of  employees  
  or  personnel  is  legal  and  just.  Section  33  of  Art.  277  (b)  of  the  Labor  Code  states  
The   Labor   Arbiter   ruled   that   there   was   neither   underpayment   nor   illegal   that:  
dismissal.   The   NLRC,   however,   reversed   the   Labor   Arbiter's   Decision.   Upon   a    
petition  for  certiorari,  the  CA  affirmed  the  decision  of  the  NLRC.   (b)   Subject   to   the   constitutional   right   of   workers   to   security   of   tenure  
  and   their   right   to   be   protected   against   dismissal   except   for   a   just   and  
ISSUES:   authorized   cause   and   without   prejudice   to   the   requirement   of   notice  
1.   Whether   EDI   has   established   by   way   of   substantial   evidence   that   Gran's   under  Art.  283  of  this  Code,  the  employer  shall  furnish  the  worker  whose  
termination   was   justifiable   by   reason   of   incompetence,   insubordination,   and   employment   is   sought   to   be   terminated   a   written   notice   containing   a  
disobedience.  Corollary  hereto,  whether  the  Prieto  vs.  NLRC  ruling,  as  applied   statement  of  the  causes  for  termination  and  shall  afford  the  latter  ample  
by  the  CA,  is  applicable  in  the  instant  case   opportunity   to   be   heard   and   to   defend   himself   with   the   assistance   of   his  
2.  Whether  Gran  was  afforded  due  process  prior  to  termination   representative   if   he   so   desires   in   accordance   with   company   rules   and  
  regulations   promulgated   pursuant   to   guidelines   set   by   the   Department  
HELD:   of  Labor  and  Employment.  Any  decision  taken  by  the  employer  shall  be  
1.  No.  In  cases  involving  OFWs,  the  rights  and  obligations  among  and  between   without   prejudice   to   the   right   of   the   workers   to   contest   the   validity   or  
 
CONFLICT  OF  LAWS                                                                                    AV  DE  TORRES   77  
ATTY.  ARIS  L.  GULAPA                                            AY  2015-­‐2016  
legality  of  his  dismissal  by  filing  a  complaint  with  the  regional  branch  of   have   provided   a   copy   of   the   company   policy,   Gran's   job   description,   or   any  
the  NLRC.  The  burden  of  proving  that  the  termination  was  for  a  valid  or   other   document   that   would   show   that   the   "Daily   Activity   Reports"   were  
authorized  cause  shall  rest  on  the  employer.  x  x  x   required   for   submission   by   the   employees,   more   particularly   by   a   Computer  
  Specialist.  
In  many  cases,  it  has  been  held  that  in  termination  disputes  or  illegal  dismissal    
cases,   the   employer   has   the   burden   of   proving   that   the   dismissal   is   for   just   and   Even   though   EDI   and/or   ESI   were   merely   the   local   employment   or  
valid  causes;  and   failure   to   do   so   would   necessarily   mean   that   the   dismissal   recruitment   agencies   and   not   the   foreign   employer,   they   should   have  
was  not  justified  and  therefore  illegal.  Taking  into  account  the  character  of  the   adduced   additional   evidence   to   convincingly   show   that   Gran's   employment  
charges   and   the   penalty   meted   to   an   employee,   the   employer   is   bound   to   was   validly   and   legally   terminated.   The   burden   devolves   not   only   upon   the  
adduce   clear,   accurate,   consistent,   and   convincing   evidence   to   prove   that   the   foreign-­‐based   employer   but   also   on   the   employment   or   recruitment   agency  
dismissal  is  valid  and  legal.   for   the   latter   is   not   only   an   agent   of   the   former,   but   is   also   solidarily   liable  
  with   the   foreign   principal   for   any   claims   or   liabilities   arising   from   the  
In   the   instant   case,   EDI   submitted   two   letters   as   evidence.   The   first   is   the   dismissal   of   the   worker.   Thus,   EDI   failed   to   prove   that   Gran   was   justifiably  
termination   letter,   addressed   to   Gran,   from   Andrea   E.   Nicolaou,   Managing   dismissed  due  to  incompetence,  insubordination,  or  willful  disobedience.  
Director  of  OAB.  The  second  is  an  unsigned  letter  from  OAB  addressed  to  EDI    
and   ESI,   which   outlined   the   reasons   why   OAB   had   terminated   Gran's   PRIETO  RULING  
employment.  EDI  claims  that  Gran  was  incompetent  for  the  Computer  Specialist   In  Prieto,  this  Court  ruled  that  "[i]t  is  presumed  that  before  their  deployment,  
position   because   he   had   "insufficient   knowledge   in   programming   and   zero   the  petitioners  were  subjected  to  trade  tests  required  by  law  to  be  conducted  
knowledge   of   [the]   ACAD   system."   Gran   was   justifiably   dismissed   due   to   by   the   recruiting   agency   to   insure   employment   of   only   technically   qualified  
insubordination   or   disobedience   because   he   continually   failed   to   submit   the   workers   for   the   foreign   principal."   The   CA,   using   the   ruling   in   the   said   case,  
required   "Daily   Activity   Reports."   However,   other   than   the   abovementioned   ruled   that   Gran   must   have   passed   the   test;   otherwise,   he   would   not   have   been  
letters,   no   other   evidence   was   presented   to   show   how   and   why   Gran   was   hired.   Therefore,   EDI   was   at   fault   when   it   deployed   Gran   who   was   allegedly  
considered   incompetent,   insubordinate,   or   disobedient.   EDI   had   clearly   failed   "incompetent"  for  the  job.  
to  overcome  the  burden  of  proving  that  Gran  was  validly  dismissed.    
  The  CA  is  correct  in  applying  Prieto  although  Gran  misrepresented  himself  in  his  
An  allegation  of  incompetence  should  have  a  factual  foundation.  Incompetence   CV  as  a  Computer  Specialist.  The  purpose  of  the  required  trade  test  is  to  weed  
may   be   shown   by   weighing   it   against   a   standard,   benchmark,   or   criterion.   out  incompetent  applicants  from  the  pool  of  available  workers.  It  is  supposed  
However,  EDI  failed  to  establish  any  such  bases.   to   reveal   applicants   with   false   educational   backgrounds,   and   expose   bogus  
  qualifications.  Since  EDI  deployed  Gran  to  Riyadh,  it  can  be  presumed  that  Gran  
With   regard   to   insubordination   and   disobedience,   the   elements   that   must   had  passed  the  required  trade  test  and  that  Gran  is  qualified  for  the  job.  Even  if  
concur   for   the   charge   of   insubordination   or   willful   disobedience   to   prosper   there   was   no   objective   trade   test   done   by   EDI,   it   was   still   EDI's   responsibility   to  
were   not   present.   It   was   held   in   Micro   Sales   Operation   Network   v.   NLRC:   For   subject  Gran  to  a  trade  test;  and  its  failure  to  do  so  only  weakened  its  position  
willful   disobedience   to   be   a   valid   cause   for   dismissal,   the   following   twin   but   should   not   in   any   way   prejudice   Gran.   In   any   case,   the   issue   is   rendered  
elements   must   concur:   (1)   the   employee's   assailed   conduct   must   have   been   moot  and  academic  because  Gran's  incompetency  is  unproved.  
willful,   that   is,   characterized   by   a   wrongful   and   perverse   attitude;   and   (2)   the    
order   violated   must   have   been   reasonable,   lawful,   made   known   to   the   2.   No.   As   discussed   earlier,   in   the   absence   of   proof   of   Saudi   laws,   Philippine  
employee   and   must   pertain   to   the   duties   which   he   had   been   engaged   to   Labor  laws  and  regulations  shall  govern  the  relationship  between  Gran  and  EDI.  
discharge.  In  the  case  at  bar,  EDI  failed  to  show  that  the  order  of  the  company   Thus,   our   laws   and   rules   on   the   requisites   of   due   process   relating   to  
which   was   violated—the   submission   of   "Daily   Activity   Reports"—was   part   of   termination  of  employment  shall  apply.  Under  the  twin  notice  requirement,  the  
Gran's   duties   as   a   Computer   Specialist.   Before   the   Labor   Arbiter,   EDI   should   employees   must   be   given   two   (2)   notices   before   their   employment   could   be  
 
CONFLICT  OF  LAWS                                                                                    AV  DE  TORRES   78  
ATTY.  ARIS  L.  GULAPA                                            AY  2015-­‐2016  
terminated:  (1)  a  first  notice  to  apprise  the  employees  of  their  fault,  and  (2)  a   either  for  their  reciprocal  benefit  or  for  the  benefit  of  a  third  person."  
second   notice   to   communicate   to   the   employees   that   their   employment   is    
being   terminated.   In   between   the   first   and   second   notice,   the   employees   The  Court  agreed  with  Tirso’s  view,  supported  by  eminent  commentators,  that  
should  be  given  a  hearing  or  opportunity  to  defend  themselves  personally  or  by   the   prohibition   of   Art.   669   is   directed   against   the   execution   of   a   joint   will,   or  
counsel  of  their  choice.   the   expression   by   two   or   more   testators   of   their   wills   in   a   single   document   and  
  by   one   act,   rather   than   against   mutual   or   reciprocal   wills,   which   may   be  
OAB's   manner   of   dismissing   Gran   fell   short   of   the   two   notice   requirement.   separately   executed.   Upon   this   premise,   however,   Tirso   argues   that   Art.   669  
While   it   furnished   Gran   the   written   notice   informing   him   of   his   dismissal,   it   has  been  repealed  by  Act.  No.  190  (Code  of  CivPro),  which  he  claims  provides  
failed   to   furnish   Gran   the   written   notice   apprising   him   of   the   charges   against   for  and  regulates  the  extrinsic  formalities  of  wills,  contending  that  whether  two  
him,   as   prescribed   by   the   Labor   Code.  Consequently,   he   was   denied   the   wills   should   be   executed   conjointly   or   separately   is   but   a   matter   of   extrinsic  
opportunity   to   respond   to   said   notice.   In   addition,   OAB   did   not   schedule   a   formality.  
hearing   or   conference   with   Gran   to   defend   himself   and   adduce   evidence   in    
support  of  his  defenses.   ISSUE:  
  Whether   Art.   669   of   the   Civil   Code   was   repealed   by   Act   No.   190   (Code   of  
  CivPro)  
 
EXCEPTIONS  TO  THE  APPLICATION  OF  FOREIGN  LAW  
  HELD:  
  No.   The   SC   applied   its   ruling   in   In   re:   Will   of   Bilbao,   wherein   it   stated   that  
50.  DACANAY  V.  FLORENDO   several  articles  of  the  Civil  Code  regarding  wills  have  not  only  been  referred  to  
G.R.  No.  L-­‐2071  |  September  19,  1950   but   have   also   been   applied   side   by   side   with   the   provisions   of   the   Code   of  
  CivPro.  The  provision  of  Art.  669  is  not  unwise  and  is  not  against  public  policy.  
Contrary  to  public  policy   The  reason  for  this  is  that  when  a  will  is  made  jointly  or  in  the  same  instrument,  
  the  spouse  who  is  more  aggressive,  stronger  in  will  or  character  and  dominant  
N.B.:  What  provision  is  relevant  today?   is   liable  to  dictate  the  terms  of  the  will  for  his  or  her  own  benefit  or  for  that  of  
Article  819.  Wills,  prohibited  by  the  preceding  article,  executed  by  Filipinos  in  a   third  persons  whom  he  or  she  desires  to  favor.  And,  where  the  will  is  not  only  
foreign  country  shall  not  be  valid  in  the  Philippines,  even  though  authorized  by   joint   but   reciprocal,   either   one   of   the   spouses   who   may   happen   to   be  
the  laws  of  the  country  where  they  may  have  been  executed.  (733a)     unscrupulous,   wicked,   faithless   or   desperate,   knowing   as   he   or   she   does   the  
  terms  of  the  will  whereby  the  whole  property  of  the  spouses  both  conjugal  and  
FACTS:   paraphernal   goes   to   the   survivor,   may   be   tempted   to   kill   or   dispose   of   the  
A   joint   and   reciprocal   will   was   executed   by   the   spouses   Isabel   Florendo   and   other.  
Tirso  Dacanay  in  1940.  After  the  death  of  Isabel,  Tirso  now  seek  to  probate  said    
will   in   CFI   La   Union,   with   the   will   providing   in   substance   that   whoever   of   the   Considering   the   wisdom   of   the   provisions   of   this   Art.   669   and   the   fact   that   it  
spouses,  joint  testators,  shall  survive  the  other,  shall  inherit  all  the  properties   has  not  been  repealed,  at  least  not  expressly,  as  well  as  the  consideration  that  
of  the  latter,  with  an  agreement  as  to  how  the  surviving  spouse  shall  dispose  of   its   provisions   are   not   incompatible   with   those   of   the   Code   of   CivPro   on   the  
the  properties  in  case  of  his  or  her  demise.   subject   of   wills,   said   Art.   669   of   the   Civil   Code   is   still   in   force.   Further,   this  
  article  has  been  reproduced  word  for  word  in  Art.  818  of  the  New  Civil  Code.  
The   relatives   of   the   deceased   Isabel   opposed   the   probate   of   said   will   on   the   The   implication   is   that   the   Philippine   Legislature   that   passed   this   Act   and  
ground  that  it  is  null  and  void  for  being  in  violation  of  Art.  669  of  the  Civil  Code:   approved   the   NCC,   including   the   members   of   the   Code   Commission   who  
"Two  or  more  persons  cannot  make  a  will  conjointly  or  in  the  same  instrument,   prepared   it,   are   of   the   opinion   that   the   provisions   of   Art.   669   are   not  
incompatible  with  those  of  the  Code  of  Civil  Procedure.  
 
CONFLICT  OF  LAWS                                                                                    AV  DE  TORRES   79  
ATTY.  ARIS  L.  GULAPA                                            AY  2015-­‐2016  
  2.   Whether   private   respondents   are   natural   children   of   Sy   Kiat   with   Asuncion  
51.  YAO  KEE,  ET  AL.  V.  SY-­‐GONZALES   Gillego  
G.R.  No.  L-­‐55960  |  November  24,  1988    
  HELD:  
DOCTRINE:   To   establish   a   valid   foreign   marriage   two   things   must   be   proven,   1.  No.  To  buttress  petitioners’  argument  that  the  marriage  of  Sy  Kiat  to  Yao  Kee  
namely:   (1)   the   existence   of   the   foreign   law   as   a   question   of   fact;   and   (2)   the   in   accordance   with   Chinese   law   and   custom   was   conclusively   proven,   they  
alleged  foreign  marriage  by  convincing  evidence.   relied  on  the  following  testimonial  and  documentary  evidence.  
   
FACTS:   (1)   the   testimony   of   Yao   Kee:   She   testified   that   she   was   married   to   Sy  
Sy   Kiat,   a   Chinese   national,   died   on   January   17,   1977   in   Caloocan   City   where   he   Kiat  in  1931  in  Fookien,  China.  She  does  not  have  a  marriage  certificate  
was   then   residing,   leaving   behind   real   and   personal   properties   here   in   the   because  the  practice  during  that  time  was  for  elders  to  agree  upon  the  
Philippines  worth  P300,000.00  more  or  less.   betrothal   of   their   children,   and   in   her   case,   her   elder   brother   was   the  
  one  who  contracted  or  entered  into  [an]  agreement  with  the  parents  of  
Thereafter,  private  respondents  (all  surnamed  Sy)  filed  a  petition  for  the  grant   her  husband.  During  the  wedding  the  document  would  be  signed  by  the  
of   letters   of   administration   of   the   then   CFI   of   Rizal,   Caloocan   City.   In   said   parents  of  the  groom  as  well  as  by  the  parents  of  the  bride  and  there  is  
petition   they   alleged   among   others   that   (a)   they   are   the   children   of   the   no   solemnizing   officer   as   is   known   in   the   Philippines.   The   parties  
deceased   with   Asuncion   Gillego;   and   (b)   to   their   knowledge   Sy   Kiat   died   themselves   do   not   sign   the   document.   As   to   the   whereabouts   of   this  
intestate;  among  others.   document,   she   and   Sy   Kiat   were   married   for   46   years   already   and   the  
  document   was   left   in   China   and   she   doubt   if   that   document   can   still   be  
The   petition   was   opposed   by   Petitioners   Yao   Kee,   Sze   Sook   Wah,   Sze   Lai   Cho   found  now.  It  was  left  in  the  possession  of  Sy  Kiat's  family.  Right  now,  she  
and   Sy   Yun   Chen   who   alleged   that:   (a)   Yao   Kee   is   the   lawful   wife   of   Sy   Kiat,   does  not  know  the  whereabouts  of  that  document  because  of  the  lapse  
whom   he   married   in   China;   (b)   the   other   oppositors   are   the   legitimate   children   of   many   years   and   because   they   left   it   in   a   certain   place   and   it   was  
of   the   deceased   with   Yao   Kee;   and,   (c)   Sze   Sook   Wah   is   the   eldest   among   them   already  eaten  by  the  termites.    
and   is   competent,   willing   and   desirous   to   become   the   administratrix   of   the   (2)  The  testimony  of  Gan  Ching,  a  younger  brother  of  Yao  Kee  who  stated  
estate  of  Sy  Kiat.   that   he   was   among   the   many   people   who   attended   the   wedding   of   his  
  sister   with   Sy   Kiat   and   that   no   marriage   certificate   is   issued   by   the  
After  hearing,  the  probate  court  ruled  in  favor  of  the  petitioners  and  found  that   Chinese  government,  a  document  signed  by  the  parents  or  elders  of  the  
(1)   Sy   Kiat   was   legally   married   to   Yao   Kee;   (2)   Petitioners   are   the   legitimate   parties  being  sufficient.  
children   of   Yao   Kee   with   Sy   Kiat;   and   (3)   private   respondents   are   the   (3)   The   statements   made   by   Asuncion   Gillego   to   the   effect   that   Sy   Kiat  
acknowledged   illegitimate   offsprings   of   Sy   Kiat   with   Asuncion   Gillego.   On   was   married   to   Yao   Kee   according   to   Chinese   custom;   and   Sy   Kiat's  
appeal   the   CA   ruled   that   Sy   Kiat   is   an   unmarried   man   who   was   living   with   admission   to   her   that   he   has   a   Chinese   wife   whom   he   married   according  
Asuncion  Gillego  without  the  benefit  of  marriage  for  many  years  and  that  the   to  Chinese  custom.  
legality   of   the   alleged   marriage   of   Sy   Kiat   to   Yao   Kee   in   China   had   not   been   (4)   Sy   Kiat's   Master   Card   of   Registered   Alien   issued   in   Caloocan   City   on  
proven  to  be  valid  to  the  laws  of  the  Chinese  People's  Republic  of  China.   October  3,  1972  where  the  following  entries  are  found:  "Marital  status—
  Married";  "If  married  give  name  of  spouses—Yao  Kee";  "Address-­‐China;  
ISSUES:   "Date  of  marriage—1931";  and  "Place  of  marriage—China"  
1.  Whether  the  marriage  of  Sy  Kiat  to  Yao  Kee  was  proven  valid  in  accordance   (5)   Sy   Kiat's   Alien   Certificate   of   Registration   issued   in   Manila   stating:  
with  laws  of  the  People's  Republic  of  China   "Civil   status—Married";   and,   'If   married,   state   name   and   address   of  
spouse—Yao  Kee  Chingkang,  China".  

 
CONFLICT  OF  LAWS                                                                                    AV  DE  TORRES   80  
ATTY.  ARIS  L.  GULAPA                                            AY  2015-­‐2016  
(6)   The   certification   issued   in   Manila   by   the   Embassy   of   the   People's    
Republic   of   China   to   the   effect   that   "according   to   the   information   Proof  of  a  written   foreign   law,  on  the  other  hand,  is  provided  for  under  Rule  
available  at  the  Embassy  Mr.  Sy  Kiat  a  Chinese  national  and  Mrs.  Yao  Kee   132  section  25,  thus:  
alias   Yui   Yip   also   Chinese   were   married   on   January   19,   1931   in   Fukien,    
the  People's  Republic  of  China.   SEC.  25.  Proof  of  public  or  official  record.—An  official  record  or  an  entry  
  therein,   when   admissible   for   any   purpose,   may   be   evidenced   by   an  
These   evidence   may   very   well   prove   the  fact   of   marriage  between   Yao   Kee   and   official  publication  thereof  or  by  a  copy  attested  by  the  officer  having  the  
Sy   Kiat.   However,   the   same   do   not   suffice   to   establish   the   validity   of   said   legal   custody   of   the   record,   or   by   his   deputy,   and   accompanied,   if   the  
marriage  in  accordance  with  Chinese  law  or  custom.   record   is   not   kept   in   the   Philippines,   with   a   certificate   that   such   officer  
  has   the   custody.   If   the   office   in   which   the   record   is   kept   is   in   a   foreign  
Custom   is   defined   as   "a   rule   of   conduct   formed   by   repetition   of   acts,   country,   the   certificate   may   be   made   by   a   secretary   of   embassy   or  
uniformly   observed   (practiced)   as   a   social   rule,   legally   binding   and   legation,  consul  general,  consul,  vice  consul,  or  consular  agent  or  by  any  
obligatory".   The   law   requires   that   "a   custom   must   be   proved   as   a   fact,   officer   in   the   foreign   service   of   the   Philippines   stationed   in   the   foreign  
according  to  the  rules  of  evidence"  [Art.  12,  NCC.]  On  this  score  the  Court  had   country  in  which  the  record  is  kept  and  authenticated  by  the  seal  of  his  
occasion   to   state   that   "a   local   custom   as   a   source   of   right   can   not   be   office.  
considered   by   a   court   of   justice   unless   such   custom   is   properly   established   by    
competent   evidence   like   any   other   fact.”  The   same   evidence,   if   not   one   of   a   The   Court   has   interpreted   §25   to   include   competent   evidence   like   the  
higher  degree,  should  be  required  of  a  foreign  custom.   testimony  of  a  witness  to  prove  the  existence  of  a  written  foreign  law.    
   
The   law   on   foreign   marriages   is   provided   by   Art.   71   of   the   Civil   Code   which   In  the  case  at  bar  petitioners  did  not  present  any  competent  evidence  relative  
states   that:   Art.   71.   All   marriages   performed   outside   the   Philippines   in   to  the  law  and  custom  of  China  on  marriage.  The  testimonies  of  Yao  and  Gan  
accordance   with   the   laws   in   force   in   the   country   where   they   were   performed   Ching  cannot  be  considered  as  proof  of  China's  law  or  custom  on  marriage  not  
and   valid   there   as   such,   shall   also   be   valid   in   this   country,   except   bigamous,   only  because  they  are  self-­‐serving  evidence,  but  more  importantly,  there  is  no  
polygamous,   or   incestuous   marriages,   as   determined   by   Philippine   law.   [Now,   showing  that  they  are  competent  to  testify  on  the  subject  matter.   For   failure  
Art.  26,  FC]   to   prove   the   foreign   law   or   custom,   and   consequently,   the   validity   of   the  
  marriage  in  accordance  with  said  law  or  custom,  the  marriage  between  Yao  Kee  
Construing   this   provision   of   law   the   Court   has   held   that   to   establish   a   valid   and  Sy  Kiat  cannot  be  recognized  in  this  jurisdiction.  
foreign  marriage  two  things  must  be  proven,  namely:  (1)  the  existence  of  the    
foreign   law   as   a   question   of   fact;   and   (2)   the   alleged   foreign   marriage   by   Petitioners   cited   the   case   of   Sy   Joc   Lieng   v.   Sy   Quia,   where   the   party   alleging  
convincing  evidence.   the   foreign   marraige   were   not   duty   bound   to   prove   the   Chinese   law   on  
  marriage   as   judicial   notice   thereof   had   been   taken   by   the   Court.   This  
EXISTENCE  OF  THE  FOREIGN  LAW   contention   is   erroneous.   Well-­‐established   in   this   jurisdiction   is   the   principle  
In  proving  a  foreign  law  the  procedure  is  provided  in  the  Rules  of  Court.  With   that   Philippine   courts   cannot   take   judicial   notice   of   foreign   laws.   They   must  
respect  to  an  unwritten  foreign  law,  Rule  130,  §45  states  that:   be   alleged   and   proved   as   any   other   fact.   Moreover   a   reading   of   said   case  
  would   show   that   the   party   alleging   the   foreign   marriage   presented   a   witness,  
SEC.  45.  Unwritten  law.—The  oral  testimony  of  witnesses,  skilled  therein,   one  Li  Ung  Bieng,  to  prove  that  matrimonial  letters  mutually  exchanged  by  the  
is   admissible   as   evidence   of   the   unwritten   law   of   a   foreign   country,   as   contracting   parties   constitute   the   essential   requisite   for   a   marriage   to   be  
are  also  printed  and  published  books  of  reports  of  decisions  of  the  courts   considered   duly   solemnized   in   China.   His   testimony   was   uniformly  
of   the   foreign   country,   if   proved   to   be   commonly   admitted   in   such   corroborated  by  authors  on  the  subject  of  Chinese  marriage.  
courts.    
 
CONFLICT  OF  LAWS                                                                                    AV  DE  TORRES   81  
ATTY.  ARIS  L.  GULAPA                                            AY  2015-­‐2016  
Further,  assuming  arguendo  that  the  Court  has  indeed  taken  judicial  notice  of   52.  BANK  OF  AMERICA  V.  AMERICAN  REALTY  CORPORATION  
the  law  of  China  on  marriage  in  the  aforecited  case,  petitioners  however  have   G.R.  No.  133876  |  December  29,  1999  
not   shown   any   proof   that   the   Chinese   law   or   custom   obtaining   at   the   time    
the  Sy  Joc  Lieng  marriage  was  celebrated  in  1847  was  still  the  law  when  the   FACTS:  
alleged  marriage  of  Sy  Kiat  to  Yao  Kee  took  place  in  1931  or  eighty-­‐four  (84)   Bank  of  America  NT  &  SA  (BANTSA)  is  an  international  banking  institution  duly  
years  later.   licensed  to  do  business  in  the  Philippines,  organized  under  the  laws  of  the  State  
  of  California.  American  Realty  (ARC)  is  a  domestic  corporation.  Bank  of  America  
EXISTENCE  OF  THE  FOREIGN  MARRIAGE   International  Limited  (BAIL)  is  a  limited  liability  company  organized  and  existing  
Petitioners   moreover   cite   the   case   of  U.S.   v.   Memoracion,   where   the   testimony   under  the  laws  of  England.  
of   one   of   the   contracting   parties   is   competent   evidence   to   show   the   fact   of    
marriage.  This  case  however  is  not  applicable  to  the  case  at  bar  as  said  case  did   BANTSA   and   BAIL   granted   three   multimillion   USD   loans   to   1.   Liberian   Transport  
not   concern   a   foreign   marriage   and   the   issue   posed   was   whether   or   not   the   2.  El  Challenger  and  3.  Eshley  Compania  (herein  referred  to  as  borrowers).  All  
oral   testimony   of   a   spouse   is   competent   evidence   to   prove   the  fact   of   are  existing  under  the  laws  of  Panama,  and  are  foreign  affiliates  of  ARC.  
marriage  in  a  complaint  for  adultery.    
  BANTSA   and   the   borrowers   signed   restructuring   agreements   due   to   their  
rd  
Accordingly,  in  the  absence  of  proof  of  the  Chinese  law  on  marriage,  it  should   default.   As   additional   security,   ARC,   as   3 party   mortgagor,   executed   2   real  
be   presumed   that   it   is   the   same   as   ours.   Since   Yao   Kee   admitted   in   her   estate   mortgages   over   land   parcels   in   San   Jose   Del   Monte,   Bulacan.   Eventually,  
testimony   that   there   was   no   solemnizing   officer   as   is   known   here   in   the   the  borrowers  defaulted  in  the  restructured  loans,  prompting  BANTSA  to  file  4  
Philippines   [See   Art.   56,   Civil   Code]   when   her   alleged   marriage   to   Sy   Kiat   was   civil  actions  before  foreign  courts  for  the  collection:  2  in  England’s  High  Court  
celebrated,   it   therefore   follows   that   her   marriage   to   Sy   Kiat,   even   if   true,   and  2  in  the  Supreme  Court  of  Hongkong  High  Court.  
cannot  be  recognized  in  this  jurisdiction.    
rd
  In   the   cases   before   foreign   courts,   ARC   as   the   3   party   mortgagor   wasn’t  
2.   Yes.   Private   respondents   on   the   other   hand   are   also   the   deceased's   impleaded   as   party-­‐defendant.   In   1992,   BANTSA   filed   an   application   for  
acknowledged  natural  children  with  Asuncion  Gillego.  They  have  in  their  favor   extrajudicial   foreclosure   of   REM   before   the   Office   of   the   Provincial   Sheriff   of  
their   father's   acknowledgment,   evidenced   by   a   compromise   agreement   Bulacan.  After  publication  and  due  notice,  the  mortgaged  real  properties  were  
entered  into  by  and  between  their  parents  and  approved  by  the  CFI,  wherein  Sy   sold,   with   Integrated   Credit   &   Corporation   Services   (ICCS)   as   the   highest   bidder  
Kiat  not  only  acknowleged  them  as  his  children  by  Asuncion  Gillego  but  likewise   for  24m.  
made  provisions  for  their  support  and  future  inheritance.    
  ARC  then  filed  before  the  Pasig  RTC  an  action  for  damages  for  BANTSA’s  act  of  
As  regards  petitioners,  they  failed  to  establish  the  marriage  of  Yao  Kee  with  Sy   extrajudicially   foreclosing   the   mortgages   despite   the   pendency   of   civil   suits  
Kiat  according  to  the  laws  of  China.  Thus,  they  cannot  be  accorded  the  status  of   before   foreign   courts   for   the   collection   of   the   principal   loan.   BANTSA   alleged  
legitimate  children  but  only  that  of  acknowledged  natural  children.  Petitioners   that   the   rule   prohibiting   the   mortgagee   from   foreclosing   the   mortgage   after   an  
are  natural  children,  it  appearing  that  at  the  time  of  their  conception  Yao  Kee   ordinary   suit   for   collection   has   been   filed   is   not   applicable   here,   because:   1)  
rd  
and  Sy  Kiat  were  not  disqualified  by  any  impediment  to  marry  one  another  [See   ARC   is   a   3 party   mortgagor   and   not   a   party   in   the   principal   restructuring  
Art.  269,  Civil  Code.]   agreements,  and  not  made  as  a  defendant  in  HKG  and  England  2)  There  is  no  
  civil  suit  for  sum  of  money  filed  in  the  Philippines  since  they  were  filed  in  HKG  
and   England,   so   such   decisions   which   may   be   rendered   are   not   enforceable  
unless   there   is   a   separate   action   and   3)   Under   English   Law,   which   is   the  
governing   law   under   the   principal   agreements,   the   mortgagee   does   not   lose   its  
security  interest  by  filing  civil  actions  for  sums  of  money.  
 
CONFLICT  OF  LAWS                                                                                    AV  DE  TORRES   82  
ATTY.  ARIS  L.  GULAPA                                            AY  2015-­‐2016  
  Thus,  when  the  foreign  law,  judgment  or  contract  is  contrary  to  a  sound  and  
ARC  filed  a  motion  for  suspension  of  the  redemption  period  because  it  cannot   established   public   policy   of   the   forum,   the   said   foreign   law,   judgment   or  
exercise   the   right   without   waiving   its   contentions   in   the   case   that   the   order  shall  not  be  applied.  
foreclosure  of  the  mortgage  on  its  properties  is  legally  improper  and  therefore    
invalid.   Additionally,   prohibitive   laws   concerning   persons,   their   acts   or   property,   and  
  those  which  have  for  their  object  public  order,  public  policy  and  good  customs  
RTC  granted  the  motion  for  suspension.  RD  of  Meycauyan  received  the  order.   shall   not   be   rendered   ineffective   by   laws   or   judgments   promulgated,   or   by  
After   a   month,   ICCS   consolidated   its   ownership   over   the   real   properties,   determinations  or  conventions  agreed  upon  in  a  foreign  country.  (Art.  17,  NCC)  
resulting  in  the  issuance  of  TCT  in  its  name.  ICCS  then  sold  them  to  Stateland    
Investment  Corporation  for  39m,  and  TCTs  were  issued  to  its  name.   The   public   policy   sought   to   be   protected   in   the   instant   case   is   the   principle  
  imbedded   in   our   jurisdiction   proscribing   the   splitting   up   of   a   single   cause   of  
The  Pasig  RTC  rendered  a  decision  in  favor  of  ARC.  CA  affirmed.   action.  
   
ISSUE:   §4,  Rule  2  of  the  1997  Rules  of  Civil  Procedure  is  pertinent—If  two  or  more  suits  
Whether  BANTSA’s  act  of  filing  a  collection  suit  against  the  principal  debtors  for   are   instituted   on   the   basis   of   the   same   cause   of   action,   the   filing   of   one   or   a  
the  recovery  of  the  loan  before  the  foreign  courts  constituted  a  waiver  of  the   judgment  upon  the  merits  in  any  one  is  available  as  a  ground  for  the  dismissal  
remedy  of  foreclosure   of  the  others.  
   
HELD:   Moreover,  foreign  law  should  not  be  applied  when  its  application  would  work  
Yes.     undeniable  injustice  to  the  citizens  or  residents  of  the  forum.  To  give  justice  is  
  the   most   important   function   of   law;   hence,   a   law,   or   judgment   or   contract  
ENGLISH  LAW  NOT  APPLICABLE   that   is   obviously   unjust   negates   the   fundamental   principles   of   Conflict   of  
BANTSA   further   alleges   that   under   English   Law,   which   according   to   it   is   the   Laws.  
governing  law  with  regard  to  the  principal  agreements,  the  mortgagee  does  not    
lose   its   security   interest   by   simply   filing   civil   actions   for   sums   of   money.   The    
Court  disagreed,  ruling  that  English  Law  is  not  applicable.  
VI.  NATURE  OF  CONFLICTS  RULES,  THE  PROBLEM  OF  
 
In  the  case  at  bench,  Philippine  law  shall  apply  notwithstanding  the  evidence    CHARACTERIZATION,  AND  RENVOI  
 
presented   by   BANTSA   to   prove   the   English   law   on   the   matter.   There   is   no    
judicial  notice  of  any  foreign  law.  A  foreign  law  must  be  properly  pleaded  and   53.  IN  THE  MATTER  OF  THE  TESTATE  ESTATE  OF  EDWARD  E.  CHRISTENSEN  
proved   as   a   fact.  Thus,   if   the   foreign   law   involved   is   not   properly   pleaded   and   G.R.  No.  L-­‐16749  |  January  31,  1963  
proved,  our  courts  will  presume  that  the  foreign  law  is  the  same  as  our  local    
or   domestic   or   internal   law.  This   is   what   we   refer   to   as   the   doctrine   of   DOCTRINE:   There   are   two   types   of   renvoi.   (1)   The   type   of   renvoi   where   a   jural  
processual  presumption.   matter   is   presented   which   the   COL   rule   of   the   forum   refers   to   a   foreign   law,   the  
  conflict-­‐of-­‐laws  rule  of  which,  in  turn,  refers  the  matter  back  again  to  the  law  of  
Assuming  arguendo  that  the  English  Law  on  the  matter  were  properly  pleaded   the   forum.   (2)   The   doctrine   of   renvoi   which   postulates   that   the   court   of   the  
and   proved   in   accordance   with   §24,   Rule   132   of   the   Rules   of   Court   and   the   forum,  in  determining  the  question  before  it,  must  take  into  account  the  whole  
jurisprudence  laid  down  in  Yao  Kee,  et  al.  v.  Sy-­‐Gonzales,  said  foreign  law  would   law  of  the  other  jurisdiction,  but  also  its  rules  as  to  COL,  and  then  apply  the  law  
still  not  find  applicability.   to   the   actual   question   which   the   rules   of   the   other   jurisdiction   prescribe.  
 
 
CONFLICT  OF  LAWS                                                                                    AV  DE  TORRES   83  
ATTY.  ARIS  L.  GULAPA                                            AY  2015-­‐2016  
According  to  this  theory  'the  law  of  a  country'  means  the  whole  of  its  law.   that   at   the   time   of   his   death   he   was   domiciled   in   the   Philippines.   Hence,   the  
  law  that  governs  the  validity  of  his  testamentary  dispositions  is  defined  in  Art.  
FACTS:   16  of  the  Civil  Code  of  the  Philippines,  which  is  as  follows:  
Edward   E.   Christensen   executed   a   will   in   Manila   on   March   5,   1951.   He   stated    
therein   that   he   has   one   child,   Mary   Lucy   Christensen   (now   Mrs.   Bernard   ART.  16.  Real  property  as  well  as  personal  property  is  subject  to  the  law  
Daney),  a  Filipino  citizen  but  a  resident  of  Los  Angeles  and  that  he  had  no  other   of  the  country  where  it  is  situated.  
living  ascendants  and  descendants.  To  her,  he  devised  and  bequeathed  all  the    
income   from   the   remainder   of   his   property   and   estate,   real,   personal   and/or   However,   intestate   and   testamentary   successions,   both   with   respect   to  
mixed,   of   whatsoever   kind   or   character,   and   wheresoever   situated.   He   also   the  order  of  succession  and  to  the  amount  of  successional  rights  and  to  
stipulated  the  he  was  devising  and  bequeathing  to  Maria  Helen  Christensen  the   the   intrinsic   validity   of   testamentary   provisions,   shall   be   regulated   by   the  
sum   of   P3,600   and   paid   to   her   in   monthly   increments   of   P100.   Thus,   the   national   law   of   the   person   whose   succession   is   under   consideration,  
executor  finalized  the  account  and  project  of  partition  as  such.   whatever   may   be   the   nature   of   the   property   and   regardless   of   the  
  country  where  said  property  may  be  found.  
Maria  Helen  opposed  the  approval  of  the  project  of  partition  as  it  deprives  her    
of   her   legitime   as   an   acknowledged   natural   child,   she   having   been   declared   the   The   application   of   the   aforementioned   in   the   case   at   bar   requires   the  
court  in  as  such  in  a  previous  case.  The  legal  grounds  of  opposition  are  (a)  that   determination  of  the  meaning  of  the  term  "national  law"  as  used  therein.  There  
the   distribution   should   be   governed   by   the   laws   of   the   Philippines,   and   (b)   that   is   no   single   American   law   governing   the   validity   of   testamentary   provisions   in  
said   order   of   distribution   is   contrary   to   law   as   it   denies   her   one-­‐half   of   the   the   US   since   each   state   has   its   own   private   law   applicable   to   its   citizens   only  
estate.   Furthermore,   she   alleged   that   the   law   that   should   govern   Edward's   and   in   force   only   within   the   state.   The   "national   law"   indicated   in   Art.   16  
estate   should   not   be   the   internal   law   of   California   alone,   but   the   entire   law   cannot,   therefore,   possibly   mean   or   apply   to   any   general   American   law.   It  
thereof  because  several  foreign  elements  are  involved.  She  alleged  further  that   refers  then  to  the  private  law  of  the  State  of  California.  The  next  question  is,  
the   forum   is   the   Philippines   and   even   if   the   case   were   decided   in   California,   what  is  the  law  in  California  governing  the  disposition  of  personal  property?  
§946   of   the   California   Civil   Code,   requires   that   the   domicile   of   the   decedent    
should  apply.   It   is   argued   on   executor's   behalf   that   as   the   deceased   Christensen   was   a   citizen  
  of   the   State   of   California,   the   internal   law   thereof,   should   govern   the  
The  court  below  ruled  that  as  Edward  was  a  citizen  of  the  US  and  of  the  State  of   determination   of   the   validity   of   the   testamentary   provisions   of   Christensen's  
California   at   the   time   of   his   death,   it  follows  then  that  the  successional  rights   will,  such  law  being  in  force  in  the  State  of  California  of  which  Christensen  was  
and   intrinsic   validity   of   the   provisions   in   his   will   are   to   be   governed   by   a   citizen.   Maria   Helen,   on   the   other   hand,   insists   that   Art.   946   should   be  
California  law.  Therefore,  it  held  that  he  had  the  right  to  dispose  of  his  property   applicable,   which   reads:  “If  there  is  no  law  to  the  contrary,   in   the   place   where  
in   the   way   he   desires   because   the   right   of   absolute   dominion   over   his   property   personal  property  is  situated,  it  is  deemed  to  follow  the  person  of  its  owner,  and  
is  sacred  and  inviolable.   is   governed   by   the   law   of   his   domicile.”   And   in   accordance   therewith   and  
  following   the   doctrine   of   the  renvoi,   the   question   of   the   validity   of   the  
ISSUE:   testamentary   provision   in   question   should   be   referred   back   to   the   law   of   the  
Whether   under   Renvoi   Doctrine   the   intrinsic   validity   of   the   testamentary   decedent's  domicile,  which  is  the  Philippines.  
disposition  of  the  distribution  should  be  governed  by  the  Philippine  Laws?    
  The   theory   of   doctrine   of   renvoi   states   the   problem   in   this   way:   "When   the  
HELD:   Conflict   of   Laws   rule   of   the   forum   refers   a   jural   matter   to   a   foreign   law   for  
Yes.   There   is   no   question   that   Edward   was   a   citizen   of   the   United   States   and   of   decision,   is   the   reference   to   the   purely   internal   rules   of   law   of   the   foreign  
the   State   of   California   at   the   time   of   his   death   and   there   is   also   no   question   system;   i.e.,   to   the   totality   of   the   foreign   law   minus   its   Conflict   of   Laws  
rules?"  
 
CONFLICT  OF  LAWS                                                                                    AV  DE  TORRES   84  
ATTY.  ARIS  L.  GULAPA                                            AY  2015-­‐2016  
  Von   Bar   presented   his   views   at   the   meeting   of   the   Institute   of   International  
In  explaining  the  logic  behind  renvoi,  the  Court  cited  this:  “The  Michigan  court   Law,  at  Neuchatel,  in  1900,  in  the  form  of  the  following  theses:  1)  Every  court  
chose  to  accept  the  renvoi,  that  is,  applied  the  Conflict  of  Laws  rule  of  Illinois   shall  observe  the  law  of  its  country  as  regards  the  application  of  foreign  laws.  
which  referred  the  matter  back  to  Michigan  law.  But  once  having  determined   (2)   Provided   that   no   express   provision   to   the   contrary   exists,   the   court   shall  
the   COL   principle   is   the   rule   looked   to,   it   is   difficult   to   see   why   the   reference   respect:  (a)  The  provisions  of  a  foreign  law  which  disclaims  the  right  to  bind  its  
back   should   not   have   been   to   Michigan   Conflict   of   Laws.   This   would   have   nationals   abroad   as   regards   their   personal   statute,   and   desires   that   said  
resulted   in   the   "endless   chain   of   references"   which   has   so   often   been   criticized   personal  statute  shall  be  determined  by  the  law  of  the  domicile,  or  even  by  the  
by  legal  writers.  The  opponents  of  the  renvoi  would  have  looked  merely  to  the   law  of  the  place  where  the  act  in  question  occurred.  (b)  The  decision  of  two  or  
internal  law  of  Illinois,  thus  rejecting  the  renvoi  or  the  reference  back.  Yet  there   more   foreign   systems   of   law,   provided   it   be   certain   that   one   of   them   is  
seems   no   compelling   logical   reason   why   the   original   reference   should   be   the   necessarily   competent,   which   agree   in   attributing   the   determination   of   a  
internal   law   rather   than   to   the   Conflict   of   Laws   rule.   It   is   true   that   such   a   question  to  the  same  system  of  law.  
solution   avoids   going   on   a   merry-­‐go-­‐round,   but   those   who   have   accepted    
the  renvoi  theory   avoid   this  inextricabilis   circulas  by   getting   off   at   the   second   Art.   946   of   the   California   Civil   Code   is   its   COL   rule,   while   the   rule   invoked   by  
reference  and  at  that  point  applying  internal  law.”   Maria   Helen   is   an   internal   law.   If   the   law   on   succession   and   the   conflict   of   laws  
  rules   of   California   are   to   be   enforced   jointly,   each   in   its   own   intended   and  
The   court   also   discussed   two   types   of   renvoi.   One   type   of   renvoi   is   where   a   appropriate   sphere,   the   internal   law   should   apply   to   citizens   living   in   the  State,  
jural  matter  is  presented  which  the  COL  rule  of  the  forum  refers  to  a  foreign   but   Article   946   should   apply   to   such   of   its   citizens   as   are   not   domiciled   in  
law,  the  conflict-­‐of-­‐laws  rule  of  which,  in  turn,  refers  the  matter  back  again  to   California  but  in  other  jurisdictions.  The  rule  laid  down  of  resorting  to  the  law  
the   law   of   the   forum.  This  is  renvoi  in  the  narrower  sense.  The  German  term   of  the  domicile  in  the  determination  of  matters  with  foreign  element  involved  
for   this   judicial   process   is   'Ruckverweisung.'   Thus,   after   a   decision   has   been   is  in  accord  with  the  general  principle  of  American  law  that  the  domiciliary  law  
arrived  that  a  foreign  law  is  to  be  resorted  to  as  governing  a  particular  case,  the   should  govern  in  most  matters  or  rights  which  follow  the  person  of  the  owner.  
further   question   may   arise:   Are   the   rules   as   to   the   COL   contained   in   such    
foreign  law  also  to  be  resorted  to?  This  is  a  question  which,  while  it  has  been   In  the  case  at  bar,  the  estate  of  Edward  through  its  executor  argues  that  what  
considered   by   the   courts   in   but   a   few   instances,   has   been   the   subject   of   Art.  16  of  the  Civil  Code  of  the  Philippines  pointed  out  as  the  national  law  is  the  
frequent   discussion   by   textwriters   and   essayists.   The   doctrine   involved   has   internal  law  of  California.  However,   the   laws   of   California   have   prescribed   two  
been   descriptively   designated   as   the   "Renvoyer"   to   send   back,   or   the   sets   of   laws   for   its   citizens,   one   for   residents   therein   and   another   for   those  
"Ruchversweisung",  or  the  "Weiterverweisung."     domiciled   in   other   jurisdictions.   Reason   then   demands   that   the   California  
  internal   law   should   be   enforced   as   it   is   prescribed   for   its   citizens   residing  
The   second   theory   is   known   as   the   "doctrine   of   renvoi."   The   doctrine   therein,   and   enforce   the   conflict   of   laws   rules   for   the   citizens   domiciled  
postulates   that   the   court   of   the   forum,   in   determining   the   question   before   it,   abroad.  If  the  court  then  must  enforce  the  law  of  California  as  in  comity  we  
must  take  into  account  the  whole  law  of  the  other  jurisdiction,  also  its  rules  as   are  bound  to  go,  as  so  declared  in  Art.  16,  then  we  must  enforce  the  law  of  
to   COL,   and   then   apply   the   law   to   the   actual   question,   which   the   rules   of   the   California   in   accordance   with   the   express   mandate   thereof   and   as   above  
other  jurisdiction  prescribe.  This  may  be  the  law  of  the  forum.  The  doctrine  of   explained,  i.e.,  apply  the  internal  law  for  residents  therein,  and  its  conflict-­‐of-­‐
the   renvoi   has   generally   been   repudiated   by   the   American   authorities.   The   laws  rule  for  those  domiciled  abroad.  
recognition  of  this  renvoi  theory  implies  that  the  rules  of  the  conflict  of  laws    
are  to  be  understood  as  incorporating  not  only  the  ordinary  or  internal  law  of   It   is   further   argued   by   the   executor   that   that   the   clause   "if   there   is   no   law   to  
the   foreign   state   or   country,   but   its   rules   of   the   conflict   of   laws   as   well.   the   contrary   in   the   place   where   the   property   is   situated"   in   Sec.   946   of   the  
According  to  this  theory  'the  law  of  a  country'  means  the  whole  of  its  law.   California   Civil   Code   refers   to   Art.   16   and   that   the   law   to   the   contrary   in   the  
  Philippines   is   the   provision   in   said   Art.   16   that   the   national   law   of   the   deceased  
should  govern.  The  court  held  that  this  contention  cannot  be  sustained.  
 
CONFLICT  OF  LAWS                                                                                    AV  DE  TORRES   85  
ATTY.  ARIS  L.  GULAPA                                            AY  2015-­‐2016  
  Helen   alleged   that   the   will   deprives   her   of   her   legitime   as   an   acknowledged  
The   national   law   mentioned   in   Art.   16   is   the   law   on   COL   in   the   California   Civil   natural  child.  She  claims  that  under  Art.  16  of  the  Civil  Code  of  the  Philippines,  
Code,  i.e.  Art.  946,  which  authorizes  the  reference  or  return  of  the  question   the  California  law  should  be  applied,  and  in  accordance  therewith  and  following  
to   the   law   of   the   testator's   domicile.   The   conflict   of   laws   rule   in   California,   the   doctrine   of   the  renvoi,   the   question   of   the   validity   of   the   testamentary  
Art.   946   precisely   refers   back   the   case,   when   a   decedent   is   not   domiciled   in   provision   in   question   should   be   referred   back   to   the   law   of   the   decedent's  
California,  to  the  law  of  his  domicile—the  Philippines  in  the  case  at  bar.  The   domicile,   which   is   the   Philippines.   The   question   of   the   validity   of   the  
court  of  the  domicile  cannot  and  should  not  refer  the  case  back  to  California;   testamentary   provision   should   thus   be   referred   back   to   the   law   of   the  
such   action   would   leave   the   issue   incapable   of   determination   because   the   decedent’s  domicile,  which  is  the  Philippines.  She  invokes  the  provisions  of  Art.  
case   will   then   be   like   a   football,   tossed   back   and   forth   between   the   two   946   of   the   Civil   Code   of   California,   which   is   as   follows:   “If   there   is   no   law   to   the  
states,   between   the   country   of   which   the   decedent   was   a   citizen   and   the   contrary,   in   the   place   where   personal   property   is   situated,   it   is   deemed   to  
country   of   his   domicile.   The   Philippine   court   must   apply   its   own   law   as   follow   the   person   of   its   owner,   and   is   governed   by   the   law   of   his   domicile.”  
directed   in   the   conflict   of   laws   rule   of   the   state   of   the   decedent,   if   the   Accordingly,   her   share   must   be   increased   in   view   of   successional   rights   of  
question  has  to  be  decided,  especially  as  the  application  of  the  internal  law  of   illegitimate  children  under  Philippine  laws.  
California   provides   no   legitime   for   children   while   the   Philippine   law,   Arts.    
887(4)   and   894,   Civil   Code   of   the   Philippines,   makes   natural   children   legally   On  the  other  hand,  the  executor  and  Lucy  argued  that  the  national  law  of  the  
acknowledged  forced  heirs  of  the  parent  recognizing  them.   deceased  must  apply,  and  thus  the  courts  must  apply  internal  law  of  California  
  on   the   matter.   Under   California   law,   there   are   no   compulsory   heirs   and  
Therefore,   the   court   found   that   the   domicile   of   Edward   although,   a   citizen   of   consequently   a   testator   may   dispose   of   his   property   by   will   in   the   form   and  
California,   is   the   Philippines.   Hence,   the   validity   of   the   provisions   of   his   will   manner  he  desires  (Kaufman  Case).  
depriving   his   acknowledged   natural   child   Maria   Helen   should   be   governed   by    
Philippine   Law  pursuant  to  Art.  946,  not  by  the  internal  law  of  California.  The   ISSUE:  
appealed   decision   is   reversed   and   the   case   is   returned   to   the   lower   court   so   Whether  Philippine  law  should  ultimately  be  applied  
that  the  partition  can  be  made  pursuant  to  Philippine  law  on  succession.    
  HELD:  
54.  AZNAR  V.  CHRISTENSEN-­‐GARCIA,  supra   Yes.  Edward  was  a  US  Citizen  and  domiciled  in  the  Philippines  at  the  time  of  his  
  death.   The   law   that   governs   the   validity   of   his   testamentary   dispositions   is  
FACTS:   defined  in  Art.  16  of  the  Civil  Code  of  the  Philippines.  
Edward  Christensen,  born  in  New  York,  migrated  to  California  where  he  resided    
and   consequently   was   considered   citizen   thereof.   He   came   to   the   Philippines   The  laws  of  California  have  prescribed  two  sets  of  laws  for  its  citizens,  one  for  
where  he  became  a  domiciliary  until  the  time  of  his  death.  However,  during  the   residents   therein   and   another   for   those   domiciled   in   other   jurisdictions.   Article  
entire  period  of  his  residence  in  this  country,  he  had  always  considered  himself   946   of   the   California   Civil   Code   is   its   conflict   of   laws   rule,   while   the   rule  
a  citizen  of  California.   applied  in  Kaufman,  is  its  internal  law.  If  the  law  on  succession  and  the  conflict  
  of  laws  rules  of  California  are  to  be  enforced  jointly,  each  in  its  own  intended  
In   his   will,   Edward   instituted   his   daughter   Maria   Lucy   Christensen   as   his   only   and   appropriate   sphere,   the   principle   cited   in   Kaufman   should   apply   to  
heir,  but  left  a  legacy  of  P3600  in  favor  of  Helen  Christensen  Garcia  who,  in  his   citizens  living  in  the  State,  but  Article  946  should  apply  to  such  of  its  citizens  
will  was  described  as  "not  in  any  way  related  to"  him  but  in  a  decision  rendered   as  are  not  domiciled  in  California  but  in  other  jurisdictions.  
by   the   SC   in   another   case   had   been   declared   as   an   acknowledged   natural    
daughter  of  his.   The   national   law   mentioned   in   Art.   16   is   the   law   on   conflict   of   laws   in   the  
  California  Civil  Code,  i.e.,  Art.  946,  which  authorizes  the  reference  or  return  of  
the   question   to   the   law   of   the   testator's   domicile.   The   conflict   of   laws   rule   in  
 
CONFLICT  OF  LAWS                                                                                    AV  DE  TORRES   86  
ATTY.  ARIS  L.  GULAPA                                            AY  2015-­‐2016  
California,   Art.   946,   Civil   Code,   precisely   refers   back   the   case,   when   a   decedent   properties  are  situated,  renvoi  would  arise,  since  the  properties  here  involved  
is  not  domiciled  in  California,  to  the  law  of  his  domicile,  which  is  the  Philippines   are  found  in  the  Philippines.  
in  the  case  at  bar.    
  Whatever   public   policy   or   good   customs   may   be   involved   in   our   System   of  
The   Philippine   court   therefore   must   apply   its   own   law   as   directed   in   the   legitimes,  Congress  has  not  intended  to  extend  the  same  to  the  succession  of  
conflict   of   laws   rule   of   the   state   of   the   decedent.   Wherefore,   the   decision   foreign  nationals.  
appealed   from   is   hereby   reversed   and   the   case   returned   to   the   lower   court   with    
instructions   that   the   partition   be   made   as   the   Philippine   law   on   succession   FACTS:  
provides.   Amos  Bellis  was  a  citizen  and  resident  of  Texas  at  the  time  of  his  death.    He  had  
  5   legitimate   children   with   his   first   wife,   Mary   Mallen,   whom   he   divorced.   He  
RENVOI  DOCTRINE   had  3  legitimate  daughters  with  his  second  wife,  Violet,  who  survived  him,  and  
A  jural  matter  is  presented  which  the  conflict-­‐of-­‐laws  rule  of  the  forum  refers   another   3   illegitimate   children   with   another   woman.   Before   he   died,   he  
to   a   foreign   law,   the   conflict-­‐of-­‐laws   rule   of   which,   in   turn,   refers   the   matter   executed   2   wills,   disposing   of   his   Texas   properties,   the   other   disposing   his  
back   again   to   the   law   of   the   forum.   This   is   renvoi   in   the   narrower   sense.   The   Philippine   properties.  In   his   will,   which   he   executed   in   the   Philippines,   he  
German  term  for  this  judicial  process  is  'Ruckverweisung.'   directed   that   after   all   taxes,   obligations,   and   expenses   of   administration   are  
  paid   for,   his   distributable   estate   should   be   divided,   in   trust,   in   the   following  
Another   theory,   known   as   the   "doctrine   of   renvoi",   has   been   advanced.   The   order   and   manner:   a)   $240,000.00   to   his   first   wife   Mary   Mallen   b)   $120,000.00  
theory  of  the  doctrine  of  renvoi  is  that  the  court  of  the  forum,  in  determining   to   his   three   illegitimate   children   Amos   Bellis,   Jr.,   Maria   Cristina   Bellis,   Miriam  
the   question   before   it,   must   take   into   account   the   whole   law   of   the   other   Palma   Bellis,   or   $40,000.00   each,   and  c)   After   foregoing   the   two   items   have  
jurisdiction,  but  also  its  rules  as  to  conflict  of  laws,  and  then  apply  the  law  to   been  satisfied,  the  remainder  shall  go  to  his  seven  surviving  children  by  his  first  
the  actual  question  which  the  rules  of  the  other  jurisdiction  prescribe.  This  may   and  second  wives.  
be  the  law  of  the  forum.    
  Maria  Cristina  Bellis  and  Miriam  Palma  Bellis,  filed  their  respective  oppositions  
Residence   Domicile   to   the   project   of   partition   on   the   ground   that   they   were   deprived   of   their  
Requires   bodily   presence   of   an   Requires  bodily  presence  in  that  place   legitimes   as   illegitimate   children   and,   therefore,   compulsory   heirs   of   the  
inhabitant  in  a  given  place   and  also  an  intention  to  make  it  one’s   deceased.  
domicile    
  The  lower  court  issued  an  order  overruling  the  oppositions  and  approving  the  
54.  BELLIS  V.  BELLIS,  supra   executor’s   final   account,   report   and   administration,   and   project   of   partition.  
  Relying   upon   Art.   16   of   the   Civil   Code,   it   applied   the   national   law   of   the  
DOCTRINE:   The  doctrine  of  renvoi  is  usually  pertinent  where  the  decedent  is  a   decedent,  which  in  this  case  did  not  provide  for  legitimes.  
national   of   one   country,   and   a   domicile   of   another.   In   the   present   case,   the    
decedent  was  both  a  national  of  Texas  and  a  domicile  thereof  at  the  time  of  his   ISSUE:  
 
death. So   that   even   assuming   Texas   has   a   conflict   of   law   rule   providing   that   the   Whether  such  illegitimate  children  of  Bellis  be  entitled  to  successional  rights  
domiciliary   system   (law   of   the   domicile)   should   govern,   the   same   would   not    
result   in   a   reference   back   (renvoi)   to   Philippine   law,   but   would   still   refer   to   HELD:  
Texas   law.   Nonetheless,   if   Texas   has   a   conflicts   rule   adopting   the   situs   theory   No.   The   said   illegitimate   children   are   not   entitled   to   their   legitimes.   Under  
(lex   rei   sitae)   calling   for   the   application   of   the   law   of   the   place   where   the   Texas   law,   there   are   no   legitimes.   Even   if   the   other   will   was   executed   in   the  
Philippines,  his  national  law,  still,  will  govern  the  properties  for  succession  even  

 
CONFLICT  OF  LAWS                                                                                    AV  DE  TORRES   87  
ATTY.  ARIS  L.  GULAPA                                            AY  2015-­‐2016  
if  it  is  stated  in  his  testate  that  it  shall  be  governed  by  the  Philippine  law.   domiciliary  system  (law  of  the  domicile)  should  govern,  the  same  would  not  
  result   in   a   reference   back   (renvoi)   to   Philippine   law,   but   would   still   refer   to  
Art.  16,  Par.  2  renders  applicable  the  national  law  of  the  decedent,  in  intestate   Texas   law.   Nonetheless,   if   Texas   has   a   conflicts   rule   adopting   the   situs   theory  
and   testamentary   successions,   with   regard   to   four   items:   (a)   the   order   of   (lex   rei   sitae)   calling   for   the   application   of   the   law   of   the   place   where   the  
succession,   (b)   the   amount   of   successional   rights,   (c)   the   intrinsic   validity   of   properties  are  situated,  renvoi  would  arise,  since  the  properties  here  involved  
provisions  of  will,  and  (d)  the  capacity  to  succeed.   are   found   in   the   Philippines.   In   the   absence,   however,   of   proof   as   to   the  
  conflict  of  law  rule  of  Texas,  it  should  not  be  presumed  different  from  ours.  The  
Appellants   however   counter   that   Art.   17,   par.   3—Prohibitive   laws   concerning   appellants’  position  is  therefore  not  rested  on  the  doctrine  of  renvoi.  As  stated,  
persons,   their   acts   or   property,   and   those   which   have   for   their   object   public   they   never   invoked   nor   even   mentioned   it   in   their   arguments.   Rather,   they  
rd
order,  public  policy  and  good  customs  shall  not  be  rendered  ineffective  by  laws   argue   that   their   case   falls   under   the   circumstances   mentioned   in   the   3   par.   of  
or  judgments  promulgated,  or  by  determinations  or  conventions  agreed  upon  in   Art.  17  in  relation  to  Art.  16  of  the  Civil  Code.  
a   foreign   country.—prevails   as   the   exception   to   Art.   16,   par.   2.   This   is   not    
correct.   55.  SAUDI  ARABIAN  AIRLINES  V.  CA,  supra  
   
Congress   deleted  the   phrase,   "notwithstanding   the   provisions   of   this   and   the   DOCTRINES:  A  factual  situation  that  cuts  across  territorial  lines  and  is  affected  
next  preceding  article"  when  they  incorporated  Art.  11  of  the  old  Civil  Code  as   by  the  diverse  laws  of  two  or  more  states  is  said  to  contain  a  “foreign  element.”  
Art.  17  of  the  new  Civil  Code,  while  reproducing  without  substantial  change  the    
second  paragraph  of  Art.  10  of  the  old  Civil  Code  as  Art.  16  in  the  new.  It  must   Characterization   or   the   doctrine   of   qualification   is   the   process   of   deciding  
have   been   their   purpose   to   make   the   second   paragraph   of   Art.   16   a   specific   whether   or   not   the   facts   relate   to   the   kind   of   question   specified   in   a   conflicts  
provision   in   itself   which   must   be   applied   in   testate   and   intestate   succession.   As   rule.   The   purpose   of   “characterization”   is   to   enable   the   forum   to   select   the  
further   indication   of   this   legislative   intent,   Congress   added   a   new   provision,   proper  law.  
under  Art.  1039,  which  decrees  that  capacity  to  succeed  is  to  be  governed  by    
the  national  law  of  the  decedent.   Choice-­‐of-­‐law   rules   invariably   consist   of   a   factual   relationship   (such   as   property  
  right,   contract   claim)   and   a   connecting   factor   or   point   of   contact,   such   as   the  
It   is   therefore   evident   that   whatever   public   policy   or   good   customs   may   be   situs  of  the  res,  the  place  of  celebration,  the  place  of  performance,  or  the  place  
involved   in   our   system   of   legitimes,   Congress   has   not   intended   to   extend   the   of  wrongdoing.  Note  that  one  or  more  circumstances  may  be  present  to  serve  
same   to   the   succession   of   foreign   nationals.   For   it   has   specifically   chosen   to   as  the  possible  test  for  the  determination  of  the  applicable  law.  
leave,  inter   alia,   the  amount  of   successional   rights,   to   the   decedent's   national    
law.   Specific   provisions   must   prevail   over   general   ones.   A   provision   in   a   These  “test  factors”  or  “points  of  contact”  or  “connecting  factors”  could  be  any  
foreigner's   will   to   the   effect   that   his   properties   shall   be   distributed   in   of  the  following:  (SAFE  IN  SF)  
accordance  with  Philippine  law  and  not  with  his  national  law,  is  illegal  and  void,   (1)  the  nationality  of  a  person,  his  domicile,  his  residence,  his  place  of  sojourn,  
for  his  national  law  cannot  be  ignored  in  regard  to  those  matters  that  Art.  10— or  his  origin;  
now  Art.  16—of  the  Civil  Code  states  said  national  law  should  govern.   (2)  the  seat  of  a  legal  or  juridical  person,  such  as  a  corporation;  
  (3)  the  situs  of  a  thing,  that  is,  the  place  where  a  thing  is,  or  is  deemed  to  be  
In   this   regard,   the   parties   do   not   submit   the   case   on,   nor   even   discuss,   the   situated.  In  particular,  the  lex  situs  is  decisive  when  real  rights  are  involved;  
doctrine  of  renvoi,  applied  by  this  Court  in  Aznar  v.  Christensen.   Said  doctrine   (4)   the   place   where   an   act   has   been   done,   the   locus   actus,   such   as   the   place  
is   usually   pertinent   where   the   decedent   is   a   national   of   one   country,   and   a   where  a  contract  has  been  made,  a  marriage  celebrated,  a  will  signed  or  a  tort  
domicile  of  another.  In  the  present  case,  it  is  not  disputed  that  the  decedent   committed.  The  lex  loci  actus  is  particularly  important  in  contracts  and  torts;  
 
was  both  a  national  of  Texas  and  a  domicile  thereof  at  the  time  of  his  death. (5)   the   place   where   an   act   is   intended   to   come   into   effect,   e.g.,   the   place   of  
So   that   even   assuming   Texas   has   a   conflict   of   law   rule   providing   that   the  
 
CONFLICT  OF  LAWS                                                                                    AV  DE  TORRES   88  
ATTY.  ARIS  L.  GULAPA                                            AY  2015-­‐2016  
performance  of  contractual  duties,  or  the  place  where  a  power  of  attorney  is  to   afraid  that  she  might  be  tricked  into  something  she  did  not  want  because  of  her  
be  exercised;   inability   to   understand   the   local   dialect.   She   also   declined   to   sign   a   blank   paper  
(6)   the   intention   of   the   contracting   parties   as   to   the   law   that   should   govern   and   a   document   written   in   the   local   dialect.   However,   she   learned   that,  
their  agreement,  the  lex  loci  intentionis;   through   the   intercession   of   the   Saudi   Arabian   government,   the   Indonesian  
(7)   the   place   where   judicial   or   administrative   proceedings   are   instituted   or   authorities   agreed   to   deport   Thamer   and   Allah   after   2   weeks   of   detention.  
done.  The  lex  fori—the  law  of  the  forum—is  particularly  important  because,  as   Eventually,  they  were  again  put  in  service  by  defendant  SAUDIA.  In  September  
we  have  seen  earlier,  matters  of  ‘procedure’  not  going  to  the  substance  of  the   1990,  SAUDIA  transferred  Morada  to  Manila.  
claim   involved   are   governed   by   it;   and   because   the   lex   fori   applies   whenever    
the   content   of   the   otherwise   applicable   foreign   law   is   excluded   from   In  1992,  Morada’s  superiors  requested  her  to  see  Mr.  Ali  Meniewy,  Chief  Legal  
application   in   a   given   case   for   the   reason   that   it   falls   under   one   of   the   Officer  of  SAUDIA,  in  Jeddah,  Saudi  Arabia.  When  she  saw  him,  he  brought  her  
exceptions  to  the  applications  of  foreign  law;  and   to   the   police   station   where   the   police   took   her   passport   and   questioned   her  
(8)   the   flag   of   a   ship,   which   in   many   cases   is   decisive   of   practically   all   legal   about  the  Jakarta  incident.  Not  until  she  agreed  to  do  so  did  the  police  return  
relationships   of   the   ship   and   of   its   master   or   owner   as   such.   It   also   covers   her  passport  and  allowed  her  to  catch  the  afternoon  flight  out  of  Jeddah.  
contractual  relationships  particularly  contracts  of  affreightment.    
  One   year   and   a   half   later,   in   Riyadh,   a   few   minutes   before   the   departure   of   her  
In   applying   the   “State   of   the   most   significant   relationship”   rule,   the   following   flight   to   Manila,   Morada   was   not   allowed   to   board   the   plane   and   instead  
contacts  are  to  be  taken  into  account  and  evaluated  according  to  their  relative   ordered   to   take   a   later   flight   to   Jeddah   to   see   Mr.   Miniewy   again.   When   she  
importance  with  respect  to  the  particular  issue:  (a)  the  place  where  the  injury   did,   she   was   brought   her   to   a   Saudi   court   where   she   was   asked   to   sign   a  
occurred;  (b)  the  place  where  the  conduct  causing  the  injury  occurred;  (c)  the   document  written  in  Arabic.  They  told  her  that  this  was  necessary  to  close  the  
domicile,   residence,   nationality,   place   of   incorporation   and   place   of   business   of   case  against  Thamer  and  Allah.  As  it  turned  out,  plaintiff  signed  a  notice  to  her  
the   parties;   and   (d)   the   place   where   the   relationship,   if   any,   between   the   to  appear  before  the  court  on  June  27,  1993.  She  then  returned  to  Manila.  
parties  is  centered.    
  Shortly   afterwards,   SAUDIA   summoned   plaintiff   to   report   to   Jeddah   once   again  
FACTS:   where  a  Saudi  judge  interrogated  her  through  an  interpreter  about  the  Jakarta  
In   1988,   SAUDIA   hired   Milagros   Morada   as   a   Flight   Attendant   for   its   airlines   incident.   After   one   hour   of   interrogation,   when   she   was   about   to   return   to  
based   in   Jeddah,   Saudi   Arabia.   In   1990,   while   on   a   lay-­‐over   in   Jakarta,   Manila,   a   SAUDIA   officer   told   her   that   the   airline   had   forbidden   her   to   take  
Indonesia,   Morada   went   to   a   disco   dance   with   fellow   crew   members   Thamer   flight  and  took  away  her  passport  and  told  her  to  remain  in  Jeddah,  at  the  crew  
Al-­‐Gazzawi   and   Allah   Al-­‐Gazzawi,   both   Saudi   nationals.   Because   it   was   almost   quarters,   until   further   orders.   After   a   few   days,   a   SAUDIA   legal   officer   again  
morning   when   they   returned   to   their   hotels,   they   agreed   to   have   breakfast   escorted  plaintiff  to  the  same  court  where  the  judge,  to  her  astonishment  and  
together  at  the  room  of  Thamer.  When  they  were  in  the  room,  Allah  left  and   shock,   rendered   a   decision   sentencing   her   to   5   months   imprisonment   and   to  
shortly   after,   Thamer   attempted   to   rape   her.   Fortunately,   a   roomboy   and   286   lashes.   Only   then   did   she   realize   that   the   Saudi   court   had   tried   her,  
several  security  personnel  heard  her  cries  for  help  and  rescued  her.  Later,  the   together  with  Thamer  and  Allah,  for  what  happened  in  Jakarta.  The  court  found  
Indonesian  police  came  and  arrested  Thamer  and  Allah  Al-­‐Gazzawi,  the  latter  as   Morada  guilty  of  (1)  adultery;  (2)  going  to  a  disco,  dancing  and  listening  to  the  
an  accomplice.   music   in   violation   of   Islamic   laws;   and   (3)   socializing   with   the   male   crew,   in  
  contravention  of  Islamic  tradition.”  
When   plaintiff   returned   to   Jeddah   a   few   days   later,   SAUDIA   officials    
interrogated   her   about   the   Jakarta   incident.   They   then   requested   her   to   go   She  was  denied  any  assistance  by  SAUDIA  thus,  she  then  asked  the  Philippine  
back   to   Jakarta   to   help   arrange   the   release   of   Thamer   and   Allah.   Morada   did   Embassy  in  Jeddah  to  help  her  while  her  case  is  on  appeal.  Meanwhile,  to  pay  
not   cooperate   in   the   negotiation   with   the   Indonesian   police   because   she   was   for   her   upkeep,   she   worked   on   the   domestic   flight   of   SAUDIA,   while   Thamer  
and   Allah   continued   to   serve   in   the   international   flights.   Because   she   was  
 
CONFLICT  OF  LAWS                                                                                    AV  DE  TORRES   89  
ATTY.  ARIS  L.  GULAPA                                            AY  2015-­‐2016  
wrongfully  convicted,  the  Prince  of  Makkah  dismissed  the  case  against  her  and   situation  to  arise.  Thus,  Morada’s  assertion  that  the  case  is  purely  domestic  is  
allowed  her  to  leave   Saudi   Arabia.   Shortly   before   her   return   to   Manila,   she   was   imprecise.   A   conflicts   problem   presents   itself   here,   and   the   question   of  
terminated   from   the   service   by   SAUDIA,   without   her   being   informed   of   the   jurisdiction  confronts  the  court  a  quo.  
cause.    
  2)   Yes.   Based   on   the   allegations   in   the   Amended   Complaint,   read   in   the   light   of  
In  November  1993,   Morada  filed  a  Complaint   for  damages  against  SAUDIA,  and   the  Rules  of  Court  on  jurisdiction,  the  Court  finds  that  the  RTC  of  Quezon  City  
Khaled   Al-­‐Balawi,   its   country   manager   in   the   trial   court.   SAUDIA   filed   an   possesses   jurisdiction   over   the   subject   matter   of   the   suit.   Its   authority   to   try  
Omnibus   MTD   which   raised   the   following   grounds,   to   wit:   (1)   that   the   and   hear   the   case   is   provided   for   under   Sec.   1   of   R.A.   No.   7691:   “RTCs   shall  
Complaint   states   no   cause   of   action   against   SAUDIA;   (2)   that   defendant   Al-­‐ exercise  exclusive  jurisdiction:  (8)  In  all  other  cases  in  which  demand,  exclusive  
Balawi  is  not  a  real  party  in  interest;  (3)  that  the  claim  or  demand  set  forth  in   of  interest,  damages  of  whatever  kind,  attorney's  fees,  litigation  expenses,  and  
the   Complaint   has   been   waived,   abandoned   or   otherwise   extinguished;   and   (4)   costs  or  the  value  of  the  property  in  controversy  exceeds  One  hundred  thousand  
that  the  trial  court  has  no  jurisdiction  to  try  the  case  on  the  basis  of  Art.  21  of   pesos   (P100,000.00)   or,   in   such   other   cases   in   Metro   Manila,   where   the  
the  NCC,  since  the  proper  law  applicable  is  the  law  of  the  KSA.  The  trial  court   demand,   exclusive   of   the   above-­‐mentioned   items   exceeds   Two   hundred  
denied  the  MTD.   Thousand  pesos  (P200,000.00).”  
   
Consequently,   SAUDIA   filed   its   Petition   for   Certiorari   and   Prohibition   with   Pragmatic  considerations,  including  the  convenience  of  the  parties,  also  weigh  
Prayer   for   Issuance   of   Writ   of   Preliminary   Injunction   and/or   TRO   with   the   CA,   heavily   in   favor   of   the   RTC   of   QC   assuming   jurisdiction.   Paramount   is   the  
which   ruled   that   the   Philippines   is   an   appropriate   forum   considering   that   the   private   interest   of   the   litigant.   Enforceability   of   a   judgment   if   one   is   obtained   is  
Amended  Complaint’s  basis  for  recovery  of  damages  is  Art.  21,  and  thus,  clearly   quite   obvious.   Relative   advantages   and   obstacles   to   a   fair   trial   are   equally  
within  the  jurisdiction  of  respondent  Court.   important.  Plaintiff  may  not,  by  choice  of  an  inconvenient  forum,  ‘vex,’  ‘harass,’  
  or   ‘oppress’   the   defendant,   e.g.   by   inflicting   upon   him   needless   expense   or  
ISSUE:   disturbance.  But  unless  the  balance  is  strongly  in  favor  of  the  defendant,  the  
1.  Whether  the  case  is  a  conflict  of  laws.   plaintiff’s  choice  of  forum  should  rarely  be  disturbed.  
2.  Whether  the  RTC  of  Quezon  City  has  jurisdiction.    
3.  Whether  the  Philippine  Law  should  govern.   Weighing   the   relative   claims   of   the   parties,   the   court   a   quo   found   it   best   to  
  hear  the  case  in  the  Philippines.  Had  it  refused  to  take  cognizance  of  the  case,  it  
HELD:   would   be   forcing   Morada   to   seek   remedial   action   elsewhere,   i.e.   in   the   KSA  
1.  Yes.  A   factual   situation   that   cuts   across   territorial   lines   and   is   affected   by   where   she   no   longer   maintains   substantial   connections.   That   would   have  
the  diverse  laws  of  two  or  more  states  is  said  to  contain  a  “foreign  element.”     caused  a  fundamental  unfairness  to  her.  Moreover,  by  hearing  the  case  in  the  
  Philippines  no  unnecessary  difficulties  and  inconvenience  have  been  shown  by  
The   forms   in   which   this   foreign   element   may   appear   are   many.   The   foreign   either  of  the  parties.  
element  may  simply  consist  in  the  fact  that  one  of  the  parties  to  a  contract  is  an    
alien   or   has   a   foreign   domicile,   or   that   a   contract   between   nationals   of   one   3)   Yes.   As   to   the   choice   of   applicable   law,   we   note   that   choice-­‐of-­‐law   problems  
State  involves  properties  situated  in  another  State.  In  other  cases,  the  foreign   seek  to  answer  two  important  questions:  (1)  What  legal  system  should  control  
element  may  assume  a  complex  form.  In  the  instant  case,  the  foreign  element   a  given  situation  where  some  of  the  significant  facts  occurred  in  two  or  more  
consisted   in   the   fact   that   Morada   is   a   resident   Philippine   national,   and   that   states;   and   (2)   to   what   extent   should   the   chosen   legal   system   regulate   the  
SAUDIA  is  a  resident  foreign  corporation.  Also,  by  virtue  of  the  employment  of   situation.  
Morada  with  the  SAUDIA  as  a  flight  stewardess,  events  did  transpire  during  her    
many   occasions   of   travel   across   national   borders,   particularly   from   Manila,   Before   a   choice   can   be   made,   it   is   necessary   for   us   to   determine   under   what  
Philippines   to   Jeddah,   Saudi   Arabia,   and   vice   versa,   that   caused   a   “conflicts”   category   a   certain   set   of   facts   or   rules   fall.   This   process   is   known   as  
 
CONFLICT  OF  LAWS                                                                                    AV  DE  TORRES   90  
ATTY.  ARIS  L.  GULAPA                                            AY  2015-­‐2016  
“characterization,”   or   the   “doctrine   of   qualification.”   It   is   the   “process   of   legal  relationships  of  the  ship  and  of  its  master  or  owner  as  such.  It  also  
deciding  whether  or  not  the  facts  relate  to  the  kind  of  question  specified  in  a   covers  contractual  relationships  particularly  contracts  of  affreightment.  
conflicts   rule.”   The   purpose   of   “characterization”   is   to   enable   the   forum   to    
select  the  proper  law.   Considering  that  the  complaint  in  the  court  a  quo  is  one  involving  torts,  the  
  “connecting  factor”  or  “point  of  contact”  could  be  the  place  or  places  where  
Our   starting   point   of   analysis   here   is   not   a   legal   relation,   but   a   factual   the   tortious   conduct   or   lex   loci   actus   occurred.   And   applying   the   torts  
situation,  event,  or  operative  fact.  An  essential  element  of  conflict  rules  is  the   principle   in   a   conflicts   case,   we   find   that   the   Philippines   could   be   said   as   a  
indication  of  a  “test”  or  “connecting  factor”  or  “point  of  contact.”  Choice-­‐of-­‐ situs   of   the   tort   (the   place   where   the   alleged   tortious   conduct   took   place).  
law   rules   invariably   consist   of   a   factual   relationship   (such   as   property   right,   This   is   because   it   is   in   the   Philippines   where   SAUDIA   allegedly   deceived  
contract   claim)   and   a   connecting   factor   or   point   of   contact,   such   as   the   situs   of   Morada,   a   Filipina   residing   and   working   here.   According   to   her,   she   had  
the   res,   the   place   of   celebration,   the   place   of   performance,   or   the   place   of   honestly   believed   that   SAUDIA   would,   in   the   exercise   of   its   rights   and   in   the  
wrongdoing.   performance  of  its  duties,  “act  with  justice,  give  her  due  and  observe  honesty  
  and   good   faith.”   Instead,   SAUDIA   failed   to   protect   her.   That   certain   acts   or  
Note  that  one  or  more  circumstances  may  be  present  to  serve  as  the  possible   parts  of  the  injury  allegedly  occurred  in  another  country  is  of  no  moment.  What  
test  for  the  determination  of  the  applicable  law.  These  “test   factors”  or  “points   is   important   here   is   the   place   where   the   over-­‐all   harm   or   the   totality   of   the  
of  contact”  or  “connecting  factors”  could  be  any  of  the  following:   alleged  injury  to  the  person,  reputation,  social  standing  and  human  rights  of  
  complainant,  had  lodged,  according  to  Morada.  All  told,  it  is  not  without  basis  
(1)   the   nationality   of   a   person,   his   domicile,   his   residence,   his   place   of   to  identify  the  Philippines  as  the  situs  of  the  alleged  tort.  
sojourn,  or  his  origin;    
(2)  the  seat  of  a  legal  or  juridical  person,  such  as  a  corporation;   Moreover,  with  the  widespread  criticism  of  the  traditional  rule  of  lex  loci  delicti  
(3)  the  situs  of  a  thing,  that  is,  the  place  where  a  thing  is,  or  is  deemed  to   commissi,   modern   theories   and   rules   on   tort   liability   have   been   advanced   to  
be   situated.   In   particular,   the   lex   situs   is   decisive   when   real   rights   are   offer  fresh  judicial  approaches  to  arrive  at  just  results.  In  keeping  abreast  with  
involved;   the   modern   theories   on   tort   liability,   we   find   here   an   occasion   to   apply   the  
(4)   the   place   where   an   act   has   been   done,   the   locus   actus,   such   as   the   “State   of   the   most   significant   relationship”   rule,  which  in  our  view  should  be  
place   where   a   contract   has   been   made,   a   marriage   celebrated,   a   will   appropriate  to  apply  now,  given  the  factual  context  of  this  case.   In   determining  
signed   or   a  tort  committed.  The  lex  loci  actus  is   particularly  important  in   the  State  which  has  the  most  significant  relationship,  the  following  contacts  
contracts  and  torts;   are   to   be   taken   into   account   and   evaluated   according   to   their   relative  
(5)  the  place  where  an  act  is  intended  to  come  into  effect,  e.g.,  the  place   importance  with  respect  to  the  particular  issue:  (a)  the  place  where  the  injury  
of   performance   of   contractual   duties,   or   the   place   where   a   power   of   occurred;   (b)   the   place   where   the   conduct   causing   the   injury   occurred;   (c)   the  
attorney  is  to  be  exercised;   domicile,  residence,  nationality,  place  of  incorporation  and  place  of  business  
(6)   the   intention   of   the   contracting   parties   as   to   the   law   that   should   of  the  parties;  and  (d)  the  place  where  the  relationship,  if  any,  between  the  
govern  their  agreement,  the  lex  loci  intentionis;   parties  is  centered.  
(7)  the  place  where  judicial  or  administrative  proceedings  are  instituted    
or   done.   The   lex   fori—the   law   of   the   forum—is   particularly   important   With  these  guidelines  in  mind,  the  trial  court  must  proceed  to  try  and  adjudge  
because,  as  we  have  seen  earlier,  matters  of  ‘procedure’  not  going  to  the   the   case   in   the   light   of   relevant   Philippine   law,   with   due   consideration   of   the  
substance  of  the  claim  involved  are  governed  by  it;  and  because  the  lex   foreign  element  or  elements  involved.  Nothing  said  herein,  of  course,  should  be  
fori   applies   whenever   the   content   of   the   otherwise   applicable   foreign   construed  as  prejudging  the  results  of  the  case  in  any  manner  whatsoever.  
law  is  excluded  from  application  in  a  given  case  for  the  reason  that  it  falls    
under  one  of  the  exceptions  to  the  applications  of  foreign  law;  and  
(8)   the   flag   of   a   ship,   which   in   many   cases   is   decisive   of   practically   all  
 
CONFLICT  OF  LAWS                                                                                    AV  DE  TORRES   91  
ATTY.  ARIS  L.  GULAPA                                            AY  2015-­‐2016  
56.  LLORENTE  V.  CA   Sur,   a   petition   for   the   probate   and   allowance   of   his   last   will   and   testament  
G.R.  No.  124371  |  November  23,  2000   wherein   Lorenzo   moved   that   Alicia   be   appointed   Special   Administratrix   of   his  
  estate.   RTC   admitted   the   will   to   probate.   Before   the   proceedings   could   be  
FACTS:   terminated,  Lorenzo  died.  
Lorenzo  Llorente  was  an  enlisted  serviceman  of  the  US  Navy.  During  his  term  of    
duty,  Lorenzo  and  Paula  Llorente  were  married  before  a  parish  priest,  Roman   Paula   filed   with   the   same   court   a   petition   for   letters   of   administration   over  
Catholic   Church,   in   Camarines   Sur.   Before   the   outbreak   of   the   Pacific   War,   Lorenzo’s   estate   in   her   favor.   Paula   contended   (1)   that   she   was   Lorenzo’s  
Lorenzo   departed   for   the   US   and   Paula   stayed   in   the   conjugal   home   in   surviving   spouse,   (2)   that   the   various   property   were   acquired   during   their  
Camarines   Sur.   Afterwards,   Lorenzo   was   naturalized   as   an   American   Citizen.   marriage,   (3)   that   Lorenzo’s   will   disposed   of   all   his   property   in   favor   of   Alicia  
Upon   the   end   of   the   WWII,   Lorenzo   was   granted   an   accrued   leave   by   the   US   and   her   children,   encroaching   on   her   legitime   and   1/2   share   in   the   conjugal  
Navy,  to  visit  his  wife  and  he  visited  the  Philippines.  He  discovered  that  his  wife   property.   Alicia   filed   in   the   testate   proceeding   a   petition   for   the   issuance   of  
Paula   was   pregnant   and   was   “living   in”   and   having   an   adulterous   relationship   letters  testamentary.  RTC  gave  due  course  to  Paula's  petition.  RTC  held  that  the  
with   his   brother,   Ceferino   Llorente.   Paula   gave   birth   to   a   boy   registered   as   divorce   decree   was   void   and   inapplicable   in   the   Philippines   and   therefore   the  
“Crisologo   Llorente,”   with   the   birth   certificate   stating   that   the   child   was   not   second  marriage  with  Alicia  was  also  void.  CA  affirmed  RTC's  decision.  
legitimate  and  the  line  for  the  father’s  name  was  left  blank.    
  ISSUE:  
Lorenzo   refused   to   forgive   Paula   and   live   with   her.   In   fact,   the   couple   drew   a   Who  are  entitled  to  inherit  from  the  late  Lorenzo  N.  Llorente?  
written   agreement   to   the   effect   that   (1)   all   the   family   allowances   allotted   by    
the   US   Navy   as   part   of   Lorenzo’s   salary   and   all   other   obligations   for   Paula’s   HELD:  
daily   maintenance   and   support   would   be   suspended;   (2)   they   would   dissolve   Alicia  and  her  three  children.  
their   marital   union   in   accordance   with   judicial   proceedings;   (3)   they   would    
make  a  separate  agreement  regarding  their  conjugal  property  acquired  during   THE  APPLICABLE  LAW  
their  marital  life;  and  (4)  Lorenzo  would  not  prosecute  Paula  for  her  adulterous   The  fact  that  Lorenzo  became  an  American  citizen  long  before  and  at  the  time  
act   since   she   voluntarily   admitted   her   fault   and   agreed   to   separate   from   of:   (1)   his   divorce   from   Paula;   (2)   marriage   to   Alicia;   (3)   execution   of   his   will;  
Lorenzo  peacefully.  The  agreement  was  signed  by  both  Lorenzo  and  Paula  and   and   (4)   death,   is   duly   established,   admitted   and   undisputed.   Thus,   as   a   rule,  
was   witnessed   by   Paula’s   father   and   stepmother.   The   agreement   was   issues   arising   from   these   incidents   are   necessarily   governed   by   foreign   law.  
notarized.   Foreign  laws  do  not  prove  themselves  in  our  jurisdiction  and  our  courts  are  not  
  authorized   to   take   judicial   notice   of   them.   Like   any   other   fact,   they   must   be  
Lorenzo   returned   to   the   US   and   filed   for   divorce   with   the   Superior   Court   of   the   alleged  and  proved.  
State   of   California   and   was   issued   an   interlocutory   judgment   of   divorce.   The    
divorce  decree  became  final  afterwards.   While  the  substance  of  the  foreign  law  was  pleaded,  the  CA  did  not  admit  the  
  foreign  law.  The  CA  and  the  RTC  called  to  the  fore  the  renvoi  doctrine,  where  
Lorenzo   returned   to   the   Philippines   and   married   Alicia   Llorente   in   Manila.   the  case  was  “referred  back”  to  the  law  of  the  decedent’s  domicile,  in  this  case,  
Apparently,  Alicia  had  no  knowledge  of  the  first  marriage  even  if  they  resided   Philippine   law.   While   the   trial   court   stated   that   the   law   of   New   York   was   not  
in  the  same  town  as  Paula,  who  did  not  oppose  the  marriage  or  cohabitation.   sufficiently   proven,   in   the   same   breath   it   made   the   categorical,   albeit   equally  
Lorenzo   and   Alicia   lived   together   as   husband   and   wife.   Their   25   year   union   unproven  statement  that  “American  law  follows  the  ‘domiciliary  theory’  hence,  
produced  three  children,  Raul,  Luz  and  Beverly,  all  surnamed  Llorente.   Philippine  law  applies  when  determining  the  validity  of  Lorenzo’s  will.  
   
Lorenzo  executed  a  Last  Will  and  Testament.  In  the  will,  Lorenzo  bequeathed  all   First,  there  is  no  such  thing  as  one  American  law.  The  "national  law"  indicated  
his  property  to  Alicia  and  their  3  children.  Lorenzo  filed  with  the  RTC  Camarines   in  Art.16  of  the  Civil  Code  cannot  possibly  apply  to  general  American  law.  There  
 
CONFLICT  OF  LAWS                                                                                    AV  DE  TORRES   92  
ATTY.  ARIS  L.  GULAPA                                            AY  2015-­‐2016  
is   no   such   law   governing   the   validity   of   testamentary   provisions   in   the   US.   Each   condition  and  legal  capacity.”  
State   of   the   union   has   its   own   law   applicable   to   its   citizens   and   in   force   only    
within  the  State.  It  can  therefore  refer  to  no  other  than  the  law  of  the  State  of   Whether   the   will   is   intrinsically   valid   and   who   shall   inherit   from   Lorenzo   are  
which   the   decedent   was   a   resident.   Second,   there   is   no   showing   that   the   issues  best  proved  by  foreign  law,  which  must  be  pleaded  and  proved.  Whether  
application  of  the  renvoi  doctrine  is  called  for  or  required  by  NY  State  law.   the  will  was  executed  in  accordance  with  the  formalities  required  is  answered  
  by  referring  to  Philippine  law.  In  fact,  the  will  was  duly  probated.  
The  hasty  application  of  Philippine  law  and  the  complete  disregard  of  the  will,    
already   probated   as   duly   executed   in   accordance   with   the   formalities   of    
Philippine  law,  is  fatal,  especially  in  light  of  the  factual  and  legal  circumstances  
PROPERTY  
here  obtaining.    
   
VALIDITY  OF  THE  FOREIGN  DIVORCE   IMMOVABLES  
Owing  to  the  nationality  principle  embodied  in  Art.15  of  the  Civil  Code—Laws    
relating   to   family   rights   and   duties,   or   to   the   status,   condition   and   legal   SCOPE  OF  LEX  SITUS:  [CFE  IME]  
capacity   of   persons   are  binding   upon   citizens   of   the   Philippines,   even   though   1. Capacity  to  take  and  transfer  immovables  
living   abroad.—only   Philippine   nationals   are   covered   by   the   policy   against   2. Formalities  of  conveyance  
absolute  divorces,  the  same  being  considered  contrary  to  our  concept  of  public   3. Essential  validity  and  effect  of  the  transfer  
policy  and  morality.  However,  aliens  may  obtain  divorces  abroad,  provided  they   4. Interpretation  and  effect  of  conveyance  
are   valid   according   to   their   national   law.   Once   proven   that   the   person   who   GR:  Lex  Situs  
initiated   the   divorce   was   no   longer   a   Filipino   citizen   when   he   obtained   the   EXC:  If  transferred  through  agreement:  1.  Lex  Loci  Intentionis,  2.  If  
divorce  from  his  spouse,  the  spouse  will  lose  her  right  to  inherit  from  him.   not,  Lex  loci  celebrationis,  or  3.  If  not,  Lex  situs  
  5. Marital  interest  in  land  
Divorce   and   its   legal   effects   may   be   recognized   in   the   Philippines   insofar   as   6. Equitable  interests  in  land  
respondent  is  concerned  in  view  of  the  nationality  principle  in  our  civil  law  on    
the  status  of  persons.  The  divorce  obtained  by  Lorenzo  from  his  first  wife  Paula   EXCEPTIONS:  [ROCSS]  
was   valid   and   recognized   in   this   jurisdiction   as   a   matter   of   comity.   Now,   the   1. Rights  and  liabilities  of  the  parties  to  a  contract  where  the  subject  matter  of  
effects  of  this  divorce  (as  to  the  succession  to  the  estate  of  the  decedent)  are   the   contract   is   an   immovable   –   governed   by   the   law   that   regulates   the  
matters  best  left  to  the  determination  of  the  trial  court.   contract  itself  
  2. Validity   and   effect   of   the   obligation   which   that   mortgage   secures   –  
VALIDIY  OF  THE  WILL   governed  by  principles  applicable  to  contracts  generally  
The  Civil  Code  provides:  “Art.  17.    The  forms   and   solemnities  of  contracts,  wills,   3. Validity   of   a   contract   to   transfer   as   opposed   to   validity   of   a   transfer   –  
and   other   public   instruments   shall   be   governed   by   the   laws   of  the   country   in   proper  law  of  the  contract  
which   they   are   executed.   When   the   acts   referred   to   are   executed   before   the   4. Problems  of  succession  –  Art.  16  and  17  
diplomatic   or   consular   officials   of   the   Republic   of   the   Philippines   in   a   foreign   5. Property   belongs   to   a   diplomat   or   to   another   sovereign   –   (Holy   See   v.  
country,   the   solemnities   established   by   Philippine   laws   shall   be   observed   in   Rosario)  
their  execution.”    
  MOVABLES  
The   clear   intent   of   Lorenzo   to   bequeath   his   property   to   his   second   wife   and   CHOSES  IN  POSSESSION  
children   by   her   is   glaringly   shown   in   the   will   he   executed.   Since   he   was   a    
foreigner,   he   is   not   covered   by   our   laws   on   “family   rights   and   duties,   status,  
 
CONFLICT  OF  LAWS                                                                                    AV  DE  TORRES   93  
ATTY.  ARIS  L.  GULAPA                                            AY  2015-­‐2016  
GR:  Lex  situs  or  the  law  of  the  State  in  which  the  property  is  located  at  the  time  of  the   57.  LAUREL  V.  GARCIA    
transaction  in  question    
  Kyna’s  notes:  
EXC:  [SHAD]   Article  16  (par  1)  is  not  the  answer  to  all  conflict  of  laws  questions  in  relation  to  
1.  SEIZURE  AND  ARREST   movables  and  immovable  i.e.  the  Roponggi  property  in  this  case.  Atty.  Gulapa  
  Law  of  the  resting  place/law  of  the  place  where  the  property  is  seized   thinks   that   there   exists   a   conflict   of   laws   issue   given   that   the   situs   of   the  
2.  DISPOSITION   property   is   in   Japan.   However,   the   lex   situs   rule   wasn’t   applied   in   this   case  
Owner:  1.  Law  of  the  resting  place,  or  2.  If  not,  Law  of  any  place  having  a   because  the  issue  in  this  case  involves  the  authority  of  the  respondent  officials  
substantial  connection  with  the  transaction  which  would  uphold  its  validity   to   validly   dispose   of   property   belonging   to   the   State.   And   the   validity   of   the  
Non-­‐owner  (when  the  one  entering  into  the  transaction  is  not  an  owner  but   procedures  adopted  to  effect  its  sale.  This  is  governed  by  Philippine  Law.  
has   authority   to   do   so):   Law   of   the   place   where   the   goods   were   actually    
situated  at  the  time  of  the  transaction  in  question   Conflict  of  law  situation  arises  only  when:  
3.  HIGH  SEAS/FOREIGN  PORT   1.  There  is  a  dispute  as  to  title  to  or  ownership  of  an  immovable,    such  that  the  
1.   Law   of   the   flag   of   the   ship,   or   2.   In   states   consisting   of   several   countries   capacity  to  take,  and  transfer  immovables,  formalities  of  conveyance,  essential  
like  the  UK,  law  of  the  place  of  registry   validity  and  effect  of  transfer,  or  the  interpretation  and  effect  of  conveyance  are  
  determined.      
CHOSES  IN  ACTION   2.   A   foreign   law   on   land   ownership   and   its   conveyance   is   asserted   to   conflict  
  with  a  domestic  law  on  the  same  manner.  Hence  the  need  to  determine  which  
DEBTS   law  should  apply.      
Voluntary/Assignment  -­‐  1.  Law  of  the  contract,  2.  If  not,  Lex  loci  intentionis,   But   according   to   Sir,   there   is   actually   a   conflict   of   laws   situation   in   this   case  
or   3.   If   not,   Law   which   has   the   most   significant   relationship   to   the   since  the  property  is  basically  Ph  gov’t  property  situated  in  Japan.    
transaction    
Involuntary/Garnishment  –  Law  of  the  State  where  jurisdiction  is  effectively   Art.  420.  The  following  things  are  property  of  public  dominion:  
exercisable  against  the  garnishee  (where  it  may  be  garnished)   (1)   Those   intended   for   public   use,   such   as   roads,   canals,   rivers,   torrents,   ports  
  and  bridges  constructed  by  the  State,  banks,  shores,  roadsteads,  and  others  of  
NEGOTIABLE  INSTRUMENTS   similar  character;  
  Law  of  the  place  where  the  instrument  is  at  the  time  of  the  transfer   (2)   Those   which   belong   to   the   State,   without   being   for   public   use,   and   are  
  intended   for   some   public   service   or   for   the   development   of   the   national  
CORPORATE  STOCKS  OR  SHARES   wealth.  (339a)  
  Between   assignor   and   assignee   –   Law   most   closely   connected   to   the    
transaction   FACTS:  
Has   the   consequence   of   changing   the   relations   of   the   parties   with   the   This   case   is   a   consolidation   of   two   petitions   for   prohibition   seeking   to   enjoin  
corporation  –  Law  of  the  place  of  incorporation   respondents   in   their   capacities   as   Head   of   the   Asset   Privatization   Trust,  
  Secretary  of  Foreign  Affairs,  Executive  Secretary,  and  members  of  the  Principal  
and   Bidding   Committees   on   the   Utilization/Disposition   of   Philippine  
Government  Properties  in  Japan  from  proceeding  with  the  bidding  for  the  sale  
of  the  “Roppongi  property”.    
 
The   subject   property   in   this   case   is   one   of   the   four   (4)   properties   in   Japan  

 
CONFLICT  OF  LAWS                                                                                    AV  DE  TORRES   94  
ATTY.  ARIS  L.  GULAPA                                            AY  2015-­‐2016  
acquired   by   the   Philippine   government   under   the   Reparations   Agreement    
entered   into   with   Japan,   the   other   lots   being:   (1)   The   Nampeidai   Property,   ISSUE:  
which   is   the   present   site   of   the   Philippine   Embassy   Chancery;   (2)   The   Kobe   Whether  or  not  Japanese  law  should  govern  the  sale  of  the  Roppongi  property  
Commercial   Property,   categorized   as   a   commercial   lot   now   being   used   as   a    
warehouse  and  parking  lot  for  the  consulate  staff;  and  (3)  The  Kobe  Residential   HELD:  
Property,  which  is  now  vacant.     No.   The   nature   of   the   Roppongi   lot   as   property   for   public   service   is   expressly  
  spelled  out.  It  is  dictated  by  the  terms  of  the  Reparations  Agreement  and  the  
The  properties  and  the  capital  goods  and  services  procured  from  the  Japanese   corresponding   contract   of   procurement   which   bind   both   the   Philippine  
government  for  national  development  projects  are  part  of  the  indemnification   government  and  the  Japanese  government.  There  can  be  no  doubt  that  it  is  of  
to   the   Filipino   people   for   their   losses   in   life   and   property   and   their   suffering   public   dominion   and   is   outside   the   commerce   of   man.   And   the   property  
during  World  War  II.     continues   to   be   part   of   the   public   domain,   not   available   for   private  
  appropriation   or   ownership   until   there   is   a   formal   declaration   on   the   part   of  
The   Roppongi   property   consists   of   the   land   and   building   "for   the   Chancery   of   the  government  to  withdraw  it  from  being  such.  
the   Philippine   Embassy".   As   intended,   it   became   the   site   of   the   Philippine    
Embassy   until   the   latter   was   transferred   to   Nampeidai   when   the   Roppongi   The  Roppongi  property  is  correctly  classified  under  paragraph  2  of  Art.  420  of  
building  needed  major  repairs.  Due  to  the  failure  of  our  government  to  provide   the  Civil  Code  as  property  belonging  to  the  State  and  intended  for  some  public  
necessary  funds,  the  Roppongi  property  has  remained  undeveloped  since  that   service.   As   property   of   public   dominion,   the   Roppongi   lot   is   outside   the  
time.     commerce  of  man.  It  cannot  be  alienated.  Its  ownership  is  a  special  collective  
  ownership  for  general  use  and  enjoyment,  an  application  to  the  satisfaction  of  
During   the   incumbency   of   President   Aquino,   a   proposal   was   made   by   the   collective  needs,  and  resides  in  the  social  group.  The  purpose  is  not  to  serve  the  
former   Philippine   Ambassador   to   Japan,   Carlos   J.   Valdez,   to   lease   the   subject   State  as  a  juridical  person,  but  the  citizens;  it  is  intended  for  the  common  public  
property   to   Kajima   Corporation,   a   Japanese   firm,   in   exchange   for   the   welfare  and  cannot  be  the  object  of  appropriation.  
construction   of   2   buildings   in   Roppongi,   1   building   in   Nampeidai,   and   the    
renovation   of   the   Philippine   Chancery   in   Nampeidai.   The   Government   did   not   It  is  not  for  the  President  to  convey  valuable  real  property  of  the  government  
act   favorably   on   said   proposal,   but   instead,   on   11   August   1986,   President   on   his   or   her   own   sole   will.   Any   such   conveyances   must   be   authorized   and  
Aquino   created   a   committee   to   study   the   disposition   or   utilization   of   Philippine   approved   by   a   law   enacted   by   the   Congress.   It   requires   executive   and  
government   properties   in   Tokyo   and   Kobe   though   AO-­‐3,   and   AO   3-­‐A   to   3-­‐D.   On   legislative  concurrence.  Moreover,  an  abandonment  of  the  intention  to  use  the  
25   July   1987,   the   President   issued   EO   296   entitling   non-­‐Filipino   citizens   or   Roppongi  property  for  public  service  and  to  make  it  patrimonial  property  under  
entities  to  avail  of  reparations'  capital  goods  and  services  in  the  event  of  sale,   Article  422  of  the  Civil  Code  must  be  definite.  Abandonment  cannot  be  inferred  
lease   or   disposition.   The   four   properties   in   Japan   including   the   Roppongi   from   the   non-­‐use   alone   specially   if   the   non-­‐use   was   attributable   not   to   the  
property   were   specifically   mentioned   in   the   first   "Whereas"   clause.   Amidst   government’s   own   deliberate   and   indubitable   will   but   to   a   lack   of   financial  
opposition   by   various   sectors,   the   Executive   branch   of   the   government   has   support  to  repair  and  improve  the  property  
been   pushing,   with   great   vigor,   its   decision   to   sell   the   reparations   properties    
starting  with  the  Roppongi  lot.     The   respondents   have   failed   to   convincingly   show   that   the   property   has  
  become   patrimonial.   The   fact   that   the   Roppongi   site   has   not   been   used   for   a  
These   petitions   claim   that   the   Roppongi   property   forms   part   of   the   public   long   time   for   actual   Embassy   service   does   not   automatically   convert   it   to  
domain  as  characterized  under  Art.  420  of  the  Civil  Code,  and  thus  cannot  be   patrimonial   property.   Any   such   conversion   happens   only   if   the   property   is  
alienated,   being   outside   the   commerce   of   man.   On   the   other   hand,   withdrawn   from   public   use.   A   property   continues   to   be   part   of   the   public  
respondents   invoke   lex   loci   rei   sitae,   or   lex   situs   and   insist   Japanese   Law   and   domain,   not   available   for   private   appropriation   or   ownership   until   there   is   a  
not  our  Civil  Code  should  apply.     formal   declaration   on   the   part   of   the   government   to   withdraw   it   from   being  
 
CONFLICT  OF  LAWS                                                                                    AV  DE  TORRES   95  
ATTY.  ARIS  L.  GULAPA                                            AY  2015-­‐2016  
such.  In  the  present  case,  the  recent  Administrative  Orders  authorizing  a  study   law   which   should   determine   who   can   acquire   the   properties   so   that   the  
of   the   status   and   conditions   of   government   properties   in   Japan   were   merely   constitutional   limitation   on   acquisition   of   lands   of   the   public   domain   to   Filipino  
directives   for   investigation   but   did   not   in   any   way   signify   a   clear   intention   to   citizens  and  entities  wholly  owned  by  Filipinos  is  inapplicable.  We  see  no  point  
dispose  of  the  properties.  Further  EO  296  does  not  declare  that  the  properties   in   belaboring   whether   or   not   this   opinion   is   correct.   Why   should   we   discuss  
lost   their   public   character,   but   merely   intends   to   make   the   properties   available   who   can   acquire   the   Roppongi   lot   when   there   is   no   showing   that   it   can   be  
to   foreigners   and   not   to   Filipinos   alone   in   case   of   a   sale,   lease   or   other   sold?  
disposition.   Ultimately,   there   is   no   law   authorizing   the   conveyance   of   the    
Roppongi  property.   58.  HOLY  SEE  V.  ROSARIO  
  G.R.  No.  101949  |  December  1,  1994  
The   respondents   try   to   get   around   the   public   dominion   character   of   the    
Roppongi   property   by   insisting   that   Japanese   law   and   not   our   Civil   Code   should   DOCTRINE:   The   right   of   a   foreign   sovereign   to   acquire   property,   real   or  
apply.   personal,  in  a  receiving  state,  necessary  for  the  creation  and  maintenance  of  its  
  diplomatic   mission,   is   recognized   in   the   1961   Vienna   Convention   on   Diplomatic  
It  is  exceedingly  strange  why  our  top  government  officials,  of  all  people,  should   Relations  (Arts.  20-­‐22).  
be   the   ones   to   insist   that   in   the   sale   of   extremely   valuable   government    
property,   Japanese   law   and   not   Philippine   law   should   prevail.   The   Japanese   The  immunity  of  the  sovereign  is  recognized  only  with  regard  to  public  acts  or  
law—its  coverage  and  effects,  when  enacted,  and  exceptions  to  its  provisions— acts  jure   imperii  of   a   state,   but   not   with   regard   to   private   acts   or   acts  jure  
is  not  presented  to  the  Court.  It  is  simply  asserted  that  the  lex  loci  rei  sitae  or   gestionis.  
Japanese   law   should   apply   without   stating   what   that   law   provides.   It   is    
assumed  on  faith  that  Japanese  law  would  allow  the  sale.   This   Court   has   considered   the   following   transactions   by   a   foreign   state   with  
  private   parties   as   acts  jure   imperii:   (1)   the   lease   by   a   foreign   government   of  
We  see  no  reason  why  a  conflict  of  law  rule  should  apply  when  no  conflict  of   apartment   buildings   for   use   of   its   military   officers;   (2)   the   conduct   of   public  
law  situation  exists.  A  conflict  of  law  situation  arises  only  when:  (1)  There   is   a   bidding  for  the  repair  of  a  wharf  at  a  United  States  Naval  Station;  and  (3)  the  
dispute  over  the  title  or  ownership  of  an  immovable,  such  that  the  capacity  to   change  of  employment  status  of  base  employees.  
take   and   transfer   immovables,   the   formalities   of   conveyance,   the   essential    
validity   and   effect   of   the   transfer,   or   the   interpretation   and   effect   of   a   FSIA   defines   a   commercial   activity   as   "either   a   regular   course   of   commercial  
conveyance,  are  to  be  determined  and  (2)  A  foreign  law  on  land  ownership  and   conduct   or   a   particular   commercial   transaction   or   act."   Furthermore,   the   law  
its  conveyance  is  asserted  to  conflict  with  a  domestic  law  on  the  same  matters.   declared  that  the  "commercial  character  of  the  activity  shall  be  determined  by  
Hence,  the  need  to  determine  which  law  should  apply.   reference   to   the   nature   of   the   course   of   conduct   or   particular   transaction   or  
In  the  instant  case,  none  of  the  above  elements  exists.   act,  rather  than  by  reference  to  its  purpose."  
The   issues   are   not   concerned   with   validity   of   ownership   or   title.   There   is   no    
question   that   the   property   belongs   to   the   Philippines.   The   issue   is   the   This  case  is  one  of  the  exceptions  to  the  lex  situs  rule.  In  this  case,  the  property  
authority  of  the  respondent  officials  to  validly  dispose  of  property  belonging   is  beyond  the  jurisdiction  of  the  state  where  such  property  is  located.  Thus,  the  
to  the  State.  And  the  validity  of  the  procedures  adopted  to  effect  its  sale.  This   lex  situs  rule  does  not  apply.  
is  governed  by  Philippine  Law.  The  rule  of  lex  situs  does  not  apply.    
The   assertion   that   the   opinion   of   the   Secretary   of   Justice   sheds   light   on   the   ALG:  The  very  nature  of  the  sale  as  imperii  was  refuted  by  the  squatters.  
relevance   of   the   lex   situs   rule   is   misplaced.   The   opinion   does   not   tackle   the    
alienability   of   the   real   properties   procured   through   reparations   nor   the  
existence   in   what   body   of   the   authority   to   sell   them.   In   discussing   who   are  
capable  of  acquiring  the  lots,  the  Secretary  merely  explains  that  it  is  the  foreign  
 
CONFLICT  OF  LAWS                                                                                    AV  DE  TORRES   96  
ATTY.  ARIS  L.  GULAPA                                            AY  2015-­‐2016  
FACTS:   accepted   principles   of   International   Law   are   adopted   by   our   Courts   and   thus  
Petitioner   is   the   Holy   See   who   exercises   sovereignty   over   the   Vatican   City   in   shall  form  part  of  the  laws  of  the  land  as  a  condition  and  consequence  of  our  
Rome,  Italy,  and  is  represented  in  the  Philippines  by  the  Papal  Nuncio;  Private   admission  in  the  society  of  nations.  
respondent,   Starbright   Sales   Enterprises,   Inc.,   is   a   domestic   corporation    
engaged   in   the   real   estate   business.   It   was   noted   in   Article   31(A)   of   the   1961   Vienna   Convention   on   Diplomatic  
This  petition  arose  from  a  controversy  over  a  parcel  of  land  consisting  of  6,000   Relations   that   diplomatic   envoy   shall   be   granted   immunity   from   civil   and  
square   meters   located   in   the   Municipality   of   Paranaque   registered   in   the   name   administrative  jurisdiction  of  the  receiving  state  over  any  real  action  relating  
of   petitioner.   Said   lot   was   contiguous   with   two   other   lots   registered   in   the   to   private   immovable   property.   The   Department   of   Foreign   Affairs   (DFA)  
name  of  the  Philippine  Realty  Corporation  (PRC).   certified   that   the   Embassy   of   the   Holy   See   is   a   duly   accredited   diplomatic  
  missionary  to  the  Republic  of  the  Philippines  and  is  thus  exempted  from  local  
The   three   lots   were   sold   to   Ramon   Licup,   through   Msgr.   Domingo   A.   Cirilos,   Jr.,   jurisdiction   and   is   entitled   to   the   immunity   rights   of   a   diplomatic   mission   or  
acting   as   agent   to   the   sellers.   Later,   Licup   assigned   his   rights   to   the   sale   to   embassy  in  this  Court.    
private   respondent.    
In   view   of   the   refusal   of   the   squatters   to   vacate   the   lots   sold   to   private   Furthermore,  it  shall  be  understood  that  in  the  case  at  bar,  the  petitioner  has  
respondent,   a   dispute   arose   as   to   who   of   the   parties   has   the   responsibility   of   bought  and  sold  lands  in  the  ordinary  course  of  real  estate  business,  surely,  the  
evicting   and   clearing   the   land   of   squatters.   Complicating   the   relations   of   the   said  transaction  can  be  categorized  as  an  act  jure  gestionis.  However,  petitioner  
parties   was   the   sale   by   petitioner   of   Lot   5-­‐A   to   Tropicana   Properties   and   has  denied  that  the  acquisition  and  subsequent  disposal  of  the  lot  were  made  
Development   Corporation   (Tropicana).   for  profit  but  claimed  that  it  acquired  said  property  for  the  site  of  its  mission  or  
Private   respondent   filed   a   complaint   with   the   RTC,   Branch   61,   Makati,   Metro   the   Apostolic   Nunciature   in   the   Philippines.  Private   respondent   failed   to  
Manila   for   annulment   of   the   sale   of   the   three   parcels   of   land,   and   specific   dispute  said  claim.  
performance  and  damages  against  petitioner,  represented  by  the  Papal  Nuncio,    
and   three   other   defendants:   namely,   Msgr.   Domingo   A.   Cirilos,   Jr.,   the   PRC   and   Lot   5-­‐A   was   acquired   by   petitioner   as   a   donation   from   the   Archdiocese   of  
Tropicana   Manila.  The  donation  was  made  not  for  commercial  purpose,  but  for  the  use  
petitioner   and   Msgr.   Cirilos   separately   moved   to   dismiss   the   complaint   —   of  petitioner  to  construct  thereon  the  official  place  of  residence  of  the  Papal  
petitioner   for   lack   of   jurisdiction   based   on   sovereign   immunity   from   suit,   and   Nuncio.   The   right   of   a   foreign   sovereign   to   acquire   property,   real   or   personal,  
Msgr.  Cirilos  for  being  an  improper  party.  An  opposition  to  the  motion  was  filed   in   a   receiving   state,   necessary   for   the   creation   and   maintenance   of   its  
by   private   respondent.   diplomatic   mission,   is   recognized   in   the   1961   Vienna   Convention   on  
the   trial   court   issued   an   order   denying,   among   others,   petitioner’s   MTD   after   Diplomatic   Relations   (Arts.   20-­‐22).   This   treaty   was   concurred   in   by   the  
finding   that   petitioner   “shed   off   [its]   sovereign   immunity   by   entering   into   the   Philippine  Senate  and  entered  into  force  in  the  Philippines  on  November  15,  
business   contract   in   question”   Petitioner   forthwith   elevated   the   matter   to   us.   1965.  
In  its  petition,  petitioner  invokes  the  privilege  of  sovereign  immunity  only  on  its    
own  behalf  and  on  behalf  of  its  official  representative,  the  Papal  Nuncio.   The  decision  to  transfer  the  property  and  the  subsequent  disposal  thereof  are  
  likewise  clothed  with  a  governmental  character.  Petitioner  did  not  sell  Lot  5-­‐A  
ISSUE:   for   profit   or   gain.   It   merely   wanted   to   dispose   off   the   same   because   the  
Whether   the   Holy   See   is   immune   from   suit   insofar   as   its   business   relations   squatters   living   thereon   made   it   almost   impossible   for   petitioner   to   use   it   for  
regarding  selling  a  lot  to  a  private  entity   the  purpose  of  the  donation.    
   
HELD:  
The   Court   held   that   Holy   See   may   properly   invoke   sovereign   immunity   for   its  
non-­‐suability.  As  expressed  in  Sec.  2,  Art  II  of  the  1987  Constitution,  generally  
 
CONFLICT  OF  LAWS                                                                                    AV  DE  TORRES   97  
ATTY.  ARIS  L.  GULAPA                                            AY  2015-­‐2016  
59.  SALVACION  V.  CENTRAL  BANK   No.  The  provisions  of  Section  113  of  Central  Bank  Circular  No.  960  and  PD  No.  
  1246,   insofar   as   it   amends   Section   8   of   R.A.   No.   6426,   are   hereby   held   to   be  
E.g.  of  involuntary  assignment   INAPPLICABLE  to  this  case  because  of  its  peculiar  circumstances.  Respondents  
  are  hereby  required  to  comply  with  the  writ  of  execution  issued  in  the  civil  case  
FACTS:     and   to   release   to   petitioners   the   dollar   deposit   of   Bartelli   in   such   amount   as  
On  February  4,  1989,  Greg  Bartelli  y  Northcott,  an  American  tourist,  coaxed  and   would  satisfy  the  judgment.  
lured   petitioner   Karen   Salvacion,   then   12   years   old   to   go   with   him   to   his    
apartment.  Therein,  Greg  Bartelli  detained  Karen  Salvacion  for  four  days,  or  up   The  questioned  law  makes  futile  the  favorable  judgment  and  award  of  damages  
to   February   7,   1989   and   was   able   to   rape   the   child   once   on   February   4,   and   that   Salvacion   and   her   parents   fully   deserve.   It   then   proceeded   to   show   that  
three  times  each  day  on  February  5,  6,  and  7,  1989.  On  February  7,  1989,  after   the  economic  basis  for  the  enactment  of  RA  No.  6426  is  not  anymore  present;  
policemen  and  people  living  nearby,  rescued  Karen,  Greg  Bartelli  was  arrested   and   even   if   it   still   exists,   the   questioned   law   still   denies   those   entitled   to   due  
and   detained   at   the   Makati   Municipal   Jail.   The   policemen   recovered   from   process   of   law   for   being   unreasonable   and   oppressive.   The   intention   of   the   law  
Bartelli  the  following  items:  1.)  Dollar  Check  No.  368,  Control  No.  021000678-­‐ may  be  good  when  enacted.  The  law  failed  to  anticipate  the  iniquitous  effects  
1166111303,   US   3,903.20;   2.)   COCOBANK   Bank   Book   No.   104-­‐108758-­‐8   (Peso   producing  outright  injustice  and  inequality  such  as  the  case  before  us.  
Acct.);   3.)   Dollar   Account   —   China   Banking   Corp.,   US$/A#54105028-­‐2;   4.)   ID-­‐  
122-­‐30-­‐8877;   5.)   Philippine   Money   (P234.00)   cash;   6.)   Door   Keys   6   pieces;   7.)   The   SC   adopted   the   comment   of   the   Solicitor   General   who   argued   that   the  
Stuffed  Doll  (Teddy  Bear)  used  in  seducing  the  complainant.     Offshore   Banking   System   and   the   Foreign   Currency   Deposit   System   were  
  designed   to   draw   deposits   from   foreign   lenders   and   investors   and,  
Basically,   Greg   Bartelli,   an   American   tourist,   was   arrested   for   committing   four   subsequently,   to   give   the   latter   protection.   However,   the   foreign   currency  
counts   of   rape   and   serious   illegal   detention   against   Karen   Salvacion.   Police   deposit  made  by  a  transient  or  a  tourist  is  not  the  kind  of  deposit  encouraged  
recovered   from   him   several   dollar   checks   and   a   dollar   account   in   the   China   by   PD   Nos.   1034   and   1035   and   given   incentives   and   protection   by   said   laws  
Banking   Corp.   He   was,   however,   able   to   escape   from   prison.   In   a   civil   case   filed   because   such   depositor   stays   only   for   a   few   days   in   the   country   and,  
against   him,   the   trial   court   awarded   Salvacion   moral,   exemplary   and   attorney’s   therefore,   will   maintain   his   deposit   in   the   bank   only   for   a   short   time.  
fees  amounting  to  almost  P1,000,000.00.     Considering   that   Bartelli   is   just   a   tourist   or   a   transient,   he   is   not   entitled   to  
  the   protection   of   Section   113   of   Central   Bank   Circular   No.   960   and   PD   No.  
Salvacion   tried   to   execute   the   judgment   on   the   dollar   deposit   of   Bartelli   with   1246  against  attachment,  garnishment  or  other  court  processes.  
the   China   Banking   Corp.   but   the   latter   refused   arguing   that   Section   11   of    
Central   Bank   Circular   No.   960   exempts   foreign   currency   deposits   from   In   fine,   the   application   of   the   law   depends   on   the   extent   of   its   justice.  
attachment,  garnishment,  or  any  other  order  or  process  of  any  court,  legislative   Eventually,  if  we  rule  that  the  questioned  Section  113  of  Central  Bank  Circular  
body,   government   agency   or   any   administrative   body   whatsoever.   Salvacion   No.   960   which   exempts   from   attachment,   garnishment,   or   any   other   order   or  
therefore  filed  this  action  for  declaratory  relief  in  the  Supreme  Court.     process  of  any  court,  legislative  body,  government  agency  or  any  administrative  
  body   whatsoever,   is   applicable   to   a   foreign   transient,   injustice   would   result  
ISSUE:   especially   to   a   citizen   aggrieved   by   a   foreign   guest   like   accused   Greg   Bartelli.  
Should  Section  113  of  Central  Bank  Circular  No.  960  and  Section  8  of  R.A.  No.   This  would  negate  Article  10  of  the  New  Civil  Code  which  provides  that  “in  case  
6426,   as   amended   by   PD   1246,   otherwise   known   as   the   Foreign   Currency   of   doubt   in   the   interpretation   or   application   of   laws,   it   is   presumed   that   the  
Deposit  Act  be  made  applicable  to  a  foreign  transient?   lawmaking  body  intended  right  and  justice  to  prevail.  
   
HELD:    
 
 
 
CONFLICT  OF  LAWS                                                                                    AV  DE  TORRES   98  
ATTY.  ARIS  L.  GULAPA                                            AY  2015-­‐2016  
  tax  on  inheritance.  Thus,  if  the  issue  of  the  case  is  not  purely  personal,  do  not  
apply  Article  16  (par  2).    
VIII.  PERSONAL  LAW,  NATIONALITY  AND  DOMICILE  
   
  FACTS:  
DOMICILE   This  is  an  appeal  from  a  final  order  of  the  CFI  Manila,  requiring  the  register  of  
  deeds   of   the   City   of   Manila   to   cancel   certificates   of   title   Nos.   20880,   28336   and  
1.  Domicile  by  origin   28331,  covering  lands  located  in  the  City  of  Manila,  Philippine  Islands,  and  issue  
EXC:   Surely   a   mere   intention   to   return   to   their   former   homes,   a   in   lieu   thereof   new   certificates   of   transfer   of   title   in   favor   of   Allison   D.   Gibbs  
consummation  every  humane  person  desires  for  them,  not  realized  and   without   requiring   him   to   present   any   document   showing   that   the   succession  
which   may   never   be   realized   should   not   prevent   them,   under   the   tax   due   under   Article   XI   of   Chapter   40   of   the   Administrative   Code   has   been  
circumstances,   from   acquiring   a   residence   for   voting   purposes.   The   paid.    
court   didn’t   consider   their   domicile   of   origin   because   the   intent   to    
return  (future  intent  to  return)  will  not  necessarily  happen.  (Alcantara   The   order   of   the   court   of   March   10,   1931,   recites   that   the   parcels   of   land  
v.  Secretary  of  Interior)   covered   by   said   certificates   of   title   formerly   belonged   to   the   conjugal  
  partnership  of  Allison  D.  Gibbs  and  Eva  Johnson  Gibbs;  that  Eva  died  intestate  
2.  Domicile  by  choice   in   Palo   Alto,   California,   on   November   28,   1929;   that   at   the   time   of   her   death  
1)  residence  or  bodily  presence  in  the  new  locality   she   and   her   husband   were   citizens   of   the   State   of   California   and   domiciled  
2)   an   intention   to   remain   there   (animus   manendi)   for   an   indefinite   therein.    
period  of  time,  and    
3)  an  intention  to  abandon  the  old  domicile.  (Gallego  v.  Vera)   It  appears  further  from  said  order  that  Allison  was  appointed  administrator  of  
  the  state  of  his  said  deceased  wife  in  a  case  in  the  same  court,  entitled  "In  the  
Abandonment  of  domicile   Matter   of   the   Intestate   Estate   of   Eva   Johnson   Gibbs,   Deceased";   that   in  
1. A  deliberate  and  provable  choice  of  a  new  domicile,   intestate  proceedings,  Allison  on  September  22,1930,  filed  an  ex  parte  petition  
2. Coupled   with   actual   residence   in   the   place   chosen,   with   a   in  which  he  alleged  "that  the  parcels  of  land  hereunder  described  belong  to  the  
declared   or   provable   intent   that   it   should   be   one's   fixed   conjugal  partnership  of  Allison  and  his  wife,  Eva",  describing  in  detail  the  3  facts  
and   permanent   place   of   abode,   one's   home.   (Vellila   v.   here  involved;  and  further  alleging  that  his  said  wife,  a  citizen  and  resident  of  
Posadas)   California,   died   on   November   28,1929;   that   in   accordance   with   the   law   of  
  California,   the   community   property   of   spouses   who   are   citizens   of   California,  
3.  Constructive  domicile   upon   the   death   of   the   wife   previous   to   that   of   the   husband,   belongs   absolutely  
  to  the  surviving  husband  without  administration;  that  the  conjugal  partnership  
60.  GIBBS  V.  GOVERNMENT   of   Allison   and   Eva   Gibbs,   deceased,   has   no   obligations   or   debts   and   no   one   will  
  be   prejudiced   by   adjucating   said   parcels   of   land   (and   17   others   not   here  
DOCTRINE:  The  second  paragraph  of  article  10  [now  article  16]  can  be  invoked   involved)   to   be   the   absolute   property   of   Allison   as   sole   owner.   The   court  
only   when   the   deceased   was   vested   with   a   descendible   interest   in   property   granted  said  petition  and  on  September  22,  1930,  entered  a  decree  adjucating  
within  the  jurisdiction  of  the  Philippine  Islands.   to  Allison  as  the  sole  and  absolute  owner  of  said  lands,  applying  section  1401  of  
  the  Civil  Code  of  California.  Gibbs  presented  this  decree  to  the  register  of  deeds  
ALG:   Why   would   art.   16   not   apply?   Art.   16   is   limited   to   purely   personal   of  Manila  and  demanded  that  the  latter  issue  to  him  a  TCT.    
relations.   The   issue   involved   is   tax   on   inheritance.   This   case   involved   merely    
some   aspect   of   personal   law—there   is   a   succession   aspect   but   the   main   issue   is   Section  1547  of  Article  XI  of  Chapter  40  of  the  Administrative  Code  provides  in  

 
CONFLICT  OF  LAWS                                                                                    AV  DE  TORRES   99  
ATTY.  ARIS  L.  GULAPA                                            AY  2015-­‐2016  
part  that:      
Registers   of   deeds   shall   not   register   in   the   registry   of   property   any   document   3.  A  parcel  of  land  in  the  City  of  Manila,  represented  by  TCT  No.  28331,  dated  
transferring  real  property  or  real  rights  therein  or  any  chattel  mortgage,  by  way   April   6,   1927,   which   it   states   "that   Allison   D.   Gibbs   married   to   Eva   Johnson  
of   gifts   mortis   causa,   legacy   or   inheritance,   unless   the   payment   of   the   tax   fixed   Gibbs"  is  the  owner  of  the  land  described  therein;  that  said  Eva  Johnson  Gibbs  
in   this   article   and   actually   due   thereon   shall   be   shown.   And   they   shall   died   intestate   on   November   28,   1929,   living   surviving   her   husband,   the  
immediately   notify   the   Collector   of   Internal   Revenue   or   the   corresponding   appellee,  and  2  sons,  Allison  J.  Gibbs,  now  age  25  and  Finley  J.  Gibbs,  now  aged  
provincial  treasurer  of  the  non-­‐payment  of  the  tax  discovered  by  them.   22,  as  her  sole  heirs  of  law.    
   
Acting   upon   the   authority   of   said   section,   the   register   of   deeds   of   the   City   of   Article   XI   of   Chapter   40   of   the   Administrative   Code   entitled   "Tax   on  
Manila,   declined   to   accept   as   binding   said   decree   of   court   of   September   inheritances,  legacies  and  other  acquisitions  mortis  causa"  provides  in  section  
22,1930,   and   refused   to   register   the   transfer   of   title   of   the   said   conjugal   1536   that   "Every   transmission   by   virtue   of   inheritance   ...   of   real   property   ...  
property   to   Allison   on   the   ground   that   the   corresponding   inheritance   tax   had   shall  be  subject  to  the  following  tax."    
not   been   paid.   Thereupon,   under   date   of   December   26,   1930,   Allison   filed   in    
the   said   court   a   petition   for   an   order   requiring   the   said   register   of   deeds   "to   ISSUE:  
issue  the  corresponding  titles"  to  Gibbs  without  requiring  previous  payment  of   WON  Eva  Johnson  Gibbs  at  the  time  of  her  death  the  owner  of  a  descendible  
any  inheritance  tax.  After  due  hearing  of  the  parties,  the  court  reaffirmed  said   interest  in  the  Philippine  lands  above-­‐mentioned?  (YES)      
order  of  September  22,  1930,  and  entered  the  order  of  March  10,  1931,  which   WON   Register   of   Deeds   erred   in   declining   to   register   the   transfer   title   of   the  
is  under  review  on  this  appeal.     conjugal  property  on  the  ground  of  unpaid  inheritance  tax?  (NO)      
   
On  January  3,  1933,  this  court  remanded  the  case  to  the  court  of  origin  for  new   HELD:  
trial   upon   additional   evidence   in   regard   to   the   pertinent   law   of   California   in   The  second  paragraph  Article  10  [now  Art.  16]  of  the  Civil  Code  provides:  
force  at  the  time  of  the  death  of  Mrs.  Gibbs,  also  authorizing  the  introduction   Nevertheless,   legal   and   testamentary   successions,   in   respect   to   the   order   of  
of   evidence   with   reference   to   the   dates   of   the   acquisition   of   the   property   succession  as  well  as  to  the  amount  of  the  successional  rights  and  the  intrinsic  
involved  in  this  suit  and  with  reference  to  the  California  law  in  force  at  the  time   validity   of   their   provisions,   shall   be   regulated   by   the   national   law   of   the   person  
of   such   acquisition.   The   case   is   now   before   us   with   the   supplementary   whose  succession  is  in  question,  whatever  may  be  the  nature  of  the  property  or  
evidence.     the  country  in  which  it  may  be  situated.  
   
For   the   purposes   of   this   case,   the   Court   considered   the   following   facts   as   The  second  paragraph  of  article  10  applies  only  when  a  legal  or  testamentary  
established  by  the  evidence  or  the  admissions  of  the  parties:  Allison  has  been   succession   has   taken   place   in   the   Philippines   and   in   accordance   with   the   law   of  
continuously,   since   the   year   1902,   a   citizen   of   the   State   of   California   and   the   Philippine   Islands;   and   the   foreign   law   is   consulted   only   in   regard   to   the  
domiciled   therein;   that   he   and   Eva   were   married   at   Columbus,   Ohio,   in   July   order  of  succession  or  the  extent  of  the  successional  rights;  in   other   words,   the  
1906;   that   there   was   no   ante-­‐nuptial   marriage   contract   between   the   parties;   second   paragraph   of   article   10   can   be   invoked   only   when   the   deceased   was  
that   during   the   existence   of   said   marriage   the   spouses   acquired   the   following   vested   with   a   descendible   interest   in   property   within   the   jurisdiction   of   the  
lands,  among  others,  in  the  Philippine  Islands,  as  conjugal  property:     Philippine  Islands.  
1.  A  parcel  of  land  in  the  City  of  Manila  represented  by  TCT  No.  20880,  dated    
March  16,  1920,  and  registered  in  the  name  of  "Allison  D.  Gibbs  casado  con  Eva   In  the  case  of  Clarke  vs.  Clarke,  the  court  said:  
Johnson   Gibbs".  2.   A   parcel   of   land   in   the   City   of   Manila,   represented   by   TCT   It  is  principle  firmly  established  that  to  the  law  of  the  state  in  which  the  land  is  
No.  28336,  dated  May  14,  1927,  in  which  it  is  certified  "that  spouses  Allison  D.   situated   we   must   look   for   the   rules   which   govern   its   descent,   alienation,   and  
Gibbs  and  Eva  Johnson  Gibbs  are  the  owners  in  fee  simple"  of  the  land  therein   transfer,  and  for  the  effect  and  construction  of  wills  and  other  conveyances.  
described.      
 
CONFLICT  OF  LAWS                                                                                    AV  DE  TORRES   100  
ATTY.  ARIS  L.  GULAPA                                            AY  2015-­‐2016  
This   fundamental   principle   is   stated   in   the   first   paragraph   of   article   10   of   our   FACTS:  
Civil  Code  as  follows:  "Personal  property  is  subject  to  the  laws  of  the  nation  of   The   petitioners   allege   that   they   are   qualified   voters   residing   at   Culion   Leper  
the   owner   thereof;   real   property   to   the   laws   of   the   country   in   which   it   is   Colony,   Culion,   Palawan,   having   voted   in   previous   elections   in   the   Philippine  
situated.”   Islands;   that   in   a   public   mass   meeting   held   on   April   5,   1935,   they   adopted   a  
  resolution   demanding   the   right   to   vote   in   the   plebiscite   and   requesting   that  
Under   this   broad   principle,   the   nature   and   extent   of   the   title   which   vested   in   electoral  precincts  be  established  within  the  radius  of  the  Culion  Leper  Colony  
Mrs.   Gibbs   at   the   time   of   the   acquisition   of   the   community   lands   here   in   in   order   that   the   qualified   voters   therein   could   register,   which   resolution   was  
question  must  be  determined  in  accordance  with  the  lex  rei  sitae.  It  is  admitted   sent   to   his   Excellency,   the   Governor-­‐General,   who   referred   it   to   the   Honorable,  
that   the   Philippine   lands   here   in   question   were   acquired   as   community   the   Secretary   of   the   Interior;   that   the   Department   of   the   Interior,   through   its  
property   of   the   conjugal   partnership   of   the   appellee   and   his   wife.   Under   the   legal  division,  ruled  that  no  new  electoral  precincts  could  be  created  at  Culion  
law   of   the   Philippine   Islands,   she   was   vested   of   a   title   equal   to   that   of   her   Leper   Colony   inasmuch   as   the   plebiscite   is   treated   as   and   considered   as   a  
husband.   It   results   that   the   wife   of   the   appellee   was,   by   the   law   of   the   special   election;   that   in   view   of   this   ruling   the   petitioners   requested,   by  
Philippine   Islands,   vested   of   a   descendible   interest,   equal   to   that   of   her   telegram,   the   Interior   Department   to   authorize   the   Balala   Electoral   Board   of  
husband,   in   the   Philippine   lands   covered   by   certificates   of   title   Nos.   20880,   Inspectors,   Culion,   Palawan,   to   register   the   qualified   voters   of   Culion   Leper  
28336  and  28331,  from  the  date  of  their  acquisition  to  the  date  of  her  death.   Colony;   that   this   request   was   refused   upon   the   ground   that   the   petitioners  
  were   not   bona   fide   residents   of   Culion,   Palawan;   that   on   April   23,   1935,   the  
The   descendible   interest   of   Eva   Johnson   Gibbs   in   the   lands   aforesaid   was   petitioners   Juan   L.   Alcantara,   Miguel   Valdes,   Adolfo   Almeda   and   Dionisio  
transmitted   to   her   heirs   by   virtue   of   inheritance   and   this   transmission   plainly   Pañgilinan,   accompanied   by   Attorney   Martin   Miras,   appeared   before   the  
falls   within   the   language   of   section   1536   of   Article   XI   of   Chapter   40   of   the   chairman   of   the   Balala   Electoral   Board   of   Inspectors   and   requested   him   to  
Administrative   Code   which   levies   a   tax   on   inheritances.   It   is   unnecessary   in   this   register   and   inscribe   them   in   the   officials   list   of   qualified   voters   in   order   that  
proceeding   to   determine   the   "order   of   succession"   or   the   "extent   of   the   they   might   vote   on   May   14,   1935,   and   that   their   request   was   denied   on   the  
successional  rights"  (article  10,  Civil  Code,  supra)  which  would  be  regulated  by   ground   that   no   specific   instructions   to   register   them   had   been   received   from  
section  1386   of   the   Civil   Code   of   California   which   was   in   effect   at   the  time  of   the  Department  of  the  Interior.    
the  death  of  Mrs.  Gibbs.    
  The  principal  allegation  of  the  respondents,  by  way  of  special  defense,  is  "that  
61.  ALCANTARA  V.  SECRETARY  OF  INTERIOR   the   herein   petitioners   are   not   qualified   voters,   because   they   shall   not   have  
  been   residents   of   Culion   for   six   months   next   preceding   the   day   of   voting,   for  
DOCTRINE:  They  are  not  permitted  to  return  to  their  former  homes  to  vote.  They   they   have   not   acquired   residence   in   Culion   as   they   are   confined   therein   as  
are   not   allowed   to   visit   their   former   homes   even   though   they   have   been   lepers   against   their   will,   and   they   have   no   intention   to   permanently   reside  
separated  from  near  and  dear  relatives  who  are  not  afflicted  as  they  are.  Surely   there  (sections  430-­‐431  of  the  Administrative  Code  as  finally  amended  by  Acts  
a   mere   intention   to   return   to   their   former   homes,   a   consummation   every   Nos.  3387,  sec.  1,  and  4112,  secs.  1  to  3);  and  in  view  thereof,  the  respondent  
humane  person  desires  for  them,  not  realized  and  which  may  never  be  realized   Secretary  of  the  Interior  has  ruled  that  the  petitioners  are  not  qualified  voters  
should  not  prevent  them,  under  the  circumstances,  from  acquiring  a  residence   and  therefore  cannot  be  registered  under  the  law."    
for  voting  purposes.      
  At   present   the   nearest   approach   to   a   constitution   that   we   have   in   the  
The   court   didn’t   consider   their   domicile   of   origin   because   the   intent   to   return   Philippines  in  our  Organic  Act,  the  Jones  Law,  enacted  August  29,  1916,  by  the  
(future  intent  to  return)  will  not  necessarily  happen.   Congress   of   the   United   States.   "The   organic   law   (or   Act)   of   a   territory   takes   the  
  place   of   a   constitution   as   the   fundamental   law   of   the   local   government."   The  
  only  provisions  contained  in  that  law  as  to  the  qualification  of  voters  reads  as  
follows:    
 
CONFLICT  OF  LAWS                                                                                    AV  DE  TORRES   101  
ATTY.  ARIS  L.  GULAPA                                            AY  2015-­‐2016  
   
SEC.   15.   That   at   the   first   election   held   pursuant   to   this   Act,   the   qualified   SEC.  432.  Disqualifications.  —  The  following  persons  shall  be  disqualified  from  
electors   shall   be   those   having   the   qualifications   of   voters   under   the   present   voting:    
law;   thereafter   and   until   otherwise   provided   by   the   Philippine   Legislature   (a)  Any  person  who,  since  the  thirteenth  day  of  August,  eighteen  hundred  and  
herein   provided   for   the   qualifications   of   voters   for   Senators   and   ninety-­‐eight,   has   been   sentenced   by   final   judgment   to   suffer   not   less   than  
Representatives  in  the  Philippines  and  all  officers  elected  by  the  people  shall  be   eighteen  months  of  imprisonment,  such  disability  not  having  been  removed  by  
as  follows:     plenary  pardon.  (b)  Any  persons  who  has  violated  an  oath  of  allegiance  taken  
  by   him   to   the   United   States.  (c)   Insane   or   feeble-­‐minded   persons.  (d)   Deaf-­‐
Every   male   person   who   is   not   a   citizen   or   subject   of   a   foreign   power   twenty-­‐   mutes  who  cannot  read  and  write.    
one  years  of  age  or  over  (except  insane  and  feeble-­‐minded  persons  and  those   (e)  Electors  registered  under  subsection  (c)  of  the  next  preceding  section  who,  
convicted   in   a   court   of   competent   jurisdiction   of   an   infamous   offense   since   the   after   failing   to   make   sworn   statement   to   the   satisfaction   of   the   board   of  
thirteenth   day   of   August,   eighteen   hundred   and   ninety-­‐eight),   who   shall   have   inspectors  at  any  of  its  two  meetings  for  registration  and  revision,  that  they  are  
been   a   resident   of   the   Philippines   for   one   year   and   of   the   municipality   in   which   incapacitated   for   preparing   their   ballots   due   to   permanent   physical   disability,  
he  shall  offer  to  voter  for  six  months  next  preceding  the  day  of  voting,  and  who   present   themselves   at   the   hour   of   voting   as   incapacitated,   irrespective   of  
is  comprised  within  one  of  the  following  classes:     whether  such  incapacity  be  real  or  feigned.    
(a)  Those  who  under  existing  law  are  legal  voters  and  have  exercised  the  right    
of  suffrage.     ISSUE:  
(b)   Those   who   own   real   property   to   the   value   of   500   pesos,   or   who   annually   Whether  or  not  the  petitioners  have  acquitted  a  residence  for  voting  purposes  
pay   30   pesos   or   more   of   the   established   taxes.  (c)   Those   who   are   able   to   red   in  the  municipality  in  which  they  desire  to  vote.    
and  write  either  Spanish,  English,  or  a  native  language.      
Under   the   authority   conferred   upon   it   by   the   above   quoted   section   the   HELD:  
Philippine  Legislature  has  prescribed  the  qualifications  and  disqualifications  of   In   some   of   the   states   there   is   a   constitutional   provision   to   the   effect   that   for  
voters  in  sections  431  and  432  of  the  Revised  Administrative  Code,  which  read   the   purpose   of   voting   no   person   shall   be   deemed   to   have   gained   or   lost   a  
as  follows:     residence  while  a  student  at  any  seminary  of  learning.  Under  such  a  provision  it  
  has   been   held   "that   a   student   does   not   acquire   a   residence   for   voting   purposes  
SEC.  431.   Qualifications  prescribed  for  voters.   —   Every   male   or   female   person   merely   by   attending   such   an   institution."   In   addition   to   such   provisions   as   to  
who  is  not  a  citizen  or  subject  of  a  foreign  power,  twenty-­‐one  years  of  age  or   students,  constitutions  of  some  states  provide  that  "For  the  purpose  of  voting,  
over,  who  shall  have  been  a  resident  of  the  Philippines  for  one  year  and  of  the   no  person  shall  be  deemed  to  have  gained  or  lost  a  residence  by  reason  shall  
municipality   in   which   he   shall   offer   to   vote   for   six   months   next   preceding   the   be   deemed   to   have   gained   or   lost   a   residence   by   reason   of   his   presence   or  
day  of  voting  is  entitled  to  vote  in  all  elections  if  comprised  within  either  of  the   absence  while  ...  kept  at  any  almshouse  or  other  asylum  at  public  expense;  ...  
following  three  classes:     ."Under   such   a   provision   the   rule   in   some   jurisdictions   is   "that   inmates   of  
  soldiers'  homes,  by  going  to  and  residing  in  such  home,  neither  lose  their  old,  
(a)   Those   who,   under   the   laws   in   force   in   the   Philippine   Islands   upon   the   nor   gain   a   new,   residence,   though   they   intend   to   reside   in   the   home  
twenty-­‐eight   day   of   August,   nineteen   hundred   and   sixteen,   were   legal   voters   permanently.   Hence   they   are   not   entitled   to   vote   except   at   their   place   of  
and   had   exercised   the   right   of   suffrage.  (b)   Male   persons   who   own   real   residence  before  becoming  such  inmates.    
property  to  the  value  of  five  hundred  pesos,  declared  in  their  name  for  taxation    
purposes  for  a  period  of  not  less  than  one  year  prior  to  the  date  of  the  election,   In  other  jurisdictions,  however,  a  contrary  conclusion  has  been  reached,  upon  
or  who  annually  pay  thirty  pesos  or  more  of  the  established  taxes.     the   theory   that   under   such   a   constitutional   provision   an   inmate   such   an  
(c)  Those  who  are  able  to  read  and  write  either  Spanish,  or  English,  or  a  native   institution  may  acquire  a  residence  at  the  home.    
language.    
 
CONFLICT  OF  LAWS                                                                                    AV  DE  TORRES   102  
ATTY.  ARIS  L.  GULAPA                                            AY  2015-­‐2016  
In   the   absence   of   such   a   constitutional   prohibition   the   rule   is   that   a   permanent   not  be  considered  as  having  resided  for  six  months  next  preceding  the  day  of  
member  of  a  soldiers'  home  has  a  residence  at  such  home  for  the  purpose  of   voting   because   "they   have   not   acquired   residence   in   Culion   as   they   are  
voting.     confined   therein   as   lepers   against   their   will,   and   they   have   no   intention   to  
  permanently  reside  there."    
There   being   no   such   provisions   or   prohibitions   in   the   Jones   Law   nor   in   the   In   ruling   out   this   objection,   and   in   holding   that   these   lepers   were   entitled   to  
sections  of  the  Revised  Administrative  Code,  quoted  above,  we  see  no  reason   vote,  this  court  said:  
for   applying   in   this   jurisdiction   the   legal   doctrine   of   the   courts   of   the   states   There  are  a  large  number  of  people  confined  in  the  Culion  Leper  Colony.  They  
which  have  adopted  such,  or  similar,  constitutional  provisions.     are   not   permitted   to   return   to   their   former   homes   to   vote.   They   are   not  
  allowed   to   visit   their   former   homes   even   though   they   have   been   separated  
There  are  a  large  number  of  people  confined  in  the  Culion  Leper  Colony.  They   from  near  and  dear  relatives  who  are  not  afflicted  as  they  are.  Why  split  hairs  
are   not   permitted   to   return   to   their   former   homes   to   vote.   They   are   not   over  the  meaning  of  residence  for  voting  purposes  under  such  circumstances?  
allowed   to   visit   their   former   homes   even   though   they   have   been   separated   Assuming  that  the  petitioners  intend  to  return  to  their  former  homes  if  at  some  
from  near  and  dear  relatives  who  are  not  afflicted  as  they  are.  Why  split  hairs   future   time   they   are   cured,   this   intention   does   not   necessarily   defeat   their  
over  the  meaning  of  residence  for  voting  purposes  under  such  circumstances?   residence   before   they   actually   do   return   if   they   have   been   residents   of   the  
Assuming  that  the  petitioners  intend  to  return  to  their  former  homes  if  at  some   Philippine   Islands   for   one   year   and   of   the   municipality   in   which   they   offer   to  
future   time   they   are   cured,   this   intention   does   not   necessarily   defeat   their   vote  for  six  months  next  preceding  the  day  of  voting.  Surely  a  mere  intention  
residence   before   they   actually   do   return   if   they   have   been   residents   "of   the   to   return   to   their   former   homes,   a   consummation   every   humane   person  
Philippine   Islands   for   one   year   and   of   the   municipality   in   which   they   offer   to   desires   for   them,   not   realized   and   which   may   never   be   realized   should   not  
vote  for  six  months  next  preceding  the  day  of  voting."  Surely  a  mere  intention   prevent   them,   under   the   circumstances,   from   acquiring   a   residence   for   voting  
to  return  to  their  former  homes,  a  consummation  every  humane  person  desires   purposes.    
for   them,   not   realized   and   which   may   never   be   realized   should   not   prevent    
them,  under  the  circumstances,  from  acquiring  a  residence  for  voting  purposes.     The  ruling  in  the  alcantara  case  is  decisive  as  to  the  right  of  the  lepers  to  vote.  
  It   was   there   said   that   they   can   vote   in   the   places   where   they   are   confined  
This  court  is  of  the  opinion  that,  under  our  liberal  law,  such  of  the  petitioners  as   provided   that   they   evince   their   desire   to   do   so   and   had   resided   there   for   at  
have  been  residents  of  the  Philippine  Islands  for  one  year  and  residents  for  six   least  a  period  of  six  months.  It  is  true  that  the  ruling  was  given  in  connection  
months   in   the   municipality   in   which   they   desire   to   vote   and   have   the   other   with  a  plebiscite,  but  this  matter  is  of  no  moment  because  the  paramount  issue  
qualifications  prescribed  for  voters  in  section  431  of  the  Revised  Administrative   therein  involved  has  reference  to  their  qualification  from  the  point  of  view  of  
Code   and   who   have   none   of   the   disqualifications   prescribed   in   section   432   of   residence.    
the  same  Code  were  entitled  to  register  and  vote  in  the  plebiscite  of  May  14,    
1935.  Having  reached  this  conclusion  and  being  unable  to  determine  from  the   62.  VELLILA  V.  POSADAS  
record   whether   the   petitioners   have   the   prescribed   qualifications   for   voters    
and   none   of   the   prescribed   disqualifications   this   court   on   May   11,   1935,   sent   DOCTRINE:   The   record   before   us   leaves   no   doubt   in   our   minds   that   the   "usual  
the  above  mentioned  telegram  to  the  parties  in  this  case.     residence"   of   this   unfortunate   man,   whom   appellant   describes   as   a   "fugitive"  
  and   "outcast",   was   in   Manila   where   he   had   lived   and   toiled   for   more   than   a  
HELD:   quarter   of   a   century,   rather   than   in   any   foreign   country   he   visited   during   his  
Petitioners,   who   were   lepers   residing   at   the   Culion   Leper   Colony,   demanded   wanderings  up  to  the  date  of  his  death  in  Calcutta.  To  effect  the  abandonment  
that   they   be   granted   the   right   to   vote   in   the   plebiscite   (on   the   vital   question   of   of   one's   domicile,   there   must   be   a   deliberate   and   provable   choice   of   a   new  
the  acceptance  or  rejection  of  the  Constitution  for  the  Commonwealth  of  the   domicile,  coupled  with  actual  residence  in  the  place  chosen,  with  a  declared  or  
Philippine  Islands)  to  be  held  on  May  14,  1935  but  was  refused  by  the  Secretary   provable   intent   that   it   should   be   one's   fixed   and   permanent   place   of   abode,  
of   Interior   on   the   ground   that   they   were   not   qualified   voters   in   that   they   could  
 
CONFLICT  OF  LAWS                                                                                    AV  DE  TORRES   103  
ATTY.  ARIS  L.  GULAPA                                            AY  2015-­‐2016  
one's  home.     the  Philippine  Islands  in  1902  or  1903  and  engaged  actively  in  business  in  these  
   Islands  up  to  the  time  of  his  death  in  Calcutta,  India,  on  February  18,  1931.      
ALG:  It  was  not  proved  that  he  acquired  another  domicile.    
  He   had   no   business   elsewhere   and   at   the   time   of   his   death   left   an   estate  
FACTS:     consisting   principally   of   bonds   and   shares   of   stock   of   corporations   organized  
Arthur   Graydon   Moody   died   in   Calcutta,   India,   on   February   18,   1931   and   under   the   laws   of   the   Philippine   Islands,   bank   deposits   and   other   intangibles  
 executed  in  the  Philippine  Islands  a  will,  by  virtue  of  which,  he  bequeathed  all   and  personal  property  valued  by  the  commissioners  of  appraisal  and  claims  at  
his   property   to   his   only   sister,   Ida   M.   Palmer,   who   then   was   and   still   is   a   citizen    P609,767.58  and  by  the  CIR  for  the  purposes  of  inheritance  tax  at  P653,657.47.  
and  resident  of  the  State  of  New  York,  USA.      
   
A   petition   for   appointment   of   special   administrator   of   the   estate   of   the   All   of   said   property   at   the   time   of   his   death   was   located   and   had   its   situs  
deceased  Arthur  Graydon  Moody  was  filed  by  W.  Maxwell  Thebaut  with  the  CFI   within   the   Philippine   Islands.  So  far  as  this  record  shows,  he  left  no  property  of  
of  Manila.   any  kind  located  anywhere  else.      
   
Subsequently   a   petition   was   filed   by   Ida   M.   Palmer,   asking   for   the   probate   of   In  his  will  executed  without  date  in  Manila  in  accordance  with  the  formalities  of  
said   will   of   the   deceased   Moody,   and   the   same   was,   after   hearing,   duly   the  Philippine  law,  in  which  he  bequeathed  all  his  property  to  his  sister,  Ida  M.  
probated  by  the  court.       Palmer,   he   stated:   "I,   Arthur   G.   Moody,   a   citizen   'of   the   United   States   of  
Ida   M.   Palmer   was   declared   to   be   the   sole   and   only   heiress   of   the   deceased   America,   residing   in   the   Philippine   Islands,   hereby   publish   and   declare   the  
Arthur   Graydon   Moody;   and   that   during   the   hearing   for   the   declaration   of   following  as  my  last  Will  and  Testament  *  *  *"  
heirs,   Ida   M.   Palmer   presented   as   evidence   a   letter   dated   February   28,   1925,   Plaintiff  contends:  "That  there  is  no  valid  law  or  regulation  of  the  Government  
and  addressed  to  her  by  Moody.   of  the  Philippine  Islands  under  or  by  virtue  of  which  any  inheritance  tax  may  be  
      levied,   assessed   or   collected   upon   transfer,   by   death   and   succession,   of  
The  property  left  by  Moody  consisted  principally  of  bonds  and  shares  of  stock   intangible   personal   properties   of   a   person   not   domiciled   in   the   Philippine  
of   corporations   organized   under   the   laws   of   the   Philippine   Islands,   bank   Islands,   and   the   levy   and   collection   by   defendant   of   inheritance   tax   xxx...  
deposits   and   other   personal   properties,   as   are   more   fully   shown   in   the   constituted   and   constitutes   the   taking   and   deprivation   of   property   without   due  
inventory  filed  by  the  special  administrator  (Thebaut).       process   of   law.   It   is   alleged   in   the   complaint   that   at   the   time   of   his   death,  
  Arthur  G.  Moody  was  a  "non-­‐resident  of  the  Philippine  Islands".    
The   BIR   prepared   for   the   estate   an   inheritance   tax   return   and   an   income   tax    
return  for  the  fractional  period  from  January  1,  1931  to  June  30,  1931       ISSUES:  
  Whether  Moody  was  legally  domiciled  in  the  Philippine  Islands  on  the  day  of  his  
The  estate  of  the  late  Moody  paid  under  protest  the  sum  of  P50,000  on  July  22,   death?  YES  
1931,  and  the  other  sum  of  P40,019.75  on  January  19,  1932,  making  a  total  of   Whether  Moody’s  estate  is  subject  to  inheritance  tax?  YES    
P90,019.75,  of  which  P77,018.39   covers   the   assessment  for  inheritance  tax  and    
the   sum   of   P13,001.41   covers   the   assessment   for   income   tax   against   said   HELD:  
estate.       The  Supreme  Court  affirmed  the  judgment  of  the  trial  court,  first,  because  the  
  property   in   the   estate   of   Arthur   G.   Moody   at   the   time   of   his   death   was   located  
The  CIR  overruled  the  protest  made  by  Ida  M.  Palmer  through  her  attorney.       and   had   its   situs   within   the   Philippine   Islands   and,   second,   because   his   legal  
  domicile  up  to  the  time  of  his  death  was  within  the  Philippine  Islands.  
In   addition   to   the   facts,   it   appears   that   Moody,   an   American   citizen,   came   to    

 
CONFLICT  OF  LAWS                                                                                    AV  DE  TORRES   104  
ATTY.  ARIS  L.  GULAPA                                            AY  2015-­‐2016  
Moody  was  never  married  and  there  is  no  doubt  that  he  had  his  legal  domicile   establishment   of   a   legal   domicile   there.   The   negative   evidence   that   he   told  
in   the   Philippine   Islands   from   1902   or   1903   forward   during   which   time   he   Cooley  that  he  did  not  intend  to  return  to  Manila  does  not  prove  that  he  had  
accumulated   a   fortune   from   his   business   in   the   Philippine   Islands.   He   lived   in   established  a  domicile  in  Paris.  His  short  stay  of  three  months  in  Paris  is  entirely  
the  Elks’  Club  in  Manila  for  many  years  and  was  living  there  up  to  the  date  he   consistent   with   the   view   that   he   was   a   transient   in   Paris   for   the   purpose   of  
left   Manila   the   latter   part   of   February,   1928,   under   the   following   receiving   treatments   at   the   Pasteur   Institute.   The   evidence   in   the   record  
circumstances:   He   was   afflicted   with   leprosy   in   an   advanced   stage   and   had   indicates   clearly   that   Moody’s   continued   absence   from   his   legal   domicile   in  
been   informed   by   Dr.   Wade   that   he   would   be   reported   to   the   Philippine   the   Philippines   was   due   to   and   reasonably   accounted   for   by   the   same   motive  
authorities  for  confinement  in  the  Culion  Leper  Colony  as  required  by  the  law.   that  caused  his  surreptitious  departure,  namely,  to  evade  confinement  in  the  
Distressed   at   the   thought   of   being   thus   segregated   and   in   violation   of   his   Culion   Leper   Colony;   for   he   doubtless   knew   that   on   his   return   he   would   be  
promise  to  Dr.  Wade  that  he  would  voluntarily  go  to  Culion,  he  surreptitiously   immediately   confined,   because   his   affliction   became   graver   while   he   was  
left   the   Islands   the   latter   part   of   February,   1928,   under   cover   of   night,   on   a   absent  than  it  was  on  the  day  of  his  precipitous  departure  and  he  could  not  
freighter,  without  ticket,  passport  or  tax  clearance  certificate.  The  record  does   conceal  himself  in  the  Philippines  where  he  was  well  known,  as  he  might  do  
not   show   where   Moody   was   during   the   remainder   of   the   year   1928.   He   lived   in  foreign  parts.    
with   a   friend   in   Paris,   France,   during   the   months   of   March   and   April   of   the   year    
1929  where  he  was  receiving  treatment  for  leprosy  at  the  Pasteur  Institute.  The   Our  Civil  Code  (art.  40)  defines  the  domicile  of  natural  persons  as  "the  place  of  
record   does   not   show   where   Moody   was   in   the   interval   between   April,   1929,   their  usual  residence."  The  record  before  us  leaves  no  doubt  in  our  minds  that  
and  November  26,  1930,  on  which  latter  date  he  wrote  a  letter  to  Harry  Wendt   the  "usual  residence"  of  this  unfortunate  man,  whom  appellant  describes  as  a  
of   Manila,   offering   to   sell   him   his   interest   in   the   Camera   Supply   Company,   a   "fugitive"   and   "outcast",   was   in   Manila   where   he   had   lived   and   toiled   for  
Philippine   corporation,   in   which   Moody   owned   599   out   of   603   shares.   In   this   more   than   a   quarter   of   a   century,   rather   than   in   any   foreign   country   he  
letter,  among  other  things,  he  states:  "Certainly  I’ll  never  return  there  to  live  or   visited  during  his  wanderings  up  to  the  date  of  his  death  in  Calcutta.  To  effect  
enter  business  again."     the   abandonment   of   one’s   domicile,   there   must   be   a   deliberate   and   provable  
  choice  of  a  new  domicile,  coupled  with  actual  residence  in  the  place  chosen,  
As   Moody   died   of   leprosy   less   than   two   months   after   these   letters   were   with   a   declared   or   provable   intent   that   it   should   be   one’s   fixed   and  
written,   there   can   be   no   doubt   that   he   would   have   been   immediately   permanent   place   of   abode,   one’s   home.   There   is   a   complete   dearth   of  
segregated   in   the   Culion   Leper   Colony   had   he   returned   to   the   Philippine   evidence  in  the  record  that  Moody  ever  established  a  new  domicile  in  a  foreign  
Islands.  He  was,  therefore,  a  fugitive,  not  from  justice,  but  from  confinement  in   country.    
the  Culion  Leper  Colony  in  accordance  with  the  law  of  the  Philippine  Islands.      
  63.  GALLEGO  V.  VERA  
There   is   no   statement   of   Moody,   oral   or   written,   in   the   record   that   he   had    
adopted   a   new   domicile   while   he   was   absent   from   Manila.   Though   he   was   DOCTRINE:   In   order   to   acquire   a   domicile   by   choice,   there   must   concur   (1)  
physically   present   for   some   months   in   Calcutta   prior   to   the   date   of   his   death   residence   or   bodily   presence   in   the   new   locality,   (2)   an   intention   to   remain  
there,  the  appellant  does  not  claim  that  Moody  had  a  domicile  there  although   there,  and  (3)  an  intention  to  abandon  the  old  domicile.  
it  was  precisely  from  Calcutta  that  he  wrote  and  cabled  that  he  wished  to  sell    
his  business  in  Manila  and  that  he  had  no  intention  to  live  there  again.  Much   In   other   words,   there   must   be   an  animus   non   revertendi  and   an  animus  
less   plausible,   it   seems   to   us,   is   the   claim   that   he   established   a   legal   domicile   in   manendi.  The  purpose  to  remain  in  or  at  the  domicile  of  choice  must  be  for  an  
Paris   in   February,   1929.   The   record   contains   no   writing   whatever   of   Moody   INDEFINITE   period   of   time.   The   acts   of   the   person   must   conform   with   his  
from   Paris.   There   is   no   evidence   as   to   where   in   Paris   he   had   any   fixed   abode   purpose.   The   change   of   residence   must   be   voluntary;   the   residence   at   the   place  
that   he   intended   to   be   his   permanent   home.   There   is   no   evidence   that   he   chosen  for  the  domicile  must  be  actual;  and  to  the  fact  of  residence  there  must  
acquired   any   property   in   Paris   or   engaged   in   any   settled   business   on   his   own   be  added  the  animus  manendi.  
account   there.   There   is   no   evidence   of   any   affirmative   factors   that   prove   the  
 
CONFLICT  OF  LAWS                                                                                    AV  DE  TORRES   105  
ATTY.  ARIS  L.  GULAPA                                            AY  2015-­‐2016  
  there,  and  (3)  an  intention  to  abandon  the  old  domicile.  In  other  words,  there  
FACTS:   must   be   an  animus   non   revertendi  and   an  animus   manendi.   The   purpose   to  
This   is   a   petition   for   certiorari   to   review   the   decision   of   the   CA   affirming   the   remain   in   or   at   the   domicile   of   choice   must   be   for   an   INDEFINITE   period   of  
decision   of   the   CFI   of   Leyte,   which   declared   illegal   the   petitioner’s   election   to   time.   The   acts   of   the   person   must   conform   with   his   purpose.   The   change   of  
the  office  of  the  municipal  mayor  of  Abuyog,  Leyte  in  the  election  of  Dec.  1940,   residence   must   be   voluntary;   the   residence   at   the   place   chosen   for   the  
on  the  ground  that  he  did  not  meet  the  residence  qualification.     domicile   must   be   actual;   and   to   the   fact   of   residence   there   must   be   added  
  the  animus  manendi.    
Gallego   is   a   native   of   Abuyog,   Leyte.   After   his   studies,   he   was   employed   as   a    
school  teacher  in  Catarman,  Samar,  as  well  as  in  some  municipalities  in  Leyte.   The   purpose   to   remain   in   the   domicile   should   be   for   an   INDEFINITE   period   of  
In   1937,   he   ran   as   municipal   mayor   in   Abuyog,   Leyte,   but   lost.   In   June   1938,   he   time.   The   court   believed   that   Gallego   had   no   intention   to   stay   in   Malaybalay  
worked   in   Malaybalay   Bukidnon   in   a   plantation   of   the   Bureau   of   Forestry   to   indefinitely  because:  
make  up  for  the  financial  drawback  caused  by  his  loss  in  the  previous  election,    
and  stayed  there  until  he  resigned  in  Sept.  1940.     1. When  he  was  employed  as  a  teacher  in  Samar,  he  always  returned  in  
  Abuyog  and  even  resigned  when  he  ran  for  office  in  1937  
Gallego   registered   himself   as   an   elector   in   Bukidnon   and   voted   there   in   the   2. His  departure  was  only  for  the  purpose  of  making  up  for  the  financial  
election   for   assemblymen   held   in   Dec.   1938,   and   in   Jan.   1940,   He   obtained   and   drawback  caused  by  his  loss  in  the  election  
paid   for   his   residence   cert.   from   the   municipal   treasurer   of   Malaybalay,   in   3. He  did  not  take  his  wife  and  children  to  Malaybalay  with  him  
which  certificate  it  was  stated  that  he  had  resided  in  the  said  municipality  for   4. He   bought   a   piece   of   land   in   Abuyog   and   did   not   avail   of   the   land   in  
1.5  yrs.     the  plantation  offered  to  him  by  the  government  ten  hectares  of  land  
  within  the  chichona  reservation  in  Malaybalay,  where  he  worked  as  a  
The  CA  declared  that  Gallego  lost  his  domicile  in  Abuyog  Leyte  at  the  time  he   nurseryman.  
was  elected  mayor  there  on  the  grounds  that:     5. During  the  short  period  of  about  two  years  he  stayed  in  Malaybalay  as  
• He  registered  as  a  voter  in  Malaybalay,  Bukidnon       a   government   employee,   he   visited   his   home   town   and   his   family   no  
• He  voted  in  Malaybalay  in  the  1938  election  for  assemblymen       less  than  three  times  notwithstanding  the  great  distance  between  the  
• He  obtained  a  residence  cert  from  the  municipality  of  Malaybalay       two  places.  
   
ISSUE:   The   court   said   that   the   manifest   intent   of   the   law   in   fixing   a   residence  
Whether   or   not   Gallego   lost   his   domicile   of   origin   in   Abuyog,   Leyte   and   qualification  is  to:  
acquired  a  new  domicile  in  Malaybalay,  Bukidnon.   “exclude   a   stranger   or   a   newcomer,   unacquainted   with   the   conditions   and  
  needs  of  a  community  and  not  identified  with  the  latter,  from  an  elective  office  
HELD:   to  serve  that  community.”  
Gallego   did   not   lose   his   domicile   in   Abuyog   by   working   in   Malaybalay   as   an    
employee,   registering   as   voter   there   and   securing   his   residence   certificate   And  the  petitioner  was  a  native  there,  had  run  for  the  same  office  before,  and  
rd
there  for  1940.     was  now  elected  with  a  majority  of  800  votes  in  a  3  class  municipality.  
   
The   term   "residence"   as   used   in   the   election   law   is   synonymous   with  
"domicile"  which  imports  not  only  intention  to  reside  in  a  fixed  place  but  also  
personal   presence   in   that   place,   coupled   with   conduct   indicative   of   such  
intention.   In   order   to   acquire   a   domicile   by   choice,   there   must   concur   (1)  
residence   or   bodily   presence   in   the   new   locality,   (2)   an   intention   to   remain  
 
CONFLICT  OF  LAWS                                                                                    AV  DE  TORRES   106  
ATTY.  ARIS  L.  GULAPA                                            AY  2015-­‐2016  
64.  VILLHERMOSA  V.  COMMISSIONER  OF  IMMIGRATION   denied   the   writ   of   habeas   corpus   requested   by   Florentina   Villahermosa   on  
G.R.  No.  L-­‐1663  |  21  March  1948   behalf  of  her  son  Delfin  Co.  She  appealed.      
   
DOCTRINES:   He   is,   at   the   time   he   entered   this   country   from   China,   was   a   ISSUE:  
Chinese  subject  to  deportation,  and  any  subsequent  change  in  his  status  can  not   Whether   on   the   strength   of   such   reacquisition   of   Philippine   citizenship   by  
erase  the  taint  of  his  unlawful,  surreptitious  entry.   Florentina,  Delfin  is  a  national  not  subject  to  deportation?  
   
 
The  status  of  an  immigrant  and  his  right  to  stay  here  is  to  be  determined  as  of   HELD:
the   time   of   his   entry   and   that   he   could   not   do   afterwards   anything   to   render   No.  There  are  two  reasons  why  Delfin  Co  must  be  returned  to  China.  First,  he  is  
valid  what  was  originally  an  illegal  entry.   not   now   a   Filipino   citizen;   and   second,   granting   that   he   is,   at   the   time   he  
  entered  this  country  from  China  he  was  a  Chinese  subject  to  deportation,  and  
ALG:   If   she   became   a   widow   ON   or   AFTER   October   21,   1963   –   she   has   to   any   subsequent   change   in   his   status   can   not   erase   the   taint   of   his   unlawful,  
repatriate   herself,   otherwise,   she   remains   a   foreigner.   She   does   not   surreptitious  entry.    
immediately  reacquire  Philippine  citizenship.  (CA  No.  63)    
  Section   1   of   Article   IV   of   the   Constitution   enumerates   those   who   are   citizens   of  
FACTS:     the  Philippines,  as  follows:  (1)  Those  who  are  citizens  of  the  Philippine  Islands  
In   1946,   Delfin,   18   y/o,   born   in   Paniqui,   Tarlac,   of   a   Chinese   father   named   Co   at  the  time  of  the  adoption  of  the  Constitution.  (2)  Those  born  in  the  Philippine  
Suy,  alias  Yu  Kui,  and  Florentina  Villahermosa,  left  the  Philippines  for  China  as  a   Islands  of  foreign  parents  who,  before  adoption  of  this  Constitution,  had  been  
Chinese  repatriate.    Due  to  financial  difficulties  in  China,  he  took  steps  to  return   elected   to   public   office   in   the   Philippine   Islands.   (3)   Those   whose   fathers   are  
to  the  Phil.  while  there  he  met  a    Chinese  (Co  Soon  Tiong),  who  informed  him   citizens   of   the   Philippines.  (4)   Those   whose   mothers   are   citizens   of   the  
of  a  plan  to  smuggle  their  compatriots  into  the  Phil.  Delfin  agreed  to  lead  the   Philippines   and   upon   reaching   the   age   of   majority,   elect   Philippine  
party  to  Ilocos  Sur  where  his  mother  had  relatives,  who  could  render  valuable   citizenship.  (5)  Those  who  are  naturalized  in  accordance  with  law.    
assistance.        
  Delfin's  claim  to  citizenship  can  only  be  predicated,  if  at  all,  on  paragraph  4  of  
In  the  night  of  March  24,  1947,  their  party  of  69  Chinese,  led  by  Delfin,  landed   the  above  section.  But,  being  a  minor  he  has  not  had  the  opportunity  to  elect  
clandestinely  on    the  shores  of  Sto.  Domingo,  Ilocos  Sur,  in  an  attempt  to  evade   Philippine   citizenship,   and   therefore   he   is   as   yet   an   alien,   his   father   being   a  
Phil.   immigration    laws.   Unfortunately,   the   immigrants   were   discovered    and   Chinese.    
apprehended   immediately   after   arrival.   Delfin   was   examined   by   the    
Commissioner  of  Immigration.  After  formal  investigation,  the  Commissioner  of   We   have   heretofore   held   that,   after   the   Constitution,   mere   birth   in   the  
Immigration    rendered   a   decision   ordering   the   deportation   of   Delfin.   After   Philippines   of   a   Chinese   father   and   Filipino   mother   does   not   ipso   facto   confer  
knowing   the   apprehension   of   her   son   Delfin,   Florentina,   filed    in   the   civil   Philippine   citizenship   and   that   jus   sanguinis   instead   off   jus   soli   is   the  
registry   of   Tarlac   under   C.A.   No.   63   an   oath   of   allegiance   for   the   purpose   of   predominating  factor  on  questions  of  citizenship.  
resuming   her   Philippine   citizenship,   which   she   had   lost   upon   her   marriage   to   Co      
Suy.   Nevertheless,   it   is   contended   that   Florentina   Villahermosa   being   a   Filipina,  
  Delfin,   should   likewise   be   a   Filipino.   C.A.   No.   63   does   not   provide   that   upon  
On  the  strength  of  such  reacquisition  of  Philippine  citizenship  by  Florentina,  it   repatriation   of   a   Filipina   her   children   acquire   Philippine   citizenship.  It  would  
was   contended   that   Delfin,   being   a   minor,   followed   the   citizenship   of   his   be  illogical  to  consider  Delfin  as  repatriated  like  his  mother,  because  he  never  
mother,  and  was  a  national  not  subject  to  deportation.  These  contentions  were   was  a  Filipino  citizen  and  could  not  have  reacquired  such  citizenship.    
overruled   by   the   immigration   authorities.   Judge   Sotero   Rodas,   (CFI   of   Manila)    
While  his  Chinese  father  lived,  Delfin  was  not  a  Filipino.  His  mother  was  not  a  
 
CONFLICT  OF  LAWS                                                                                    AV  DE  TORRES   107  
ATTY.  ARIS  L.  GULAPA                                            AY  2015-­‐2016  
Filipina;   she   was   Chinese.   After   the   death   of   such   father,   Villahermosa   Petitioner  Saludo  is  a  Filipino,  a  member  of  the  House  of  Reps  and  resident  of  
continued  to  be  a  Chinese,  until  she  reacquired  her  Filipino  citizenship  in  April,   S.Leyte.  Respondent  AMEX  is  a  corporation  doing  business  in  the  Phil,  engaged  
1947.  After  that  reacquisition  Delfin  could  claim  that  his  mother  was  a  Filipina   in  providing  credit  and  credit  facilities  &  allied  services  with  an  office  in  Makati.    
within   the   meaning   of   paragraph   4,   section   1   of   Article   IV   of   the   Constitution;    
but,  according  to  that  same  Organic  Act,  he  had  to  elect  Philippine  citizenship   Saludo’s   cause   of   action   stemmed   from   the   wrongful   dishonor   of   his   AMEX  
upon  attaining  his  majority.  Until  he  becomes  of  age  and  makes  the  election,   credit   card   &   supplementary   card   issued   to   his   daughter.   The   first   dishonor  
he  is  the  Chinese  citizen  that  he  was  at  the  time  of  his  father's  demise.     happened   when   Saludo’s   daughter   used   her   card   to   pay   for   her   purchases   in  
  the  US  in  April  2000.  The  second  happened  when  Saludo  used  his  card  to  pay  
Moreover,  in  Juan  Co  vs.  Rafferty,  Court  said  that  the  status  of  an  immigrant   his  account  in  Hotel  Okawa  (Tokyo)  while  there  with  other  delegates  from  the  
and  his  right  to  stay  here  is  to  be  determined  as  of  the  time  of  his  entry  and   Phil   to   attend   the   Congressional   Recognition   in   honor   of   Mr.   Tanaka.   Allegedly,  
that  he  could  not  do  afterwards  anything  to  render  valid  what  was  originally   these   dishonors   resulted   from   AMEX’s   unilateral   act   of   suspending   petitioner  
an   illegal   entry.   A   Chinese   person,   not   a   merchant   at   the   time   he   applies   to   Saludo’s  account  for  his  failure  to  pay  the  balance  covering  the  period  of  March  
enter  the  Islands,  will  not  be  permitted  to  remain  here  upon  the  theory  that   2000.  Saludo  denied  receiving  the  statement  of  account.  Further,  he  alleged  a  
he   became   a   merchant   during   the   time   he   was   waiting   for   the   decision   of   the   wrongful   charge   for   late   payment   in   June   2000,   and   subsequent   cancellation   of  
proper   authorities.   SC   declared   that   Delfin   Co   is   not   now   a   Filipino   and   he,   the  cards  in  the  following  month.    
having  entered  this  country  surreptitiously,  is  subject  to  deportation.      
  AMEX   raised   the   defenses   of   lack   of   cause   of   action   and   improper   venue.   On  
71.  SALUDO  V.  AMERICAN  EXPRESS   the   latter,   they   said   that   the   residents   are   not   residents   of   S.Leyte,   and   even  
G.R.  No.  159507  |  19  April  2006   Saludo   was   allegedly   not   a   resident   thereof   as   evinced   by   his   community   tax  
  certificate  (CTC)  which  was  issued  in  Pasay.  Saludo  responded  that  at  the  time  
DOCTRINE:   “Resides”   means   the   place   of   abode,   whether   permanent   or   of   the   filing   of   the   complaint,   he   was   already   a   resident   of   S.   Leyte;   that   his  
temporary   of   the   plaintiff   of   the   defendant,   as   distinguished   from   “domicile”   CTCs  were  issued  in  Pasay  was  only  because  he  had  an  office  there.    
which  denotes  a  fixed  permanent  residence  to  which,  when  absent  one  has  the    
intention  of  returning.    
  ISSUE:  
Residence  is  not  domicile,  but  domicile  is  residence  coupled  with  the  intention   Did   the   appellate   court   err   in   declaring   an   improper   venue,   because   none   of  
to  remain  for  an  unlimited  time.  A  man  can  have  but  one  domicile  for  one  and   the  parties  was  a  resident  of  S.  Leyte  at  the  time  of  the  filing  of  the  complaint?  
the  same  purpose  at  any  time,  but  he  may  have  numerous  places  of  residence.    
   
Residence   simply   requires   bodily   presence   as   an   inhabitant   in   a   given   place,   HELD:  
while   domicile   requires   bodily   presence   in   that   place   and   also   an   intention   to   Yes.   This   complaint   is   a   personal   action,   governed   by   Sec   2,   Rule   4   of   ROC  
make   it   one's   domicile.   No   particular   length   of   time   of   residence   is   required   (Venue  of  Actions).  The  choice  of  venue  for  personal  actions  cognizable  by  the  
though;  however,  the  residence  must  be  more  than  temporary.  We   note   that   RTC   is   given   to   the   plaintiff.   Jurisprudence   has   held   that   the   term   “resides”  
Sec  2,  Rule  4  uses  the  term  “residence,”  not  domicile.   means  the  place  of  abode,  whether  permanent  or  temporary  of  the  plaintiff  
  of   the   defendant,   as   distinguished   from   “domicile”   which   denotes   a   fixed  
FACTS:   permanent   residence   to   which,   when   absent   one   has   the   intention   of  
Saludo   filed   a   complaint   for   damages   against   American   Express   Inc.   (AMEX)   returning.  Similarly,  in  Conflict  of  Laws,  residence  applies  to  a  temporary  stay  
and/or   its   officers   Fish   (Vice   President   &   Country   Manager)   &   Mascrinas   of   a   person   in   a   given   place,   while   domicile   refers   to   the   relatively   more  
(Operations   Head)   with   Branch   25,   RTC   Maasin,   Southern   Leyte   (S.Leyte).   permanent  abode  of  a  person.  This  distinction  is  very  well  emphasized  in  cases  

 
CONFLICT  OF  LAWS                                                                                    AV  DE  TORRES   108  
ATTY.  ARIS  L.  GULAPA                                            AY  2015-­‐2016  
where   the   Domiciliary   Theory   must   necessarily   supplant   the   Nationality   Theory   particular  place;  and  (2)  personal  or  physical  presence  in  that  place,  coupled  
in  cases  involving  stateless  persons.   with  conduct  indicative  of  such  intention.  As  the  Court  elucidated,  "the  place  
  where  a  party  actually  or  constructively  has  a  permanent  home,  where  he,  no  
Residence  is  not  domicile,  but  domicile  is  residence  coupled  with  the  intention   matter  where  he  may  be  found  at  any  given  time,  eventually  intends  to  return  
to  remain  for  an  unlimited  time.  A  man  can  have  but  one  domicile  for  one  and   and   remain,   i.e.,   his   domicile,   is   that   to   which   the   Constitution   refers   when   it  
the   same   purpose   at   any   time,   but   he   may   have   numerous   places   of   residence.   speaks  of  residence  for  the  purposes  of  election  law."    
His   place   of   residence   generally   is   his   place   of   domicile,   but   is   not   by   any    
means,   necessarily   so   since   no   length   of   residence   without   intention   of   On   the   other   hand,   for   purposes   of   venue,   the   less   technical   definition   of  
remaining   will   constitute   domicile.   Residence   simply   requires   bodily   presence   "residence"  is  adopted.  Thus,  it  is  understood  to  mean  as  "the  personal,  actual  
as   an   inhabitant   in   a   given   place,   while   domicile   requires   bodily   presence   in   or   physical   habitation   of   a   person,   actual   residence   or   place   of   abode.   It  
that  place  and  also  an  intention  to  make  it  one's  domicile.  No  particular  length   signifies   physical   presence   in   a   place   and   actual   stay   thereat.   In   this   popular  
of  time  of  residence  is  required  though;  however,  the  residence  must  be  more   sense,  the  term  means  merely  residence,  that  is,  personal  residence,  not  legal  
than   temporary.   We   note   that   Sec   2,   Rule   4   uses   the   term   “residence,”   not   residence   or   domicile.   Residence   simply   requires   bodily   presence   as   an  
domicile.   inhabitant   in   a   given   place,   while   domicile   requires   bodily   presence   in   that  
  place  and  also  an  intention  to  make  it  one's  domicile."  
There   is   no   dispute   that   petitioner   Saludo   was   the   congressman   or   the      
representative  of  the  lone  district  of  Southern  Leyte  at  the  time  of  filing  of  his   Since   petitioner   Saludo,   as   congressman   or   the   lone   representative   of   the  
complaint  with  the  court  a  quo.  Even  the  appellate  court  admits  this  fact  as  it   district  of  Southern  Leyte,  had  his  residence  (or  domicile)  therein  as  the  term  is  
states  that  "it  may  be  conceded  that  private  respondent  ever  so  often  travels   construed   in   relation   to   election   laws,   necessarily,   he   is   also   deemed   to   have  
to   Maasin   City,   Southern   Leyte,   because   he   is   its   representative   in   the   lower   had  his  residence  therein  for  purposes  of  venue  for  filing  personal  actions.  Put  
house.”   in   another   manner,   Southern   Leyte,   as   the   domicile   of   petitioner   Saludo,   was  
    also   his   residence,   as   the   term   is   understood   in   its   popular   sense.   This   is  
As  a  member  of  the  House  of  Representatives,  petitioner  Saludo  was  correctly   because  "residence  is  not  domicile,  but  domicile  is  residence  coupled  with  the  
deemed   by   the   court   a   quo   as   possessing   the   requirements   for   the   said   intention  to  remain  for  an  unlimited  time."  
position,   including   that   he   was   then   a   resident   of   the   district   which   he   was    
representing,   i.e.,   Southern   Leyte.   Significantly,   for   purposes   of   election   law,   S.   Leyte,   as   the   domicile   of   petitioner   Saludo,   was   also   his   residence,   as   the  
the  term  "residence"  is  synonymous  with  "domicile,"  thus:   term   is   understood   in   its   popular   sense.   This   is   because   "residence   is   not  
x  x  x  [T]he  Court  held  that  "domicile"  and  "residence"  are  synonymous.  The   domicile,  but  domicile  is  residence  coupled  with  the  intention  to  remain  for  an  
term  "residence,"  as  used  in  the  election  law,  imports  not  only  an  intention  to   unlimited   time."   Further,   petitioner   Saludo's   residence   in   S.   Leyte   could  
reside  in  a  fixed  place  but  also  personal  presence  in  that  place,  coupled  with   likewise  be  properly  taken  judicial  notice  of  by  the  court  a  quo.  It  is  bound  to  
conduct   indicative   of   such   intention.   "Domicile"   denotes   a   fixed   permanent   know  that,  under  the  Constitution,  one  of  the  qualifications  of  a  congressman  
residence  to  which  when  absent  for  business  or  pleasure,  or  for  like  reasons,   or  representative  to  the  House  of  Representatives  is  having  a  residence  in  the  
one  intends  to  return.  x  x  x   district  in  which  he  shall  be  elected.  Petition  GRANTED.  
   
It   can   be   readily   gleaned   that   the   definition   of   "residence"   for   purposes   of  
election   law   is   more   stringent   in   that   it   is   equated   with   the   term   "domicile."  
Hence,   for   the   said   purpose,   the   term   "residence"   imports   "not   only   an  
intention   to   reside   in   a   fixed   place   but   also   personal   presence   in   that   place,  
coupled   with   conduct   indicative   of   such   intention."   When   parsed,   therefore,  
the   term   "residence"   requires   two   elements:   (1)   intention   to   reside   in   the  
 
CONFLICT  OF  LAWS                                                                                    AV  DE  TORRES   109  
ATTY.  ARIS  L.  GULAPA                                            AY  2015-­‐2016  
NATIONALITY   5.  And  is  not  in  any  way  disqualified  under  the  provisions  of  the  Act.  
  6.  The  petition  shall  also  set  forth  the  names  and  post  office  addresses  of  such  
  witnesses  as  the  petitioner  may  desire  to  introduce  at  the  hearing  of  the  case  
MINORS    
Are  Filipinos  if:   A  petition  not  so  verified   by   at  least   two  persons   who  are   citizens   is   not   merely  
1. a.  Minor  children  of  persons  naturalized  under  the  law;  and   voidable   but   void.   The   Court   went   further   and   said   that   such   petition   could   not  
b.  Born  in  the  Philippines  "shall  be  considered  citizens  thereof".   be  amended.  
2. a.  Foreign-­‐born  minor  child,  b.  If  dwelling  in  the  Philippines  at  the    
time   of   the   naturalization   of   the   parents,   "shall   automatically   The  strong  legal  presumption  that  the  applicant  was  born  in  wedlock—that  his  
become  a  Filipino  citizen".   parent   were   lawful   husband   and   wife—cannot   be   destroyed   by   evidence   so  
  slim  and  shaky.  
No   conditions   are   exacted;   citizenship   of   said   minor   children   is    
conferred   by   the   law   itself,   without   further   proceedings   and   as   a   Gulapa:   The   SC   seemed   to   imply   in   Villahermoso   that   for   a   child   to   elect  
matter  of  course.  An  alien  wife  of  a  Filipino  does  not  fit  into  either   Philippine   citizenship,   the   mother   should   be   a   Filipino   citizen   at   the   time   of  
of  the  categories  just  mentioned.  Legal  action  has  to  be  taken  to   such  election.  If  we  were  to  follow  this  interpretation,  very  few  children  could  
make  her  a  citizen.   avail   themselves   of   the   option,   for   in   many   instance   (under   the   provisions   of  
  the  1935  Constitution)  the  mother  would  follow  the  husband’s  nationality  and  
ALIEN  WIFE   thus   lose   Philippine   Citizenship.   The   same   result   is   reached   if   we   require  
An  alien  woman  married  to  a  Filipino  who  desires  to  be  a  citizen  of  this   Filipino   citizenship   on   the   part   of   the   mother   at   the   time   of   the   birth   of   the  
country  must  apply  therefor  by  filing  a  petition  for  citizenship  reciting   child.   The   more   realistic   approach   would   be   to   consider   it   sufficient   that   the  
that  she  possesses  all  the  qualifications  set  forth  in  Section  2,  and  none   mother   was   a   Filipino   citizen   at   the   time   of   her   marriage   to   a   foreigner.  
of   the   disqualifications   under   Section   4,   both   of   the   Revised   Organically,  the  mother  did  not  do  anything  to  become  an  alien  except  to  get  
Naturalization  Law.   married.  This  is  in  effect,  is  the  impression  drawn  from  the  1951  decision  of  the  
  SC  in  the  case  of  In  re  Robert  Cu.    
64.  IN  RE:  ROBERT  CU    
G.R.  No.  L-­‐3018  |  18  July  1951   FACTS:  
  Robert   Cu   filed   a   petition   for   naturalization   setting   forth   facts   required   and  
DOCTRINE:  If  the  applicant's  parents  were  legally  married,  which  is  however  to   appropriate  for  that  purpose,  but  at  the  hearing  he  said  that  he  was  a  citizen  of  
be  presumed,  then  he  was  born  a  Chinese  citizen  and  continued  to  be  so,  unless   the   Philippines;   and   upon  the  conclusion  of  the  trial,  the  CFI  of  Rizal  found  him  
upon  the  age  of  majority  he  elected  Philippine  citizenship  which  he  confessedly   "to   be   a   Filipino   citizen,   both   by   right   of   birth   and   by   right   of   selection,"   and  
did  not  do.   dismissed   the   petition   for   naturalization,   holding   impliedly   that   being   already   a  
  Philippine  citizen  he  did  not  have  to  be  naturalized.  The  CFI's  pronouncement  is  
According  to  this  provision,  the  witnesses  must  be:  [CDR2  QN]   based  solely  on  the  applicant’s  testimony  that  he  was  born  in  Angat,  Bulacan  in  
1.  citizens  of  the  Philippines  and   1913.  That  he  was  a  subject  of  the  Phil.  and  that  when  he  was  a  kid  of  about  5  
2.   "personally   know   the   petitioner   to   be   a   resident   of   the   Philippines   for   the   months   old,   his   mother,   who   was   a   Filipina,   died.   Then   his   father   allegedly  
period   of   time   required   by   this   Act,"   which   in   cases   of   petitioners   born   in   the   brought   him   to   China   right   after   that   but   at   the   age   of   5,   they   left   China   and  
Philippines  is  5  years  (Sec.  3)  and  in  other  cases  10  years  (Sec.  2,  par.  2)   Robert  was  given  to  the  care  of  Doña  Margarita  Mangahas.    
3.  Petitioner  must  be  a  person  of  good  repute  and  morally  irreproachable,    
4.  He  has  all  the  qualifications  necessary  to  become  a  citizen  of  the  Philippines   Upon  motion  of  the  attorney  for  the  Government,  who  protested  that  the  last  

 
CONFLICT  OF  LAWS                                                                                    AV  DE  TORRES   110  
ATTY.  ARIS  L.  GULAPA                                            AY  2015-­‐2016  
answer  (that  Robert  is  a  Filipino  citizen)  was  a  mere  conclusion  of  the  witness   He   admitted   that   he   did   not   filed   any   citizenship   application   by   election   in  
(Robert   himself),   the   testimony   was   ordered   stricken   out.   But   Cu   proceeded   writing.  Although  he  was  a  member  of  the  reserve  force  of  the  Philippine  Army;  
that   he   considered   himself   a   Filipino   citizen   on   account   of   the   fact   that   his   an   ROTC   trainee;   trained   in   the   Philippine   Army;   was   called   during   the   war;  
mother  was  a  Filipina  and  he  was  born  in  the  Philippines.  His  only  fault  was  that   currently   a   government   employee;   a   member   of   the   faculty   of   UP;   and   a  
he  failed  to  file  his  application  to  elect  Philippine  citizenship  and  so  he  filed  his   resident  physician  of  the  Philippine  General  Hospital.  But  these  circumstances  
application  now.  He  was  asked  whether  his  mother  was  legally  married  to  his   alone   made   this   witness   neither   a   citizen   of   this   country   nor   eligible   as   a  
father,  to  which  he  answered  “The  way  I  know  it,  they  are  not  legally  married.”   vouching  witness  in  a  proceeding  of  this  character.  As   to   the   other   witnesses,  
  Dr.   Pastor   Gomez,   testified   that   he   had   known   Mr.   Cu   since   liberation,   about  
ISSUE:   August,   1945.   But   after   this   answer   was   given,   the   counsel   for   the   Govt.  
Whether   the   Cu   is   entitled   to   be   admitted   to   Philippine   citizenship   under   the   objected  to  the  witness'  testifying  any  further,  and  the  objection  having  been  
present  application?     sustained,  Dr.  Gomez  was  withdrawn.    
   
HELD:   US   Jurisprudence   provides   that   a   petition   not   so   verified   by   at   least   two  
No.   As   observed,   Cu’s   statements   make   plain   that   the   he   was   at   best   uncertain   persons   who   are   citizens   is   not   merely   voidable   but   void.   The   Court   went  
that   his   parents   were   unmarried   to   each   other   and   are   therefore   utterly   further   and   said   that   such   petition   could   not   be   amended.   Also,   the   Courts  
inadequate   to   serve   as   basis   for   declaring   him   a   Philippine   citizen.   If   the   cannot   be   expected   to   possess   acquaintance   with   the   candidates   with   the  
applicant's   parents   were   legally   married,   which   is   however   to   be   presumed,   presenting  themselves  for  naturalization  —  in  fact,  no  duty  rests  upon  them  in  
then  he  was  born  a  Chinese  citizen  and  continued  to  be  so,  unless  upon  the   this   particular;   so   that   witnesses   appearing   before   them   are   in   a   way   insures   of  
age  of  majority  he  elected  Philippine  citizenship  which  he  confessedly  did  not   the   character   of   the   candidate   concerned,   and   on   their   testimony   the   courts  
do.     are   of   necessity   compelled   to   rely.   A   witness   who   is   incompetent   renders   an  
  application   void.   The   question   of   a   witness'   qualifications   in   naturalization  
Sec.   7   of   the   Revised   Naturalization   Law   provides   that   the   petition   for   proceedings  is  therefore  a  matter  of  more  than  usual  importance.    
citizenship,  besides  stating  the  petitioner's  qualifications  as  enumerated  in  the    
Act,   "must   be   signed   by   the   applicant   in   his   own   handwriting   and   be   The   above   US   rulings   are   not   binding   upon   this   Court,   but   it   is   a   rational   rule   of  
supported   by   the   affidavit   of   at   least   two   credible   persons,  stating  that  they   statutory   construction   that   a   statute   adopted   from   another   state   or   country  
are   citizens   of   the   Philippines   and   personally   know   the   petitioner   to   be   a   will   be   presumed   to   be   adopted   with   the   construction   placed   upon   it   by   the  
resident   of   the   Philippines   for   the   period   of   time   required   by   this   Act   and   a   courts   of   that   state   or   country   before   its   adoption.   Such   construction   is  
person   of   good   repute   and   morally   irreproachable,   and   that   said   petitioner   has   regarded   as   of   great   weight,   or   at   least   persuasive,   and   will   generally   be  
in   their   opinion   all   the   qualifications   necessary   to   become   a   citizen   of   the   followed   if   found   reasonable,   and   in   harmony   with   justice   and   public   policy,  
Philippines  and  is  not  in  any  way  disqualified  under  the  provisions  of  the  Act.   and   with   other   laws   of   the   adopting   jurisdiction   on   the   subject.   We   find   the  
The  petition  shall  also  set  forth  the  names   and   post   office   addresses   of   such   United  States  courts'  reasoning  to  be  sound  and  reasonable  and  we  make  it  our  
witnesses  as  the  petitioner  may  desire  to  introduce  at  the  hearing  of  the  case."   own.    
According  to  this  provision,  the  witnesses  must  be  citizens  of  the  Philippines    
and  "personally  know  the  petitioner  to  be  a  resident  of  the  Philippines  for  the   It   is   unnecessary   to   consider   whether   the   application   could   be   granted   if  
period  of  time  required  by  this  Act,"  which  in  cases  of  petitioners  born  in  the   witnesses,  other  than  the  vouching  witnesses,  who  were  Philippine  citizens  and  
Philippines  is  5  years  (Sec.  3)  and  in  other  cases  10  years  (Sec.  2,  par.  2).     knew   the   applicant   for   the   time   required   by   the   statute,   had   testified   and  
  established   the   petitioner's   qualifications   for   admission   to   citizenship;   as  
By  their  testimony,  the  two  witnesses  who  made  affidavits  and  gave  evidence   already   indicated,   no   such   witnesses   were   introduced   in   support   of   the  
in   support   of   the   application   were   not   qualified   for   this   role.   Dr.   Jose   Ku   Yeg   petition.  The  appealed  decision  is  affirmed  in  so  far  as  it  dismissed  the  petition  
Keng   admitted   that   his   father   was   a   Chinese   national   and   his   mother   a   Filipina.   for   naturalization   and   reversed   in   so   far   as   it   declared   the   applicant   a   citizen   of  
 
CONFLICT  OF  LAWS                                                                                    AV  DE  TORRES   111  
ATTY.  ARIS  L.  GULAPA                                            AY  2015-­‐2016  
the  Philippines.  This  dismissal,  however,  will  be  without  prejudice  to  the  right   Yes.  Edward  was  a  US  Citizen  and  domiciled  in  the  Philippines  at  the  time  of  his  
of  the  Robert  Cu  to  file  a  new  application  for  naturalization     death.   The   law   that   governs   the   validity   of   his   testamentary   dispositions   is  
  defined  in  Art.  16,  CC,  which  is  as  follows:  
65.  AZNAR  V.  CHRISTENSEN-­‐GARCIA,  SUPRA      
  ART.  16.  Real  property  as  well  as  personal  property  is  subject  to  the  law  
FACTS:   of  the  country  where  it  is  situated.  
Edward  Christensen,  born  in  New  York,  migrated  to  California  where  he  resided    
and   consequently   was   considered   citizen   thereof.   He   came   to   the   Philippines   However,   intestate   and   testamentary   successions,   both   with   respect   to  
where  he  became  a  domiciliary  until  the  time  of  his  death.  However,  during  the   the  order  of  succession  and  to  the  amount  of  successional  rights  and  to  
entire   period   of   his   residence   in   this   country,   he   had   always   considered   himself   the   intrinsic   validity   of   testamentary   provisions,   shall   be   regulated   by   the  
a  citizen  of  California.   national   law   of   the   person   whose   succession   is   under   consideration,  
  whatever   may   be   the   nature   of   the   property   and   regardless   of   the  
In   his   will,   Edward   instituted   his   daughter   Maria   Lucy   Christensen   as   his   only   country  where  said  property  may  be  found.  
heir,  but  left  a  legacy  of  P3600  in  favor  of  Helen  Christensen  Garcia  who,  in  his    
will  was  described  as  "not  in  any  way  related  to"  him  but  in  a  decision  rendered   The  laws  of  California  have  prescribed  two  sets  of  laws  for  its  citizens,  one  for  
by   the   SC   in   another   case   had   been   declared   as   an   acknowledged   natural   residents  therein  and  another  for  those  domiciled  in  other  jurisdictions.  Art.  
daughter  of  his.   946   of   the   California   Civil   Code   is   its   conflict   of   laws   rule,   while   the   rule  
  applied  in  Kaufman,  is  its  internal  law.  If  the  law  on  succession  and  the  conflict  
Helen   alleged   that   the   will   deprives   her   of   her   legitime   as   an   acknowledged   of  laws  rules  of  California  are  to  be  enforced  jointly,  each  in  its  own  intended  
natural  child.  She  claims  that  under  Art.  16  of  the  Civil  Code,  the  California  law   and   appropriate   sphere,   the   principle   cited   in   Kaufman   should   apply   to  
should   be   applied,   and   in   accordance   therewith   and   following   the   doctrine   of   citizens  living  in  the  State,  but  Art.  946  should  apply  to  such  of  its  citizens  as  
renvoi,   the   question   of   the   validity   of   the   testamentary   provision   in   question   are  not  domiciled  in  California  but  in  other  jurisdictions.  
should   be   referred   back   to   the   law   of   the   decedent's   domicile,   which   is   the    
Philippines.   She   invokes   the   provisions   of   Art.   946   of   the   Civil   Code   of   The   national   law   mentioned   in   Art.   16   is   the   law   on   conflict   of   laws   in   the  
California,  which  is  as  follows:  “If  there  is  no  law  to  the  contrary,  in  the  place   California  Civil  Code,  i.e.,  Art.  946,  which  authorizes  the  reference  or  return  of  
where  personal  property  is  situated,  it  is  deemed  to  follow  the  person  of  its   the  question  to  the  law  of  the  testator's  domicile.  The   conflict   of   laws   rule   in  
owner,   and   is   governed   by   the   law   of   his   domicile.”   Accordingly,   her   share   California  precisely  refers  back  the  case,  when  a  decedent  is  not  domiciled  in  
must   be   increased   in   view   of   successional   rights   of   illegitimate   children   under   California,   to   the   law   of   his   domicile,   which   is   the   Philippines   in   the   case   at  
Philippine  laws.   bar.  
   
On   the   other   hand,   the   executor   and   Lucy   argue   that   the   national   law   of   the   The   Philippine   court   therefore   must   apply   its   own   law   as   directed   in   the  
deceased  must  apply,  and  thus  the  courts  must  apply  internal  law  of  California   conflict   of   laws   rule   of   the   state   of   the   decedent.   Wherefore,   the   decision  
on   the   matter.   Under   California   law,   there   are   no   compulsory   heirs   and   appealed   from   is   hereby   reversed   and   the   case   returned   to   the   lower   court   with  
consequently  a  testator  may  dispose  of  his  property  by  will  in  the  form  and   instructions   that   the   partition   be   made   as   the   Philippine   law   on   succession  
manner  he  desires  (Kaufman  Case).   provides.  
   
ISSUE:  
Whether  Philippine  law  should  ultimately  be  applied  
 
HELD:  
 
CONFLICT  OF  LAWS                                                                                    AV  DE  TORRES   112  
ATTY.  ARIS  L.  GULAPA                                            AY  2015-­‐2016  
68.  LAO  CHAY  V.  GALANG    
G.R.  No.  L-­‐19977  |  30  October  1964   ISSUE:  
  Whether   the   wife   of   a   Chinese   who   obtained   papers   of   Philippine   citizenship,  
DOCTRINE:  It   is   now   settled   that   under   this   provision,   an   alien   woman,   who   is   automatically   follows   the   citizenship   of   her   husband   if   not   otherwise  
married   to   a   citizen   of   the   Philippines   acquires   the   citizenship   of   her   husband   disqualified  under  the  Naturalization  Law  
only   if   she   has   all   the   qualifications   prescribed   in   Section   2   and   none   of   the    
disqualifications  provided  in  Section  4  of  the  law.  Since  Ng  Siu  Luan  admittedly   HELD:  
does   not   possess   the   qualifications   for   naturalization,   her   marriage   to   Lao   Chay   No.   Sec.   15   of   the   Revised   Naturalization   Law   provides:   Effect   of   the  
be  deemed  as  automatically  vesting  in  her  Filipino  citizenship.  Duty  of  consorts   naturalization  on  wife  and  children  -­‐  Any  woman  who  is  now  or  may  here-­‐after  
to   live   together   is   irrelevant   to   the   issue   which   concerns   only   the   right   of   a   be   married   to   a   citizen   of   the   Philippines,   and   who   might   herself   be   lawfully  
sovereign   state   to   determine   what   aliens   can   remain   within   its   territory   and   naturalized,  shall  be  deemed  a  citizen  of  the  Philippines.  
under  what  conditions  they  can  stay  therein.    
  It  is  now  settled  that  under  this  provision,  an  alien  woman,  who  is  married  to  
FACTS:   a  citizen  of  the  Philippines  acquires  the  citizenship  of  her  husband  only  if  she  
Ng   Siu   Luan   and   her   three   children,   who   are   all   of   minor   age,   came   to   the   has   all   the   qualifications   prescribed   in   Section   2   and   none   of   the  
Philippines  on  January  19,  1960  as  temporary  visitors,  having  been  allowed  to   disqualifications   provided   in   Section   4   of   the   law.   Since   Ng   Siu   Luan  
stay   in   this   country   until   January   26,   1961.   Instead   of   departing   on   that   date,   admittedly   does   not   possess   the   qualifications   for   naturalization,   her  
however,   appellees   asked   the   Bureau   of   Immigration   for   the   cancellation   of   marriage   to   Lao   Chay   be   deemed   as   automatically   vesting   in   her   Filipino  
their   alien   certificates   of   registration   as   well   as   those   of   their   children   on   the   citizenship.  
basis  of  Lao  Chay's  admission  to  Philippine  citizenship  on  December  12,  1960.      
  Anent  appellees'  claim  that  a  difference  in  the  citizenship  of  husband  and  wife  
On   January   20,   1961,   appellant   Commissioner   of   Immigration   granted   the   is   subversive   of   family   solidarity,   this   Court   has   already   said   that   the   duty   of  
petition  as  far  as  Lao  Chay  and  the  three  children  were  concerned,  but  denied   consorts   to   live   together   is   irrelevant   to   the   issue   which   concerns   only   the  
the  same  with  respect  to  Ng  Siu  Luan  on  the  ground  that  "she  is  not  qualified  to   right   of   a   sovereign   state   to   determine   what   aliens   can   remain   within   its  
acquire  Philippine  citizenship  of  her  husband  under  the  provision  of  paragraph   territory  and  under  what  conditions  they  can  stay  therein.  
1,  Section  15  of  CA  No.  473,  as  she  lacks  the  requirements  provided  for  under    
paragraph  2  of  the  same  Act."  He  therefore  asked  her  to  leave  the  country  on   69.  ZITA  NGO  BURCA  V.  REPUBLIC  
January  26,  1961.  The  Immigration  Commissioner  denied  a  subsequent  motion   G.R.  No.  L-­‐24252  |  30  January  1967  
for  reconsideration,  although  he  gave  Ng  Siu  Luan  a  five-­‐day  extension  within    
which  to  arrange  for  her  departure.     DOCTRINE:  Minor  children  of  persons  naturalized  under  the  law  who  were  born  
  in  the  Philippines  "shall  be  considered  citizens  thereof".  Similarly,  a  foreign-­‐born  
To  stop  the  threatened  deportation  of  Ng  Siu  Luan  appellees  filed  a  petition  for   minor  child,  if  dwelling  in  the  Philippines  at  the  time  of  the  naturalization  of  the  
mandamus  and  prohibition  in  the  CFI  and  secured  from  it  a  writ  of  preliminary   parents,   "shall   automatically   become   a   Filipino   citizen".  No   conditions   are  
injunction.     exacted;  citizenship  of  said  minor  children  is  conferred  by  the  law  itself,  without  
  further  proceedings  and  as  a  matter  of  course.  An  alien  wife  of  a  Filipino  does  
After   trial,   the   court   granted   the   petition,   and   held   that   the   law   does   not   not   fit   into   either   of   the   categories   just   mentioned.   Legal   action   has   to   be   taken  
require  that  an  alien  wife  should  have  the  same  qualifications  as  those  required   to  make  her  a  citizen.    
of   applicants   for   naturalization,   it   being   enough   that   she   is   not   otherwise    
disqualified.    

 
CONFLICT  OF  LAWS                                                                                    AV  DE  TORRES   113  
ATTY.  ARIS  L.  GULAPA                                            AY  2015-­‐2016  
(1)   An   alien   woman   married   to   a   Filipino   who   desires   to   be   a   citizen   of   this   Immigration.     Conceivably,  absence  of  clear  legal  direction  on  the  matter  could  
country  must  apply  therefor  by  filing  a  petition  for  citizenship  reciting  that  she   have  given  rise  to  divergence  of  views.  We  should  aim  at  drying  up  sources  of  
possesses   all   the   qualifications   set   forth   in   Section   2,   and   none   of   the   doubt.   Parties   interested   should   not   be   enmeshed   in   jurisdictional  
disqualifications   under   Section   4,   both   of   the   Revised   Naturalization   Law;   (2)   entanglements.   Public   policy   and   sound   practice,   therefore,   suggest   that   a  
Said   petition   must   be   filed   in   the   Court   of   First   Instance   where   petitioner   has   clear-­‐cut  ruling  be  made  on  this  subject.  
resided   at   least   one   year   immediately   preceding   the   filing   of   the   petition;   and   If   an   alien   woman   married   to   a   Filipino   does   not   become  ipso   facto  a   citizen,  
(3)   Any   action   by   any   other   office,   agency,   board   or   official,   administrative   or   then   she   must   have   to   file   a  "petition   for   citizenship"  in   order   that   she   may  
otherwise   —   other   than   the   judgment   of   a   competent   court   of   justice   —   acquire  the  status  of  a  Filipino  citizen.  Authority  for  this  view  is  Section  7  of  the  
certifying  or  declaring  that  an  alien  wife  of  the  Filipino  citizen  is  also  a  Filipino   Revised   Naturalization   Law   in   which   the   plain   language   is:   "Any   person  
citizen,  is  hereby  declared  null  and  void.   desiring  to  acquire  Philippine  citizenship,  shall  file  with  the  competent  court"  a  
  petition  for  the  purpose.  And  this,  because  such  alien  woman  is  not  a  citizen,  
Gulapa:   How   did   Burca   modify   Lao   Chay?   A   wife   must   file   a   petition   with   the   and   she   desires   to  acquire  it.   The   proper   forum,   Section   8   of   the   same   law  
competent  courts  for  purposes  of  naturalization.   points   out,   is   the   Court   of   First   Instance   of   the   province   where   the   petitioner  
  has  resided  "at  least  one  year  immediately  preceding  the  filing  of  the  petition".  
We   part   from   the   premise   that   such   an   alien   woman   does   not,   by   the   fact   of   It   is   quite   plain   that   the   determination   of   whether   said   alien   wife   should   be  
marriage,   acquire   Philippine   citizenship.   The   statute   heretofore   quoted   (Sec.   given  the  status  of  a  citizen  should  fall  within  the  area  allocated  to  competent  
15,   Revised   Naturalization   Law),   we   repeat,   recites   that   she   "shall   be   deemed   a   courts.   That   this   is   so,   is   exemplified   by   the   fact   that   this   Court   has   taken  
citizen  of  the  Philippines"  if  she  "might  herself  be  lawfully  naturalized".   jurisdiction  in  one  such  case  originating  from  the  court  of  first  instance,  where  
  an  alien  woman  had  directly  sought  naturalization  in  her  favor.  
How  then  shall  she  be  "deemed"  a  citizen  of  the  Philippines?  An  examination  of    
the  Revised  Naturalization  Law  is  quite  revealing.  For  instance,  minor  children   And,  as  nothing  in  the  Revised  Naturalization  Law  empowers  any  other  office,  
of  persons  naturalized  under  the  law  who  were  born  in  the  Philippines  "shall  be   agency,  board  or  official,  to  determine  such  question,  we  are  persuaded  to  say  
considered   citizens   thereof".   Similarly,   a   foreign-­‐born   minor   child,   if   dwelling   in   that  resolution  thereof  rests  exclusively  with  the  competent  courts.  
the   Philippines   at   the   time   of   the   naturalization   of   the   parents,   "shall    
automatically  become  a  Filipino  citizen".  No  conditions  are  exacted;  citizenship   We   accordingly   rule   that:   (1)   An   alien   woman   married   to   a   Filipino   who  
of   said   minor   children   is   conferred   by   the   law   itself,   without   further   desires  to  be  a  citizen  of  this  country  must  apply  therefor  by  filing  a  petition  
proceedings  and  as  a  matter  of  course.  An  alien  wife  of  a  Filipino  does  not  fit   for   citizenship   reciting   that   she   possesses   all   the   qualifications   set   forth   in  
into  either  of  the  categories  just  mentioned.  Legal  action  has  to  be  taken  to   Section   2,   and   none   of   the   disqualifications   under   Section   4,   both   of   the  
make  her  a  citizen.   Revised  Naturalization  Law;  (2)  Said  petition  must  be  filed  in  the  Court  of  First  
  Instance   where   petitioner   has   resided   at   least   one   year   immediately  
There   is   no   law   or   rule   which   authorizes   a   declaration   of   Filipino   citizenship.   preceding   the   filing   of   the   petition;   and   (3)   Any   action   by   any   other   office,  
Citizenship   is   not   an   appropriate   subject   for   declaratory   judgment   agency,   board   or   official,   administrative   or   otherwise   —   other   than   the  
proceedings.  And   in   one   case,   we   held   that   citizenship   of   an   alien   woman   judgment   of   a   competent   court   of   justice   —   certifying   or   declaring   that   an  
married  to  a  Filipino  must  be  determined  in  an  "appropriate  proceeding".   alien   wife   of   the   Filipino   citizen   is   also   a   Filipino   citizen,   is   hereby   declared  
  null  and  void.  
Speculations  arise  as  to  the  import  of  the  term  "appropriate  proceeding".  The    
record  of  this  case  disclose  that,  in  some  quarters,  opinion  is  advanced  that  the  
determination   of   whether   an   alien   woman   married   to   a   Filipino   shall   be  
deemed   a   Filipino   citizen,   may   be   made   by   the   Commissioner   of  

 
CONFLICT  OF  LAWS                                                                                    AV  DE  TORRES   114  
ATTY.  ARIS  L.  GULAPA                                            AY  2015-­‐2016  
70.  MO  YA  YIM  YAO  V.  CIR   Ping  for  1  month.  She  was  permitted  to  come  and  stay  in  the  Phils.  until  April  
G.R.  No.  L-­‐21289  |  1971   13,  1961.  Upon  her  arrival,  one  Asher  Cheng  filed  a  P1Million  bond  to  ensure  
  that  she  would  actually  depart  from  the  country  on  or  before  the  expiration  of  
DOCTRINE:   Under   Section   15   of   Commonwealth   Act   473,   (“…any   woman   her   authorized   period   of   stay   or   within   the   period   the   COI   or   his   authorized  
married   to   a   citizen   of   the   Philippines,   and   who   might   herself   be   lawfully   representative   might   allow.   Repeatedly,   LY   Yeung   was   allowed   to   stay   in   the  
naturalized,  is  deemed  a  citizen  of  the  Philippines.”)  an  alien  woman  marrying  a   Philippines  up  to  February  13,  1962.    
Filipino,  native  born  or  naturalized,  becomes  ipso  facto  a  Filipina  provided  she  is    
not   disqualified   to   be   a   citizen   of   the   Philippines   under   Section   4   of   the   same   But   days   before,   on   January   25,   1962,   she   married   Moy   Ya   Lim   Yao   alias  
law.   Likewise,   an   alien   woman   married   to   an   alien   who   is   subsequently   Edilberto  Aguinaldo  Lim,  a  Filipino.  The  COI  ordered  her  arrest,  deportation  and  
naturalized   here   follows   the   Philippine   citizenship   of   her   husband   the   moment   confiscation  of  her  bond.  It  was  noted  that  she  could  not  write  either  English  or  
he  takes  his  oath  as  Filipino  citizen,  provided  that  she  does  not  suffer  from  any   Tagalog,   and   could   speak   only   a   few   words   in   those   language.   She   could   not  
of   the   disqualifications   under   said   Section   4.   If   the   Court   will   interpret   it   such   name   any   Filipino   neighbor,   with   a   Filipino   name   except   one,   Rosa.   She   did   not  
that   the   alien   wife   must   prove   the   qualifications   prescribed   by   the   law   (Sec   2   of   know   the   names   of   her   brothers-­‐   in-­‐law,   or   sisters-­‐in-­‐law.   LY   Yeung   and   her  
CA  473),  the  privilege  granted  to  alien  wives  would  become  illusory.   husband  Lim  contended  that  she  has  become  a  Filipino  through  her  marriage,  
  thus  prayed  for  the  court  to  enjoin  the  COI’s  order.    
Gulapa:   How   did   this   modify   Burca   and   Lao   Chay?   What   was   the   latin   term?    
Ipso  facto.  No  need  to  file  a  petition  BUT  for  practical  purposes,  she  should  file   The   COI   and   Office   of   Solicitor   General   (OSG)   won   in   the   lower   court   on   the  
with  the  Bureau  of  Immigration.   following  basis:    
   
An  alien  woman  married  to  a  Filipino  citizen  does  not  have  to  prove  in  a  judicial   1.   It   is   evident   that   said   marriage   was   effected   merely   for   convenience   to  
proceeding   that   she   does   not   suffer   from   any   of   the   disqualifications   under   defeat  or  avoid  her  then  impending  compulsory  departure  [or  deportation].    
Section   4   of   the   Naturalization   Law,   who   will   determine   whether   she   has    
automatically   become   a   Filipino   citizen   by   the   mere   fact   of   marriage   to   a   2.  She  must  not  only  be  not  among  those  disqualified  but  also  she  must  prove  
Filipino  National?  Evidently,  her  on  determination  will  not  bind  the  government   she   is   qualified.   Based   on   Sec.15   of   the   Revised   Naturalization   Law   which  
agencies.  She  still  has  to  prove  before  some  agency  of  the  Government  that  she   provides:   “Effect   of   the   naturalization   on   wife   and   children.   —   Any   woman   who  
is   not   disqualified   to   become   a   Filipino   citizen   by   naturalization.   Since   the   is   now   or   may   hereafter   be   married   to   a   citizen   of   the   Philippines,   and   who  
current   position   of   the   SC   is   that   there   can   be   no   independent   action   for   the   might   herself   be   lawfully   naturalized   shall   be   deemed   a   citizen   of   the  
judicial   declaration   of   the   citizenship   of   an   individual   -­‐     whether   in   an   action   for   Philippines,”   COI   asserted   that   the   clause   "who   might   herself   be   lawfully  
declaratory  relief  or  in  a  summary  action  for  a  change  or  correction  in  the  Civil   naturalized"   incontestably   implies   that   an   alien   woman   may   be   deemed   a  
Registry   –   the   determination   must   be   made   by   some   administrative   agency,   citizen  of  the  Philippines  by  virtue  of  her  marriage  to  a  Filipino  citizen  only  if  she  
such  as  the  Immigration  Office  or  the  Department  of  Foreign  affairs.   possesses   all   the   qualifications   and   none   of   the   disqualifications   specified   in   the  
  law,  because  these  are  the  explicit  requisites  provided  by  law  for  an  alien  to  be  
Case:   Chinese   woman   married   a   Filipino   citizen.   Court   held   she   became   a   naturalized.  Thus,  LY  Yeung  while  claiming  not  to  be  disqualified,  cannot  allege  
Filipino  citizen  through  marriage.     that  she  possesses  all  the  qualifications  to  be  naturalized,  because  (1)  she  has  
Facts:  On  February  8,  1961,  Lau  Yuen  Yeung  (LY  Yeung)  applied  for  a  passport   been  admitted  as  a  temporary  visitor  only,  and  (2)  it  is  obvious  at  once  that  she  
visa   to   enter   the   Philippines   as   a   non-­‐immigrant.   In   the   interview   for   her   lacks  at  least,  the  req’d  length  of  residence  in  the  Philippines.  If  the  intention  of  
application   for   a   temporary   visitor's   visa   to   enter   the   Philippines,   she   stated   the   law   that   that   the   alien   need   only   be   not   disqualified,   it   would   have   been  
that   she   was   a   Chinese   residing   at   Kowloon,   Hongkong,   and   that   she   desired   to   worded   "and   who   herself   is   not   disqualified   to   become   a   citizen   of   the  
take  a  pleasure  trip  to  the  Philippines  to  visit  her  great  (grand)  uncle  Lau  Ching   Philippines."    
 
 
CONFLICT  OF  LAWS                                                                                    AV  DE  TORRES   115  
ATTY.  ARIS  L.  GULAPA                                            AY  2015-­‐2016  
3.  There  was  deliberate  and  voluntary  representation  by  LY  Yeung  that  she  will   have  been  recognized  by  the  respondent  as  a  Filipino  citizen  in  the  instant  case,  
enter  and  stay  only  for  a  period  of  1  month  and  was  able  to  secure  a  visa,  thus,   without   requiring   her   to   submit   to   the   usual   proceedings   for   naturalization.  
she   cannot   go   back   on   her   representation   to   stay   permanently   without   first   Hence,  LY  Yeung  is  not  among  those  disqualified  under  Section  4  of  CA  473,  and  
departing  from  the  Philippines  as  she  had  promised.  Based  on  Sec.  9  (g)  of  the   “may  be  lawfully  naturalized.”  
Philippine   Immigration   Act   of   1940,   “An   alien   who   is   admitted   as   a   non-­‐    
immigrant  cannot  remain  in  the  Philippines  permanently.  To  obtain  permanent   As   under   any   other   law   rich   in   benefits   for   those   coming   under   it,   doubtless  
admission,   a   non-­‐immigrant   alien   must   depart   voluntarily   to   some   foreign   there   will   be   instances   where   unscrupulous   persons   will   attempt   to   take  
country   and  procure   from   the   appropriate   Philippine   Consul   the   proper   visa   and   advantage  of  this  provision  of  law  by  entering  into  fake  and  fictitious  marriages  
thereafter  undergo  examination  by  the  Officers  of  the  Bureau  of  Immigration  at   or  mala  fide  matrimonies.  We  cannot  as  a  matter  of  law  hold  that  just  because  
a   Philippine   port   of   entry   for   determination   of   his   admissibility   in   accordance   of   these   possibilities,   the   construction   of   the   provision   should   be   otherwise  
with  the  requirements  of  this  Act.”     than  as  dictated  inexorably  by  more  ponderous  relevant  considerations,  legal,  
  juridical   and   practical.   There   can   always   be   means   of   discovering   such  
4.   COI,   under   Sec.3   of   the   Commonwealth   Act   613,   is   charged   with   the   undesirable  practice  and  every  case  can  be  dealt  with  accordingly  as  it  arises.  
administration   of   all   laws   relating   to   immigration   is   given   the   authority   to    
perform   the   quasi-­‐judicial   function   in   determining   cases   presented   to   him   in  
IX.  MARRIAGE,  ADOPTION,  AND  FAMILY  RELATIONS  
relation  to  alien  immigrants      
   
ISSUE:   MARRIAGE  
Whether  marriage  of  a  foreigner  woman  to  a  Filipino  citizen  confers  ipso  facto    
to  her  Philippine  citizenship?     Art.  17  &  Art.  26  –  Lex  loci  celebrationis  is  an  imperative  rule.  
   
HELD:   EXC:  
Yes.  Accordingly,  We  now  hold,  all  previous  decisions  of  this  Court  indicating   35   (1)   -­‐   Those   contracted   by   any   party   below   eighteen   years   of   age  
otherwise  notwithstanding,  that  under  Section  15  of  Commonwealth  Act  473,   even  with  the  consent  of  parents  or  guardians;    
an   alien   woman   marrying   a   Filipino,   native   born   or   naturalized,   becomes  ipso    
facto  a   Filipina   provided   she   is   not   disqualified   to   be   a   citizen   of   the   35   (4)   -­‐   Those   bigamous   or   polygamous   marriages   not   failing   under  
Philippines   under   Section   4   of   the   same   law.   Likewise,   an   alien   woman   Article  41;    
married   to   an   alien   who   is   subsequently   naturalized   here   follows   the   Philippine    
citizenship   of   her   husband   the   moment   he   takes   his   oath   as   Filipino   citizen,   35  (5)    -­‐   Those  contracted  through  mistake  of  one  contracting  party  as  
provided  that  she  does  not  suffer  from  any  of  the  disqualifications  under  said   to  the  identity  of  the  other;  and    
Section  4.    
    35  (6)  -­‐  Those  subsequent  marriages  that  are  void  under  Article  53;  
If   the   Court   will   interpret   it   such   that   the   alien   wife   must   prove   the      
qualifications  prescribed  by  the  law  (Sec  2  of  CA  473),  the  privilege  granted  to    
alien  wives  would  become  illusory.   The   wife   is   required   to   prove   only   that   she   36   -­‐   A   marriage   contracted   by   any   party   who,   at   the   time   of   the  
may  herself  be  lawfully  naturalized  (or  not  disqualified)  in  order  to  establish  her   celebration,   was   psychologically   incapacitated   to   comply   with   the  
citizenship  status  as  a  fact.  For  the  phrase  "herself  may  be  lawfully  naturalized,"   essential  marital  obligations  of  marriage,  shall  likewise  be  void  even  if  
the  wife  need  no  longer  prove  qualifications.  The  OSG  has  implicitly  conceded   such  incapacity  becomes  manifest  only  after  its  solemnization.    
that  had  it  been  established  in  the  proceedings  that  LY  Yeung  possesses  all  the    
qualifications   required   by   the   law   of   applicants   for   naturalization,   she   would   37  -­‐  Marriages  between  the  following  are  incestuous  and  void  from  the  
 
CONFLICT  OF  LAWS                                                                                    AV  DE  TORRES   116  
ATTY.  ARIS  L.  GULAPA                                            AY  2015-­‐2016  
beginning,   whether   relationship   between   the   parties   be   legitimate   or   prove   (2)   that   the   foreign   marriage   took   place   in   accordance   with   the   foreign  
illegitimate:     law.  
(1)   Between   ascendants   and   descendants   of   any   degree;   and  (2)    
Between  brothers  and  sisters,  whether  of  the  full  or  half  blood.     "Priest"   and   "minister   of   the   Gospel"   means   all   clergymen   of   every  
  denomination   and   faith.   A   Mohammedan   Iman   is   a   "priest   or   minister   of   the  
38   -­‐   The   following   marriages   shall   be   void   from   the   beginning   for   Gospel,"  and  Mohammedanism  is  a  "denomination,"  within  the  meaning  of  the  
reasons  of  public  policy:  [Co  SLASS  CAK]   Marriage  Law.  
(1)   Between   collateral   blood   relatives   whether   legitimate   or    
illegitimate,  up  to  the  fourth  civil  degree;  (2)  Between  step-­‐parents  and   Here  the  consequences  entailed  in  holding  that  the  marriage  of  Adong  and  the  
step-­‐children;  (3)   Between   parents-­‐in-­‐law   and   children-­‐in-­‐law;   (4)   deceased,   in   conformity   with   the   Mohammedan   religion   and   Moro   customs,  
Between   the   adopting   parent   and   the   adopted   child;  (5)   Between   the   was  void,  would  be  far  reaching  in  disastrous  result  because  there  are  at  least  
surviving   spouse   of   the   adopting   parent   and   the   adopted   child;  (6)   150K  Moros  who  have  been  married  according  to  local  custom.  
Between  the  surviving  spouse  of  the  adopted  child  and  the  adopter;  (7)    
Between   an   adopted   child   and   a   legitimate   child   of   the   adopter;   (8)   Kyna’s  notes:  
Between   adopted   children   of   the   same   adopter;   and  (9)   Between   Now,  the  validity  of  Muslims  Marriages  is  based  on  the  Muslim  Code.  
parties   where   one,   with   the   intention   to   marry   the   other,   killed   that    
other  person's  spouse,  or  his  or  her  own  spouse.  (82)     N.B.:   Art.   26.   All   marriages   solemnized   outside   the   Philippines,   in   accordance  
  with   the   laws   in   force   in   the   country   where   they   were   solemnized,   and   valid  
Art.  2,  FC:  ESSENTIAL  REQUISITIES:  [LC]   there  as  such,  shall  also  be  valid  in  this  country,  except  those  prohibited  under  
1. Legal   capacity  of  the  contracting  parties  who  must  be  a  male  and  a   Articles  35  (1),  (4),  (5)  and  (6),  36,  37  and  38.    
female    
2. Consent  freely  given  in  the  presence  of  the  solemnizing  officer   Where   a   marriage   between   a   Filipino   citizen   and   a   foreigner   is   validly  
  celebrated   and   a   divorce   is   thereafter   validly   obtained   abroad   by   the   alien  
Art.  3,  FC:  FORMAL  REQUISITIES  [ALC]   spouse   capacitating   him   or   her   to   remarry,   the   Filipino   spouse   shall   have  
1. Authority  of  the  solemnizing  officer   capacity  to  remarry  under  Philippine  law.  (As  amended  by  Executive  Order  227)    
2. A  valid  marriage  license  except  in  cases  provided  in  Chapter  2  of  this    
Title   FACTS:  
3. A   marriage   ceremony   which   takes   place   with   the   appearance   of   the   Cheong   Boo,   a   native   of   China,   died   intestate   in   Zamboanga.   He   left   property  
contracting   parties   before   the   solemnizing   officer   and   their   personal   worth   nearly   100K.   The   estate   of   the   deceased   was   claimed   by   2   parties:   (1)  
declaration   that   they   take   each   other   as   husband   and   wife   in   the   Cheong  Seng  Gee,  who  alleged  that  he  was  a  legitimate  child  by  the  marriage  of  
presence  of  not  less  than  2  witnesses  of  legal  age   Cheong   Boo   with   Tan   Dit   in   China   in   1895,   and   (2)   Mora   Adong,   who   alleged  
  that  she  had  been  lawfully  married  to  Cheong  Boo  in  1896  in  Basilan,  according  
73.  ADONG  V.  CHEONG  SENG  GEE   to  the  ceremonies  prescribed  by  the  book  on  marriage  of  the  Koran,  with  her  
G.R.  No.  18081  |  March  3,  1922   daughters  Payang  and  Rosalia.  
   
DOCTRINES:   To   establish   a   valid   foreign   marriage   pursuant   to   this   comity   The  conflicting  claims  to  the  estate  of  Cheong  Boo  were  ventilated  in  the  CFI  of  
provision,   it   is   first   necessary   to   prove   before   the   courts   of   the   Islands   (1)   the   Zamboanga.   The   trial   judge   Abeto   reached   the   conclusion   that   the   proof   did  
existence   of   the   foreign   law   as   a   question   of   fact,   and   it   is   then   necessary   to   not   sufficiently   establish   the   Chinese   marriage   but   that   because   Cheong   Seng  
Gee  had  been  admitted  to  the  Philippine  Islands  as  the  son  of  the  deceased,  he  

 
CONFLICT  OF  LAWS                                                                                    AV  DE  TORRES   117  
ATTY.  ARIS  L.  GULAPA                                            AY  2015-­‐2016  
should  share  in  the  estate  as  a  natural  child.  As  to  the  allegations  of  Adong,  the   executed,   including   decrees   of   registration,   Cheong   Boo   stated   that   he   was  
conclusion  was  that  the  marriage  between  Adong  and  the  deceased  had  been   married   to   Adong   and   he   gave   written   consent   to   the   marriage   of   his   minor  
adequately  proved  but  that  under  the  laws  of  the  Philippine  Islands  it  could  not   daughter,  Payang.  
be   held   to   be   a   lawful   marriage;   so,   the   daughters   Payang   and   Rosalia   would   3   sections   of   the   Marriage   Law   (General   Order   No.   68)   must   be   taken   into  
inherit   as   natural   children.   The   order   of   the   judge   was   for   there   to   be   a   consideration.  Sec.  IX  provides  "No  marriage  heretofore  solemnized  before  any  
partition   of   the   property   of   the   deceased   Cheong   Boo   between   the   natural   person   professing   to   have   authority   therefor   shall   be   invalid   for   want   of   such  
children,  Cheong  Seng  Gee,  Payang,  and  Rosalia.  Both  parties  appealed.     authority   or   on   account   of   any   informality,   irregularity,   or   omission,   if   it   was  
  celebrated  with  the  belief  of  the  parties,  or  either  of  them,  that  he  had  authority  
ISSUES:   and  that  they  have  been  lawfully  married."  Marriage  in  this  jurisdiction  is  both  
1.  Whether  the  marriage  contracted  in  China,  and  proven  mainly  by  an  alleged   a  civil  contract  and  a  new  relation,  an  institution  in  the  maintenance  of  which  
matrimonial  letter,  is  valid  in  the  Philippines   the   public   is   deeply   interested.   Consequently,   every   intendment   of   the   law  
2.  Whether  the  marriage  performed  in  the  Philippines  according  to  the  rites  of   leans   toward   legalizing   matrimony.   Persons   dwelling   together   in   apparent  
the  Mohammedan  religion  is  valid   matrimony   are   presumed,   in   the   absence   of   contrary   evidence,   to   be   in   fact  
  married.   Sec.   IX   is   in   the   nature   of   a   curative   provision   intended   to   safeguard  
HELD:   society  by  legalizing  prior  marriages.  The  courts  can  properly  incline  the  scales  
Validity  of  the  Chinese  Marriage   of  their  decisions  in  favor  of  the  solution  which  will  more  effectively  promote  
The   immigration   documents   only   go   to   show   the   relation   of   parent   and   child   the  public  policy.  Here  the  consequences  entailed  in  holding  that  the  marriage  
existing   between   the   Cheong   Boo   and   son   Cheong   Seng   Gee   but   do   not   of  the  Adong  and  the  deceased,  in  conformity  with  the  Mohammedan  religion  
establish  the  marriage  between  the  deceased  and  Seng  Gee’s  mother.  Sec.  IV   and   Moro   customs,   was   void,   would   be   far   reaching   in   disastrous   result  
of  Marriage  Law  (General  Order  No.  68)  provides  that  "All  marriages  contracted   because   there   are   at   least   150K   Moros   who   have   been   married   according   to  
without  these  Islands,  which  would  be  valid  by  the  laws  of  the  country  in  which   local  custom.  The  court  has  the  power  either  to  nullify  or  to  validate  all  of  these  
the   same   were   contracted,   are   valid   in   these   Islands."   To   establish   a   valid   marriages;  either  to  make  all  of  the  children  born  of  these  unions  bastards  or  to  
foreign   marriage   pursuant   to   this   comity   provision,   it   is   first   necessary   to   make   them   legitimate.   The   court   held   that   the   evidence   produced   a   moral  
prove   before   the   courts   of   the   Islands   the   existence   of   the   foreign   law   as   a   conviction   of   the   existence   of   the   Mohammedan   marriage   and   regarded   the  
question   of   fact,   and   it   is   then   necessary   to   prove   the   alleged   foreign   provisions  of  Sec.  IX  as  validating  marriages  performed  according  to  the  rites  of  
marriage   by   convincing   evidence.   In   the   case   at   bar   there   is   no   competent   the  Mohammedan  religion.  Thus,  the  Mohammedan  marriage  is  valid,  giving  to  
testimony   as   to   what   the   laws   of   Amoy,   China   concerning   marriage   were   in   the  widow  and  the  2  legitimate  children  the  rights  accruing  to  them  under  the  
1895.  There  is  lacking  proof  so  clear,  strong,  and  unequivocal  as  to  produce  a   law.  
moral  conviction  of  the  existence  of  the  alleged  prior  Chinese  marriage.   Also,  Sec.  V  provides  that  "Marriage  may  be  solemnized  by  either  a  judge  of  
  any  court  inferior  to  the  SC,  justice  of  the  peace,  or  priest  or  minister  of  the  
Validity  of  the  Mohammedan  Marriage   Gospel  of  any  denomination..."  "Priest"  and  "minister  of  the  Gospel"  means  
Adong  claimed  that  a  marriage  ceremony  took  place  in  Basilan  according  to  the   all   clergymen   of   every   denomination   and   faith.   A   Mohammedan   Iman   is   a  
rites   of   Mohammedan   religion.   This   is   established   by   the   Iman   who   solemnized   "priest   or   minister   of   the   Gospel,"   and   Mohammedanism   is   a  
the  marriage,  and  by  other  eyewitnesses,  such  as  Adong’s  father  and  the  chief   "denomination,"  within  the  meaning  of  the  Marriage  Law.  
of   the   rancheria.   The   groom   complied   with   Quranic   law   by   giving   to   the   bride   a   Lastly,  Sec.  VI  provides  that  "No  particular  form  for  the  ceremony  of  marriage  is  
dowry  of  P250  in  money  and  P250  in  goods.  From  then  day  until  the  death  of   required,   but   the   parties   must   declare,   in   the   presence   of   the   person  
Cheong   Boo,   they   cohabited   as   husband   and   wife.   They   had   5   children,   2   are   solemnizing   the   marriage,   that   they   take   each   other   as   husband   and   wife."   The  
living   at   the   time   of   this   case.   In   his   relations   w/   3rd   persons,   Cheong   Boo   2   essentials   of   a   valid   marriage   are   capacity   and   consent.   The   latter   element  
treated   Adong   as   his   lawful   wife.   He   admitted   this   relationship   in   several   may   be   inferred   from   the   ceremony   performed,   the   acts   of   the   parties,   and  
private   and   public   documents.   Thus,   when   different   legal   documents   were   habit  or  repute.  In  this  instance,  there  is  no  question  of  capacity  nor  consent.  
 
CONFLICT  OF  LAWS                                                                                    AV  DE  TORRES   118  
ATTY.  ARIS  L.  GULAPA                                            AY  2015-­‐2016  
While   it   is   true   that   during   the   Mohammedan   ceremony,   the   remarks   of   the   (2)  The  Presiding  Justice  and  the  Justices  of  the  Court  of  Appeals;  
priest   were   addressed   more   to   the   elders   than   to   the   participants,   it   is   likewise   (3)  Judges  of  the  Courts  of  First  Instance;  
true   that   Cheong   Boo   and   Adong   did   in   fact   take   each   other   to   be   husband   and   (4)  Mayors  of  cities  and  municipalities;  
wife  and  did  thereafter  live  together  as  husband  and  wife.   (5)  Municipal  judges  and  justices  of  the  peace;  
  (6)   Priests,   rabbis,   ministers   of   the   gospel   of   any   denomination,  church,  religion  
78.  WONG  WOO  YUI  V.  VIVO   or  sect,  duly  registered,  as  provided  in  Article  92;  and  
G.R.  No.  L-­‐21076  |  March  31,  1965   (7)   Ship   captains,   airplane   chiefs,   military   commanders,   and   consuls   and   vice-­‐
  consuls  in  special  cases  provided  in  Articles  74  and  75.  (4a)  
DOCTRINE:   Since   our   law   only   recognizes   a   marriage   celebrated   before   any   of    
the  officers  mentioned  therein,  and  a  village  leader  is  not  one  of  them,  it  is  clear   FACTS:    
that  petitioner’s  marriage,  even  if  true,  cannot  be  recognized  in  this  jurisdiction.   In   proceedings   held   before   the   Board   of   Special   Inquiry   in   June,   1961,   Wong  
  Woo  Yiu  declared  that  she  came  to  the  Philippines  in  1961  for  the  first  time  to  
The  fact  of  marriage  was  proved  but  not  the  Chinese  Law  on  marriage.  Even  if   join  her  husband,  Perfecto  Bias,  a  Filipino  citizen  to  whom  she  was  married  in  
the   petitioners   proved   that   a   marriage   was   celebrated,   the   marriage   is   still   void   Chingkang,   China   on   January   15,   1929,   that   they   had   several   children   all   of  
because  the  marriage  was  not  celebrated  before  any  of  the  officers  mentioned   whom   are   not   located   in   the   Philippines,   and   that   their   marriage   was  
in  the  law  (Philippine  law).     celebrated  by  one  Chua  Tio,  a  village  leader.    
   
ALG:   However,   the   present   Family   Code   changed   this,   particularly   Article   26.   On  June  28,  1961,  the  Board  of  Special  Inquiry  No.  3  rendered  a  decision  finding  
Absence   or   lack   of   authority   by   the   solemnizing   officers   is   NOT   one   of   the   petitioner   to   be   legally   married   to   Perfecto   Bias,   thus   declaring   legal   her  
exceptions   of   void   marriages.   Thus,   the   foreign   marriage   is   still   valid,   admission   into   the   country   as   a   non-­‐quota   immigrant.   This   decision   was  
notwithstanding  such  lack  of  authority.   affirmed   by   the   Board   of   Commissioners   on   July   12,   1961   of   which   petitioner  
  was   duly   informed   in   a   letter   sent   on   the   same   date   by   the   Secretary   of   the  
Kyna’s  notes:   Board.    
But  how  come  gay  marriages  are  void  even  if  not  expressly  prohibited  by  Article    
26  of  the  FC?   However,  on  June  28,  1962,  the  same  Board  of  Commissioners,  but  composed  
  entirely  of  a  new  set  of  members,  rendered  a  new  decision  contrary  to  that  of  
See  à  Article  1.  Marriage  is  a  special  contract  of  permanent  union  between  a   the  Board  of  Special  Inquiry  No.  3  and  ordering  petitioner  to  be  excluded  from  
man   and   a   woman  entered  into  in  accordance  with  law  for  the  establishment   the   country,   after   discrepancies   were   found   in   the   statements   made   by  
of  conjugal  and  family  life.   petitioner   and   her   alleged   husband   during   several   investigation   conducted   by  
  the   immigration   authorities   concerning   the   alleged   marriage   before   a   village  
What’s   wrong   with   this   pronouncement?   There   is   a   presumption   that   the   leader  in  China  in  1929,  thus  concluding  that  the  petitioner's  claim  that  she  is  
marriage   was   celebrated   abroad   but   because   the   parties   failed   to   prove   the   the  lawful  wife  of  Perfecto  Bias  was  without  basis  in  evidence  as  it  was  "a  mass  
foreign   law   on   marriage,   the   courts   applied   the   Doctrine   of   Processual   of  oral  and  documentary  evidence  bereft  of  substantial  proof  of  husband-­‐wife  
Presumption  à  argument  becomes  CIRCULAR   relationship,”   the   Board   of   Commissioners   motu   proprio   reviewed   the   record  
  concerning  the  admission  of  petitioner  into  the  country  resulting  in  its  finding  
How  does  the  SC  step  back?  –  See  next  case   that  she  was  improperly  admitted.    
   
*SEE  Art.  56.  Marriage  may  be  solemnized  by:   ISSUE:  
(1)  The  Chief  Justice  and  Associate  Justices  of  the  Supreme  Court;   Whether   Wong   Woo   Yiu's   marriage   to   Perfecto   Blas   is   valid   and   making   her  

 
CONFLICT  OF  LAWS                                                                                    AV  DE  TORRES   119  
ATTY.  ARIS  L.  GULAPA                                            AY  2015-­‐2016  
admission  into  the  country  legal   Custom  is  defined  as  "a  rule  of  conduct  formed  by  repetition  of  acts,  uniformly  
  observed  (practiced)  as  a  social  rule,  legally  binding  and  obligatory".  The  law  
HELD:   requires   that   "a   custom   must   be   proved   as   a   fact,   according   to   the   rules   of  
No.   SC   affirmed   the   latter   Board's   decision.   Indeed,   not   only   is   there   no   evidence"  [Art.  12,  NCC.]  
documentary   evidence   to   support   the   alleged   marriage   of   petitioner   to    
Perfecto  Bias  but  the  record  is  punctured  with  so  many  inconsistencies  which   In   proving   a   foreign   law   the   procedure   is   provided   in   the   Rules   of   Court.   With  
cannot  but  lead  one  to  doubt  their  veracity  concerning  the  pretended  marriage   respect  to  an  unwritten  foreign  law,  Rule  130,  §45  states  that:  
in  China  in  1929.  This  claim  cannot  also  be  entertained  under  our  law  on  family    
relations.  Thus,  Article  15  of  our  new  Civil  Code  provides  that  laws  relating  to   SEC.   45.   Unwritten   law.—The   oral   testimony   of   witnesses,   skilled   therein,   is  
family   rights   or   to   the   status   of   persons   are   binding   upon   citizens   of   the   admissible   as   evidence   of   the   unwritten   law   of   a   foreign   country,   as   are   also  
Philippines,   even   though   living   abroad,   and   it   is   well-­‐known   that   in   1929   in   printed  and  published  books  of  reports  of  decisions  of  the  courts  of  the  foreign  
order   that   a   marriage   celebrated   in   the   Philippines   may   be   valid   it   must   be   country,  if  proved  to  be  commonly  admitted  in  such  courts.  
solemnized   either   by   a   judge   of   any   court   inferior   to   the   Supreme   Court,   a    
justice  of  the  peace,  or  a  priest  or  minister  of  the  gospel  of  any  denomination   Proof   of   a   written   foreign   law,   on   the   other   hand,   is   provided   for   under   Rule  
duly   registered   in   the   Philippine   Library   and   Museum   (Public   Act   3412,   132  section  25,  thus:  
Section  2).  But  it  may  be  contended  that  under  Section  4  of  General  orders  No.    
68,  as  reproduced  in  Section  19  of  Act  No.  3613,  which  is  now  Article  71  of  our   SEC.  25.  Proof  of  public  or  official  record.—An  official  record  or  an  entry  therein,  
new  Civil  Code,  a  marriage  contracted  outside  of  the  Philippines  which  is  valid   when   admissible   for   any   purpose,   may   be   evidenced   by   an   official   publication  
under   the   law   of   the   country   in   which   it   was   celebrated   is   also   valid   in   the   thereof   or   by   a   copy   attested   by   the   officer   having   the   legal   custody   of   the  
Philippines.  But  no  validity  can  be  given  to  this  contention  because  no  proof   record,   or   by   his   deputy,   and   accompanied,   if   the   record   is   not   kept   in   the  
was  presented  relative  to  the  law  of  marriage  in  China.  Such  being  the  case,   Philippines,   with   a   certificate   that   such   officer   has   the   custody.   If   the   office   in  
we  should  apply  the  general  rule  that  in  the  absence  of  proof  of  the  law  of  a   which  the  record  is  kept  is  in  a  foreign  country,  the  certificate  may  be  made  by  a  
foreign  country  it  should  be  presumed  that  it  is  the  same  as  our  own.     secretary  of  embassy  or  legation,  consul  general,  consul,  vice  consul,  or  consular  
  agent  or  by  any  officer  in  the  foreign  service  of  the  Philippines  stationed  in  the  
The  statutes  of  other  countries  or  states  must  be  pleaded  and  proved  the  same   foreign  country  in  which  the  record  is  kept  and  authenticated  by  the  seal  of  his  
as  any  other  fact.  Courts  cannot  take  judicial  notice  of  what  such  laws  are.  In   office.  
the  absence  of  pleading  and  proof  the  laws  of  a  foreign  country  or  state  will  be    
presumed   to   be   the   same   as   our   own.   Since   our   law   only   recognizes   a   The   Court   has   interpreted   §25   to   include   competent   evidence   like   the   testimony  
marriage   celebrated   before   any   of   the   officers   mentioned   therein,   and   a   of  a  witness  to  prove  the  existence  of  a  written  foreign  law.    
village  leader  is  not  one  of  them,  it  is  clear  that  petitioner’s  marriage,  even  if   *1.  Official  publication,  or  
true,  cannot  be  recognized  in  this  jurisdiction.     2.  Copy  
  a.   attested   by   the   officer   having   the   legal   custody   of   the   record,   or   by   his  
83.  YAO  KEE,  ET  AL.  V.  SY-­‐GONZALES,  SUPRA   deputy,  
  b.   and   accompanied,   if   the   record   is   not   kept   in   the   Philippines,   with   a  
DOCTRINE:   To   establish   a   valid   foreign   marriage   two   things   must   be   proven,   certificate   that   such   officer   has   the   custody.   /   If   the   office   in   which   the   record   is  
namely:   (1)   the   existence   of   the   foreign   law   as   a   question   of   fact;   and   (2)   the   kept   is   in   a   foreign   country,   the   certificate   may   be   made   by   a   secretary   of  
alleged  foreign  marriage  by  convincing  evidence.   embassy  or  legation,  consul  general,  consul,  vice  consul,  or  consular  agent  or  by  
  any   officer   in   the   foreign   service   of   the   Philippines   stationed   in   the   foreign  
country  in  which  the  record  is  kept  and  
c.  authenticated  by  the  seal  of  his  office.  
 
CONFLICT  OF  LAWS                                                                                    AV  DE  TORRES   120  
ATTY.  ARIS  L.  GULAPA                                            AY  2015-­‐2016  
  deceased   with   Asuncion   Gillego;   and   (b)   to   their   knowledge   Sy   Kiat   died  
In  the  case  at  bar  petitioners  did  not  present  any  competent  evidence  relative  to   intestate;  among  others.  
the   law   and   custom   of   China   on   marriage.   The   testimonies   of   Yao   and   Gan    
Ching  cannot  be  considered  as  proof  of  China's  law  or  custom  on  marriage  not   The   petition   was   opposed   by   Petitioners   Yao   Kee,   Sze   Sook   Wah,   Sze   Lai   Cho  
only  because   they  are  self-­‐serving  evidence,  but  more  importantly,  there  is  no   and   Sy   Yun   Chen   who   alleged   that:   (a)   Yao   Kee   is   the   lawful   wife   of   Sy   Kiat,  
showing  that  they  are  competent  to  testify  on  the  subject  matter.   whom   he   married   in   China;   (b)   the   other   oppositors   are   the   legitimate   children  
  of   the   deceased   with   Yao   Kee;   and,   (c)   Sze   Sook   Wah   is   the   eldest   among   them  
ALG:   Fact   of   marriage   was   proven   in   this   case,   but   the   law   was   not   proved   that   and   is   competent,   willing   and   desirous   to   become   the   administratrix   of   the  
it  was  done  in  accordance  with  Chinese  laws.     estate  of  Sy  Kiat.  
   
But  it  was  said  that  the  absence  or  lack  of  authority  by  the  solemnizing  officers   After  hearing,  the  probate  court  ruled  in  favor  of  the  petitioners  and  found  that  
is   NOT   one   of   the   exceptions   of   void   marriages   under   Article   26.   Thus,   the   (1)   Sy   Kiat   was   legally   married   to   Yao   Kee;   (2)   Petitioners   are   the   legitimate  
foreign   marriage   is   still   valid,   notwithstanding   such   lack   of   authority   by   the   children   of   Yao   Kee   with   Sy   Kiat;   and   (3)   private   respondents   are   the  
solemnizing  officer.   acknowledged   illegitimate   offsprings   of   Sy   Kiat   with   Asuncion   Gillego.   On  
  appeal   the   CA   ruled   that   Sy   Kiat   is   an   unmarried   man   who   was   living   with  
Here,   it   can   be   concluded   that   Yao   Kee   (and   even   Wong   Woo)   is   still   the   correct   Asuncion  Gillego  without  the  benefit  of  marriage  for  many  years  and  that  the  
ruling  despite  Article  26  (which  does  not  expressly  provide  that  lack  of  authority   legality   of   the   alleged   marriage   of   Sy   Kiat   to   Yao   Kee   in   China   had   not   been  
of   solemnizing   officer   invalidates   a   marriage).   Article   26   presupposes   that   a   proven  to  be  valid  to  the  laws  of  the  Chinese  People's  Republic  of  China.  
marriage   that   is   validly   celebrated   in   one   state.   The   marriages   in   these   two    
cases   could   not   have   complied   with   Article   26   because   these   marriages   were   ISSUES:  
not  proven.  Thus  Article  26  becomes  immaterial  in  these  two  cases.     1.  Whether  the  marriage  of  Sy  Kiat  to  Yao  Kee  was  proven  valid  in  accordance  
  with  laws  of  the  People's  Republic  of  China  
*Av:  This  is  confusing  to  me.  Art.  26  refers  to  marriages  celebrated  outside  the   2.   Whether   private   respondents   are   natural   children   of   Sy   Kiat   with   Asuncion  
Philippines,   such   as   the   case   at   bar.   In   order   for   them   to   be   valid,   it   must   be   Gillego  
valid   where   it   was   executed   but   due   to   failure   to   prove   the   foreign   law,   the   law    
is   presumed   to   be   the   same   as   the   Phil.   Law.   In   the   Phil,   Art.   56   then   should   HELD:  
govern   with   regard   to   solemnizing   officer,   no   longer   Art.   26.   In   any   case,   one   1.  No.  To  buttress  petitioners’  argument  that  the  marriage  of  Sy  Kiat  to  Yao  Kee  
can  argue  simply  that  as  ruled  in  Wong  Woo  and  Yao  Kee,  the  court  applied  Art.   in   accordance   with   Chinese   law   and   custom   was   conclusively   proven,   they  
56  not  Art.  26  anymore.     relied  on  the  following  testimonial  and  documentary  evidence.  
   
FACTS:   (1)   the   testimony   of   Yao   Kee:   She   testified   that   she   was   married   to   Sy  
Sy   Kiat,   a   Chinese   national,   died   on   January  17,   1977   in   Caloocan   City   where   he   Kiat  in  1931  in  Fookien,  China.  She  does  not  have  a  marriage  certificate  
was   then   residing,   leaving   behind   real   and   personal   properties   here   in   the   because  the  practice  during  that  time  was  for  elders  to  agree  upon  the  
Philippines  worth  P300,000.00  more  or  less.   betrothal   of   their   children,   and   in   her   case,   her   elder   brother   was   the  
  one  who  contracted  or  entered  into  [an]  agreement  with  the  parents  of  
Thereafter,  private  respondents  (all  surnamed  Sy)  filed  a  petition  for  the  grant   her  husband.  During  the  wedding  the  document  would  be  signed  by  the  
of   letters   of   administration   of   the   then   CFI   of   Rizal,   Caloocan   City.   In   said   parents  of  the  groom  as  well  as  by  the  parents  of  the  bride  and  there  is  
petition   they   alleged   among   others   that   (a)   they   are   the   children   of   the   no   solemnizing   officer   as   is   known   in   the   Philippines.   The   parties  
themselves   do   not   sign   the   document.   As   to   the   whereabouts   of   this  
document,   she   and   Sy   Kiat   were   married   for   46   years   already   and   the  
 
CONFLICT  OF  LAWS                                                                                    AV  DE  TORRES   121  
ATTY.  ARIS  L.  GULAPA                                            AY  2015-­‐2016  
document   was   left   in   China   and   she   doubt   if   that   document   can   still   be   The   law   on   foreign   marriages   is   provided   by   Art.   71   of   the   Civil   Code   which  
found  now.  It  was  left  in  the  possession  of  Sy  Kiat's  family.  Right  now,  she   states   that:   Art.   71.   All   marriages   performed   outside   the   Philippines   in  
does  not  know  the  whereabouts  of  that  document  because  of  the  lapse   accordance   with   the   laws   in   force   in   the   country   where   they   were   performed  
of   many   years   and   because   they   left   it   in   a   certain   place   and   it   was   and   valid   there   as   such,   shall   also   be   valid   in   this   country,   except   bigamous,  
already  eaten  by  the  termites.     polygamous,   or   incestuous   marriages,   as   determined   by   Philippine   law.   [Now,  
(2)  The  testimony  of  Gan  Ching,  a  younger  brother  of  Yao  Kee  who  stated   Art.  26,  FC]  
that   he   was   among   the   many   people   who   attended   the   wedding   of   his    
sister   with   Sy   Kiat   and   that   no   marriage   certificate   is   issued   by   the   Construing   this   provision   of   law   the   Court   has   held   that   to   establish   a   valid  
Chinese  government,  a  document  signed  by  the  parents  or  elders  of  the   foreign  marriage  two  things  must  be  proven,  namely:  (1)  the  existence  of  the  
parties  being  sufficient.   foreign   law   as   a   question   of   fact;   and   (2)   the   alleged   foreign   marriage   by  
(3)   The   statements   made   by   Asuncion   Gillego   to   the   effect   that   Sy   Kiat   convincing  evidence.  
was   married   to   Yao   Kee   according   to   Chinese   custom;   and   Sy   Kiat's    
admission   to   her   that   he   has   a   Chinese   wife   whom   he   married   according   EXISTENCE  OF  THE  FOREIGN  LAW  
to  Chinese  custom.   In  proving  a  foreign  law  the  procedure  is  provided  in  the  Rules  of  Court.  With  
(4)   Sy   Kiat's   Master   Card   of   Registered   Alien   issued   in   Caloocan   City   on   respect  to  an  unwritten  foreign  law,  Rule  130,  §45  states  that:  
October  3,  1972  where  the  following  entries  are  found:  "Marital  status—  
Married";  "If  married  give  name  of  spouses—Yao  Kee";  "Address-­‐China;   SEC.  45.  Unwritten  law.—The  oral  testimony  of  witnesses,  skilled  therein,  
"Date  of  marriage—1931";  and  "Place  of  marriage—China"   is   admissible   as   evidence   of   the   unwritten   law   of   a   foreign   country,   as  
(5)   Sy   Kiat's   Alien   Certificate   of   Registration   issued   in   Manila   stating:   are  also  printed  and  published  books  of  reports  of  decisions  of  the  courts  
"Civil   status—Married";   and,   'If   married,   state   name   and   address   of   of   the   foreign   country,   if   proved   to   be   commonly   admitted   in   such  
spouse—Yao  Kee  Chingkang,  China".   courts.  
(6)   The   certification   issued   in   Manila   by   the   Embassy   of   the   People's    
Republic   of   China   to   the   effect   that   "according   to   the   information   Proof  of  a  written   foreign   law,  on  the  other  hand,  is  provided  for  under  Rule  
available  at  the  Embassy  Mr.  Sy  Kiat  a  Chinese  national  and  Mrs.  Yao  Kee   132  section  25,  thus:  
alias   Yui   Yip   also   Chinese   were   married   on   January   19,   1931   in   Fukien,    
the  People's  Republic  of  China.   SEC.  25.  Proof  of  public  or  official  record.—An  official  record  or  an  entry  
  therein,   when   admissible   for   any   purpose,   may   be   evidenced   by   an  
These   evidence   may   very   well   prove   the  fact   of   marriage  between   Yao   Kee   and   official  publication  thereof  or  by  a  copy  attested  by  the  officer  having  the  
Sy   Kiat.   However,   the   same   do   not   suffice   to   establish   the   validity   of   said   legal   custody   of   the   record,   or   by   his   deputy,   and   accompanied,   if   the  
marriage  in  accordance  with  Chinese  law  or  custom.   record   is   not   kept   in   the   Philippines,   with   a   certificate   that   such   officer  
  has   the   custody.   If   the   office   in   which   the   record   is   kept   is   in   a   foreign  
Custom   is   defined   as   "a   rule   of   conduct   formed   by   repetition   of   acts,   country,   the   certificate   may   be   made   by   a   secretary   of   embassy   or  
uniformly   observed   (practiced)   as   a   social   rule,   legally   binding   and   legation,  consul  general,  consul,  vice  consul,  or  consular  agent  or  by  any  
obligatory".   The   law   requires   that   "a   custom   must   be   proved   as   a   fact,   officer   in   the   foreign   service   of   the   Philippines   stationed   in   the   foreign  
according  to  the  rules  of  evidence"  [Art.  12,  NCC.]  On  this  score  the  Court  had   country  in  which  the  record  is  kept  and  authenticated  by  the  seal  of  his  
occasion   to   state   that   "a   local   custom   as   a   source   of   right   can   not   be   office.  
considered   by   a   court   of   justice   unless   such   custom   is   properly   established   by    
competent   evidence   like   any   other   fact.”  The   same   evidence,   if   not   one   of   a   The   Court   has   interpreted   §25   to   include   competent   evidence   like   the  
higher  degree,  should  be  required  of  a  foreign  custom.   testimony  of  a  witness  to  prove  the  existence  of  a  written  foreign  law.    
   
 
CONFLICT  OF  LAWS                                                                                    AV  DE  TORRES   122  
ATTY.  ARIS  L.  GULAPA                                            AY  2015-­‐2016  
In  the  case  at  bar  petitioners  did  not  present  any  competent  evidence  relative   celebrated,   it   therefore   follows   that   her   marriage   to   Sy   Kiat,   even   if   true,  
to  the  law  and  custom  of  China  on  marriage.  The  testimonies  of  Yao  and  Gan   cannot  be  recognized  in  this  jurisdiction.  
Ching  cannot  be  considered  as  proof  of  China's  law  or  custom  on  marriage  not    
only  because  they  are  self-­‐serving  evidence,  but  more  importantly,  there  is  no   2.   Yes.   Private   respondents   on   the   other   hand   are   also   the   deceased's  
showing  that  they  are  competent  to  testify  on  the  subject  matter.   For   failure   acknowledged  natural  children  with  Asuncion  Gillego.  They  have  in  their  favor  
to   prove   the   foreign   law   or   custom,   and   consequently,   the   validity   of   the   their   father's   acknowledgment,   evidenced   by   a   compromise   agreement  
marriage  in  accordance  with  said  law  or  custom,  the  marriage  between  Yao  Kee   entered  into  by  and  between  their  parents  and  approved  by  the  CFI,  wherein  Sy  
and  Sy  Kiat  cannot  be  recognized  in  this  jurisdiction.   Kiat  not  only  acknowleged  them  as  his  children  by  Asuncion  Gillego  but  likewise  
  made  provisions  for  their  support  and  future  inheritance.  
Petitioners   cited   the   case   of   Sy   Joc   Lieng   v.   Sy   Quia,   where   the   party   alleging    
the   foreign   marraige   were   not   duty   bound   to   prove   the   Chinese   law   on   As  regards  petitioners,  they  failed  to  establish  the  marriage  of  Yao  Kee  with  Sy  
marriage   as   judicial   notice   thereof   had   been   taken   by   the   Court.   This   Kiat  according  to  the  laws  of  China.  Thus,  they  cannot  be  accorded  the  status  of  
contention   is   erroneous.   Well-­‐established   in   this   jurisdiction   is   the   principle   legitimate  children  but  only  that  of  acknowledged  natural  children.  Petitioners  
that   Philippine   courts   cannot   take   judicial   notice   of   foreign   laws.   They   must   are  natural  children,  it  appearing  that  at  the  time  of  their  conception  Yao  Kee  
be   alleged   and   proved   as   any   other   fact.   Moreover   a   reading   of   said   case   and  Sy  Kiat  were  not  disqualified  by  any  impediment  to  marry  one  another  [See  
would   show   that   the   party   alleging   the   foreign   marriage   presented   a   witness,   Art.  269,  Civil  Code.]  
one  Li  Ung  Bieng,  to  prove  that  matrimonial  letters  mutually  exchanged  by  the    
contracting   parties   constitute   the   essential   requisite   for   a   marriage   to   be   85.  BOARD  OF  COMMISSIONERS  V.  DELA  ROSA  
considered   duly   solemnized   in   China.   His   testimony   was   uniformly   G.R.  Nos.  95122-­‐23  |  31  May  1991  
corroborated  by  authors  on  the  subject  of  Chinese  marriage.    
  DOCTRINE:  "In   case   of   doubt,   all   presumptions   favor   the   solidarity   of   the   family.  
Further,  assuming  arguendo  that  the  Court  has  indeed  taken  judicial  notice  of   Thus,  every   intendment   of   law   or   facts   leans   toward   the   validity   of   marriage,  
the  law  of  China  on  marriage  in  the  aforecited  case,  petitioners  however  have   the   indissolubility   of   the   marriage   bonds,  the   legitimacy   of   children,  the  
not   shown   any   proof   that   the   Chinese   law   or   custom   obtaining   at   the   time   community   of   property   during   marriage,   the   authority   of   parents   over   their  
the  Sy  Joc  Lieng  marriage  was  celebrated  in  1847  was  still  the  law  when  the   children,   and   the   validity   of   defense   for   any   member   of   the   family   in   case   of  
alleged  marriage  of  Sy  Kiat  to  Yao  Kee  took  place  in  1931  or  eighty-­‐four  (84)   unlawful  aggression."  
years  later.    
  Kyna’s  notes:  
EXISTENCE  OF  THE  FOREIGN  MARRIAGE   ALG:  What  was  not  proved  is  the  CHINESE  law.  The  issue  of  deportation  would  
Petitioners   moreover   cite   the   case   of  U.S.   v.   Memoracion,   where   the   testimony   be  resolved  by  filiation  of  the  father  (meaning  there  is  a  need  to  establish  the  
of   one   of   the   contracting   parties   is   competent   evidence   to   show   the   fact   of   legitimate   filiation   with   the   father   by   proving   the   Chinese   law   relating   to   the  
marriage.  This  case  however  is  not  applicable  to  the  case  at  bar  as  said  case  did   marriage.   (Constitution   in   1935   says   “…of   Filipino   FATHERS.   If   mother,   there  
not   concern   a   foreign   marriage   and   the   issue   posed   was   whether   or   not   the   should  selection  at  the  age  of  majority).    
oral   testimony   of   a   spouse   is   competent   evidence   to   prove   the  fact   of    
marriage  in  a  complaint  for  adultery.   In  short,  William  didn’t  need  to  prove  fact  of  marriage  and  law  at  that  point  (he  
  was  only  12).  SC  didn’t  overturn  Adiong,  Yao  Kee  and  Wong  Wee.  Why?  
Accordingly,  in  the  absence  of  proof  of  the  Chinese  law  on  marriage,  it  should    
be   presumed   that   it   is   the   same   as   ours.   Since   Yao   Kee   admitted   in   her   1.  Deportation  proceeding  –  technical  rules  of  evidence  won’t  apply  
testimony   that   there   was   no   solemnizing   officer   as   is   known   here   in   the  
Philippines   [See   Art.   56,   Civil   Code]   when   her   alleged   marriage   to   Sy   Kiat   was  
 
CONFLICT  OF  LAWS                                                                                    AV  DE  TORRES   123  
ATTY.  ARIS  L.  GULAPA                                            AY  2015-­‐2016  
2.   It   was   not   William’s   fault   -­‐   The   lack   of   proof   of   Chinese   law   on   the   matter   2)  and  a  warrant  of  exclusion  also  dated  July  6,  1962  was  issued.    
cannot   be   blamed   on   Santiago   Gatchalian   much   more   on   respondent   William    
Gatchalian   who   was   then   a   twelve-­‐year   old   minor.   The   fact   is,   as   records   William  and  the  others  covered  by  the  warrant  of  exclusion  filed  a  motion  for  
indicate,   Santiago   was   not   pressed   by   the   Citizenship   Investigation   Board   to   re-­‐   hearing   with   the   BOSI   where   the   deportation   case   was   assigned.   The   Acting  
prove   the   laws   of   China   relating   to   marriage,   having   been   content   with   the   Commissioner  reaffirmed  Decision  1  and  recalled  the  warrant  of  arrest  against  
testimony   of   Santiago   that   the   Marriage   Certificate   was   lost   or   destroyed   William.    
during   the   Japanese   occupation   of   China.   Neither   was   Francisco   Gatchalian's    
testimony  subjected  to  the  same  scrutiny  by  the  Board  of  Special  Inquiry.     However,   in   1990,   the   NBI   Acting   Director   recommended   that   William   be  
  charged   with   a   violation   of   the   Immigration   Act   of   1940   to   the   Justice  
3.   Nevertheless,   the   testimonies   of   Santiago   Gatchalian   and   Francisco   Secretary,   who   then   indorsed   the   recommendation   to   the   Commissioner   of  
Gatchalian   before   the   Philippine   consular   and   immigration   authorities   Immigration  for  investigation  and  immediate  action.  William  was  arrested  but  
regarding   their   marriages,   birth   and   relationship   to   each   other   are   not   self-­‐ was  also  released  on  the  same  day  upon  posting  a  P200K  cash  bond.    
serving  but  are  admissible  in  evidence  as  statements  or  declarations  regarding    
family  reputation  or  tradition  in  matters  of  pedigree  (Sec.  34,  Rule  130).   William   then   filed   a   petition   for   certiorari   and   prohibition   before   the   Manila  
  RTC  (Judge  dela  Rosa  presided).  BOC  filed  a  motion  to  dismiss  alleging  that  the  
Regarding   burden   of   proof   (“…he   who   asserts   that   the   marriage   is   not   valid   judge  had  no  jurisdiction  over  the  BOC  and/or  the  BOSI.  The  MTD  was  denied  
under   our   law   bears   the   burden   of   proof   to   present   the   foreign   law.)   This   is   a   and  the  judge  restrained  the  BOC  from  continuing  with  any  of  the  proceedings  
circumstance   exclusively   applied   in   this   case.   The   general   rule   still   stands   that   that  would  lead  to  the  deportation  of  William.    
one  still  has  to  prove  the  law  and  fact  of  marriage.    
  Two   days   later,   William’s   wife   and   minor   children   filed   an   injunction   case  
FACTS:   before   the   Valenzuela   RTC   alleging   that   BOC   acted   without   or   in   excess   of  
In   1960,   Santiago   Gatchalian   was   recognized   by   the   Bureau   of   Immigration   as   a   jurisdiction  in  the  institution  of  the  deportation  proceedings  against  William.  A  
native   born   Filipino   citizen   following   the   citizenship   of   his   natural   mother.   He   TRO   was   issued   restraining   the   BOC   from   continuing   with   the   deportation  
had   5   children   with   his   wife,   Chu   Gim   Tee:   Jose,   Gloria,   Francisco,   Elena   and   proceedings  against  William.    
Benjamin.      
  In  the  present  petition  for  certiorari  and  prohibition,  BOC  seeks  to  set  aside  the  
On   June   27,   1961,   2   of   Santiago’s   children   (Gloria   and   Francisco),   together   with   abovementioned  TRO  and  resolutions  of  the  Manila  and  Valenzuela  RTC.    
Francisco’s   sons   (William   and   Johnson),   arrived   in   Manila   from   Hong   Kong    
seeking   admission   as   Filipino   citizens.   They   had   with   them   Certificates   of   ISSUES:  
Registration   and   Identity   issued   by   the   Philippine   Consulate   in   HK.   After   Whether  William  (respondent)  is  a  Filipino  citizen  and  therefore  should  NOT  be  
investigation,   the   Board   of   Special   Inquiry   No.   1   (BOSI)   rendered   a   decision   deported.  
dated  July  6,  1961  (Decision  1)  admitting  William  and  his  companions  as  Filipino    
citizens.  They  were  then  issued  an  Identification  Certificate  by  the  immigration   HELD:  
authorities.     Petitioners,  on  the  other  hand,  claim  that  respondent  is  an  alien.  In  support  of  
  their   position,   petitioners   point   out   that   Santiago   Gatchalian's   marriage   with  
The   following   year,   the   DOJ   Secretary   directed   the   Board   of   Commissioners   Chu   Gim   Tee   in   China   as   well   as   the   marriage   of   Francisco   (father   of   William)  
(BOC)   to   review   all   cases   where   entry   was   allowed   on   the   ground   that   the   Gatchalian   to   Ong   Chiu   Kiok,   likewise   in   China,   were   not   supported   by   any  
entrant  was  a  Philippine  citizen,  including  the  case  of  William  and  his  relatives.   evidence   other   than   their   own   self-­‐serving   testimony   nor   was   there   any  
On  July  6,  1962,  the  BOC  reversed  the  decision  of  the  BOSI  (Reversal  =  Decision   showing   what   the   laws   of   China   were.   It   is   the   postulate   advanced   by  
petitioners  that  for  the  said  marriages  to  be  valid  in  this  country,  it  should  have  
 
CONFLICT  OF  LAWS                                                                                    AV  DE  TORRES   124  
ATTY.  ARIS  L.  GULAPA                                            AY  2015-­‐2016  
been  shown  that  they  were  valid  by  the  laws  of  China  wherein  the  same  were   This   forecloses   any   further   question   about   the   Philippine   citizenship   of  
contracted.   There   being   none,   petitioners   conclude   that   the   aforesaid   respondent  William  Gatchalian.  
marriages   cannot   be   considered   valid.   Hence,   Santiago's   children,   including   The   Court   is   not   unaware   of  Woong   Woo   Yiu   vs.  Vivo  (13   SCRA   552   [1965])  
Francisco,  followed  the  citizenship  of  their  mother,  having  been  born  outside   relied  upon  by  petitioners.  The  ruling  arrived  thereat,  however,  cannot  apply  in  
of   a   valid   marriage.   Similarly,   the   validity   of   the   Francisco's   marriage   not   the  case  at  bar  for  the  simple  reason  that  the  parties  therein  testified  to  have  
having   been   demonstrated,   William   and   Johnson   followed   the   citizenship   of   been   married   in   China   by   a   village   leader,   which   undoubtedly   is   not   among  
their  mother,  a  Chinese  national.   those   authorized   to   solemnize   marriage   as   provided   in   Art.   56   of   the   Civil   Code  
  (now  Art.  7,  Family  Code).  
Philippine   law,   following   the  lex   loci   celebrationis,   adheres   to   the   rule   that   a    
marriage   formally   valid   where   celebrated   is   valid   everywhere.   Referring   to   84.  EUGENIO  V.  VELEZ  
marriages   contracted   abroad,   Art.   71   of   the   Civil   Code   (now   Art.   26   of   the   1990  
Family   Code)   provides   that   "(a)ll   marriages   performed   outside   of   the    
Philippines  in  accordance  with  the  laws  in  force  in  the  country  where  they  were   DOCTRINE:   The   provisions   of   the   Civil   Code,   unless   expressly   providing   to   the  
performed,  and  valid  there  as  such,  shall  also  be  valid  in  this  country  .  .  ."  And   contrary  as  in  Article  144,  when  referring  to  a  "spouse"  contemplate  a  lawfully  
any  doubt  as  to  the  validity  of  the  matrimonial  unity  and  the  extent  as  to  how   wedded  spouse.    
far  the  validity  of  such  marriage  may  be  extended  to  the  consequences  of  the    
coverture  is  answered  by  Art.  220  of  the  Civil  Code  in  this  manner:  "In  case  of   ALG:  GR:  Common  law  marriages  are  unrecognized  in  the  Philippines.    
doubt,   all   presumptions   favor   the   solidarity   of   the   family.   Thus,  every   EXC:  Unless  the  law  expressly  provides  that  it  involves  common  law  marriages:  
intendment   of   law   or   facts   leans   toward   the   validity   of   marriage,   the   1.  Article  144  of  the  Family  Code  -­‐  When  a  man  and  a  woman  live  together  as  
indissolubility   of   the   marriage   bonds,  the   legitimacy   of   children,  the   husband  and  wife,  but  they  are  not  married,  or  their  marriage  is  void  from  the  
community   of   property   during   marriage,   the   authority   of   parents   over   their   beginning,  the  property  acquired  by  either  or  both  of  them  through  their  work  
children,  and  the  validity  of  defense  for  any  member  of  the  family  in  case  of   or   industry   or   their   wages   and   salaries   shall   be   governed   by   the   rules   on   co-­‐
unlawful   aggression."   (Emphasis   supplied).   Bearing   in   mind   the   "processual   ownership.    
presumption"  enunciated  in  Miciano  and  other  cases,  he  who  asserts  that  the    
marriage   is   not   valid   under   our   law   bears   the   burden   of   proof   to   present   the   2.  Art.  332  of  the  Revised  Penal  Code  -­‐  Persons  exempt  from  criminal  liability.  —  
foreign  law.   No   criminal,   but   only   civil   liability,   shall   result   from   the   commission   of   the   crime  
  of   theft,   swindling   or   malicious   mischief   committed   or   caused   mutually   by   the  
Having   declared   the   assailed   marriages   as   valid,   respondent   William   Gatchalian   following  persons:  
follows   the   citizenship   of   his   father   Francisco,   a   Filipino,   as   a   legitimate   child   of    
the   latter.   Francisco,   in   turn   is   likewise   a   Filipino   being   the   legitimate   child   of   1.   Spouses,   ascendants   and   descendants,   or   relatives   by   affinity   in   the   same  
Santiago   Gatchalian   who   (the   latter)   is   admittedly   a   Filipino   citizen   whose   line.  x  x  x    
Philippine  citizenship  was  recognized  by  the  Bureau  of  Immigration  in  an  order    
dated  July  12,  1960.   3.  Article  34  of  the  Family  Code  -­‐  Art.  34.  No  license  shall  be  necessary  for  the  
  marriage   of   a   man   and   a   woman   who   have   lived   together   as   husband   and   wife  
Finally,   respondent   William   Gatchalian   belongs   to   the   class   of   Filipino   citizens   for   at   least   five   years   and   without   any   legal   impediment   to   marry   each   other.  
contemplated  under  Sec.  1,  Article  IV  of  the  Constitution,  which  provides:   The   contracting   parties   shall   state   the   foregoing   facts   in   an   affidavit   before   any  
Sec.  1.  The  following  are  citizens  of  the  Philippines:   person   authorized   by   law   to   administer   oaths.   The   solemnizing   officer   shall   also  
(1)   Those   who   are   citizens   of   the   Philippines   at   the   time   of   the   adoption   of   this   state   under   oath   that   he   ascertained   the   qualifications   of   the   contracting  
Constitution.  .  .  .   parties  are  found  no  legal  impediment  to  the  marriage.  (76a)    

 
CONFLICT  OF  LAWS                                                                                    AV  DE  TORRES   125  
ATTY.  ARIS  L.  GULAPA                                            AY  2015-­‐2016  
  support  under  Art.  294  was  used  as  the  basis  of  the  award.  Since  there  was  no  
FACTS:   surviving   spouse,   ascendants   or   descendants,   the   brothers   and   sisters   were  
Unaware   of   Vitaliana's   death   on   August   28,   1988,   her   full   blood   brothers   and   preferred   over   petitioner   who   was   merely   a   common   law   spouse,   the   latter  
sisters   (Vargases)   filed   on   September   27,   1988,   a   petition   for   habeas   corpus   being  himself  legally  married  to  another  woman.    
before  the  RTC  of  Misamis  Oriental.  They  alleged  that  Vitaliana  was  forcefully    
taken  by  Eugenio  and  was  detained  in  his  palacial  residence  in  Jasaan,  Misamis   ISSUES:  
Oriental.  At  the  time  the  petition  was  filed,  it  was  alleged  that  Vitaliana  was  25,   1)  Whether  the  court  has  jurisdiction  over  the  subject  matter  and  nature  of  the  
single,   and   living   with   Eugenio.   The   next   day,   the   court   issued   the   writ   of   proceedings  (YES)  
habeas  corpus  but  the  writ  was  returned  unsatisfied.  Counsel  of  Eugenio  stated   2)  Whether  Eugenio  can  be  considered  as  the  spouse  of  Vitaliana.  (NO)    
that   Vitaliana   already   died   on   Aug.   28   because   of   heart   failure   due   to   her    
pregnancy   and   that   Eugenio   refused   to   surrender   the   corpse   on   the   ground   HELD:  
that  a  corpse  cannot  be  the  subject  of  habeas  corpus  and  that  he  had  already   Full  blood  brothers  and  sisters  of  Vitaliana  (deceased)  alleged  that  Eugenio  was  
obtained  a  burial  permit.  Furthermore,  since  he  is  the  common  law  spouse,  he   in  no  way  related  to  Vitaliana  and  that  he  was  wrongfully  interfering  with  the  
has   the   right   to   the   custody   of   the   body   and   to   perform   the   burial   rights.   Vargases'  duty  to  bury  her.  They  invoked  the  Civil  Code  provisions  on  asserting  
Eugenio   filed   an   urgent   motion   to   dismiss   based   on   lack   of   jurisdiction   over   the   that   they   are   to   be   considered   as   the   next   of   kin   in   the   Philippines   and  
nature  of  the  action.     therefore   they   are   the   legal   custodians   of   the   corpse.   Satisfied   with   its  
  jurisdiction,   the   respondent   court   then   proceeded   to   the   matter   of   rightful  
A  special  proceeding  for  habeas  corpus,  Eugenio  argued,  is  not  applicable  to  a   custody   over   the   dead   body,   (for   purposes   of   burial   thereof).   The   order   of  
dead  person  but  extends  only  to  all  cases  of  illegal  confinement  or  detention  of   preference  to  give  support  under  Art.  294  was  used  as  the  basis  of  the  award.  
a  live  person.  On  the  side  of  the  Vargases,  they  were  granted  leave  of  court  to   Since  there  was  no  surviving  spouse,  ascendants  or  descendants,  the  brothers  
amend   their   petition.   They   alleged   that   Eugenio   was   in   no   way   related   to   and   sisters   were   preferred   over   petitioner   who   was   merely   a   common   law  
Vitaliana  and  that  he  was  wrongfully  interfering  with  the  Vargases'  duty  to  bury   spouse,  the  latter  being  himself  legally  married  to  another  woman.  
her.   They   invoked   the   Civil   Code   provisions   on   asserting   that   they   are   to   be    
considered  as  the  next  of  kin  in  the  Philippines  and  therefore  they  are  the  legal   The   court   held   that   the   custody   of   the   dead   body   of   Vitaliana   was   correctly  
custodians   of   the   corpse.   The   motion   to   dismiss   was   denied.   Thereafter,   the   awarded  to  the  surviving  brothers  and  sisters  pursuant  to  Section  1103  of  the  
court   a   quo   proceeded   as   in   or   civil   cases   and,   in   due   course,   rendered   a   Revised  Administrative  Code  which  provides:  
decision,   resolving   the   main   issue   of   whether   or   not   said   court   acquired   “Persons   charged   with   duty   of   burial-­‐   if   the   deceased   was   an  
jurisdiction   over   the   case   by   treating   it   as   an   action   for   custody   of   a   dead   body,   unmarried  man  or  woman  or  a  child  and  left  any  kin;  the  duty  of  the  
without  the  petitioners  having  to  file  a  separate  civil  action  for  such  relief,  and   burial  shall  devolve  upon  the  nearest  kin  of  the  deceased.  
without  the  Court  first  dismissing  the  original  petition  for  habeas  corpus.      
  Petitioner   claims   he   is   the  spouse  contemplated   under   Art.   294   of   the   Civil  
The  amendments  to  the  petition  were  but  elaborations  but  the  ultimate  facts   Code,   the   term   spouse   used   therein   not   being   preceded   by   any   qualification;  
remained   the   same,   hence,   this   court   strongly   finds   that   this   court   has  ample   hence,   in   the   absence   of   such   qualification,   he   is   the   rightful   custodian   of  
jurisdiction  to  entertain  and  sit  on  this  case  as  an  action  for  custody  and  burial   Vitaliana's   body.   Vitaliana's   brothers   and   sisters   contend   otherwise.   Indeed,  
of  the  dead  body  because  the  body  of  the  petition  controls  and  is  binding  and   Philippine  Law  does  not  recognize  common  law  marriages.  A  man  and  woman  
since  this  case  was  raffled  to  this  court  to  the  exclusion  of  all  other  courts,  it  is   not   legally   married   who   cohabit   for   many   years   as   husband   and   wife,   who  
the  primary  duty  of  this  court  to  decide  and  dispose  of  this  case.     represent  themselves  to  the  public  as  husband  and  wife,  and  who  are  reputed  
  to  be  husband  and  wife  in  the  community  where  they  live  may  be  considered  
Satisfied   with   its   jurisdiction,   the   respondent   court   then   proceeded   to   the   legally  mauled  in  common  law  jurisdictions  but  not  in  the  Philippines.  
matter  of  rightful  custody  over  the  dead  body.  The  order  of  preference  to  give    
 
CONFLICT  OF  LAWS                                                                                    AV  DE  TORRES   126  
ATTY.  ARIS  L.  GULAPA                                            AY  2015-­‐2016  
While   it   is   true   that   our   laws   do   not   just   brush   aside   the   fact   that   such   Termination  of  marriage  relations  –  void  
relationships   are   present   in   our   society,   and   that   they   produce   a   community   of    
properties  and  interests  which  is  governed  by  law,  authority  exists  in  case  law   After   marriage   à   void   in   both   circumstances.   So   characterization   would   have  
to  the  effect  that  such  form  of  co-­‐ownership  requires  that  the  man  and  woman   not  happened  at  all.    
living  together  must  not  in  any  way  be  incapacitated  to  contract  marriage.     In    
any   case,   herein   petitioner   has   a   subsisting   marriage   with   another   woman,   a   FACTS:  
legal  impediment  which  disqualified  him  from  even  legally  marrying  Vitaliana.   Oct.  18,  1969  –  Private  respondent  Jose  Vicente  De  Leon  and  petitioner  Sylvia  
In  Santero  vs.  CFI  of  Cavite,  the  Court,  thru  Mr.  Justice  Paras,  interpreting  Art.   Lichauco   De   Leon   were   united   in   wedlock   before   the   Municipal   Mayor   of  
188   of   the   Civil   Code   (Support   of   Surviving   Spouse   and   Children   During   Binangonan  Rizal    
Liquidation   of   Inventoried   Property)   stated:   "Be   it   noted   however   that   with   Aug.  28,  1971  –  Susana  L.  De  Leon  were  born  from  this  union  
respect  to  'spouse',  the  same  must  be  the  legitimate  'spouse'  (not  common-­‐law   Oct.   1972   –   De   facto   separation   between   the   spouses   occurred   due   to  
spouses)."   irreconcilable  marital  differences;  Sylvia  left  the  conjugal  home  
  March  1973  –  Sylvia  went  to  the  US  and  obtained  American  citizenship  
There   is   a   view   that   under   Article   332   of   the   Revised   Penal   Code,   the   term   Nov.  29,  1973  –  Sylvia  filed  with  the  Superior  Court  of  California,  a  petition  for  
"spouse"   embraces   common   law   relation   for   purposes   of   exemption   from   dissolution   of   marriage   against   Jose   Vicente;   also   filed   claims   for   support   and  
criminal  liability  in  cases  of  theft,  swindling  and  malicious  mischief  committed   distribution  of  properties/  but  since  Jose  Vicente  was  a  Philippine  resident  and  
or   caused   mutually   by   spouses.   The   Penal   Code   article,   it   is   said,   makes   no   did   not   have   any   assets   in   the   US,   Sylvia   held   the   divorce   proceedings   in  
distinction  between  a  couple  whose  cohabitation  is  sanctioned  by  a  sacrament   abeyance   and   concentrated   her   efforts   to   obtain   property   settlements   with  
or   legal   tie   and   another   who   are   husband   and   wife  de   facto.   But   this   view   Jose  Vicente    
cannot   even   apply   to   the   facts   of   the   case   at   bar.   We   hold   that   the   provisions   March   16,   1977   –   Sylvia   entered   into   a   Letter-­‐Agreement   with   private  
of  the  Civil  Code,  unless  expressly  providing  to  the  contrary  as  in  Article  144,   respondent  Macaria  De  Leon  (her  mother-­‐in-­‐law)    
when   referring   to   a   "spouse"   contemplate   a   lawfully   wedded   spouse.   March   30,   1977   -­‐   Sylvia   and   Jose   Vicente   filed   before   the   CFI   of   Rizal   a   joint  
Petitioner  vis-­‐a-­‐vis  Vitaliana  was  not  a  lawfully-­‐wedded  spouse  to  her;  in  fact,   petition  for  judicial  approval  of  dissolution  of  their  conjugal  partnership    
he  was  not  legally  capacitated  to  marry  her  in  her  lifetime.   Trial  court:  declared  that  the  conjugal  partnership  of  the  spouses  is  DISSOLVED  
  March  17,  1980  –  Sylvia  moved  for  the  execution  of  the  Order  of  the  trial  court;    
81.  DE  LEON  V.  CA   Vicente  moved  for  MR  
186  SCRA  347  (1990)   April   20,   1980   –   Macaria   filed   with   the   trial   court   a   motion   for   leave   to  
  intervene,   alleging   that   she   is   the   owner   of   the   properties   involved   in   the   case;  
DOCTRINE:   But   marriage   is   NOT   a   mere   contract   but   a   sacred   social   assailed   the   validity   and   legality   of   the   Letter-­‐Agreement   which   had   for   its  
institution.  Thus,  Art.  52  of  the  Civil  Code  provides:  Art.  52.  Marriage  is  not  a   purpose,  the  termination  of  the  marital  relations  of  the  spouses    
mere   contract   but   an   inviolable   social   institution.   Its   nature,   consequences   CA:  affirmed  the  decision  of  the  RTC  
and  incidents  are  governed  by  law  and  not  subject  to  stipulations...    
  ISSUE:  
Kyna’s  notes:   Whether  or  not  the  Letter-­‐Agreement  is  valid  
ALG:   What   is   the   issue   in   COL?   IT   IS   A   CHARACTERIZATION   EXAMPLE   IN    
RELATION  TO  MARRIAGE.   HELD:  
  No.  The  only  basis  by  which  Sylvia  may  lay  claim  to  the  properties  which  are  the  
Agreement  executed  before  marriage     subject   matter   of   the   Letter-­‐Agreement,   is   the   Letter-­‐Agreement   itself.   Sylvia  
Termination  of  property  relation  –  would  have  been  valid   insists   that   the   consideration   for   her   execution   of   the   Letter-­‐Agreement   was  

 
CONFLICT  OF  LAWS                                                                                    AV  DE  TORRES   127  
ATTY.  ARIS  L.  GULAPA                                            AY  2015-­‐2016  
the  termination  of  property  relations  with  her  husband.  Indeed,  Sylvia  and  Jose   citizen  and  a  foreigner  is  validly  celebrated  and  a  divorce  is  thereafter  
Vicente   subsequently   filed   a   joint   petition   for   judicial   approval   of   the   validly  obtained  abroad  by  the  alien  spouse  capacitating  him  or  her  to  
dissolution  of  their  conjugal  partnership,  sanctioned  by  Article  191  of  the  Civil   remarry,   the   Filipino   spouse   shall   have   capacity   to   remarry   under  
Code.   On   the   other   hand,   Macaria   and   Jose   Vicente   assert   that   the   Philippine  law.  
consideration  was  the  termination  of  marital  relationship.  
  2.  CASE  OF  VAN  DORN  (FOREIGN  SPOUSE  IS  ESTOPPED  FROM  DENYING  
RTC  said  that  the  cause  or  consideration  for  the  intervenor  Macaria  De  Leon  in   THE   DIVORCE   BECAUSE   HE/SHE   HIMSELF/HERSELF   OBTAINED   THE  
having   executed   the   Letter-­‐Agreement   was   the   termination   of   the   marital   DIVORCE  DECREE  IN  ANOTHER  STATE)  
relationship   between   her   son   Jose   Vicente   De   Leon   and   Sylvia   Lichauco   de   3.   NECESSARY   RECOGNITION   (IF   BOTH   SPOUSES   ARE   ALIENS   –   THEY  
Leon.   ARE  GOVERNED  BY  THEIR  NATIONAL  LAW  ACCDG  TO  ART.  15)  
   
But  marriage  is  NOT  a  mere  contract  but  a  sacred  social  institution.   Thus,  Art.   WHAT  GOVERNS  ANNULMENT?  
52  of  the  Civil  Code  provides:  Art.  52.  Marriage  is  not  a  mere  contract  but  an   LAW   GOVERNING   THE   FORMAL   AND   SUBSTANTIVE   VALIDITY   OF  
inviolable   social   institution.   Its   nature,   consequences   and   incidents   are   MARRIAGE  
governed  by  law  and  not  subject  to  stipulations...    
  75.  HIX  V.  FLUEMER  
From  the  foregoing  provisions  of  the  Civil  Code,  the  court  believes  that  Macaria   G.R.  No.  L-­‐34259  |  March  21,  1931  
De   Leon’s   undertaking   that   the   termination   of   marital   relationship   is   not   only    
contrary   to   law   but   also   to   Filipino   morals   and   public   policy.   As   such,   any   DOCTRINES:   It   is   not   the   citizenship   of   the   plaintiff   for   divorce,   which   confers  
agreement   or   obligations   based   on   such   unlawful   consideration   and   which   is   jurisdiction   upon   a   court,   but   his   legal   residence   within   the   State   where   he  
contrary  to  public  policy  should  be  deemed  null  and  void.   applies  for  a  divorce.  
   
Even   granting   that   the   consideration   of   the   Letter-­‐Agreement   was   the   Hix   himself   believed   he   had   relinquished   his   former   legal   residence   in   West  
termination  of  property  relations,  the  Court  said  that:   Virginia,   of   which   he   was   a   citizen,   upon   establishing   his   marriage   domicile   in  
  the   City   of   Manila,   Philippine   Islands,   is   shown   by   the   fact   that   he   had   to  
Art.  221.  The  following  shall  be  void  and  of  no  effect:   reestablish  his  residence  in  said  State  for  the  length  of  time  fixed  by  the  law  
(1)  Any  contract  for  personal  separation  between  husband  and  wife;     in  order  to  be  able  to  file  his  complaint  for  a  divorce.  
(2)   Every   extra-­‐judicial   agreement,   during   marriage,   for   the   dissolution   of   the    
conjugal   partnership   of   gains   or   of   the   absolute   community   of   property   (1)   That   the   residence   acquired   in   a   state   of   the   American   Union   by   a   husband,  
between  husband  and  wife   who,  for  the  purpose  of  obtaining  a  divorce,  abandons  the  country  wherein  are  
  his  matrimonial  domicile  and  his  wife,  who  is  living  apart  from  him  by  mutual  
DIVORCE   consent,   and   then   returns   to   said   matrimonial   domicile   after   obtaining   a  
  divorce,  continues  residing,  therein  and  engaging  in  business,  is  not  bona  fide  
  residence,  and  does  not  confer  jurisdiction  upon  the  court  even  if  he  alleges  in  
DIVORCE  IS  THE  LEGAL  DISSOLUTION  OF  THE  MARRIAGE  BOND  RENDERED  BY  A   the  complaint  for  divorce  that  he  intends  to  reside  permanently  in  said  state;  (2)  
COMPETENT   COURT   FOR   CAUSES   DEFINED   BY   LAW   WHICH   AROSE   AFTER   that  the  summons  by  publication  in  a  complaint  for  divorce,  filed  in  a  state  by  
MARRIAGE.  IT  PRESUPPOSES  THAT  MARRIAGE  IS  VALID.   the  husband  who  has  gone  to  said  state,  abandoning  his  matrimonial  domicile  
  where  his  wife  continues  to  reside,  does  not  confer  jurisdiction  upon  the  court  
METHODS  BY  WHICH  PHIL.  LAW  RECOGNIZES  DIVORCE:   over   the   person   of   said   wife   when   she   has   not   entered   an   appearance   in   the  
1.   FAMILY   CODE,   ARTICLE   26(2)   -­‐   Where   a   marriage   between   a   Filipino  
 
CONFLICT  OF  LAWS                                                                                    AV  DE  TORRES   128  
ATTY.  ARIS  L.  GULAPA                                            AY  2015-­‐2016  
case,  and  the  decree  issued  by  said  court  dissolving  the  marriage  is  not  binding   On   March   16,   1919,   the   appellant   left   for   Canada,   where   she   remained   with  
upon   her;   and   (3)   that   a   decree   of   divorce   issued   by   a   court   of   any   state   or   their   child   until   February,   1921,   when   she   returned   to   Manila   in   a   very  
territory  of  the  American  Union,  or  of  a  foreign  country,  may  be  impeached  in   precarious   condition   of   health   and   was   given   medical   treatment   in   the   St.  
another   case   for   lack   of   jurisdiction   in   said   court   over   the   subject   matter,   or   Paul's   Hospital   at   the   expense   of   her   husband.   After   she   regained   her   health,  
over  the  person  of  the  defendant,  or  for  fraud  in  obtaining  it  on  the  part  of  the   she  lived  apart  from  her  husband  by  mutual  consent.    
person  procuring  it.      
  On   December   7,   1922,   the   appellant   instituted   an   action   in   the   CFI   of   Manila  
Av:  Phil.  Law  does  not  prohibit  aliens  from  obtaining  divorce  decrees.  (Art.  26)   against   her   husband,   E.   Randolph   Hix,   for   the   purpose   of   compelling   him   to  
As   long   as   the   divorce   is   validly   obtained   abroad,   it   shall   be   recognized   in   the   provide  adequate  support  for  herself  and  her  son,  Preston  Randolf  Hix.  The  trial  
Phil.    HOWEVER,  how  is  divorce  validly  obtained  abroad  by  an  alien  spouse?   court   adjudicated   the   case   in   her   favor   and   ordered   the   defendant   E.   Randolph  
  Hix   to   pay   her   the   sum   of   P500   in   advance   on   or   before   the   5th   day   of   each  
1.   Divorce   must   have   been   valid   under   the   national   law   of   the   foreign   spouse   month  for  the  maintenance  of  herself  and  her  son.    
(Plead  and  prove  national  law)    
2.   The   court   granting   the   divorce   decree   must   have   had   jurisdiction   to   render   In  the  month  of  May,  1925,  (12  years  after  they  established  their  domicile  in  
such.  The  court  acquires  jurisdiction  if  the  state  of  the  court  is  the  matrimonial   Manila)  that  is,  one  year  after  his  arrival  at  Elkins,  West  Virginia,  the  deceased  
domicile  of  the  spouses.  (Hix  v.  Fluemer)   filed   a   complaint   for   a   divorce   with   the   Circuit   Court   of   Randolph   County,  
  West   Virginia,  alleging,  among  other  things,  that  he  was  a  citizen  of  the  US,  and  
FACTS:   of  the  State  of  West  Virginia,  and  had  been  for  more  than  one  year  prior  to  the  
E.  Randolph  Hix  was  born  in  the  year  1866  in  Union,  South  Carolina,  where  he   date   of   the   institution   of   the   suit,   an   actual   bona   fide   citizen   and   resident   of  
lived  with  his  parents  until  the  age  of  15.  They  then  moved  to  Rye,  Westchester   Randolph   County,   West   Virginia;   that   Annie   Cousins   Hix   was   a   resident   of   the  
County,   New   York.   A   few   years   later,   he   was   sent   to   the   University   of   Lehigh,   City  of  Peking,  China;  that  on  December  1,  1921,  his  wife  had  abandoned  and  
and   to   the   Massachusetts   Institute   of   Technology,   leaving   the   latter   before   deserted   him,   taking   up   a   separate   residence   and   declining   to   live   or   have  
graduating,  to  accept  employment  with  the  Edison  Company  where  he  worked   anything   to   do   with   him;   that   he,   Hix,   freely,   voluntarily,   and   adequately  
for  about  three  years.  After  resigning  from  his  position  he  opened  an  office  and   supported   his   wife   and   child,   paying   her   the   sum   of   $175   per   month;   that   he  
engaged   in   private   work   as   consulting   engineer   and   contractor   until   the   year   intended  to  reside  permanently  in  the  US,  and  that  it  was  with  such  intention  
1895,   when   he   moved   to   Wheeling,   West   Virginia,   to   engage   in   the   general   that   he   had   returned   to   West   Virginia;   that   he   and   his   wife   had   been   living  
engineering  business  as  a  member  of  the  firm  of  Hogg  &  Hix,  surveyors.     apart   for   three   years,   and   that   she   had   rejected   his   offer   of   reconciliation.   As  
  the   appellant   was   not   a   resident   of   the   State   of   West   Virginia,   she   was  
After   fifteen   years   of   residence   in   Wheeling,   he   took   an   examination   and   summoned   upon   the   complaint   for   divorce   by   publication,   and   not   having  
received  an  appointment  as  coal  expert  for  the  Philippine  Government,  arriving   entered  an  appearance  in  the  case,  either  personally  or  by  counsel  within  the  
at  Manila  some  time  during  the  year  1910.  While  E.  Randolph  Hix  was  living  in   term   fixed,   the   Circuit   Court   of   Randolph   County,   West   Virginia,   rendered  
Manila   in   1912,   he   met   appellant   Annie   and   married   her   in   Shanghai,   China,   on   judgment   against   her   in   1925   declaring   her   marriage   with   the   plaintiff  
or   about   June   24,   1913,   returning   to   Manila   where   they   established   their   dissolved.  Having  procured  the  divorce,  E.  Randolph  Hix  returned  to  Manila  in  
domicile.   A   son   was   born   of   this   union   in   Boston,   Massachusetts,   on   July   1,   1927,   where   he   continued   to   live   and   engaged   in   business   up   to   the   time   of   his  
1915,   named   Preston   Randolph   Hix,   while   she   was   in   the   US   where   she   had   death  in  the  year  1929.    
gone   on   the   month   of   May   of   the   same   year   to   visit   her   family   and   the   mother    
and   sister   of   her   husband.   The   appellant   returned   to   Manila   in   November,   ISSUE:  
1916,  and  continued  to  live  with  the  deceased  as  husband  and  wife.     Whether   the   Circuit   Court   of   Randolph   County   in   West   Virginia   acquired  
  jurisdiction  to  take  cognizance  of  the  complaint  for  divorce  filed  by  E  .Randolph  

 
CONFLICT  OF  LAWS                                                                                    AV  DE  TORRES   129  
ATTY.  ARIS  L.  GULAPA                                            AY  2015-­‐2016  
Hix   and   to   render   a   valid   and   binding   judgment   against   the   petitioner   and   applies   for   a   divorce.   That   E.   Randolph   Hix   himself   believed   he   had  
appellant,  Annie  Cousins  Hix.   relinquished   his   former   legal   residence   in   West   Virginia,   of   which   he   was   a  
  citizen,   upon   establishing   his   marriage   domicile   in   the   City   of   Manila,  
HELD:   Philippine   Islands,   is   shown   by   the   fact   that   he   had   to   reestablish   his  
In  order  that  a  judgment  of  a  court  or  judge  of  any  state  of  the  American  Union   residence  in  said  State  for  the  length  of  time  fixed  by  the  law  in  order  to  be  
with   respect   to   the   personal   or   legal   condition   of   a   particular   person   may   be   able  to  file  his  complaint  for  a  divorce.  
conclusive   and   constitute  res   judicata,   it   is   essential   that   the   court   have    
jurisdiction,  and  such  jurisdiction  is  presumed  in  the  absence  of  evidence  to  the   Since  E.  Randolph  Hix  was  not  a  bona  fide  resident  of  the  State  of  West  Virginia,  
contrary.   the   divorce   decree   he   obtained   from   the   Circuit   Court   of   Randolph   County,   is  
  null   and   void,   said   court   having   failed   to   acquire   jurisdiction   over   the   subject  
Section  312  of  the  Code  of  Civil  Procedure  provides:   matter.  
SEC.  312.  How  Judicial  Record  May  be  Impeached.  —  Any  judicial  record  may  be    
impeached  by  evidence  of  a  want  of  jurisdiction  in  the  court  or  judicial  officer,   That  the  residence  acquired  in  a  state  of  the  American  Union  by  a  husband,  
of   collusion   between   the   parties,   or   of   fraud   in   the   party   offering   the   record,   in   who,   for   the   purpose   of   obtaining   a   divorce,   abandons   the   country   wherein  
respect  to  the  proceedings.   are   his   matrimonial   domicile   and   his   wife,   who   is   living   apart   from   him   by  
  mutual   consent,   and   then   returns   to   said   matrimonial   domicile   after  
One  of  the  conditions  for  the  validity  of  a  decree  of  absolute  divorce  is  that   obtaining   a   divorce,   continues   residing,   therein   and   engaging   in   business,   is  
the  court  granting  it  has  acquired  jurisdiction  over  the  subject  matter,  and  to   not  bona   fide   residence,   and   does   not   confer   jurisdiction   upon   the   court   even  
this   end   the   plaintiff   must   be   domiciled   in   good   faith,   and   for   the   length   of   if   he   alleges   in   the   complaint   for   divorce   that   he   intends   to   reside  
time   fixed   by   the   law,   in   the   state   in   which   it   was   granted.   E.   Randolph   Hix   permanently  in  said  state.  
was   domiciled   in   the   City   of   Manila   where   he   lived   apart   from   his   wife   and    
child,   by   mutual   consent,   and   here   he   had   his   business.   He   removed   to   the   77.  ARCA  V.  JAVIER  
State   of   West   Virginia   leaving   his   aforesaid   wife   and   child   and   his   business   G.R.  No.  L-­‐6768  |  July  31,  1954  
behind,   for   the   purpose   of   obtaining   an   absolute   divorce,   which   he   did   in    
1925,   returning   in   the   year   1927   to   reside   in   the   City   of   Manila,   and   DOCTRINES:   One   of   the   essential   conditions   for   the   validity   of   a   decree   of  
continuing  his  business.   divorce  is  that  the  court  must  have  jurisdiction  over  the  subject  matter  and  in  
  order  that  this  may  be  acquired,  plaintiff  must  be  domiciled  in  good  faith  in  the  
Although  the  opponent  and  appellee  attempted  to  show  that  E.  Randolph  Hix   State  in  which  it  is  granted.  So  it  has  been  held  that  "it  is  not  ...  the  citizenship  
went   to   West   Virginia   with   the   intention   of   residing   there   permanently,   as   of  the  plaintiff  for  divorce  which  confers  jurisdiction  upon  a  court,  but  his  legal  
allege  in  the  complaint  for  divorce,  such  an  intention  was  contradicted  by  the   residence  within  the  State."    
fact   that   before   leaving   the   City   of   Manila,   he   did   not   liquidate   his   business   but    
placed  it  under  the  management  of  said  opponent,  and  once  having  obtained   [DVBD]  
his   divorce,   he   returned   to   the   City   of   Manila   to   take   up   his   residence   and   to   1.  Neither  domiciled  +  merely  to  obtain  divorce  =  no  jurisdiction  to  determine  
continue  his  aforesaid  business,  and  that  his  purpose  in  going  to  West  Virginia   martial  status  
was  to  obtain  a  divorce.   2.  Voluntary  appearance  =/=  jurisdiction  
  3.  Bona  fide  residence  of  the  plaintiff  =  jurisdiction  
This   ruling   has   not   been   weakened   in   the   present   case  by   the   fact   that   E.   4.  Matrimonial  domicile  governs  over  separate  domicile  in  good  faith  
Randolph   Hix   was   a   citizen   of   the   United   States   and   of   the   State   of   West    
Virginia,  since  it  is  not  the  citizenship  of  the  plaintiff  for  divorce  which  confers   The   courts   in   the   Philippines   can   grant   a   divorce   only   on   the   ground   of  
jurisdiction   upon   a   court,   but   his   legal   residence   within   the   State   where   he  
 
CONFLICT  OF  LAWS                                                                                    AV  DE  TORRES   130  
ATTY.  ARIS  L.  GULAPA                                            AY  2015-­‐2016  
"adultery  on  the  part  of  the  wife  or  concubinage  on  the  part  of  the  husband"  as   Philippines,   arriving   on   February   13,   1950.   After   his   arrival   in   the   Philippines,  
provided  for  under  section  1  of  Act  No.  2710.   armed   with   two   decrees   of   divorce   issued   by   the   Circuit   Court   of   Mobile  
  County,  State  of  Alabama,  USA,  Alfredo  Javier  married  Maria  Odvina  before  the  
Gulapa:   Divorce   was   filed   in   1940   and   was   issued   in   1941.   The   governing   law   Municipal  Court  of  Manila  on  April  19,  1950.  
was  Act  2710  (Divorce  Law).  The  latter  only  allowed  divorce  on  the  grounds  of    
concubinage  and  adultery.     At   the   instance   of   plaintiff   Salud,   an   information   for   bigamy   was   filed   by   the  
Divorce  decree  here  relevant:   City  Fiscal  of  Manila  on  July  25,  1950  against  defendant  Alfredo  Javier  with  the  
Filipinos  –  can  only  get  divorced  in  adultery  and  concubinage   CFI  of  Manila.    
Foreigners  –  any  ground  as  long  as  it  is  validly  obtained    
  ISSUE:  
In  this  case,  the  ground  alleged  by  Javier  was  abandonment  by  the  wife.     Whether  the  divorce  decree  has  a  valid  effect  in  this  jurisdiction    
   
History  of  divorce  law  in  the  Philippines:   HELD:  
Divorce   law   (1917)   à   Amended   to   include   other   grounds   (1942   during   the   No.  One   of   the   essential   conditions   for   the   validity   of   a   decree   of   divorce   is  
Japanese  occupation)  à  It  was  amended  again  before  the  enactment  of  the  old   that   the   court   must   have   jurisdiction   over   the   subject   matter   and   in   order  
Civil  Code  à  U.S.  came  back  and  Philippines  went  back  to  the  Divorce  Law  à   that   this   may   be   acquired,   plaintiff   must   be   domiciled   in   good   faith   in   the  
Divorce  Law  repealed  by  the  Civil  Code  (1950).       State  in  which  it  is  granted.  So  it  has  been  held  that  "it  is  not  ...  the  citizenship  
  of   the   plaintiff   for   divorce   which   confers   jurisdiction   upon   a   court,   but   his  
  legal  residence  within  the  State."    
FACTS:    
Alfredo  Javier  was  a  native  born  citizen  of  the  Philippines  who,  in  1937,  married   It  is  true  that  Salud  filed  an  answer  in  the  divorce  case  instituted  at  the  Mobile  
Salud   R.   Arca,   another   Filipino   citizen.   Before   their   marriage   they   had   already   a   County   in   view   of   the   summons   served   upon   her   in   this   jurisdiction,   but   this  
child,   Alfredo   Javier,   Jr.,   who   thereby   became   legitimated.   In   1927   appellant   action  cannot  be  interpreted  as  placing  her  under  the  jurisdiction  of  the  court  
enlisted   in   the   U.S.   Navy   and   in   1938   (a   year   after   being   married)   sailed   for   the   because   it   was   a   special   appearance,   the   only   purpose   of   which   was   to   impugn  
United  States  aboard  a  navy  ship  in  connection  with  his  service  leaving  behind   the   claim   of   appellant   that   his   domicile   or   legal   residence   at   that   time   was  
his   wife   and   child,   and   on   August   13,   1940   (2   years   after   leaving),   he   filed   an   Mobile  County,  and  to  show  that  the  ground  of  desertion  imputed  to  her  was  
action   for   divorce   in   the   Circuit   Court   of   Mobile   County,   Alabama,   U.S.A.,   baseless  and  false.    
alleging   as   ground   abandonment   by   his   wife.   Having   received   a   copy   of   the    
complaint,   Salud   R.   Arca   filed   an   answer   alleging,   among   other   things,   that   In  several  cases  this  court  laid  down  the  following  doctrines:    
appellant  was  not  a  resident  of  Mobile  County,  but  of  Naic,  Cavite,  Philippines,   The  court   of   a   country   in   which   neither   of   the   spouses   is   domiciled   and  
and   that   it   was   not   true   that   the   cause   of   their   separation   was   abandonment   to   which   one   or   both   of   them   may   resort   merely   for   the   purpose   of  
on  her  part  but  that  appellant  was  in  the  US,  without  her,  because  he  was  then   obtaining  a  divorce  has  no  jurisdiction  to  determine  their  matrimonial  
enlisted   in   the   U.S.   Navy.   Nevertheless,   the   Circuit   Court   of   Mobile   County   status;   and   a   divorce   granted   by   such   a   court   is   not   entitled   to  
granted  a  decree  of  divorce  on  April  9,  1941.     recognition   elsewhere.   The   voluntary   appearance   of   the   defendant  
  before  such  a  tribunal  does  not  vest  the  court  with  jurisdiction.    
In   July,   1941,   Alfredo   Javier   married   Thelma   Francis,   an   American   citizen,   and    
bought  a  house  and  lot  at  248  Brooklyn,  NY  City.  In  1949,  Thelma  obtained  a   It   follows   that,   to   give   a   court   jurisdiction   on   the   ground   of   the   plaintiff's  
divorce   from   him   for   reasons   not   disclosed   by   the   evidence,   and,   later   on,   residence   in   the   State   or   country   of   the   judicial   forum,   his   residence  
having   retired   from   the   US   Navy,   defendant   Alfredo   Javier   returned   to   the   must   be   bona   fide.  If   a   spouse   leaves   the   family   domicile   and   goes   to  
another  State  for  the  sole  purpose  of  obtaining  a  divorce,  and  with  no  
 
CONFLICT  OF  LAWS                                                                                    AV  DE  TORRES   131  
ATTY.  ARIS  L.  GULAPA                                            AY  2015-­‐2016  
intention   of   remaining,   his   residence   there   is   not   sufficient   to   confer   decree  cannot  be  enforced  in  this  jurisdiction.  The  divorce  decree  in  question  
jurisdiction  on  the  courts  of  the  State.  This  is  especially  true  where  the   was  granted  on  the  ground  of  desertion,  clearly  not  a  cause  for  divorce  under  
cause   of   divorce   is   one   not   recognized   by   the   laws   of   the   State   of   his   our  laws.    
own  domicile.    
  The   above   pronouncement   is   sound   as   it   is   in   keeping   with   the   well   known  
But  even  if  his  residence  had  been  taken  up  in  good  faith,  and  the  court   principle   of   Private   International   Law   which   prohibits   the   extension   of   a  
had   acquired   jurisdiction   to   take   cognizance   of   the   divorce   suit,   the   foreign   judgment,   or   the   law   affecting   the   same,   if   it   is   contrary   to   the   law   or  
decree   issued   in   his   favor   is   not   binding   upon   the   appellant;   for   the   fundamental   policy   of   the   State   of   the   forum.   It   is   also   in   keeping   with   our  
matrimonial   domicile   of   the   spouses   being   the   City   of   Manila,   and   no   concept   of   moral   values   which   has   always   looked   upon   marriage   as   an  
new   domicile   having   been   acquired   in   West   Virginia,   the   summons   institution.   And   such   concept   has   actually   crystallized   in   a   more   tangible  
made   by   publication,   she   not   having   entered   an   appearance   in   the   manner  when  in  the  new  Civil  Code  our  people,  through  Congress,  decided  to  
case,   either   personally   or   by   counsel,   did   not   confer   jurisdiction   upon   eliminate   altogether   our   law   relative   to   divorce.   Because   of   such   concept   we  
said  court  over  her  person.   cannot  but  react  adversely  to  any  attempt  to  extend  here  the  effect  of  a  decree  
  which  is  not  in  consonance  with  our  customs,  morals,  and  traditions.    
At  all  times  the  matrimonial  domicile  of  this  couple  has  been  within  the    
Philippine  Islands  and  the  residence  acquired  in  the  State  of  Nevada  by   The   courts   in   the   Philippines   can   grant   a   divorce   only   on   the   ground   of  
the   husband   for   the   purpose   of   securing   a   divorce   was   not   a   bona   fide   "adultery  on  the  part  of  the  wife  or  concubinage  on  the  part  of  the  husband"  as  
residence  and  did  not  confer  jurisdiction  upon  the  court  of  the  State  to   provided   for   under   section   1   of   Act   No.   2710.   The   divorce   decree   in   question  
dissolve  the  bonds  of  matrimony  in  which  he  had  entered  in  1919.     was  granted  on  the  ground  of  desertion,  clearly  not  a  cause  for  divorce  under  
  our  laws.  That  our  divorce  law,  Act  No.  2710,  is  too  strict  or  too  liberal  is  not  for  
In   the   light   of   the   foregoing   authorities,   it   cannot   therefore   be   said   that   the   this   court   decide.   (Barretto   Gonzales  vs.   Gonzales,  supra).   The   allotment   of  
Mobile  County  Court  of  Alabama  had  acquired  jurisdiction  over  the  case  for  the   powers   between   the   different   governmental   agencies   restricts   the   judiciary  
simple  reason  that  at  the  time  it  was  filed  appellant's  legal  residence  was  then   within  the  confines  of  interpretation,  not  of  legislation.  The  legislative  policy  on  
in   the   Philippines.   He   could   not   have   acquired   legal   residence   or   domicile   at   the  matter  of  divorce  in  this  jurisdiction  is  clearly  set  forth  in  Act  No.  2710  and  
Mobile  County  when  he  moved  to  that  place  in  1938  because  at  that  time  he   has  been  upheld  by  this  court.  
was   still   in   the   service   of   the   U.S.   Navy   and   merely   rented   a   room   where   he    
used   to   stay   during   his   occasional   shore   leave   for   shift   duty.   That   he   never   76.  BARRETTO  V.  GONZALES  
intended   to   live   there   permanently   is   shown   by   the   fact   that   after   his   marriage   G.R.  No.  37048  |  March  7,  1933    
to   Thelma   Francis   in   1941,   he   moved   to   New   York   where   he   bought   a   house    
and   a   lot,   and   after   his   divorce   from   Thelma   in   1949   and   his   retirement   from   DOCTRINE:  Litigants   by   mutual   agreement  cannot   compel   the   courts   to   approve  
the  U.S.  Navy,  he  returned  to  the  Philippines  and  married  Maria  Odvina  of  Naic,   of   their   own   actions   or   permit   the   personal   relations   of   the   citizens   of   these  
Cavite,  where  he  lived  ever  since.  It   may   therefore   be   said   that   appellant   went   Islands   to   be   affected   by   decrees   of   foreign   courts   in   a   manner   which   our  
to   Mobile   County,   not   with   the   intention   of   permanently   residing   there,   or   of   Government  believes  is  contrary  to  public  order  and  good  morals.  
considering   that   place   as   his   permanent   abode,   but   for   the   sole   purpose   of    
obtaining   divorce   from   his   wife.   Such   residence   is   not   sufficient   to   confer   *N.B.:   Article   15.  Laws   relating   to   family   rights   and   duties,   or   to   the   status,  
jurisdiction  on  the  court.     condition   and   legal   capacity   of   persons   are   binding   upon   citizens   of   the  
  Philippines,  even  though  living  abroad.  (9a)  
Under   section   1   of   Act   No.   2710,   the   courts   in   the   Philippines   can   grant   divorce    
only  on  the  ground  of  adultery  on  the  part  of  the  wife  or  concubinage  on  the  
part   of   the   husband,   and   if   the   decree   is   predicated   on   another   ground,   that  
 
CONFLICT  OF  LAWS                                                                                    AV  DE  TORRES   132  
ATTY.  ARIS  L.  GULAPA                                            AY  2015-­‐2016  
Article  17.  Prohibitive   laws   concerning   persons,   their   acts   or   property,   and   those    
which   have   for   their   object   public   order,   public   policy   and   good   customs   shall   ISSUE:  
not   be   rendered   ineffective   by   laws   or   judgments   promulgated,   or   by   Whether  the  decree  of  divorce  could  be  enforced  in  the  Philippines  
determinations  or  conventions  agreed  upon  in  a  foreign  country    
  HELD:  
FACTS:   No.   The   SC   said   that   the   action   of   the   husband   was   clearly   to   circumvent   the  
Plaintiff   and   defendants   are   citizens   of   the   Phil.,   residents   of   Manila   and   laws  of  the  Philippine  Islands  regarding  divorce  and  to  secure  for  themselves  a  
married   in   the   Phil.   They   voluntarily   separated   and   not   lived   as   husband   and   change  of  status  for  reasons  and  under  conditions  not  authorized  by  our  law.  
wife.  They  have  4  children,  who  are  11,  10,  8  and  6  years  of  age.  They  agreed  to   At   all   times   the   matrimonial   domicile   of   this   couple   has   been   within   the  
allow  the  husband  for   the  wife’s  support  and  their  children,  500  pesos  monthly   Philippine   Islands   and   the   residence   acquired   in   the   State   of   Nevada   by   the  
and  to  be  increased  during  illness  and  necessity,  and  the  title  of  properties  be   husband  for  the  purpose  of  securing  a  divorce  was  not  a  bona  fide  residence  
put   in   her   name.   After   the   agreement,   husband   left   the   Philippines   and   resided   and   did   not   confer   jurisdiction   upon   the   Court   of   that   State   to   dissolve   the  
in  Reno,  Nevada  and  secured  an  absolute  divorce  on  the  ground  of  desertion,   bonds  of  matrimony  in  which  he  had  entered  in  1919.  While  the  decisions  of  
which   was   approved   in   November   1927.   During   that   time,   he   married   a   Filipina   this   court   heretofore   in   refusing   to   recognize   the   validity   of   foreign   divorce   has  
and  had  4  children  as  a  result  of  that  marriage.  Defendant  reduced  the  amount   usually   been   expressed   in   the   negative   and   have   been   based   upon   lack   of  
he   agreed   for   the   support   and   has   not   made   payments   fixed   in   the   divorce   matrimonial   domicile   or   fraud   or   collusion,   we   have   not   overlooked   the  
decree.     provisions  of  the  Civil  Code  now  in  force  in  these  Islands.  Article  9  thereof  reads  
  as   follows:   The   laws   relating   to   family   rights   and   duties,   or   to   the   status,  
When   he   went   back   to   Philippines,   the   wife   brought   an   action   before   the   CFI   condition   and   legal   capacity   of   persons,   are   binding   upon   Spaniards   even  
Manila   requesting   that   the   court   confirm   and   ratify   the   decree   of   divorce   though  they  reside  in  a  foreign  country.    
issued  by  the  state  of  Nevada.  And  that  section  9  of  Act  No.  2710,  which  reads    
as  follows:  “The  decree  of  divorce  shall  dissolve  the  community  of  property  as   And   article   11,   the   last   part   of   which   reads:...   the   prohibitive   laws   concerning  
soon   as   such   decree   becomes   final,   but   shall   not   dissolve   the   bonds   of   persons,   their   acts   and   their   property,   and   those   intended   to   promote   public  
matrimony   until   one   year   thereafter.   The   bonds   of   matrimony   shall   not   be   order  and  good  morals,  shall  not  be  rendered  without  effect  by  any  foreign  laws  
considered   as   dissolved   with   regard   to   the   spouse   who,   having   legitimate   or   judgments   or   by   anything   done   or   any   agreements   entered   into   a   foreign  
children,   has   not   delivered   to   each   of   them   or   to   the   guardian   appointed   by   the   country.    
court,  within  said  period  of  one  year,  the  equivalent  of  what  would  have  been    
due   to   them   as   their   legal   portion   if   said   spouse   had   died   intestate   immediately   It   is   therefore   a   serious   question   whether   any   foreign   divorce   relating   to  
after  the  dissolution  of  the  community  of  property.”  be  enforced,  and  that  she   citizens  of  the  Philippine  Islands,  will  be  recognized  in  this  jurisdiction,  except  it  
and   the   defendant   deliver   to   the   guardian   ad   litem   the   equivalent   of   what   be  for  a  cause,  and  under  conditions  for  which  the  courts  of  Philippine  Islands  
would   have   been   due   to   their   children.   It   is   also   prayed   that   the   community   would  grant  a  divorce.  The  lower  court  in  granting  relief  as  prayed  for  frankly  
existing   between   plaintiff   and   defendant   be   declared   dissolved   and   the   stated  that  the  securing  of  the  divorce,  the  contracting  of  another  marriage  and  
defendant   be   ordered   to   render   an   accounting   and   to   deliver   to   the   plaintiff   the  bringing  into  the  world  of  innocent  children  brings  about  such  a  condition  
her  share  of  the  community  property,  that  the  defendant  be  ordered  to  pay  the   that   the   court   must   grant   relief.   The   hardships   of   the   existing   divorce   laws   of  
plaintiff  alimony  at  the  rate  of  P500  per  month.   the  Philippine  Islands  are  well  known  to  the  members  of  the  Legislature.  It  is  of  
  no   moment   in   this   litigation   what   the   personal   views   of   the   writer   on   the  
A  guardian  ad  litem  was  appointed  for  the  minor  children,  and  they  appeared   subject   of   divorce   may   be.   It   is   the   duty   of   the   courts   to   enforce   the   laws   of  
as   intervenors   and   join   their   mother   in   these   proceedings.   CFI   rendered   divorce  as  written  by  the  Legislature  if  they  are  constitutional.  Courts  have  no  
judgment  against  the  defendant  husband.   right  to  say  that  such  laws  are  too  strict  or  too  liberal.    
 
 
CONFLICT  OF  LAWS                                                                                    AV  DE  TORRES   133  
ATTY.  ARIS  L.  GULAPA                                            AY  2015-­‐2016  
Litigants  by  mutual  agreement  cannot  compel  the  courts  to  approve  of  their   Escano   parents   wanted   them   to   have   a   church   wedding   but   Escano   changed  
own  actions  or  permit  the  personal  relations  of  the  citizens  of  these  Islands  to   her   mind   when   she   received   an   anonymous   letter   stating   that   Tenchavez   had  
be  affected  by  decrees  of  foreign  courts  in  a  manner  which  our  Government   another   woman.   Escano   parents   did   not   agree   as   well.   Vicenta   continued   living  
believes  is  contrary  to  public  order  and  good  morals.  Holding  the  above  views   with  her  parents  while  Pastor  returned  to  his  job  in  Manila.  They  continued  to  
it   becomes   unnecessary   to   discuss   the   serious   constitutional   question   exchange   letters   that   were   not   as   endearing   as   before.  Vicenta   went   to  
presented  by  appellant  in  his  first  assignment  of  error.     Misamis   Occidental,   to   escape   from   the   scandal   that   her   marriage   stirred   in  
  Cebu  society.  There,  a  lawyer  filed  for  her  a  petition,  drafted    
79.  TENCHAVEZ  V.  ESCANO   by  then  Senator  Emmanuel  Pelaez,  to  annul  her  marriage.  She  did  not  sign  the  
G.R.  No.  L-­‐19671  |  29  November  1965   petition   however   and   the   case   was   dismissed   without   prejudice   because   of  
  her  non-­‐appearance  at  the  hearing.    
DOCTRINE:   It   is   irrelevant   that   appellant   Pastor   Tenchavez   should   have    
appeared  in  the  Nevada  divorce  court.  Primarily  because  the  policy  of  our  law   She  applied  for  a  passport,  indicating  in  her  application  that  she  was  single,  that  
cannot   be   nullified   by   acts   of   private   parties   (Civil   Code,   Art.   17)   and   her   purpose   was   to   study,   and   she   was   domiciled   in   Cebu   City,   and   that   she  
additionally,   because   the   mere   appearance   of   a   non-­‐resident   consort   cannot   intended   to   return   after   2   yrs.   In   the   US,   Escano   filed   divorce   against  
confer  jurisdiction  where  the  court  originally  had  none  (Arca  v.  Javier).   Tenchavez  in  the  Second  Judicial  District  Court  of  the  State  of  Nevada  in  and  for  
  the   County   of   Washoe,   on   the   ground   of   "extreme   cruelty,   entirely   mental   in  
N.B.:   character."   Oct/21/1950  -­‐  decree  of  divorce,  "final  and  absolute",  was  issued  
The   divorce   decree   was   obtained   in   the   year   1950.   At   this   time,   the   New   Civil   in  open  court  by  the  said  tribunal.    
Code   was   already   in   force.   In   this   case,   the   marriage   between   Escano   and    
Tenchavez   was   declared   still   subsisting   by   the   court   because   at   the   time   the   Later,   Escano   married   an   American,   Russell   Leo   Moran,   in   Nevada.   She   now  
divorce   decree   was   obtained   by   Escano   and   issued   by   the   courts   of   Nevada,   lives   with   him   in   California,   and,   by   him,   has   begotten   children.   Aug/8/1958   -­‐  
Escano  was  still  a  Filipino  Citizen.     She   acquired   American   citizenship.  Jul/30/1955   amended   on   May   31,   1956   -­‐  
  Tenchavez   had   initiated   the   proceedings   at   bar   against   Vicenta  F.  Escaño,   legal  
FACTS:   separation  for  bigamy,  against  Escano   Parents,  whom  he  charged  with  having  
Vicenta   Escaño   is   27   y.o.   scion   of   a   well-­‐to-­‐do   and   socially   prominent   Filipino   dissuaded   and   discouraged   Vicenta   from   joining   her   husband,   and   alienating  
family   of   Spanish   ancestry   and   a   "sheltered   colegiala".   Pastor   Tenchavez   is   a   32   her  affections,  and  against  the  Roman  Catholic  Church,  for  having,  through  its  
years  of  age,  an  engineer,  ex-­‐army  officer  and  of  undistinguished  stock.   Diocesan  Tribunal,  decreed  the  annulment  of  the  marriage,  and  asked  for  legal  
  separation  and  one  million  pesos  in  damages.    
They  exchanged  marriage  vows  without  the  knowledge  of  her  parents,  before    
a  Catholic  chaplain,  Lt.  Moises  Lavares.  The  marriage  was  the  culmination  of  a   Escano  –  claimed  a  valid  divorce  from  plaintiff  and  an  equally  valid  marriage  to  
previous  love  affair  and  was  duly  registered  with  the  local  civil  register.  They   her  present  husband,  Russell  Leo  Moran    
planned   to   get   married   and   then   elope.   Elopement   did   not,   however,   Escano   Parents   -­‐   denied   that   they   had   in   any   way   influenced   their   daughter's  
materialize  because  when  Vicente  went  back  to  her  classes  after  the  marriage,   acts,  and  counterclaimed  for  moral  damages.    
her   mother   was   already   waiting   for   her   at   the   college.   Vicenta   was   taken   home   CFI,   Cebu   -­‐   did   not   decree   a   legal   separation,   but   freed   the   plaintiff   from  
where  she  admitted  that  she  had  already  married  Pastor.  Mamerto  and  Mena   supporting   his   wife   and   to   acquire   property   to   the   exclusion   of   his   wife.   It  
Escaño   (Escaño   parents)   were   surprised,   because   Pastor   never   asked   for   the   allowed  the  counterclaim  of  Mamerto  Escaño  and  Mena  Escaño  for  moral  and  
hand   of   Vicenta,   and   were   disgusted   because   of   the   great   scandal   that   the   exemplary   damages   and   attorney's   fees   against   the   plaintiff-­‐appellant  
clandestine  marriage  would  provoke.     (P45,000.00).  Thus,  plaintiff  resorted  directly  to  this  Court.    
   
 
 
CONFLICT  OF  LAWS                                                                                    AV  DE  TORRES   134  
ATTY.  ARIS  L.  GULAPA                                            AY  2015-­‐2016  
ISSUE:   74.  RAMIREZ  V.  GMUR  
Whether  Escano  was  still  a  Filipino  citizen  when  the  divorce  decree  was  issued   G.R.  No.  L-­‐11796  |  August  5,  1918  
   
HELD:   DOCTRINES:  It   is   established   by   the   great   weight   of   authority   that   the   court   of   a  
Yes.  At  the  time  the   divorce  decree  was  issued,  Escano  like  her  husband,  was   country  in  which  neither  of  the  spouses  is  domiciled  and  to  which  one  or  both  
still   a   Filipino   citizen.   She   was   then   subject   to   Philippine   law   under   Art.   15   of   of   them   may   resort   merely   for   the   purpose   of   obtaining   a   divorce   has   no  
the   NCC.   Philippine   law,   under   the   NCC   then   now   in   force,   does   not   admit   jurisdiction   to   determine   their   matrimonial   status;   and   a   divorce   granted   by  
absolute  divorce  but  only  provides  for  legal  separation.   such  a  court  is  not  entitled  to  recognition  elsewhere.  The  voluntary  appearance  
  of   the   defendant   before   such   a   tribunal   does   not   vest   the   court   with  
NCC  does  not  allow  absolute  divorce,  quo  ad  vinculo  matrimonii;  and  does  not   jurisdiction.    
even   use   that   term,   to   further   emphasize   its   restrictive   policy   on   the   matter,   in    
contrast  to  the  preceding  legislation  that  admitted  absolute  divorce  on  grounds   It   follows   that,   to   give   a   court   jurisdiction   on   the   ground   of   the   plaintiff's  
of   adultery   of   the   wife   or   concubinage   of   the   husband   (Act   2710).   Instead   of   residence   in   the   State   or   country   of   the   judicial   forum,   his   residence   must   be  
divorce,   the   NCC   only   provides   for   legal   separation   (Title   IV,   Book   1,   Arts.   97   to   bona  fide.  If  a  spouse  leaves  the  family  domicile  and  goes  to  another  State  for  
108),   and,   even   in   that   case,   it   expressly   prescribes   that   "the   marriage   bonds   the  sole  purpose  of  obtaining  a  divorce,  and  with  no  intention  of  remaining,  his  
shall  not  be  severed"   residence   there   is   not   sufficient   to   confer   jurisdiction   on   the   courts   of   that  
  State.  This  is  especially  true  where  the  cause  of  divorce  is  one  not  recognized  
For   Phil.   courts   to   recognize   foreign   divorce   decrees   bet.   Filipino   citizens   would   by  the  laws  of  the  State  of  his  own  domicile.    
be  a   patent   violation   of   the   declared   policy   of   the   State,   especially   in   view   of    
the   3rd   par.   of   Art.   17,   NCC.   Moreover,   recognition   would   give   rise   to   FACTS:  
scandalous  discrimination  in  favor  of  wealthy  citizens  to  the  detriment  of  those   Samuel   Bischoff   Werthmuller,   a   native   of   Switzerland   but   a   resident   of   the  
members   of   our   society   whose   means   do   not   permit   them   to   sojourn   abroad   Philippines,   died   in   Iloilo   City   on   June   29,   1913,   leaving   valuable   asset   which   he  
and  obtain  absolute  divorce  outside  the  Phils.   disposed   by   will.   His   will   was   offered   for   probate   in   the   CFI   of   Iloilo,   which  
  allowed   it.   Werthmuller’s   widow,   Doña   Ana   M.   Ramirez,   was   named   as  
Therefore,  a  foreign  divorce  bet.  Filipino  citizens,  sought  and  decreed  after  the   executrix.   Everything   was   given   to   the   widow,   as   the   will   provided,   except   for   a  
effectivity  of  the  NCC,  is  not  entitled  to  recognition  as  valid  in  this  jurisdiction.   piece  of  real   property   in   the   City   of   Thun,   Switzerland,  which  was  devised  to  
  Werthmuller’s   brothers   and   sisters.   Werthmuller   seemed   to   have   ignored   in  
From   this   point   of   view,   it   is   irrelevant   that   appellant   Pastor   Tenchavez   making  his  will  that  he  had  heirs  from  his  natural  daughter,  Leona  Castro.    
should   have   appeared   in   the   Nevada   divorce   court.   Primarily   because   the    
policy  of  our  law  cannot  be  nullified  by  acts  of  private  parties  (Civil  Code,Art.   Leona  Castro  was  the  daughter  of  Felisa  Castro  and  an  unknown  father.  There  
17)  and  additionally,  because  the  mere  appearance  of  a  non-­‐resident  consort   was   an   annotation   on   the   margin   of   the   original   baptismal   entry   of   Leona  
cannot   confer   jurisdiction   where   the   court   originally   had   none   (Arca   vs.   Castro  that  a  public  document  (an  “escritura”)  states  that  she  was  recognized  
Javier,  95  Phil.  579).   by  Samuel  Bischoff  on  June  22,  1877.  This  annotation  was  authenticated  by  the  
  signature   of   Father   Ferrero,   whose   deposition   was   taken   in   this   case.   Father  
Ferrero   testified   that   the   word   “escritura”   in   this   entry   means   a   public  
document;   and   he   says   that   such   document   was   exhibited   to   him   when   the  
marginal   note   was   added   to   the   baptismal   record   and   supplied   the   basis   of   the  
annotation  in  the  entry.    
 

 
CONFLICT  OF  LAWS                                                                                    AV  DE  TORRES   135  
ATTY.  ARIS  L.  GULAPA                                            AY  2015-­‐2016  
Samuel  Bischoff  tacitly  recognized  Leona  Castro  as  his  daughter  and  treated  her    
as  such.  Leona  Castro  was  later  married   to   Frederick   von   Kauffman,   a   British   ISSUE:  
subject,  born  in  HK  and  lived  in  Iloilo  City.  Leona  Castro  and  von  Kauffman  had   1.   WON   the   divorce   decree   granted   by   the   Paris   court   can   be   recognized   in   the  
3   children   (Elene,   Federico   and   Ernesto).   Leona   Castro   was   then   brought   to   Philippines  
Thun,   Switzerland   to   recuperate   her   health.   Years   later,   Leona   Castro   informed   2.  WON  the  Mory  claimants  are  entitled  to  the  estate  
von  Kauffman  that  she  does  not  want  to  stay  married  with  the  latter.      
  HELD:  
Von  Kauffman  later  obtained  a  divorce  decree  in  Paris,  France.  It  showed  that   No.   The   divorce   decree   in   question   cannot   be   recognized   as   valid   in   the  
Leona   Castro   lived   in   Paris,   though   there   is   no   evidence   showing   that   she   Philippines.   The   French   tribunal   has   no   jurisdiction   to   entertain   an   action   for  
acquired  permanent  domicile  in  Paris.     the   dissolution   of   a   marriage   contracted   in   the   Philippines   by   a   person  
  domiciled   here;   such   marriage   being   indissoluble   under   the   laws   then  
The   estrangement   of   von   Kauffman   and   Leona   Castro   was   because   Leona   prevailing  in  this  country.  
Castro   was   attracted   to   Dr.   Ernest   Emil   Mory,   the   physician   in   charge   of   the    
sanitarium   in   Switzerland   where   Leona   Castro   was   brought.   Dr.   Mory   and   The   evidence   shows   that   both   Kauffman   and   Leona   are   domiciled   in   Iloilo,  
Leona  Castro  was  later  married  in  London,  England.  It  appears  that  Dr.  Mory   Philippines   and   that   their   departure   to   Switzerland   was   for   medical   purpose,  
was  already  married  to  a  certain  Helena  Wolpman,  but  had  divorced  her.     and   that   Kauffman   went   to   Paris   in   1904   to   obtain   a   divorce   without   an  
  intention  to  establish  permanent  residence  in  that  city.  
Before  Dr.  Mory  and  Leona  Castro  got  married,  they  begot  a  daughter  named    
nd  
Leontina   Elizabeth   in   Thun,   Switzerland.   A   2 daughter,   Carmen   Maria,   was   A  court,  where  neither  of  the  spouses  is  domiciled,  and  to  which  one  or  both  of  
born   in   Berne,   Switzerland,   and   a   3rd   daughter,   Esther.   On   October   6,   1910,   them   may   resort   merely   for   the   purpose   of   obtaining   a   divorce,   has   no  
Leona  Castro  died.     jurisdiction   to   determine   their   matrimonial   status;   and   a   divorce   granted   by  
  such  a  court  is  not  entitled  to  recognition  elsewhere.  
Now,   the   2   sets   of   children   are   claiming   from   the   estate   of   Samuel    
Werthmuller.  Otto   Gmur   is   the   guardian   of   the   3   Mory   daughters.   Frederick   2.  No.  With  regard  to  the  Mory  claimants,  their  rights  principally  depend  upon  
von  Kauffman  appeared  as  guardian  for  his  own  children.     the   effect   to   be   given   by   this   court   to   divorce   degree   granted   to   Kauffman   in  
  Paris.  If  divorce  decree  is  valid,  the  subsequent  marriage  of  Leona  and  Mory  is  
Ana  Ramirez  insists,  as  against  the  Mory  daughters,  that  the  Leona  Castro  had   also  valid.  
never  been  recognized  by  Samuel  Werthmuller  at  all.      
  Since  the  divorce  granted  by  the  French  court  cannot  be  recognized,  it  follows  
As   to   the   Mory   daughters,   Leontina   Elizabeth   is   considered   an   illegitimate   that  the  marriage  between  Mory  and  Leona  in  London  could  not  legalize  their  
daughter   which   was   legitimated   by   the   subsequent   marriage   of   Dr.   Mory   and   relations.   The   claims   of   the   Mory   children   to   participate   in   the   estate   of  
Leona  Castro.  Carmen  Maria  and  Esther  Renate,  on  the  other  hand,  are  to  be   Bischoff   must   be   rejected.   The   right   to   inherit   is   limited   to   legitimate,  
considered   legitimate   offspring   of   Leona   Castro   since   the   latter’s   marriage   to   legitimated,  and  acknowledged  natural  children.    
von   Kauffman   was   already   divorced   when   they   were   born   and   Leona   was    
already  married  to  Dr.  Mory.    
 
The   von   Kauffman   children   insists   that   the   divorce   decree   was   wholly   invalid;  
that   the   Mory   daughters   are   the   offspring   of   an   adulterous   relationship;   and  
that  the  von  Kauffman  daughters  alone  should  be  entitled  to  participate  in  the  
division  of  the  estate.    
 
CONFLICT  OF  LAWS                                                                                    AV  DE  TORRES   136  
ATTY.  ARIS  L.  GULAPA                                            AY  2015-­‐2016  
80.  VAN  DORN  V.  ROMILLO    
139  SCRA  139  (1985)   HELD:  
  Yes.  There  can  be  no  question  as  to  the  validity  of  that  Nevada  divorce  in  any  of  
DOCTRINE:   However,   aliens   may   obtain   divorces   abroad,   which   may   be   the   States   of   the   United   States.   The   decree   is   binding   on   private   respondent   as  
recognized   in   the   Philippines,   provided   they   are   valid   according   to   their   an  American  citizen.  For  instance,  private  respondent  cannot  sue  petitioner,  as  
national  law.  Thus,  pursuant  to  his  national  law,  private  respondent  is  no  longer   her   husband,   in   any   State   of   the   Union.   What   he   is   contending   in   this   case   is  
the   husband   of   petitioner.   He   would   have   no   standing   to   sue   in   the   case   below   that   the   divorce   is   not   valid   and   binding   in   this   jurisdiction,   the   same   being  
as  petitioner's  husband  entitled  to  exercise  control  over  conjugal  assets.  As  he   contrary  to  local  law  and  public  policy.  
is   bound   by   the   Decision   of   his   own   country's   Court,   which   validly   exercised    
jurisdiction   over   him,   and   whose   decision   he   does   not   repudiate,   he   is   It  is  true  that  owing  to  the  nationality  principle  embodied  in  Article  15  of  the  
estopped  by  his  own  representation  before  said  Court  from  asserting  his  right   Civil   Code,  only   Philippine   nationals   are   covered   by   the   policy   against  
over  the  alleged  conjugal  property.   absolute   divorces   the   same   being   considered   contrary   to   our   concept   of  
  public   police   and   morality   (art.   17).   However,   aliens   may   obtain   divorces  
Kyna’s  Notes:   abroad,  which  may  be  recognized  in  the  Philippines,  provided  they  are  valid  
ALG:  1.  Divorce  obtained  abroad  is  allowed  and  recognized  in  the  Philippines  as   according   to   their   national   law.  In   this   case,   the   divorce   in   Nevada   released  
long  as  the  foreigner  participated  in  the  proceedings  –  TRUE   private   respondent   from   the   marriage   from   the   standards   of   American   law,  
2.   Divorce   obtained   by   a   Filipino   where   foreigner   spouse   participated   is   valid   under  which  divorce  dissolves  the  marriage.  As  stated  by  the  Federal  Supreme  
here  in  the  Philippines  –  FALSE   Court  of  the  United  States  in  Atherton  vs.  Atherton,  45  L.  Ed.  794,  799:  
  The  purpose  and  effect  of  a  decree  of  divorce  from  the  bond  
FACTS:   of   matrimony   by   a   court   of   competent   jurisdiction   are   to  
Alice   Reyes   Van   Dorn,   a   Filipino   Citizen   and   private   respondent,   Richard   Upton,   change   the   existing   status   or   domestic   relation   of   husband  
a   US   citizen,   was   married   in   Hong   Kong   in   1979.   They   established   their   and  wife,  and  to  free  them  both  from  the  bond.  The  marriage  
residence  in  the  Philippines  and  had  2  children.  They  were  divorced  in  Nevada,   tie  when  thus  severed  as  to  one  party,  ceases  to  bind  either.  
USA  in  1982  and  petitioner  remarried,  this  time  with  Theodore  Van  Dorn.  A  suit   A   husband   without   a   wife,   or   a   wife   without   a   husband,   is  
against  petitioner  was  filed  on  June  8,  1983,  stating  that  petitioner’s  business  in   unknown   to   the   law.   When   the   law   provides,   in   the   nature   of  
Ermita   Manila,   the   Galleon   Shop,   is   a   conjugal   property   with   Upton   and   prayed   a   penalty   that   the   guilty   party   shall   not   marry   again,   that  
therein  that  Alice  be  ordered  to  render  an  accounting  of  the  business  and  he  be   party,   as   well   as   the   other,   is   still   absolutely   freed   from   the  
declared   as   the   administrator   of   the   said   property.   Petitioner   moved   to   dismiss   bond  of  the  former  marriage.  
the  case  on  the  ground  that  the  cause  of  action  is  barred  by  previous  judgment    
in   the   divorce   proceedings   before   the   Nevada   Court   wherein   respondent   had   Thus,  pursuant  to  his  national  law,  private  respondent  is  no  longer  the  husband  
acknowledged  that  he  and  petitioner  had  "no  community  property"  as  of  June   of   petitioner.   He   would   have   no   standing   to   sue   in   the   case   below   as  
11,  1982.  The  Court  below  denied  the  Motion  to  Dismiss  in  the  mentioned  case   petitioner's  husband  entitled  to  exercise  control  over  conjugal  assets.  As  he  is  
on   the   ground   that   the   property   involved   is   located   in   the   Philippines   so   that   bound   by   the   Decision   of   his   own   country's   Court,   which   validly   exercised  
the  Divorce  Decree  has  no  bearing  in  the  case.     jurisdiction   over   him,   and   whose   decision   he   does   not   repudiate,   he   is  
  estopped  by  his  own  representation  before  said  Court  from  asserting  his  right  
ISSUE:   over  the  alleged  conjugal  property.  
Whether   or   not   the   foreign   divorce   between   the   petitioner   and   private    
respondent  in  Nevada  is  binding  in  the  Philippines  where  petitioner  is  a  Filipino   To  maintain,  as  private  respondent  does,  that,  under  our  laws,  petitioner  has  to  
citizen?     be   considered   still   married   to   private   respondent   and   still   subject   to   a   wife's  
obligations   under   Article   109,  et.   seq.   of   the   Civil   Code   cannot   be   just.  
 
CONFLICT  OF  LAWS                                                                                    AV  DE  TORRES   137  
ATTY.  ARIS  L.  GULAPA                                            AY  2015-­‐2016  
Petitioner   should   not   be   obliged   to   live   together   with,   observe   respect   and   discovered  that  his  wife  Paula  was  pregnant  and  was  “living  in”  and  having  an  
fidelity,   and   render   support   to   private   respondent.   The   latter   should   not   adulterous  relationship  with  his  brother,  Ceferino  Llorente.  Paula  gave  birth  to  
continue   to   be   one   of   her   heirs   with   possible   rights   to   conjugal   property.   She   a  boy  registered  as  “Crisologo  Llorente,”  with  the  birth  certificate  stating  that  
should  not  be  discriminated  against  in  her  own  country  if  the  ends  of  justice  are   the  child  was  not  legitimate  and  the  line  for  the  father’s  name  was  left  blank.    
to  be  served.    
  Lorenzo   refused   to   forgive   Paula   and   live   with   her.   In   fact,   the   couple   drew   a  
Husband  in  this  case  is  estopped  –  this  is  one  way  to  recognize  divorce  decrees   written   agreement   to   the   effect   that   (1)   all   the   family   allowances   allotted   by  
  the  United  States  Navy  as  part  of  Lorenzo’s  salary  and  all  other  obligations  for  
71.  LLORENTE  V.  CA,  SUPRA   Paula’s   daily   maintenance   and   support   would   be   suspended;   (2)   they   would  
  dissolve   their   marital   union   in   accordance   with   judicial   proceedings;   (3)   they  
DOCTRINE:  Only  Philippine  nationals  are  covered  by  the  policy  against  absolute   would  make  a  separate  agreement  regarding  their  conjugal  property  acquired  
divorces,   the   same   being   considered   contrary   to   our   concept   of   public   policy   during   their   marital   life;   and   (4)   Lorenzo   would   not   prosecute   Paula   for   her  
and  morality.    In  the  same  case,  the  Court  ruled  that  aliens  may  obtain  divorces   adulterous  act  since  she  voluntarily  admitted  her  fault  and  agreed  to  separate  
abroad,  provided  they  are  valid  according  to  their  national  law.   from  Lorenzo  peacefully.  The  agreement  was  signed  by  both  Lorenzo  and  Paula  
  and   was   witnessed   by   Paula’s   father   and   stepmother.   The   agreement   was  
ALG:   notarized.    
1991  –  Celebration  of  marriage    
1992  –  Filipino  went  to  the  U.S.   Lorenzo   returned   to   the   US   and   filed   for   divorce   with   the   Superior   Court   of   the  
1993  –  Obtains  a  divorce  decree  in  U.S.   State   of   California   and   was   issued   an   interlocutory   judgment   of   divorce.   The  
  divorce  decree  became  final  afterwards.    
The  divorce  decree  is  not  valid.  The  Filipino  remained  a  Filipino  citizen  and  not    
a  citizen  of  the  U.S.  So  the  governing  law  over  the  person  of  the  Filipino  is  still   Lorenzo   returned   to   the   Philippines   and   married   Alicia   Llorente   in   Manila.  
Philippine  law.     Apparently,  Alicia  had  no  knowledge  of  the  first  marriage  even  if  they  resided  
  in  the  same  town  as  Paula,  who  did  not  oppose  the  marriage  or  cohabitation.  
1991  –  Celebration  of  marriage   Lorenzo   and   Alicia   lived   together   as   husband   and   wife.   Their   25   year   union  
1992  –  Filipino  went  to  the  U.S.  and  became  an  American  citizen   produced  three  children,  Raul,  Luz  and  Beverly,  all  surnamed  Llorente.    
1993  –  Obtains  a  divorce  decree  in  U.S.    
  Lorenzo  executed  a  Last  Will  and  Testament.  In  the  will,  Lorenzo  bequeathed  all  
The   divorce   decree   is   valid.   It   is   the   time   of   getting   the   divorce   which   is   his   property   to   Alicia   and   their   three   children.   Lorenzo   filed   with   the   RTC  
material.   Camarines   Sur,   a   petition   for   the   probate   and   allowance   of   his   last   will   and  
  testament   wherein   Lorenzo   moved   that   Alicia   be   appointed   Special  
FACTS:   Administratrix   of   his   estate.   RTC   admitted   the   will   to   probate.   Before   the  
Llorente  was  an  enlisted  serviceman  of  the  United  States  Navy.  During  his  term   proceedings  could  be  terminated,  Lorenzo  died.    
of   duty,   Lorenzo   and   Paula   Llorente   were   married   before   a   parish   priest,    
Roman   Catholic   Church,   in   Camarines   Sur.   Before   the   outbreak   of   the   Pacific   Paula   filed   with   the   same   court   a   petition   for   letters   of   administration   over  
War,  Lorenzo  departed  for  the  United  States  and  Paula  stayed  in  the  conjugal   Lorenzo’s   estate   in   her   favor.   Paula   contended   (1)   that   she   was   Lorenzo’s  
home   in   Camarines   Sur.   Afterwards,   Lorenzo   was   naturalized   as   an   American   surviving   spouse,   (2)   that   the   various   property   were   acquired   during   their  
Citizen.   Upon   the   end   of   the   World   War   II,   Lorenzo   was   granted   an   accrued   marriage,   (3)   that   Lorenzo’s   will   disposed   of   all   his   property   in   favor   of   Alicia  
leave   by   the   US   Navy,   to   visit   his   wife   and   he   visited   the   Philippines.   He   and   her   children,   encroaching   on   her   legitime   and   1/2   share   in   the   conjugal  
property.   Alicia   filed   in   the   testate   proceeding   a   petition   for   the   issuance   of  
 
CONFLICT  OF  LAWS                                                                                    AV  DE  TORRES   138  
ATTY.  ARIS  L.  GULAPA                                            AY  2015-­‐2016  
letters  testamentary.  RTC  gave  due  course  to  Paula's  petition.  RTC  held  that  the   proven  that  respondent  was  no  longer  a  Filipino  citizen  when  he  obtained  the  
divorce  decree  was  and  void  and  inapplicable  in  the  Philippines  and  therefore   divorce   from   petitioner,   the   ruling   in  Van   Dorn  would   become   applicable   and  
the  second  marriage  with  Alicia  was  also  void.  CA  affirmed  RTC's  decision.     petitioner  could  “very  well  lose  her  right  to  inherit”  from  him.  
   
ISSUE:   In  Pilapil   v.   Ibay-­‐Somera,   we   recognized   the   divorce   obtained   by   the  
Whether  Lorenzo’s  first  marriage  has  been  validly  dissolved  through  a  divorce   respondent  in  his  country,  the  Federal  Republic  of  Germany.    There,  we  stated  
decree,  which  capacitated  him  to  remarry  Alicia.     that  divorce  and  its  legal  effects  may  be  recognized  in  the  Philippines  insofar  as  
  respondent  is  concerned  in  view  of  the  nationality  principle  in  our  civil  law  on  
HELD:   the  status  of  persons.  
The  Applicable  Law    
The   fact   that   the   late   Lorenzo   N.   Llorente   became   an   American   citizen   long   For  failing  to  apply  these  doctrines,  the  decision  of  the  Court  of  Appeals  must  
before  and  at  the  time  of:  (1)  his  divorce  from  Paula;  (2)  marriage  to  Alicia;  (3)   be  reversed.  We  hold  that  the  divorce  obtained  by  Lorenzo  H.  Llorente  from  his  
execution   of   his   will;   and   (4)   death,   is   duly   established,   admitted   and   first   wife   Paula   was   valid   and   recognized   in   this   jurisdiction   as   a   matter   of  
undisputed.  Thus,  as  a  rule,  issues  arising  from  these  incidents  are  necessarily   comity.    Now,  the  effects  of  this  divorce  (as  to  the  succession  to  the  estate  of  
governed  by  foreign  law.   the  decedent)  are  matters  best  left  to  the  determination  of  the  trial  court.  
   
The  Civil  Code  clearly  provides:  
ADOPTION  
“Art.   15.    Laws   relating   to   family   rights   and   duties,   or   to   the   status,   condition    
and  legal  capacity  of  persons  are  binding  upon  citizens  of  the  Philippines,  even    
though  living  abroad.   ADOPTION  IS  AN  ACT  WHICH  RENDERS  A  CHILD  LEGITIMATE  IN  RELATION  TO  THE  
  ADOPTING  PARENTS  TO  WHOM  THE  CHILD  MAY  OR  MAY  NOT  BE  RELATED.  
“Art.   16.    Real   property   as   well   as   personal   property   is   subject   to   the   law   of   the    
country  where  it  is  situated.   ISSUES  IN  COL  RELATE  TO:  
  1. JURISDICTION  TO  GRANT  
“However,   intestate   and   testamentary   succession,   both   with   respect   to   the   2. CAPACITY  OF  ALIEN  TO  ADOPT  
order   of   succession   and   to   the   amount   of   successional   rights   and   to   the   3. EFFECTS  OF  CONSEQUENCES  OF  ADOPTION  
intrinsic  validity  of  testamentary  provisions,shall  be  regulated  by  the  national    
law  of  the  person  whose  succession  is  under  consideration,  whatever  may  be   CHOICE  OF  LAW    
the   nature   of   the   property   and   regardless   of   the   country   wherein   said   property       Personal  law  of  both  the  adopter  and  adoptee  
may  be  found.”     Adopter:  to  determine  his  eligibility  to  adopt      
  Adoptee:  to  determine  his  eligibility  to  be    adopted      
Validity  of  the  Foreign  Divorce    
In  Van   Dorn   v.   Romillo,   Jr.   we   held   that   owing   to   the   nationality   principle   EFFECTS:  SROPALS2  
embodied  in  Article  15  of  the  Civil  Code,  only  Philippine  nationals  are  covered   1. CREATION   OF   SUCCESSIONAL   RIGHTS   AND   OBLIGAITONS   BETWEEN  
by  the  policy  against  absolute  divorces,  the  same  being  considered  contrary  to   ADOPTERS  AND  ADOPTEE  
our   concept   of   public   policy   and   morality.    In   the   same   case,   the   Court   ruled   2. PARTIES   ARE   VESTED   WITH   RIGHTS—CIVIL   RIGHTS   FOR   THE   ADOPTERS,  
that  aliens  may   obtain   divorces   abroad,   provided   they   are   valid   according   to   SUCH   AS   PARENTAL   AUTHORITY   AND   THE   ADOPTEE’S   LEGITIMACY,   AND  
their  national  law.   THE  RIGHT  TO  USE  THE  ADOPTER’S  SURNAME  FOR  THE  ADOPTEE  
  3. ALL  LEGAL  TIES  BETWEEN  BIOLOGICAL  PARENTS  AND  THE  ADOPTEE  SHALL  
Citing  this  landmark  case,  the  Court  held  in  Quita  v.  Court  of  Appeals,  that  once   BE  SEVERED  
 
CONFLICT  OF  LAWS                                                                                    AV  DE  TORRES   139  
ATTY.  ARIS  L.  GULAPA                                            AY  2015-­‐2016  
BUT—Adoptee   may   still   claim   inheritance   from   natural   3. Illegitimate   child,   by   a   qualified   adopter   to   raise   the   status   of   the  
parents       former  to  that  of  legitimacy  
  4. A  child  whose  adoption  has  been  previously  rescinded  
DOMESTIC  ADOPTION  ACT  OF  1998   5. A  child  whose  biological  or  adoptive  parents  have  died,  after  6  months  
  from  such  death  
WHO  MAY  ADOPT:   6. Any  person  below  18   y/o   who  has  been  judicially  or  administratively  
  declared  available  for  adoption  
ANY  FILIPINO  CITIZEN  WHO:  [CLEGS  16]    
1. In  possession  of  full  civil  capacity  and  legal  rights   INTER-­‐COUNTRY  ADOPTION  ACT  OF  1995  
2. Of  legal  age    
3. Emotionally  and  psychologically  capable  of  caring  for  children   PETITIONER  
4. Of  good  moral  character   1. Adopter  or  
5. In  a  position  to  support  and  care  for  his  children  in  keeping  with  the   2. Inter-­‐country  adoption  body  of  the  adopter’s  state  
means  of  the  family    
6. At  least  16  years  older  than  the  adoptee   WHO  MAY  ADOPT:  [DA2C2E  JoMS  27]  
  1. Country  has  diplomatic  relations  with  the  Phil.  
ANY  ALIEN  POSSESSING  THE  SAME  QUALIFICATIONS  AS  ABOVE,  PROVIDED:  [3  DiCE]   2. Whose  government  has  a  foreign  adoption  agency  
1. Living  in  the  Phil.  For  at  least  3  continuous  years  prior  to  the  filing  of   3. Whose  laws  allow  adoption  
the   application   for   adoption   and   maintains   such   residence   until   the   4. Capacity  to  act  and  assume  all  the  rights  and  responsibilities  incidental  
adoption  decree  is  entered   to  parental  authority  under  his/her  national  law  
2. His  country  has  diplomatic  relations  with  the  Phil.   5. Undergone   appropriate   counseling   from   an   accredited   counselor   in  
3. Certified   by   his   diplomatic   or   consular   office   or   any   appropriate   his/her  country  
government  agency  to  have  legal  capacity  to  adopt  in  his  country,  and   6. Is  eligible  to  adopt  under  his/her  national  law  
that   his   government   allows   the   adoptee   to   enter   his   country   as   his   7. Files   jointly   with   his/her   spouse,   if   any,   who   shall   have   the   same  
adopted  child   qualifications  and  none  of  the  disqualifications  to  adopt  as  prescribed  
  above  
THE   GUARDIAN   WITH   RESPECT   TO   THE   WARD   AFTER   THE   TERMINATION   OF   THE   8. Has  not  been  convicted  of  a  crime  involving  moral  turpitude  
GUARDIANSHIP  AND  CLEARANCE  OF  HIS  FINANCIAL  ACCOUTABILITIES   9. Can  provide  the  proper  care  and  support  and  give  the  necessary  moral  
  values  and  example  to  the  child  and  in  the  proper  case  to  all  his/her  
GR:  Spouses  shall  jointly  adopt.   other  children  
EXC:  [other’s  LC,  own  IC  with  consent,  leg  sep]   10. At   least   27   y/o   and   is   at   least   16   years   older   than   the   child   to   be  
1. If  one  spouse  seeks  to  adopt  the  legitimate  child  of  the  other   adopted  at  the  time  of  the  filing  of  the  application,  unless  the  applicant  
2. If  one  spouse  seeks  to  adopt  his  own  illegitimate  child,  provided  that   is  the  parent  by  nature  of  the  child  to  be  adopted  or  is  the  spouse  of  
the  other  spouse  has  signified  his  consent  thereto   such  parent  by  nature  
3. If  the  spouses  are  legally  separate  from  each  other    
  WHO  MAY  BE  ADOPTED:  
WHO  MAY  BE  ADOPTED:  [CLIRD2  18]   1. A  child  voluntarily  or  involuntarily  committed  to  DSWD  as  dependent,  
1. Person   who   has   been   consistently   considered   and   treated   by   the   abandoned   or   neglected   pursuant   to   the   provisions   of   the   Child   and  
adopters  as  their  own  child  since  minority   Youth  Welfare  Code  
2. Legitimate  child  of  the  spouse  of  the  prospective  adopter   2. Below  15  y/o  
 
CONFLICT  OF  LAWS                                                                                    AV  DE  TORRES   140  
ATTY.  ARIS  L.  GULAPA                                            AY  2015-­‐2016  
  Whether  Caraballo  is  qualified  to  adopt  
65.  CARABALLO  V.  REPUBLIC    
G.R.  No.  L-­‐15080  |  25  April  1962   HELD:  
  No.   (Law   in   effect   at   that   time)  Article   335   of   the   Civil   Code   provides:  The  
DOCTRINE:   Ricardo   R.   Caraballo,   the   petitioner,   an   American   citizen   who   now   following   cannot   adopt   -­‐  (1)   Those   who   have   legitimate,   legitimated,  
lives  in  Clark  Field,  municipality  of  Angeles,  province  of  Pampanga,  Republic  of   acknowledged   natural   children,   or   natural   children   by   legal   fiction;  (2)   The  
the  Philippines,  because  of  his  assignment  as  staff  sergeant  in  the  United  States   guardian,   with   respect   to   the   ward,   before   the   final   approval   of   his   accounts;  
Air  Force  –  his  stay  in  the  Philippines  then  being  temporary  –  is  a  non-­‐resident   (3)   A   married   person,   without   the   consent   of   the   other   spouse;  (4)   Non-­‐
alien   who,   pursuant   to   clause   4   of   the   above   quoted   article   of   the   Civil   Code,   is   resident  aliens;  (5)  Resident  aliens  with  whose  government  the  Republic  of  the  
disqualified  to  adopt  a  child  in  the  Philippines.     Philippines   has   broken   diplomatic   relations;  (6)   Any   person   who   has   been  
  convicted  of  a  crime  involving  moral  turpitude,  when  the  penalty  imposed  was  
N.B.:   Under   the   Inter-­‐Country   Adoption   Act,   a   non-­‐resident   alien   may   now   six  months’  imprisonment  or  more.    
adopt  a  child  in  the  Philippines  subject  to  certain  qualifications  [DA2C2E  JoMS    
27]   A   person   is   deemed   a   resident   of   a   place   in   a   country   or   state   where   he   has   his  
  abode   and   lives   there   permanently.   It   is   a   place   chosen   by   him   freely   and  
FACTS:   voluntarily,   although   he   may   later   on   change   his   mind   and   live   elsewhere.   A  
Ricardo   R.   Caraballo,   an   American   citizen,   was   living   with   his   wife   Graciela   place  in  a  country  or  state  where  he  lives  and  stays  permanently  and  to  which  
Caraballo  in  Clark  Field,  Angeles,  Pampanga.  He  enlisted  in  the  US  Air  Force  as   he   intends   to   return   after   a   temporary   absence,   no   matter   how   long,   is   his  
staff   sergeant   detailed   in   Clark   through   a   verified   petition   before   the   CFI   of   domicile.   A   sojourn   such   as   a   tourist   though   actually   present   at   a   place   of   his  
Pampanga.     free  choice  cannot  be  deemed  a  resident  of  that  place.  A  foreigner  who  has  a  
  business  or  interest  therein  or  property  located  in  a  country  or  state  and  goes  
He  alleged  that  he  and  his  wife  have  no  legitimate,  legitimated,  acknowledged   and   stays   in   that   country   or   state   to   look   after   his   business   or   property   or   to  
natural   children,   natural   children   by   legal   fiction   or   any   other   descendants,   and   check  up  the  manner  or  way  his  business  or  property  is  being  conducted  or  run  
that   with   his   wife’s   written   consent,   he   desires   to   adopt   Norma   Lee   Caber,   a   by  his  manager  but  does  not  intend  to  remain  in  the  country  indefinitely  cannot  
five-­‐year   old   daughter   of   Mercedes   Caber   of   an   unknown   father.   Caber   gave   be  deemed  a  resident  of  such  country.  Actual  or  physical  presence  or  stay  of  a  
her  consent  in  a  sworn  statement.     person   in   a   place,   not   of   his   free   and   voluntary   choice   and   without   intent   to  
  remain  there  indefinitely,  does  not  make  him  a  resident  of  the  place.    
The   verified   petition   was   ordered   to   be   published,   but   no   one   appeared   to    
contest   the   adoption.   Provincial   and   Assistant   Provincial   Fiscal   of   Pampanga   Looking  after  the  welfare  of  a  minor  to  be  adopted  the  law  has  surrounded  him  
moved  to  dismiss  the  petition  since  the  petitioner,  being  a  non-­‐resident  alien,  is   with  safeguards  to  achieve  and  insure  such  welfare.  It  cannot  be  gainsaid  that  
not  qualified  to  adopt.  MD  denied.     an   adopted   minor   may   be   removed   from   the   country   by   the   adopter,   who   is  
  not  a  resident  of  the  Philippines,  and  placed  beyond  the  reach  and  protection  
The   CFI   granted   the   adoption   of   Norma   Lee   Caber   after   finding   the   following   of  the  country  of  his  birth.    
facts:  petitioner  is  32  years  old,  the  child  is  3  months  old,  he  had  the  child  in  his    
household   since   the   day   following   her   birth   and   has   developed   fondness   for   Ricardo   R.   Caraballo,   the   petitioner,   an   American   citizen   who   now   lives   in  
her.  He  is  a  staff  sergeant  in  the  US  Air  Force  and  is  adequately  compensated,   Clark   Field,   municipality   of   Angeles,   province   of   Pampanga,   Republic   of   the  
and  he  has  never  been  convicted  of  a  crime  involving  moral  turpitude.     Philippines,  because  of  his  assignment  as  staff  sergeant  in  the  United  States  
  Air  Force  –  his  stay  in  the  Philippines  then  being  temporary  –  is  a  non-­‐resident  
ISSUE:   alien  who,  pursuant  to  clause  4  of  the  above  quoted  article  of  the  Civil  Code,  
is  disqualified  to  adopt  a  child  in  the  Philippines.    
 
CONFLICT  OF  LAWS                                                                                    AV  DE  TORRES   141  
ATTY.  ARIS  L.  GULAPA                                            AY  2015-­‐2016  
LEX  FORI  
SUCCESSION  
   
  INTERPRETATION  OF  WILLS  
SUCCESSION   IS   A   MODE   OF   ACQUISITION   BY   VIRUTE   OF   WHICH   THE   PROPERTY,   1.  INTENT  OF  THE  PARTIES  SHALL  GOVERN,  OR  IN  DEFAULT  OF  WHICH,  
RIGHTS,   AND   OBLIGATIONS   TO   THE   EXTENT   OF   THE   VALUE   OF   THE   2.  INTERPRETATION  THAT  WILL  GIVE  EFFECT  TO  EVERY  STIPULATION  
INHERITANCE,   OF   A   PERSON   ARE   TRANSMITTED   THROUGH   HIS   DEATH   TO   WHERE  DISPOSITION  IS  OPERATIVE  IS  PREFERRED  
ANOTHER  OR  OTHERS  EITHER  BY  HIS  WILL  OR  BY  OPEARTION  OF  LAW.   WHERE  INTESTACY  IS  PREVENTED  IS  PREFERRED  
   
TWO  THEORIES   ADMIN  OF  ESTATE  
UNIVERSAL  SUCCESSION     LEX  FORI  
WHATEVER   PERSONAL   LAW   GOVERNED   THE   DECEDENT    
GOVERNS   SUCCESSION   TO   HIS   ENTIRE   ESTATE   AFTER   HIS   ACTUAL  PARTITION  
DEATH;  PH  ADHERES  TO  THIS   LEX  SITEI  
  SPLIT  OR  DIVISION  SYSTEM    
SEEKS   TO   DISTINGUISH   BETWEEN   IMMOVALES   AND   86.  IN  RE  ESTATE  OF  JOHNSON  
MOVABLES;   LEX   SITUS   –   IMMOVABLES;   DOMICILE   -­‐   G.R.  No.  L-­‐12767  |  November  16,  1918  
MOVABLES    
  Upon  this  point  it  is  sufficient  to  say  that  the  probate  of  the  will  does  not  affect  
CAPACITY  TO  SUCCEED   the   intrinsic   validity   of   its   provisions,   the   decree   of   probate   being   conclusive  
ART.  1039  -­‐   NATIONAL  LAW   only  as  regards  the  due  execution  of  the  will.  The  will  was  properly  admissible  
  to  probate.  
INTRINSIC  VALIDITY    
ART.  16  -­‐      AT  THE  TIME  OF  DEATH   N.B.:  Due  execution  –  [FM  VIPS]  
  1.  Formalities  2.  T  was  of  sound  and  disposing  mind  3.  No  Vitiation  of  consent  4.  
EXTRINSIC  OR  FORMAL  VALIDITY   Not  procured  by  improper  Influence  or  Pressure  5.  Signature  is  genuine  
ART.  17  -­‐     AT  THE  TIME  OF  EXECUTION    
  The  following  articles  govern  foreign  wills  now:  
  FIL     ABROAD     LAW   OF   THE   COUNTRY   IN   WHICH   HE   MAY    
BE   Article   17.   The   forms   and   solemnities   of   contracts,   wills,   and   other   public  
  ALIEN   PHIL     NATIONAL  LAW   instruments   shall   be   governed   by   the   laws   of   the   country   in   which   they   are  
  ALIEN   ABROAD     [RNP]  LAW  OF  RESIDENCE,  NATL  LAW,  PHIL   executed.  
LAW    
  HOLO  WILL     WRITTEN,  DATED,  SIGNED  ENTIRELY   Article  816.  The  will  of  an  alien  who  is  abroad  produces  effect  in  the  Philippines  
  FIL   JOINT  WILLS   VOID   if   made   with   the   formalities   prescribed   by   the   law   of   the   place   in   which   he  
  resides,   or   according   to   the   formalities   observed   in   his   country,   or   in   conformity  
REVOCATION   with  those  which  this  Code  prescribes.  
  NONRES  ABROAD     [MD]  WILL  WAS  MADE,  DOMICILIARY  LAW    
  RES   ABROAD     [PR]  PHIL  LAW,  PLACE  OF  REVOCATION   At  the  time  of  the  decision  of  this  case,  Phil.  courts  can  take  judicial  notice  of  US  
  Federal   Laws   because   it   was   then   a   colony.   Now,   the   Philippine   courts   cannot  
PROBATE  
 
CONFLICT  OF  LAWS                                                                                    AV  DE  TORRES   142  
ATTY.  ARIS  L.  GULAPA                                            AY  2015-­‐2016  
take  judicial  notice  of  any  US  Law  except  under  certain  exceptions.   accordance  with  the  law  of  the  state  or  country  of  which  he  is  a  citizen  or  
  subject,  and  which  might  be  proved  and  allowed  by  the  law  of  his  own  
ALG:   At   that   time,   our   courts   can   only   take   judicial   notice   of   federal   law   but   not   state  or  country,  may  be  proved,  allowed,  and  recorded  in  the  Philippine  
state  law.  The  country  was  a  US  protectorate  then.   Islands,   and   shall   have   the   same   effect   as   if   executed   according   to   the  
  laws  of  these  Islands.  
Summary  of  Facts:    
Emil   Johnson   died   in   the   City   of   Manila;   however   he   was   a   naturalized   The  hearing  on  said  application  was  set  and  three  weeks  publication  of  notice  
American  citizen.  He  left  a  will  through  which  he  disposed  an  estate  valued  at   was   ordered   in   the   "Manila   Daily   Bulletin."   Witnesses   were   examined   relative  
P231,800.   However,   the   will   was   signed   by   two   witnesses   instead   of   the   3   to  the  execution  of  the  will.  Thereafter  the  document  was  declared  to  be  legal  
required   by   Sec.   618   of   the   Code   of   Civil   Procedure.   However,   a   petition   was   and   was   admitted   to   probate.   Victor   Johnson   was   appointed   sole  
made   stating   that   the   will   was   made   in   conformity   with   US   Law,   thus   valid   in   administrator.  
the   Philippines   as   provided   for   in   Section   636   in   the   Code   of   Civil   Procedure.    
The   will   was   later   probated   and   declared   legal,   however   the   testator's   By   virtue   of   the   will,   the   testator   bequeath   shares   of   the   corporate   stock   in   the  
daughter,  Ebba  Ingeborg  entered  an  appearance,  claiming  that  as  a  legitimate   Johnson-­‐Pickett  Rope  Company  to  his  brother  Victor,  to  his  father  and  mother  
child   she   cannot   be   deprived   of   the   legitime   which   she   is   entitled   to   as   in  Sweden,  to  his  daughter  Ebba  Ingeborg,  to  his  wife,  Alejandra  Ibañez,  if  she  
provided  by  Philippine  law.  She  moved  to  annul  the  decree  of  probate  and  put   remains  single  and  to  Simeona  Ibañez,  spinster,  if  she  remains  single.  The  rest  
the  estate  into  intestate  administration  in  order  for  her  to  claim  the  estate  as   of   the   property   is   left   to   the   testator's   five   children   -­‐   Mercedes,   Encarnacion,  
the  sole  legitimate  heir  of  her  father.   Victor,  Eleonor  and  Alberto.    
   
FACTS:   The  biographical  facts  relative  to  the  deceased  necessary  to  an  understanding  
Emil   H.   Johnson,   a   native   of   Sweden   and   a   naturalized   citizen   of   the   United   of  the  case  are  these:  Emil  H.  Johnson  was  born  in  Sweden,  May  25,  1877,  from  
States,   died   in   the   city   of   Manila,   leaving   a   will   by   which   he   disposed   of   an   which  country  he  emigrated  to  the  United  States  and  lived  in  Chicago,  Illinois,  
estate,   the   value   of   which   was   P231,800.   This   document   is   a   holographic   from   1893   to   1898.   On   May   9,   1898,   at   Chicago,   he   was   married   to   Rosalie  
instrument,   being   written   in   the   testator's   own   handwriting,   and   is   signed   by   Ackeson,  and  immediately  thereafter  embarked  for  the  Philippine  Islands  as  a  
himself  and  two  witnesses  only,  instead  of  three  witnesses  required  by  section   soldier   in   the   Army   of   the   United   States.   As   a   result   of   relations   between  
618   of   the   Code   of   Civil   Procedure.   This   will,   therefore,   was   not   executed   in   Johnson  and  Rosalie  Ackeson  a  daughter,  named  Ebba  Ingeborg,  was  born.  
conformity  with  the  provisions  of  law  generally  applicable  to  wills  executed  by    
inhabitants   of   these   Islands,   and   hence   could   not   have   been   proved   under   After   Johnson   was   discharged   as   a   soldier   from   the   service   of   the   United   States  
section  618.   he  continued  to  live  in  the  Philippine  Islands,  and  on  November  20,  1902,  the  
  wife,  Rosalie  Johnson,  was  granted  a  decree  of  divorce  from  him  in  the  Circuit  
However,   a   petition   was   presented   in   the   CFI   of   the   city   of   Manila   for   the   Court   of   Cook   County,   Illinois,   on   the   ground   of   desertion.   A   little   later   Johnson  
probate  of  this  will,  on  the  ground  that  Johnson  was  at  the  time  of  his  death  a   appeared   in   the   United   States   on   a   visit   and   on   January   10,   1903,   procured   a  
citizen   of   the   State   of   Illinois,   USA;   that   the   will   was   duly   executed   in   certificate   of   naturalization   at   Chicago.   After   a   short   trip   to   Sweden,   the  
accordance  with  the  laws  of  that  State;  and  hence  could  properly  be  probated   deceased  returned  to  Manila,  where  he  prospered  in  business  and  continued  to  
here  pursuant  to  section  636  of  the  Code  of  Civil  Procedure.  This  section  reads   live  until  his  death.  
as  follows:    
  In   this   city   he   appears   to   have   entered   into   marital   relations   with   Alejandra  
Will  made  here  by  alien.  -­‐  A  will  made  within  the  Philippine  Islands  by  a   Ibañez,  by  whom  he  had  three  children:  Mercedes,  Encarnacion  and  Victor.  The  
citizen   or   subject   of   another   state   or   country,   which   is   executed   in   other  two  children  (Eleonor  and  Alberto)  mentioned  in  the  will  were  borne  to  
the  deceased  by  Simeona  Ibañez.  
 
CONFLICT  OF  LAWS                                                                                    AV  DE  TORRES   143  
ATTY.  ARIS  L.  GULAPA                                            AY  2015-­‐2016  
   
About  three   months   after   the   will   had   been  probated,  the  attorneys  for  Ebba   It   does   not   affirmatively   appear   from   the   transaction   of   the   testimony   adduced  
Ingeborg   Johnson   entered   an   appearance   in   her   behalf   and   noted   an   exception   in   the   trial   court   that   any   witness   was   examined   with   reference   to   the   law   of  
to   the   other   admitting   the   will   to   probate.   The   purpose   of   the   proceeding   on   Illinois   on   the   subject   of   the   execution   of   will.   The   trial   judge   no   doubt   was  
behalf   of   the   petitioner   is   to   annul   the   decree   of   probate   and   put   the   estate   satisfied  that  the  will  was  properly  executed  by  examining  section  1874  of  the  
into  intestate  administration,  thus  preparing  the  way  for  the  establishment  of   Revised  Statutes  of  Illinois  and  he  may  have  assumed  that  he  could  take  judicial  
the  claim  of  the  petitioner  as  the  sole  legitimate  heir  of  her  father.  At  the  time   notice  of  the  laws  of  Illinois  under  section  275  of  the  Code  of  Civil  Procedure.  If  
the   court   made   the   order   of   publication,   it   was   apprised   of   the   fact   that   the   so,   he   was   in   our   opinion   mistaken   that   section   authorizes   the   courts   here   to  
petitioner   lived   in   the   United   States   and   that   as   daughter   and   heir   she   was   take   judicial   notice,   among   other   things,   of   the   acts   of   the   legislative  
necessarily   interested   in   the   probate   of   the   will.   It   is,   therefore,   insisted   that   department  of  the  United  States.  These  words  clearly  have  reference  to  Acts  of  
the  court  should  have  appointed  a  date  for  the  probate  of  the  will  sufficiently   the   Congress   of   the   US;   and   we   would   hesitate   to   hold   that   our   courts   can,  
far  in  the  future  to  permit  the  petitioner  to  be  present  either  in  person  or  by   under  this  provision,  take  judicial  notice  of  the  multifarious  laws  of  the  various  
representation;  and  it  is  said  that  the  failure  of  the  court  thus  to  postpone  the   American  States.  Nor  do  we  think  that  any  such  authority  can  be  derived  from  
probate   of   the   will   constitutes   an   infringement   of   that   provision   of   the   the  broader  language,  used  in  the  same  action,  where  it  is  said  that  our  courts  
Philippine   Bill   which   declared   that   property   shall   not   be   taken   without   due   may   take   judicial   notice   of   matters   of   public   knowledge   "similar"   to   those  
process  of  law.   therein   enumerated.   The   proper   rule   we   think   is   to   require   proof   of   the  
  statutes   of   the   States   of   the   American   Union   whenever   their   provisions   are  
The   grounds   upon   which   the   petitioner   seeks   to   avoid   the   probate   are:   (1)   Emil   determinative  of  the  issues  in  any  action  litigated  in  the  Philippine  courts.  
was   a   resident   of   the   city   of   Manila   and   not   a   resident   of   the   State   of   Illinois   at    
the   time   the   will   in   question   was   executed;   (2)   The   will   is   invalid   and   Nevertheless,   even   supposing   that   the   trial   court   may   have   erred   in   taking  
inadequate   to   pass   real   and   personal   property   in   the   State   of   Illinois;   (3)   The   judicial  notice  of  the  law  of  Illinois  on  the  point  in  question,  such  error  is  not  
order  admitting  the  will  to  probate  was  made  without  notice  to  the  petitioner;   now  available  to  the  petitioner,  first,  because  the  petition  does  not  state  any  
and  (4)  The  order  in  question  was  beyond  the  jurisdiction  of  the  court.   fact   from   which   it   would   appear   that   the   law   of   Illinois   is   different   from   what  
  the   court   found,   and,   secondly,   because   the   assignment   of   error   and  
ISSUES:   argument   for   the   appellant   in   this   court   raises   no   question   based   on   such  
Whether  the  testator  was  a  citizen  of  the  State  of  Illinois  and  that  the  will  was   supposed  error.  
executed   in   conformity   with   the   laws   of   that   State   and   was   necessarily   and    
properly  admitted  to  probate   The   petitioner,   it   is   true,   states   in   general   terms   that   the   will   in   question   is  
  invalid   and   inadequate   to   pass   real   and   personal   property   in   the   State   of  
HELD:   Illinois,   but   this   is   merely   a   conclusion   of   law.   The   affidavits   by   which   the  
Yes.   The   proof   adduced   before   the   trial   court   must   therefore   be   taken   as   petition  is  accompanied  contain  no  reference  to  the  subject,  and  we  are  cited  
showing  that,  at  the  time  the  will  was  executed,  the  testator  was,  as  stated  in   to  no  authority  in  the  appellant's  brief  which  might  tend  to  raise  a  doubt  as  to  
the  order  of  probate,  a  citizen  of  the  State  of  Illinois.  This,  in  connection  with   the   correctness   of   the   conclusion   of   the   trial   court.   It   is   very   clear,   therefore,  
the   circumstance   that   the   petition   does   not   even   so   much   as   deny   such   that  this  point  cannot  be  urged  as  of  serious  moment.  
citizenship   but   only   asserts   that   the   testator   was   a   resident   of   the   Philippine    
Islands,  demonstrates  the  impossibility  of  setting  the  probate  aside  for  lack  of   But  it  is  insisted  in  the  brief  for  the  appellant  that  the  will  in  question  was  not  
the   necessary   citizenship   on   the   part   of   the   testator.   But   residence   in   the   properly   admissible   to   probate   because   it   contains   provisions   which   cannot  
Philippine  Islands  is  compatible  with  citizenship  in  Illinois.  As  already  observed,   be   given   effect   consistently   with   the   laws   of   the   Philippine   Islands;   and   it   is  
the   allegation   of   the   petition   on   this   point   is   wholly   insufficient   to   justify   any   suggested  that  as  the  petitioner  is  a  legitimate  heir  of  the  testator  she  cannot  
relief  whatever.   be  deprived  of  the  legitime  to  which  she  is  entitled  under  the  law  governing  
 
CONFLICT  OF  LAWS                                                                                    AV  DE  TORRES   144  
ATTY.  ARIS  L.  GULAPA                                            AY  2015-­‐2016  
testamentary   successions   in   these   Islands.   Upon   this   point   it   is   sufficient   to    
say   that   the   probate   of   the   will   does   not   affect   the   intrinsic   validity   of   its   N.B.:  
provisions,   the   decree   of   probate   being   conclusive   only   as   regards   the   due   Art.   873.   Impossible   conditions   and   those   contrary   to   law   or   good   customs   shall  
execution  of  the  will.   be   considered   as   not   imposed   and   shall   in   no   manner   prejudice   the   heir,   even   if  
  the  testator  should  otherwise  provide.  
If,   therefore,   upon   the   distribution   of   this   estate,   it   should   appear   that   any    
legacy  given  by  the  will  or  other  disposition  made  therein  is  contrary  to  the  law    
applicable  in  such  case,  the  will  must  necessarily  yield  upon  that  point  and  the   FACTS:  
law   must   prevail.   Nevertheless,   it   should   not   be   forgotten   that   the   intrinsic   The   judicial   administrator   of   the   estate   of   deceased   Joseph   Brimo   filed   a  
validity  of  the  provisions  of  this  will  must  be  determined  by  the  law  of  Illinois   scheme  of  partition,  which  was  approved.  Andre  Brimo,  one  of  the  brothers  of  
and   not,   as   the   appellant   apparently   assumes.   It   is   settled   that   in   legal   and   the  deceased,  opposed  it  based  on  the  fact  that  the  partition  in  question  puts  
testamentary  successions,  with  regard  to  the  order  of  succession,  as  well  as  to   into  effect  the  provisions  of  Joseph’s  will  which  are  not  in  accordance  with  the  
the   amount   of   the   successional   rights   and   to   the   intrinsic   validity   of   their   laws   of   his   Turkish   nationality,   for   which   reason   they   are   void   as   being   in  
provisions,   shall   be   regulated   by   the   laws   of   the   nation   of   the   person   whose   violation  or  article  10  (now  article  16)  of  the  Civil  Code.  
succession  is  in  question,  whatever  may  be  the  nature  of  the  property  and  the    
country  where  it  may  be  situate.     ISSUE:  
  Whether  the  approval  of  the  scheme  partition  was  erroneous  
Given   the   matters   discussed   hereinabove,   the   trial   court   committed   no   error   in    
denying  the  relief  sought.     HELD:  
  No,  but  modified  to  include  Andre.  The  law  then  in  force  was  the  Old  Civil  Code  
87.  MICIANO  V.  BRIMO   of   the   Philippines   which,   among   other   things,   provides:   Nevertheless,   legal   and  
G.R.  No.  L-­‐22595  |  November  1,  1927   testamentary   successions,   in   respect   to   the   order   of   succession   as   well   as   to   the  
  amount   of   the   successional   rights   and   the   intrinsic   validity   of   their   provisions,  
DOCTRINE:   The   institution   of   legatees   in   this   will   is   conditional,   and   the   shall   be   regulated   by   the   national   law   of   the   person   whose   succession   is   in  
condition   is   that   the   instituted   legatees   must   respect   the   testator's   will   to   question,  whatever  may  be  the  nature  of  the  property  or  the  country  in  which  it  
distribute  his  property,  not  in  accordance  with  the  laws  of  his  nationality,  but  in   may  be  situated.  
accordance   with   the   laws   of   the   Philippines.   If   this   condition   were   legal   and    
valid,  any  legatee  who  fails  to  comply  with  it,  as  Andre  who,  by  his  attitude  in   The  fact  is  that  Andre  did  not  prove  that  Joseph’s  testamentary  dispositions  are  
these  proceedings  has  not  respected  the  will  of  the  testator,  is  prevented  from   not   in   accordance   with   Turkish   laws,   inasmuch   as   he   did   not   present   any  
receiving  his  legacy.  However,  said  condition  is  void,  being  contrary  to  law,  for   evidence  showing  what  the  Turkish  laws  are  on  the  matter.  In   the   absence   of  
Art  792  of  the  Old  Civil  Code  provides  that  conditions  contrary  to  law  or  good   evidence   on   such   laws,   they   are   presumed   to   be   the   same   as   those   of   the  
morals   shall   be   considered   as   not   imposed   and   shall   not   prejudice   the   heir   or   Philippines.  Andre,  himself,  acknowledges  that  said  laws  have  not  been  proven  
legatee  in  any  manner  whatsoever,  even  if  the  testator  otherwise  provides.   in   these   proceedings   when   he   asked   the   court   to   be   given   an   opportunity   to  
  present   evidence   on   this   point;   so   much   so   that   he   assigns   as   an   error   of   the  
Said   condition   is   contrary   to   law   because   it   expressly   ignores   the   testator's   court  in  not  having  deferred  the  approval  of  the  scheme  of  partition  until  the  
national  law  when,  according  to  Art  10,  such  national  law  of  the  testator  is  the   receipt   of   certain   testimony   requested   regarding   the   Turkish   laws   on   the  
one   to   govern   his   testamentary   dispositions.   Said   condition   then   is   considered   matter.   The   refusal   to   give   Andre   another   opportunity   to   prove   such   laws   does  
unwritten,   and   the   institution   of   legatees   in   said   will   is   unconditional   and   is   not   an   error.   It   is   discretionary   with   the   trial   court,   and,   taking   into  
consequently  valid  and  effective  even  as  to  the  Andre.   consideration   that   Andre   was   granted   ample   opportunity   to   introduce  

 
CONFLICT  OF  LAWS                                                                                    AV  DE  TORRES   145  
ATTY.  ARIS  L.  GULAPA                                            AY  2015-­‐2016  
evidence,   there   was   no   abuse   of   discretion   on   the   part   of   the   court   in   this   it,   and   to   the   condition   imposed   upon   the   legatees,   is   null   and   void,   being  
particular.  There   is,   therefore,   no   evidence   in   the   record   that   the   national   law   contrary   to   law.   All   of   the   remaining   clauses   of   the   will   with   all   their  
of   the   testator   was   violated   in   the   testamentary   dispositions   in   question   dispositions   and   requests   are   perfectly   valid   and   effective   it   not   appearing   that  
which,   not   being   contrary   to   our   laws   in   force,   must   be   complied   with   and   said  clauses  are  contrary  to  the  testator's  national  law.  
executed.   Therefore,   the   approval   of   the   scheme   of   partition   in   this   respect   Therefore,  the  orders  appealed  from  were  modified  and  distribution  of  Joseph  
was  not  erroneous.   Brimo’s  estate  was  directed  to  be  made  in  such  a  manner  as  to  include  Andre  
  Brimo   as   one   of   the   legatees,   and   the   scheme   of   partition   submitted   by   the  
In   regard   to   the   1st   assignment   of   error,   which   deals   with   the   exclusion   of   Miciano  was  approved  in  all  other  respects.  
Andre  as  a  legatee,  inasmuch  as  he  is  one  of  the  persons  designated  as  such  in    
will,  it  must  be  taken  into  consideration  that  such  exclusion  is  based  on  the  last   88.  VALERA  V.  CALDERON  
part  of  the  2nd  clause  of  the  will,  which  says:   G.R.  No.  L-­‐36342  |  October  8,  1932  
   
Second.   I   like   desire   to   state   that   although   by   law,   I   am   a   Turkish   citizen,   this   DOCTRINE:  The  court  which  originally  took  cognizance  of  the  case  decided  that  
citizenship  having  been  conferred  upon  me  by  conquest  and  not  by  free  choice,   such  circumstance  does  not  invalidate  the  will.  We  concur  in  said  opinion  and  
nor   by   nationality   and,   on   the   other   hand,   having   resided   for   a   considerable   hold  that  a  clause  drawn  up  in  such  manner  is  superfluous  and  does  not  affect  
length   of   time   in   the   Philippine   Islands   where   I   succeeded   in   acquiring   all   of   the   in   any   way   the   essential   requisites   prescribed   for   holographic   wills   by   the  
property   that   I   now   possess,   it  is  my  wish  that  the  distribution  of  my  property   French  law,  and,  consequently,  it  has  not  invalidated  the  will  nor  deprived  it  of  
and   everything   in   connection   with   this,  my   will,   be   made   and   disposed   of   in   its  holographic  character.  In  reaching  this  conclusion,  we  base  our  opinion  not  
accordance   with   the   laws   in   force   in   the   Philippine   islands,   requesting   all   of   only   on   the   clear   and   conclusive   provisions   of   article   970   of   the   French   Civil  
my   relatives   to   respect   this   wish,   otherwise,   I   annul   and   cancel   beforehand   Code   and   on   the   decisions   of   the   French   CA   cited   in   the   appelee's   brief,   but  
whatever  disposition  found  in  this  will  favorable  to  the  person  or  persons  who   principally   on   the   fact   established   in   the   depositions   made   by   practicing  
fail  to  comply  with  this  request.   attorneys   F.   de   Roussy   de   Sales,   Gething   C.   Miller   and   Henri   Gadd   of   Paris,  
  France,   who   emphatically   declared   that   the   will   in   question   did   not   lose   its  
The  institution  of  legatees  in  this  will  is  conditional,  and  the  condition  is  that   holographic  character  by  the  addition  of  the  aforementioned  attestation  clause  
the   instituted   legatees   must   respect   the   testator's   will   to   distribute   his   and  that  it  may  be  allowed  to  probate  in  conformity  with  the  French  laws  under  
property,  not  in  accordance  with  the  laws  of  his  nationality,  but  in  accordance   which  it  had  been  made  and  executed.  
with   the   laws   of   the   Philippines.   If   this   condition   were   legal   and   valid,   any    
legatee   who   fails   to   comply   with   it,   as   Andre   who,   by   his   attitude   in   these   FACTS:  
proceedings   has   not   respected   the   will   of   the   testator,   is   prevented   from   The   deceased   Francisco   Varela   Calderon,   a   physician   by   profession,   was   a  
receiving   his   legacy.   However,   said   condition   is   void,   being   contrary   to   law,   Filipino   citizen   resident   of   the   City   of   Manila   where   he   owned   real   properties  
for  Art  792  of  the  Old  Civil  Code  provides  that  conditions  contrary  to  law  or   assessed   at   P188,017.81.   He   traveled   abroad   for   his   health   and   temporarily  
good   morals   shall   be   considered   as   not   imposed   and   shall   not   prejudice   the   resided   in   Hendaye-­‐Plage,   France.   Not   feeling   very   well,   but   in   the   full  
heir   or   legatee   in   any   manner   whatsoever,   even   if   the   testator   otherwise   enjoyment   of   his   mental   faculties,   he   decided   to   make   his   last   will   and  
provides.   testament,  on  April  14,  1930,  in  Paris,  France,  with  the  assistance  of  attorneys  
Said   condition   is   contrary   to   law   because   it   expressly   ignores   the   testator's   F.   de   Roussy   de   Sales,   Gething   C.   Miller   and   Henri   Gadd.   Sometime   later,   he  
national   law   when,   according   to   Art   10,   such   national   law   of   the   testator   is   died  in  the  Grand-­‐Hotel  de  Leysin  Sanatorium  in  Switzerland.  
the   one   to   govern   his   testamentary   dispositions.   Said   condition   then   is    
considered   unwritten,   and   the   institution   of   legatees   in   said   will   is   On   September   20,   1930,   the   herein   petitioner-­‐appellee,   Francisco   Carmelo  
unconditional   and   consequently   valid   and   effective   even   as   to   the   Andre.   Varela,  filed  a  petition  in  the  CFI  of  the  City  of  Manila,  praying  that  said  will  be  
From  all  this,  the  second  clause  of  the  will  regarding  the  law  which  shall  govern  
 
CONFLICT  OF  LAWS                                                                                    AV  DE  TORRES   146  
ATTY.  ARIS  L.  GULAPA                                            AY  2015-­‐2016  
admitted   to   probate.   Said   petition   was   opposed   by   the   deceased's   brother   allowed   and   admitted   to   probate   therein,   may,   also   be   proved,   allowed   and  
Angel,   Jesus,   Trinidad,   Paula,   Pilar   and   Maria,   surnamed   Varela   Calderon,   on   recorded  in  the  Philippine  Islands  in  the  same  manner  and  with  the  same  effect  
the   grounds:   (a)   That   the   will   sought   to   be   probate   was   not   holographic   in   as  if  executed  in  the  latter  country.  Both  provisions  of  law  literally  copied  from  
character   and   did   not   comply   with   the   requisites   prescribed   by   article   970   of   the  English  text,  read  as  follows:  
the   French   Civil   Code;   (b)   that   the   witnesses   to   the   will   did   not   possess   the    
qualifications   required   by   article   980   of   the   French   Civil   Code;   (c)   that   for   not   Article   970,   French   Civil   Code   à   A   holographic   will   is   not   valid   unless   it   is  
having   complied   with   the   requisites   prescribed   by   the   French   law,   said   will   is   entirely   written,   dated,   and   signed   by   the   testator.   No   other   formality   is  
null  and  void;  (d)  that  neither  has  it  the  character  of  an  open  will,  not  having   required.  
been  executed  in  accordance  with  article  1001  of  the  French  Civil  Code;  and  (e)   Article  635,  Code  of  Civil  Procedure  à  Will  made  out  of  the  Philippine  Islands.  
that  the  provisions  of  article  1007  of  the  same  Code  relative  to  the  recording  of   —  A  will  made  out  of  the  Philippine  Islands  which  might  be  proved  and  allowed  
wills  were  not  complied  with  in  connection  with  the  will  in  question.   by   the   laws   of   the   state   or   country   in   which   it   was   made,   may   be   proved,  
  allowed,  and  recorded  in  the  Philippine  Islands,  and  shall  have  the  same  effect  
The   original   will   was   executed   in   the   French   language   and   had   been   written,   as  if  executed  according  to  the  laws  of  these  Islands.  
dated  and  signed  by  the  testator  with  his  own  hand,  with  the  exception  of  the    
attestation   clause   which   appears   at   the   bottom   of   the   document.   This   fact   is   It   is   an   admitted   fact   that   the   will   was   written,   dated   and   signed   by   the  
proved  by  the  testimony  of  the  appellee  and  his  other  witnesses,  including  the   deceased  testator,  for  which  reason,  there  is  no  doubt  that  it  had  been  made  
depositions,  and  is  admitted  by  the  appellants.   and   executed   in   accordance   with   article   970   of   the   French   Civil   Code   were   it  
  not  for  the  attestation  clause  which  appears  at  the  bottom  of  the  document.  
The   petition   for   the   allowance   and   probate   of   said   will   is   based   on   the    
provisions  of  article  970  of  the  French  Civil  Code,  to  wit:  “A  holographic  will  is   The  appellants  contend  that  the  addition  of  said  of  clause  has  entirely  vitiated  
not   valid   unless   it   is   entirely   written,   dated,   and   signed   by   the   testator.   No   the  will,  because  it  ceased  to  be  a  holographic  will,  neither  does  it  possess  the  
other   formality   is   required.”   and   on   section   635   of   the   Code   of   Civil   Procedure,   requisites   of   a   public   or   open   will   in   accordance   with   the   French   law.  The  court  
to   wit:   “Will   made   out   of   the   Philippine   Islands.   —   A   will   made   out   of   the   which  originally  took  cognizance  of  the  case  decided  that  such  circumstance  
Philippine  Islands  which  might  be  proved  and  allowed  by  the  laws  of  the  state   does   not   invalidate   the   will.   We   concur   in   said   opinion   and  hold   that   a   clause  
or  country  in  which  it  was  made,  may  be  proved,  allowed,  and  recorded  in  the   drawn  up  in  such  manner  is  superfluous  and  does  not  affect  in  any  way  the  
Philippine   Islands,   and   shall   have   the   same   effect   as   if   executed   according   to   essential   requisites   prescribed   for   holographic   wills   by   the   French   law,   and,  
the  laws  of  these  Islands.”   consequently,   it   has   not   invalidated   the   will   nor   deprived   it   of   its   holographic  
  character.   In   reaching   this   conclusion,   we   base   our   opinion   not   only   on   the  
ISSUE:   clear  and  conclusive  provisions  of  article  970  of  the  French  Civil  Code  and  on  
Whether  the  holographic  will  was  made  and  executed,  in  accordance  with  the   the  decisions  of  the  French  Court  of  Appeals  cited  in  the  appelee's  brief,  but  
laws  of  the  French  Republic  and  thus,  validly  allowed  and  admitted   principally   on   the   fact   established   in   the   depositions   made   by   practicing  
  attorneys   F.   de   Roussy   de   Sales,   Gething   C.   Miller   and   Henri   Gadd   of   Paris,  
HELD:   France,   who   emphatically   declared   that   the   will   in   question   did   not   lose   its  
Yes.   The   petition   for   the   allowance   and   probate   of   said   will   is   based   on   the   holographic   character   by   the   addition   of   the   aforementioned   attestation  
provisions   of   article   970   of   the   French   Civil   Code   which   considers   as   a   clause   and   that   it   may   be   allowed   to   probate   in   conformity   with   the   French  
holographic   will   that   which   is   made   or   executed,   dated   and   signed   by   the   laws  under  which  it  had  been  made  and  executed.  
testator   in   his   own   handwriting   without   the   necessity   of   any   other   formality,    
and   on   section   635   of   the   Code   of   Civil   Procedure   in   force   in   this   jurisdiction   89.  GIBBS  V.  GOVERNMENT  
which  provides  that  a  will  made  out  of  the  Philippine  Islands  in  accordance  with   G.R.  No.  L-­‐35694  |  December  23,  1933  
the   laws   in   force   in   the   country   in   which   it   was   made   and   which   may   be    
 
CONFLICT  OF  LAWS                                                                                    AV  DE  TORRES   147  
ATTY.  ARIS  L.  GULAPA                                            AY  2015-­‐2016  
Kyna’s  notes:   partnership  of  Allison  D.  Gibbs  and  Eva  Johnson  Gibbs;  that  Eva  died  intestate  
The   issue   involved   is   tax   on   inheritance.   Thus,   if   the   issue   of   the   case   is   not   in   Palo   Alto,   California,   on   November   28,   1929;   that   at   the   time   of   her   death  
purely  personal,  do  not  apply  Article  16  (par  2).   she   and   her   husband   were   citizens   of   the   State   of   California   and   domiciled  
  therein.    
Av:   Under   California   Law,   spouses   only   have   inchoate   rights   over   properties    
acquired   jointly.   Thus,   upon   the   death   of   the   wife,   the   entire   community   It  appears  further  from  said  order  that  Allison  was  appointed  administrator  of  
property   without   administration   belongs   to   the   surviving   husband;   that   he   is   the  state  of  his  said  deceased  wife  in  a  case  in  the  same  court,  entitled  "In  the  
the   absolute   owner   of   all   the   community   property   from   the   moment   of   the   Matter   of   the   Intestate   Estate   of   Eva   Johnson   Gibbs,   Deceased";   that   in  
death   of   his   wife,   not   by   virtue   of   succession   or   by   virtue   of   her   death,   but   by   intestate  proceedings,  Allison  on  September  22,  1930,  filed  an  ex  parte  petition  
virtue   of   the   fact   that   when   the   death   of   the   wife   precedes   that   of   the   husband   in  which  he  alleged  "that  the  parcels  of  land  hereunder  described  belong  to  the  
he  acquires  the  community  property,  not  as  an  heir  or  as  the  beneficiary  of  his   conjugal  partnership  of  Allison  and  his  wife,  Eva",  describing  in  detail  the  3  facts  
deceased   wife,   but   because   she   never   had   more   than   an   inchoate   interest   or   here  involved;  and  further  alleging  that  his  said  wife,  a  citizen  and  resident  of  
expentancy  which  is  extinguished  upon  her  death.   California,   died   on   November   28,   1929;   that   in   accordance   with   the   law   of  
  California,  the  community  property  of  spouses  who  are  citizens  of  California,  
However,  since  the  Phil.  Adheres  to  the  lex  situs  rule  with  regard  to  properties   upon   the   death   of   the   wife   previous   to   that   of   the   husband,   belongs  
and   the   properties  were  in  the  Phil.,   under  Phil.  Law,   both   spouses   are   absolute   absolutely  to  the  surviving  husband  without  administration;  that  the  conjugal  
owners  of  the  property.  It  results  that  the  wife  of  Allison  was,  by  the  law  of  the   partnership  of  Allison  and  Eva  Gibbs,  deceased,  has  no  obligations  or  debts  and  
Philippine   Islands,   vested   of   a   descendible   interest,   equal   to   that   of   her   no   one   will   be   prejudiced   by   adjudicating   said   parcels   of   land   (and   17   others  
husband,  in  the  Philippine  lands.   not   here   involved)   to   be   the   absolute   property   of   Allison   as   sole   owner.   The  
  court   granted   said   petition   and   on   September   22,   1930,   entered   a   decree  
DOCTRINE:  The  issue  in  this  case  is  tax  on  inheritance  not  capacity  to  succeed,   adjucating   to   Allison   as   the   sole   and   absolute   owner   of   said   lands,   applying  
intrinsic   validity,   amount   of   successional   rights,   or   order   of   succession   [CIAO].   section  1401  of  the  Civil  Code  of  California.  Gibbs  presented  this  decree  to  the  
Hence,  Art.  16(2)  which  prescribes  national  law  should  not  have  been  applied.   register   of   deeds   of   Manila   and   demanded   that   the   latter   issue   to   him   a   TCT.  
(The   California   Law   in   this   case   states   that   there   was   no   inheritance.)   The   Acting   upon   the   authority   of   Section   1547   of   Article   XI   of   Chapter   40   of   the  
issue—which   is   inheritance   tax—not   being   purely   personal,   Art.   16(2)   should   Administrative   Code,   the   register   of   deeds   refused   to   register   the   transfer   of  
not  be  applied.  Rather,  Lex  Situs  should  govern.  (The  Phil.  Law  provides  that  in   title   of   the   said   conjugal   property   to   Allison   on   the   ground   that   the  
this  case,  there  was  inheritance.)   corresponding   inheritance   tax   had   not   been   paid.   Thereupon,   under   date   of  
  December   26,   1930,   Allison   filed   in   the   said   court   a   petition   for   an   order  
FACTS:   requiring  the  said  register  of  deeds  "to  issue  the  corresponding  titles"  to  Gibbs  
This  is  an  appeal  from  a  final  order  of  the  CFI  Manila,  requiring  the  register  of   without   requiring   previous   payment   of   any   inheritance   tax.   After   due   hearing  
deeds   of   the   City   of   Manila   to   cancel   certificates   of   title   Nos.   20880,   28336   and   of   the   parties,   the   court   reaffirmed   said   order   of   September   22,   1930,   and  
28331,  covering  lands  located  in  the  City  of  Manila,  Philippine  Islands,  and  issue   entered  the  order  of  March  10,  1931,  which  is  under  review  on  this  appeal.    
in   lieu   thereof   new   certificates   of   transfer   of   title   in   favor   of   Allison   D.   Gibbs    
without   requiring   him   to   present   any   document   showing   that   the   succession   For   the   purposes   of   this   case,   the   Court   considered   the   following   facts   as  
tax   due   under   Article   XI   of   Chapter   40   of   the   Administrative   Code   has   been   established  by  the  evidence  or  the  admissions  of  the  parties:  Allison  has  been  
paid.     continuously,   since   the   year   1902,   a   citizen   of   the   State   of   California   and  
  domiciled   therein;   that   he   and   Eva   were   married   at   Columbus,   Ohio,   in   July  
The   order   of   the   court   of   March   10,   1931,   recites   that   the   parcels   of   land   1906;  that  there  was  no  ante-­‐nuptial  marriage  contract  between  the  parties.    
covered   by   said   certificates   of   title   formerly   belonged   to   the   conjugal    
Article   XI   of   Chapter   40   of   the   Administrative   Code   entitled   "Tax   on  
 
CONFLICT  OF  LAWS                                                                                    AV  DE  TORRES   148  
ATTY.  ARIS  L.  GULAPA                                            AY  2015-­‐2016  
inheritances,  legacies  and  other  acquisitions  mortis  causa"  provides  in  section   grants   practical   autonomy   to   the   Government   of   the   Philippine   Islands.   This  
1536   that   "Every   transmission   by   virtue   of   inheritance   ...   of   real   property   ...   Government,   therefore,   may   apply   the   principles   and   rules   of   private  
shall  be  subject  to  the  following  tax."     international   law   (conflicts   of   laws)   on   the   same   footing   as   an   organized  
  territory   or   state   of   the   United   States.   We  should,  therefore,  resort  to  the  law  
ISSUES:   of   California,   the   nationality   and   domicile   of   Mrs.   Gibbs,   to   ascertain   the  
1.   Whether   Eva   Johnson   Gibbs   at   the   time   of   her   death   is   the   owner   of   a   norm  which  would  be  applied  here  as  law  were  there  any  question  as  to  her  
descendible  interest  in  the  Philippine  lands  above-­‐mentioned?   status.    
2.  Whether  the  Register  of  Deeds  erred  in  declining  to  register  the  transfer  title    
of  the  conjugal  property  on  the  ground  of  unpaid  inheritance  tax?  (NO)       But   the   appellant's   chief   argument   and   the   sole   basis   of   the   lower   court's  
nd
  decision  rests  upon  the  2  paragraph  of  article  10  of  the  Civil  Code  which  is  as  
HELD:   follows:   Nevertheless,   legal   and   testamentary   successions,   in   respect   to   the  
1.  Yes.  Allison  contends  that  the  law  of  California  should  determine  the  nature   order  of  succession  as  well  as  to  the  amount  of  the  successional  rights  and  the  
and   extent   of   the   title,   if   any,   that   vested   in   Eva   under   the   3   certificates   of   title   intrinsic  validity  of  their  provisions,  shall  be  regulated  by  the  national  law  of  the  
Nos.   20880,   28336   and   28331   above   referred   to,   citing   article   9   of   the   Civil   person   whose   succession   is   in   question,   whatever   may   be   the   nature   of   the  
Code  (The  laws  relating  to  family  rights  and  duties,  or  to  the  status,  condition,   property  or  the  country  in  which  it  may  be  situated.    
and   legal   capacity   of   persons,   are   binding   upon   Spaniards   even   though   they    
reside  in  a  foreign  country.)  But  that,  even  if  the  nature  and  extent  of  her  title   In   construing   the   above   language   the   Court   met   at   the   outset   with   some  
under   said   certificates   be   governed   by   the   law   of   the   Philippine   Islands,   the   difficulty   by   the   expression   "the   national   law   of   the   person   whose   succession   is  
nd
laws  of  California  govern  the  succession  to  such  title,  citing  the  2  paragraph  of   in   question",   by   reason   of   the   rather   anomalous   political   status   of   the  
article  10  of  the  Civil  Code.     Philippine  Islands.  The  Court  encountered  no  difficulty  in  applying  article  10  in  
  the  case  of  a  citizen  of  Turkey.  (Miciano  v.  Brimo)  Having  regard  to  the  practical  
It   is   argued   that   the   conjugal   right   of   the   California   wife   in   community   real   autonomy  of  the  Philippine  Islands,  as  above  stated,  the  Court  concluded  that  if  
estate   in   the   Philippine   Islands   is   a   personal   right   and   must,   therefore,   be   article   10   is   applicable   and   the   estate   in   question   is   that   of   a   deceased  
settled  by  the  law  governing  her  personal  status,  that  is,  the  law  of  California.   American   citizen,   the   succession   shall   be   regulated   in   accordance   with   the  
But  our  attention  has  not  been  called  to  any  law  of  California  that  incapacitates   norms  of  the  State  of  his  domicile  in  the  United  States.  
a   married   woman   from   acquiring   or   holding   land   in   a   foreign   jurisdiction   in    
accordance   with   the   lex   rei   sitae.   There   is   not   the   slightest   doubt   that   a   The  trial  court  found  that  under  the  law  of  California,  upon  the  death  of  the  
California  married  woman  can  acquire  title  to  land  in  a  common  law  jurisdiction   wife,   the   entire   community   property   without   administration   belongs   to   the  
like  the  State  of  Illinois  or  the  District  of  Columbia,  subject  to  the  common-­‐law   surviving   husband;   that   he   is   the   absolute   owner   of   all   the   community  
estate  by  the  courtesy  which  would  vest  in  her  husband.  Nor  is  there  any  doubt   property   from   the   moment   of   the   death   of   his   wife,   not   by   virtue   of  
that  if  a  California  husband  acquired  land  in  such  a  jurisdiction  his  wife  would   succession  or  by  virtue  of  her  death,  but  by  virtue  of  the  fact  that  when  the  
be   vested   with   the   common   law   right   of   dower,   the   prerequisite   conditions   death   of   the   wife   precedes   that   of   the   husband   he   acquires   the   community  
obtaining.   Article   9   of   the   Civil   Code   treats   of   purely   personal   relations   and   property,   not   as   an   heir   or   as   the   beneficiary   of   his   deceased   wife,   but  
status   and   capacity   for   juristic   acts,   the   rules   relating   to   property,   both   because  she  never  had  more  than  an  inchoate  interest  or  expentancy  which  is  
personal   and   real,   being   governed   by   article   10   of   the   Civil   Code.   extinguished   upon   her   death.  Quoting  the  case  of  Estate  of  Klumpke,  the  court  
Furthermore,   article   9,   by   its   very   terms,   is   applicable   only   to   "Spaniards"   said:   "The   decisions   under   this   section   (1401   Civil   Code   of   California)   are  
(now,  by  construction,  to  citizens  of  the  Philippine  Islands).     uniform  to  the  effect  that  the  husband  does  not  take  the  community  property  
  upon   the   death   of   the   wife   by   succession,   but   that   he   holds   it   all   from   the  
The   Organic   Act   of   the   Philippine   Islands   (Act   of   Congress,   August   29,   1916,   moment  of  her  death  as  though  required  by  himself.  ...  It  never  belonged  to  the  
known   as   the   "Jones   Law")   as   regards   the   determination   of   private   rights,   estate  of  the  deceased  wife."    
 
CONFLICT  OF  LAWS                                                                                    AV  DE  TORRES   149  
ATTY.  ARIS  L.  GULAPA                                            AY  2015-­‐2016  
  provides  that  "the  husband  may  dispose  by  will  of  his  half  only  of  the  property  
The  argument  of  Allison  apparently  leads  to  this  dilemma:  If  he  takes  nothing   of   the   conjugal   partnership."   Article   1426   provides   that   upon   dissolution   of   the  
nd  
by  succession  from  his  deceased  wife,  how  can  the  2 paragraph  of  article  10   conjugal   partnership   and   after   inventory   and   liquidation,   "the   net   remainder   of  
be  invoked?  Can  Allison  be  heard  to  say  that  there  is  a  legal  succession  under   the   partnership   property   shall   be   divided   share   and   share   alike   between   the  
the   law   of   the   Philippine   Islands   and   no   legal   succession   under   the   law   of   husband  and  wife,  or  their  respective  heirs."  Under  the  provisions  of  the  Civil  
nd  
California?  It  seems  clear  that  the  2 paragraph  of  article  10  applies  only  when   Code   and   the   jurisprudence   prevailing   here,   the   wife,   upon   the   acquisition   of  
a   legal   or   testamentary   succession   has   taken   place   in   the   Philippines   and   in   any  conjugal  property,  becomes  immediately  vested  with  an  interest  and  title  
accordance   with   the   law   of   the   Philippine   Islands;   and   the   foreign   law   is   therein  equal  to  that  of  her  husband,  subject  to  the  power  of  management  and  
consulted   only   in   regard   to   the   order   of   succession   or   the   extent   of   the   disposition   which   the   law   vests   in   the   husband.   Immediately   upon   her   death,   if  
nd  
successional   rights;   in   other   words,   the   2 paragraph   of   article   10   can   be   there   are   no   obligations   of   the   decedent,   as   is   true   in   the   present   case,   her  
invoked   only   when   the   deceased   was   vested   with   a   descendible   interest   in   share   in   the   conjugal   property   is   transmitted   to   her   heirs   by   succession.  
property  within  the  jurisdiction  of  the  Philippine  Islands.     (Articles  657,  659,  661,  Civil  Code)    
   
In  the  case  of  Clarke  v.  Clarke,  the  court  said:  It  is  principle  firmly  established   It   results   that   the   wife   of   Allison   was,   by   the   law   of   the   Philippine   Islands,  
that  to  the  law  of  the  state  in  which  the  land  is  situated  we  must  look  for  the   vested  of  a  descendible  interest,  equal  to  that  of  her  husband,  in  the  Philippine  
rules  which  govern  its  descent,  alienation,  and  transfer,  and  for  the  effect  and   lands   covered   by   certificates   of   title   Nos.   20880,   28336   and   28331,   from   the  
construction   of   wills   and   other   conveyances.   This   fundamental   principle   is   date  of  their  acquisition  to  the  date  of  her  death.  Allison  himself  believed  that  
stated   in   the   first   paragraph   of   article   10   of   our   Civil   Code   as   follows:   "Personal   his   wife   was   vested   of   such   a   title   and   interest   in   manifest   from   the   end   of   said  
property  is  subject  to  the  laws  of  the  nation  of  the  owner  thereof;  real  property   certificates,  No.  28336,  dated  May  14,  1927,  introduced  by  him  in  evidence,  in  
to  the  laws  of  the  country  in  which  it  is  situated.     which   it   is   certified   that   "the   spouses   Allison   D.   Gibbs   and   Eva   Johnson   Gibbs  
  are  the  owners  in  fee  simple  of  the  conjugal  lands  therein  described."    
It   is   stated   in   5   Cal.   Jur.,   478:   In   accord   with   the   rule   that   real   property   is    
subject  to  the  lex  rei  sitae,  the  respective  rights  of  husband  and  wife  in  such   The  descendible  interest  of  Eva  in  the  lands  aforesaid  was  transmitted  to  her  
property,   in   the   absence   of   an   antenuptial   contract,   are   determined   by   the   heirs   by   virtue   of   inheritance   and   this   transmission   plainly   falls   within   the  
law  of  the  place  where  the  property  is  situated,  irrespective  of  the  domicile  of   language   of   section   1536   of   Article   XI   of   Chapter   40   of   the   Administrative  
the  parties  or  to  the  place  where  the  marriage  was  celebrated.     Code  which  levies  a  tax  on  inheritances.  It  is  unnecessary  in  this  proceeding  to  
  determine  the  "order  of  succession"  or  the  "extent  of  the  successional  rights"  
Under   this   broad   principle,   the   nature   and   extent   of   the   title   which   vested   in   (article  10,  Civil  Code,  supra)  which  would  be  regulated  by  section  1386  of  the  
Mrs.   Gibbs   at   the   time   of   the   acquisition   of   the   community   lands   here   in   Civil   Code   of   California   which   was   in   effect   at   the   time   of   the   death   of   Mrs.  
question  must  be  determined  in  accordance  with  the  lex  rei  sitae.  It  is  admitted   Gibbs.    
that   the   Philippine   lands   here   in   question   were   acquired   as   community    
property  of  the  conjugal  partnership  of  Allison  and  his  wife.  Under  the  law  of   The   record   does   not   show   what   the   proper   amount   of   the   inheritance   tax   in  
the   Philippine   Islands,   she   was   vested   of   a   title   equal   to   that   of   her   husband.   this   case   would   be   nor   that   Allison   in   any   way   challenged   the   power   of   the  
Article  1407  of  the  Civil  Code  provides:    All  the  property  of  the  spouses  shall  be   Government   to   levy   an   inheritance   tax   or   the   validity   of   the   statute   under  
deemed  partnership  property  in  the  absence  of  proof  that  it  belongs  exclusively   which   the   register   of   deeds   refused   to   issue   a   certificate   of   transfer   reciting  
to  the  husband  or  to  the  wife.     that   Allison   is   the   exclusive   owner   of   the   Philippine   lands   included   in   the   3  
  certificates  of  title  here  involved.    
Article   1395   provides:   "The   conjugal   partnership   shall   be   governed   by   the   rules    
of   law   applicable   to   the   contract   of   partnership   in   all   matters   in   which   such   The  judgment  of  the  court  below  of  March  10,  1931,  is  reversed  with  directions  
rules  do  not  conflict  with  the  express  provisions  of  this  chapter."  Article  1414   to  dismiss  the  petition,  without  special  pronouncement  as  to  the  costs.    
 
CONFLICT  OF  LAWS                                                                                    AV  DE  TORRES   150  
ATTY.  ARIS  L.  GULAPA                                            AY  2015-­‐2016  
  upon  the  filing  of  a  bond  in  the  sum  of  P10,000.00,  let  letters  testamentary  be  
90.  PHILIPPINE  TRUST  V.  BOHANAN   issued   and   after   taking   the   prescribed   oath,   it   may   enter   upon   the   execution  
G.R.  No.  L-­‐12105  |  30  January  1960   and  performance  of  its  trust.    
   
DOCTRINES:  As   in   accordance   with   Article   10   of   the   old   Civil   Code,   the   validity   of   It   does   not   appear   that   the   order   granting   probate   was   ever   questioned   on  
testamentary   dispositions   are   to   be   governed   by   the   national   law   of   the   appeal.   The   executor   filed   a   project   of   partition   dated   January   24,   1956,  
testator,   and   as   it   has   been   decided   and   it   is   not   disputed   that   the   national   law   making,   in   accordance   with   the   provisions   of   the   will,   the   following  
of  the  testator  is  that  of  the  State  of  Nevada,  which  allows  a  testator  to  dispose   adjudications:   (1)   one-­‐half   of   the   residuary   estate,   to   the   Farmers   and  
of   all   his   property   according   to   his   will,   as   in   the   case   at   bar,   the   order   of   the   Merchants  National  Bank  of  Los  Angeles,  California,  U.S.A.  in  trust  only  for  the  
court   approving   the   project   of   partition   made   in   accordance   with   the   benefit   of   testator's   grandson   Edward   George   Bohanan,   which   consists   of  
testamentary   provisions,   must   be,   as   it   is   hereby   affirmed,   with   costs   against   several   mining   companies;   (2)   the   other   half   of   the   residuary   estate   to   the  
appellants.     testator's   brother,   F.L.   Bohanan,   and   his   sister,   Mrs.   M.   B.   Galbraith,   share  
  and   share   alike.   This   consist   in   the   same   amount   of   cash   and   of   shares   of  
Since  no  right  to  share  in  the  inheritance  in  favor  of  a  divorced  wife  exists  in  the   mining  stock  similar  to  those  given  to  testator's  grandson;  (3)  legacies  of  P6,000  
State  of  Nevada  and  since  the  court  below  had  already  found  that  there  was  no   each  to  his  (testator)  son,  Edward  Gilbert  Bohana,  and  his  daughter,  Mary  Lydia  
conjugal   property   between   the   testator   and   Magdalena,   the   latter   can   now   Bohanan,  to  be  paid  in  three  yearly  installments;  (4)  legacies  to  Clara  Daen,  in  
have  no  longer  claim  to  pay  portion  of  the  estate  left  by  the  testator.   the  amount  of  P10,000.00;  Katherine  Woodward,  P2,000;  Beulah  Fox,  P4,000;  
  and  Elizabeth  Hastings,  P2,000;  It  will  be  seen  from  the  above  that  out  of  the  
FACTS:   total   estate   (after   deducting   administration   expenses)   of   P211,639.33   in   cash,  
On   April   24,   1950,   the   CFI   of   Manila,   admitted   to   probate   a   last   will   and   the   testator   gave   his   grandson   P90,819.67   and   one-­‐half   of   all   shares  of   stock   of  
testament  of  C.  O.  Bohanan,  executed  by  him  on  April  23,  1944  in  Manila.     several   mining   companies   and   to   his   brother   and   sister   the   same   amount.   To  
  his  children  he  gave  a  legacy  of  only  P6,000  each,  or  a  total  of  P12,000.    
According  to  the  evidence  of  the  opponents  the  testator  was   born  in  Nebraska    
and   therefore   a   citizen   of   that   state,   or  at  least  a  citizen   of   California   where   The   wife   Magadalena   C.   Bohanan   and   her   two   children   questioned   the  
some   of   his   properties   are   located.   This   contention   in   untenable.   validity   of   the   testamentary   provisions,   claiming   that   they   have   been  
Notwithstanding  the  long  residence  of  the  decedent  in  the  Philippines,  his  stay   deprived  of  the  legitime  that  the  laws  of  the  form  concede  to  them.    
here  was  merely  temporary,  and  he  continued  and  remained  to  be  a  citizen  of    
the  US  and  of  the  state  of  his  pertinent  residence  to  spend  the  rest  of  his  days   ISSUE:  
in  that  state.  His  permanent  residence  or  domicile  in  the  US  depended  upon   Whether  Magdalena  and  her  two  children  have  been  deprived  of  the  legitime  
his   personal   intent   or   desire,   and   he   selected   Nevada   as   his   domicile   and   due  to  them?    
therefore  at  the  time  of  his  death,  he  was  a  citizen  of  that  state.  Nobody  can    
choose   his   domicile   or   permanent   residence   for   him.   That   is   his   exclusive   HELD:  
personal  right.     No.   The   first   question   refers   to   the   share   that   the   wife   of   the   testator,  
  Magdalena  C.  Bohanan,  should  be  entitled  to  receive.  The  will  has  not  given  her  
Wherefore,  the  court  finds  that  the  testator  C.  O.  Bohanan  was  at  the  time  of   any   share   in   the   estate   left   by   the   testator.   It   is   argued   that   it   was   error   for   the  
his  death  a  citizen  of  the  US  and  of  the  State  of  Nevada  and  declares  that  his   trial  court  to  have  recognized  the  Reno  divorce  secured  by  the  testator  from  his  
will  and  testament,  is  fully  in  accordance  with  the  laws  of  the  state  of  Nevada   Filipino  wife  Magdalena  C.  Bohanan,  and  that  said  divorce  should  be  declared  a  
and   admits   the   same   to   probate.   Accordingly,   the   Philippine   Trust   Company,   nullity  in  this  jurisdiction.    
named   as   the   executor   of   the   will,   is   hereby   appointed   to   such   executor   and    
The  court  below  refused  to  recognize  the  claim  of  the  widow  on  the  ground  
 
CONFLICT  OF  LAWS                                                                                    AV  DE  TORRES   151  
ATTY.  ARIS  L.  GULAPA                                            AY  2015-­‐2016  
that   the   laws   of   Nevada,   of   which   the   deceased   was   a   citizen,   allow   him   to   successions,  in  respect  to  the  order  of  succession  as  well  as  to  the  extent  of  the  
dispose  of  all  of  his  properties  without  requiring  him  to  leave  any  portion  of   successional   rights   and   the   intrinsic   validity   of   their   provisions,   shall   be  
his   estate   to   his   wife.   Section   9905   of   Nevada   Compiled   Laws   of   1925   regulated   by   the   national   law   of   the   person   whose   succession   is   in   question,  
provides:  Every  person  over  the  age  of  eighteen  years,  of  sound  mind,  may,  by   whatever   may   be   the   nature   of   the   property   and   the   country   in   which   it   is  
last   will,   dispose   of   all   his   or   her   estate,   real   and   personal,   the   same   being   found.    
chargeable  with  the  payment  of  the  testator's  debts.      
  In   the   proceedings   for   the   probate   of   the   will,   it   was   found   out   and   it   was  
Besides,  the  right  of  Magdalena  to  a  share  in  the  testator's  estate  had  already   decided  that  the  testator  was  a  citizen  of  the  State  of  Nevada  because  he  had  
been  passed  upon  adversely  against  her  in  an  order  dated  June  19,  1955,  which   selected  this  as  his  domicile  and  his  permanent  residence.  So  the  question  at  
had  become  final,  as  Magdalena  does  not  appear  to  have  appealed  therefrom   issue  is  whether  the  testementary  dispositions,  especially  those  for  the  children  
to   question   its   validity.   On   December   16,   1953,   the   said   former   wife   filed   a   which  are  short  of  the  legitime  given  them  by  the  Civil  Code  of  the  Philippines,  
motion   to   withdraw   the   sum   of   P20,000   from   the   funds   of   the   estate,   are  valid.  It  is  not  disputed  that  the  laws  of  Nevada  allow  a  testator  to  dispose  
chargeable  against  her  share  in  the  conjugal  property,  and  the  court  in  its  said   of  all  his  properties  by  will.  It  does  not  appear  that  at  time  of  the  hearing  of  the  
error   found   that   there   exists   no   community   property   owned   by   the   decedent   project  of  partition,  the  above-­‐quoted  provision  was  introduced  in  evidence,  as  
and   his   former   wife   at   the   time   the   decree   of   divorce   was   issued.   As   already   it  was  the  executor's  duly  to  do.    
and   Magdalena   C.   Bohanan   may   no   longer   question   the   fact   contained   therein,    
i.e.   that   there   was   no   community   property   acquired   by   the   testator   and   The  law  of  Nevada,  being  a  foreign  law  can  only  be  proved  in  our  courts  in  the  
Magdalena  C.  Bohanan  during  their  converture.     form   and   manner   provided   for   by   our   Rules,   which   are   as   follows:  SEC.   41.  
  Proof  of  public  or  official  record.  —  An  official  record  or  an  entry  therein,  when  
Moreover,  the  court  below  had  found  that  the  testator  and  Magdalena  were   admissible  for  any  purpose,  may  be  evidenced  by  an  official  publication  thereof  
married  on  January  30,  1909,  and  that  divorce  was  granted  to  him  on  May  20,   or  by  a  copy  tested  by  the  officer  having  the  legal  custody  of  the  record,  or  by  
1922;   that   sometime   in   1925,   Magdalena   married   Carl   Aaron   and   this   his  deputy,  and  accompanied,  if  the  record  is  not  kept  in  the  Philippines,  with  a  
marriage   was   subsisting   at   the   time   of   the   death   of   the   testator.   Since   no   certificate  that  such  officer  has  the  custody.    
right  to  share  in  the  inheritance  in  favor  of  a  divorced  wife  exists  in  the  State    
of   Nevada   and   since   the   court   below   had   already   found   that   there   was   no   We   have,   however,   consulted   the   records   of   the   case   in   the   court   below   and  
conjugal   property   between   the   testator   and   Magdalena,   the   latter   can   now   we   have   found   that   during   the   hearing   on   October   4,   1954   of   the   motion   of   for  
have  no  longer  claim  to  pay  portion  of  the  estate  left  by  the  testator.     withdrawal   of   P20,000   as   her   share,   the   foreign   law,   especially   Section   9905,  
  Compiled   Nevada   Laws   was   introduced   in   evidence   by   appellant's   counsel.   In  
The   most   important   issue   is   the   claim   of   the   testator's   children,   Edward   and   addition,   the   other   appellants,   children   of   the   testator,   do   not   dispute   the  
Mary   Lydia,   who   had   received   legacies   in   the   amount   of   P6,000   each   only,   and,   above-­‐   quoted   provision   of   the   laws   of   the   State   of   Nevada.   Under   all   the  
therefore,  have  not  been  given  their  shares  in  the  estate  which,  in  accordance   above   circumstances,   we   are   constrained   to   hold   that   the   pertinent   law   of  
with   the   laws   of   the   forum,   should   be   two-­‐thirds   of   the   estate   left   by   the   Nevada,  especially  Section  9905  of  the  Compiled  Nevada  Laws  of  1925,  can  be  
testator.   Is   the   failure   old   the   testator   to   give   his   children   two-­‐thirds   of   the   taken   judicial   notice   of   by   us,   without   proof   of   such   law   having   been   offered   at  
estate  left  by  him  at  the  time  of  his  death,  in  accordance  with  the  laws  of  the   the  hearing  of  the  project  of  partition.    
forum  valid?      
  As   in   accordance   with   Article   10   of   the   old   Civil   Code,   the   validity   of  
The  old  Civil  Code,  which  is  applicable  to  this  case  because  the  testator  died  in   testamentary   dispositions   are   to   be   governed   by   the   national   law   of   the  
1944,  expressly  provides  that  successional  rights  to  personal  property  are  to   testator,   and   as   it   has   been   decided   and   it   is   not   disputed   that   the   national  
be   governed   by   the   national   law   of   the   person   whose   succession   is   in   law  of  the  testator  is  that  of  the  State  of  Nevada,  which  allows  a  testator  to  
question.   Says   the   law   on   this   point:   Nevertheless,   legal   and   testamentary   dispose   of   all   his   property   according   to   his   will,  as  in  the  case  at  bar,  the  order  
 
CONFLICT  OF  LAWS                                                                                    AV  DE  TORRES   152  
ATTY.  ARIS  L.  GULAPA                                            AY  2015-­‐2016  
of   the   court   approving   the   project   of   partition   made   in   accordance   with   the   said  three  witnesses  signed  their  names  on  the  last  page  after  the  attestation  
testamentary   provisions,   must   be,   as   it   is   hereby   affirmed,   with   costs   against   clause   in   his   presence   and   in   the   presence   of   each   other.   The   oppositors   did  
appellants.     not   submit   any   evidence.   The   trial   court   found   and   declared   the   will   to   be   a  
  holographic   will;   that   it   was   in   the   handwriting   of   the   testator   and   that  
94.  ENRIQUEZ  V.  ABADIA   although   at   the   time   it   was   executed   and   at   the   time   of   the   testator's   death,  
G.R.  No.  L-­‐7188  |  August  9,  1954     holographic   wills   were   not   permitted   by   law   still,   because   at   the   time   of   the  
  hearing  and  when  the  case  was  to  be  decided  the  new  Civil  Code  was  already  in  
DOCTRINES:   The   extrinsic   validity   of   a   will   is   to   be   judged   not   by   the   law   force,  which  Code  permitted  the  execution  of  holographic  wills,  under  a  liberal  
enforce  at  the  time  of  the  testator's  death  or  at  the  time  the  supposed  will  is   view,   and   to   carry   out   the   intention   of   the   testator   which   according   to   the   trial  
presented  in  court  for  probate  or  when  the  petition  is  decided  by  the  court  but   court   is   the   controlling   factor   and   may   override   any   defect   in   form,   said   trial  
at   the   time   the   instrument   was   executed   (Art.   795   of   the   Civil   Code).   One   court   admitted   to   probate   the   Last   Will   and   Testament   of   Father   Sancho  
reason  in  support  of  the  rule  is  that  although  the  will  operates  upon  and  after   Abadia.  The  oppositors  appealed  from  that  decision.    
the  death  of  the  testator,  the  wishes  of  the  testator  about  the  disposition  of  his    
estate   among   his   heirs   and   among   the   legatees   is   given   solemn   expression   at   ISSUE:  
the   time   the   will   is   executed,   and   in   reality,   the   legacy   or   bequest   then   Whether   the   holographic   will   should   be   allowed   despite   the   fact   that   when   it  
becomes  a  completed  act.   was  executed  the  civil  code  proscribes  the  execution  of  such  wills  
   
The  general  rule  is  that  the  Legislature  cannot  validate  void  wills.   HELD:  
  No.   The   new   Civil   Code   (Republic   Act   No.   386)   under   article   810   thereof  
N.B.:   Art.   795.   The   validity   of   a   will   as   to   its   form   depends   upon   the   observance   provides  that  a  person  may  execute  a  holographic  will  which  must  be  entirely  
of  the  law  in  force  at  the  time  it  is  made.   written,  dated  and  signed  by  the  testator  himself  and  need  not  be  witnessed.  It  
  is  a  fact,  however,  that  at  the  time  that  Exhibit  "A"  was  executed  in  1923  and  at  
FACTS:   the  time  that  Father  Abadia  died  in  1943,  holographic  wills  were  not  permitted,  
On   September  6,  1923,   Father   Sancho   Abadia,   parish   priest   and   also   a   resident   and   the   law   at   the   time   imposed   certain   requirements   for   the   execution   of  
of   Talisay,   Cebu,   executed   a   document   purporting   to   be   his   Last   Will   and   wills,   such   as   numbering   correlatively   each   page   (not   folio   or   sheet)   in   letters  
Testament.   He   died   on   January   14,   1943,   in   the   municipality   of   Aloguinsan,   and  signing  on  the  left  hand  margin  by  the  testator  and  by  the  three  attesting  
Cebu,   where   he   was   an   evacuee.   He   left   properties   estimated   at   P8,000   in   witnesses,  requirements  which  were  not  complied  with  in  Exhibit  "A"  because  
value.  On  October  2,  1946,  Andres  Enriquez,  one  of  the  legatees  filed  a  petition   the  back  pages  of  the  first  two  folios  of  the  will  were  not  signed  by  any  one,  not  
for  the  probate  of  the  will  in  the  CFI  of  Cebu.  Some  cousins  and  nephews  who   even  by  the  testator  and  were  not  numbered,  and  as  to  the  three  front  pages,  
would  inherit  the  estate  of  the  deceased  if  he  left  no  will,  filed  opposition.   they  were  signed  only  by  the  testator.  
   
During   the   hearing   one   of   the   attesting   witnesses,   the   other   two   being   dead,   The   validity   of   a   will   is   to   be   judged   not   by   the   law   enforce   at   the   time   of   the  
testified  without  contradiction  that  in  his  presence  and  in  the  presence  of  his   testator's   death   or   at   the   time   the   supposed   will   is   presented   in   court   for  
co-­‐witnesses,   Father   Sancho   wrote   out   in   longhand   the   will   in   Spanish   which   probate   or   when   the   petition   is   decided   by   the   court   but   at   the   time   the  
the  testator  spoke  and  understood;  that  he  signed  on  The  left  hand  margin  of   instrument  was  executed  (Art.  795  of  the  Civil  Code).  One  reason  in  support  
the  front  page  of  each  of  the  three  folios  or  sheets  of  which  the  document  is   of  the  rule  is  that  although  the  will  operates  upon  and  after  the  death  of  the  
composed,   and   numbered   the   same   with   Arabic   numerals,   and   finally   signed   testator,   the   wishes   of   the   testator   about   the   disposition   of   his   estate   among  
his  name  at  the  end  of  his  writing  at  the  last  page,  all  this,  in  the  presence  of   his  heirs  and  among  the  legatees  is  given  solemn  expression  at  the  time  the  
the  three  attesting  witnesses  after  telling  that  it  was  his  last  will  and  that  the   will   is   executed,   and   in   reality,   the   legacy   or   bequest   then   becomes   a  
completed  act.  
 
CONFLICT  OF  LAWS                                                                                    AV  DE  TORRES   153  
ATTY.  ARIS  L.  GULAPA                                            AY  2015-­‐2016  
  will  was  described  as  "not  in  any  way  related  to"  him  but  in  a  decision  rendered  
Of   course,   there   is   the   view   that   the   intention   of   the   testator   should   be   the   by   the   SC   in   another   case   had   been   declared   as   an   acknowledged   natural  
ruling  and  controlling  factor  and  that  all  adequate  remedies  and  interpretations   daughter  of  his.  
should   be   resorted   to   in   order   to   carry   out   said   intention,   and   that   when    
statutes   passed   after   the   execution   of   the   will   and   after   the   death   of   the   Helen   alleged   that   the   will   deprives   her   of   her   legitime   as   an   acknowledged  
testator   lessen   the   formalities   required   by   law   for   the   execution   of   wills,   said   natural  child.  She  claims  that  under  Art.  16  of  the  Civil  Code,  the  California  law  
subsequent   statutes   should   be   applied   so   as   to   validate   wills   defectively   should   be   applied,   and   in   accordance   therewith   and   following   the   doctrine   of  
executed  according  to  the  law  in  force  at  the  time  of  execution.  However,  we   renvoi,   the   question   of   the   validity   of   the   testamentary   provision   in   question  
should  not  forget  that  from  the  day  of  the  death  of  the  testator,  if  he  leaves  a   should   be   referred   back   to   the   law   of   the   decedent's   domicile,   which   is   the  
will,   the   title   of   the   legatees   and   devisees   under   it   becomes   a   vested   right,   Philippines.   She   invokes   the   provisions   of   Art.   946   of   the   Civil   Code   of  
protected   under   the   due   process   clause   of   the   constitution   against   a   California,  which  is  as  follows:  “If  there  is  no  law  to  the  contrary,  in  the  place  
subsequent  change  in  the  statute  adding  new  legal  requirements  of  execution   where  personal  property  is  situated,  it  is  deemed  to  follow  the  person  of  its  
of   wills   which   would   invalidate   such   a   will.   By   parity   of   reasoning,   when   one   owner,   and   is   governed   by   the   law   of   his   domicile.”   Accordingly,   her   share  
executes   a   will   which   is   invalid   for   failure   to   observe   and   follow   the   legal   must   be   increased   in   view   of   successional   rights   of   illegitimate   children   under  
requirements   at   the   time   of   its   execution   then   upon   his   death   he   should   be   Philippine  laws.  
regarded  and  declared  as  having  died  intestate,  and  his  heirs  will  then  inherit    
by  intestate  succession,  and  no  subsequent  law  with  more  liberal  requirements   On   the   other   hand,   the   executor   and   Lucy   argue   that   the   national   law   of   the  
or  which  dispenses  with  such  requirements  as  to  execution  should  be  allowed   deceased  must  apply,  and  thus  the  courts  must  apply  internal  law  of  California  
to   validate   a   defective   will   and   thereby   divest   the   heirs   of   their   vested   rights   in   on   the   matter.   Under   California   law,   there   are   no   compulsory   heirs   and  
the   estate   by   intestate   succession.   The   general   rule   is   that   the   Legislature   consequently  a  testator  may  dispose  of  his  property  by  will  in  the  form  and  
cannot  validate  void  wills     manner  he  desires  (Kaufman  Case).  
   
91.  AZNAR  V.  CHRISTENSES-­‐GARCIA   ISSUE:  
G.R.  No.  L-­‐16749  |  January  31,  1963   Whether  Philippine  law  should  ultimately  be  applied  
   
DOCTRINE:  The  recognition  of  the  renvoi  theory  implies  that  the  rules  of  conflict   HELD:  
of  laws  are  to  be  understood  as  incorporating  not  only  the  ordinary  or  internal   Yes.  Edward  was  a  US  Citizen  and  domiciled  in  the  Philippines  at  the  time  of  his  
laws   of   the   foreign   state   or   country   but   its   rules   of   conflict   of   laws   as   well.   death.   The   law   that   governs   the   validity   of   his   testamentary   dispositions   is  
According  to  this  theory,  the  law  of  a  country  means  the  whole  of  its  law.     defined  in  Art.  16,  CC,  which  is  as  follows:  
   
FACTS:   ART.  16.  Real  property  as  well  as  personal  property  is  subject  to  the  law  
Edward  Christensen,  born  in  New  York,  migrated  to  California  where  he  resided   of  the  country  where  it  is  situated.  
and   consequently   was   considered   citizen   thereof.   He   came   to   the   Philippines    
where  he  became  a  domiciliary  until  the  time  of  his  death.  However,  during  the   However,   intestate   and   testamentary   successions,   both   with   respect   to  
entire   period   of   his   residence   in   this   country,   he   had   always   considered   himself   the  order  of  succession  and  to  the  amount  of  successional  rights  and  to  
a  citizen  of  California.   the   intrinsic   validity   of   testamentary   provisions,   shall   be   regulated   by   the  
  national   law   of   the   person   whose   succession   is   under   consideration,  
In   his   will,   Edward   instituted   his   daughter   Maria   Lucy   Christensen   as   his   only   whatever   may   be   the   nature   of   the   property   and   regardless   of   the  
heir,  but  left  a  legacy  of  P3600  in  favor  of  Helen  Christensen  Garcia  who,  in  his   country  where  said  property  may  be  found.  
 
 
CONFLICT  OF  LAWS                                                                                    AV  DE  TORRES   154  
ATTY.  ARIS  L.  GULAPA                                            AY  2015-­‐2016  
The  laws  of  California  have  prescribed  two  sets  of  laws  for  its  citizens,  one  for   Maria   Cristina   Bellis   and   Miriam   Palma   Bellis,   filed   their   respective  
residents  therein  and  another  for  those  domiciled  in  other  jurisdictions.  Art.   oppositions  to  the  project  of  partition  on  the  ground  that  they  were  deprived  
946   of   the   California   Civil   Code   is   its   conflict   of   laws   rule,   while   the   rule   of  their  legitimes  as  illegitimate  children  and,  therefore,  compulsory  heirs  of  
applied  in  Kaufman,  is  its  internal  law.  If  the  law  on  succession  and  the  conflict   the  deceased.  
of  laws  rules  of  California  are  to  be  enforced  jointly,  each  in  its  own  intended    
and   appropriate   sphere,   the   principle   cited   in   Kaufman   should   apply   to   The  lower  court  issued  an  order  overruling  the  oppositions  and  approving  the  
citizens  living  in  the  State,  but  Art.  946  should  apply  to  such  of  its  citizens  as   executor’s   final   account,   report   and   administration,   and   project   of   partition.  
are  not  domiciled  in  California  but  in  other  jurisdictions.   Relying   upon   Art.   16   of   the   Civil   Code,   it   applied   the   national   law   of   the  
  decedent,  which  in  this  case  did  not  provide  for  legitimes.  
The   national   law   mentioned   in   Art.   16   is   the   law   on   conflict   of   laws   in   the    
California  Civil  Code,  i.e.,  Art.  946,  which  authorizes  the  reference  or  return  of   ISSUE:  
the  question  to  the  law  of  the  testator's  domicile.  The   conflict   of   laws   rule   in   Whether  such  illegitimate  children  of  Bellis  be  entitled  to  successional  rights  
California  precisely  refers  back  the  case,  when  a  decedent  is  not  domiciled  in    
California,   to   the   law   of   his   domicile,   which   is   the   Philippines   in   the   case   at   HELD:  
bar.   No.   The   said   illegitimate   children   are   not   entitled   to   their   legitimes.   Under  
  Texas  law,  there  are  no  legitimes.  Even  if  the  other  will  was  executed  in  the  
The   Philippine   court   therefore   must   apply   its   own   law   as   directed   in   the   Philippines,   his   national   law,   still,   will   govern   the   properties   for   succession  
conflict   of   laws   rule   of   the   state   of   the   decedent.   Wherefore,   the   decision   even   if   it   is   stated   in   his   testate   that   it   shall   be   governed   by   the   Philippine  
appealed   from   is   hereby   reversed   and   the   case   returned   to   the   lower   court   with   law.  
instructions   that   the   partition   be   made   as   the   Philippine   law   on   succession    
provides.   Art.  16,  Par.  2  renders  applicable  the  national  law  of  the  decedent,  in  intestate  
  and   testamentary   successions,   with   regard   to   four   items:   (a)   the   order   of  
92.  BELLIS  V.  BELLIS   succession,   (b)   the   amount   of   successional   rights,   (c)   the   intrinsic   validity   of  
G.R.  No.  L-­‐23678  |  June  6,  1967   provisions  of  will,  and  (d)  the  capacity  to  succeed.  
   
FACTS:   ART.16  Real  property  as  well  as  personal  property  is  subject  to  the  law  of  
Amos  Bellis  was  a  citizen  and  resident  of  Texas  at  the  time  of  his  death.         He   the  country  to  where  it  is  situated.  
had  5  legitimate  children  with  his  first  wife,  Mary  Mallen,  whom  he  divorced.    
He  had  3  legitimate  daughters  with  his  second  wife,  Violet,  who  survived  him,   However,   intestate   and   testamentary   successions,   both   with   respect   to  
and   another   3   illegitimate   children   with   another   woman.   Before   he   died,   he   the  order  of  successions  and  to  the  amount  of  successional  rights  and  to  
executed   2   wills—the   first   one   disposing   of   his   Texas   properties,   the   other   the   intrinsic   validity   of   testamentary   provisions,   shall   be   regulated   by   the  
disposing   his   Philippine   properties.  In   his   will,   which   he   executed   in   the   national   law   of   the   person   whose   succession   is   under   consideration,  
Philippines,   he   directed   that   after   all   taxes,   obligations,   and   expenses   of   whatever   may   be   the   nature   of   the   property   and   regardless   of   the  
administration  are  paid  for,  his  distributable  estate  should  be  divided,  in  trust,   country  wherein  said  property  may  be  found.  
in  the  following  order  and  manner:  a)  $240,000.00  to  his  first  wife  Mary  Mallen    
b)  $120,000.00  to  his  three  illegitimate  children  Amos  Bellis,  Jr.,  Maria  Cristina   As   a   counter-­‐argument,   appellants   invoked   Art.   17,   Par.   3,   of   the   Civil   Code,  
Bellis,  Miriam  Palma  Bellis,  or  $40,000.00  each,  and  c)  After  foregoing  the  two   stating—  
items  have  been  satisfied,  the  remainder  shall  go  to  his  seven  surviving  children    
by  his  first  and  second  wives.   Prohibitive   laws   concerning   persons,   their   acts   or   property,   and   those  
  which  have  for  their  object  public  order,  public  policy  and  good  customs  
 
CONFLICT  OF  LAWS                                                                                    AV  DE  TORRES   155  
ATTY.  ARIS  L.  GULAPA                                            AY  2015-­‐2016  
shall   not   be   rendered   ineffective   by   laws   or   judgments   promulgated,   or   persons   whom   he   or   she   desires   to   favor.   And,   where   the   will   is   not   only   joint  
by  determinations  or  conventions  agreed  upon  in  a  foreign  country.   but  reciprocal,  either  one  of  the  spouses  who  may  happen  to  be  unscrupulous,  
  wicked,  faithless,  or  desperate,  knowing  as  he  or  she  does  the  terms  of  the  will  
However,   it   is   evident   that   whatever   public   policy   or   good   customs   may   be   whereby  the  whole  property  of  the  spouses  both  conjugal  and  paraphernal  goes  
involved  in  our  System  of  legitimes  (that  is  the  public  policy  that  a  compulsory   to  the  survivor,  may  be  tempted  to  kill  or  dispose  of  the  other.  
heir  cannot  be  deprived  of  his  legitime),  Congress  has  not  intended  to  extend    
the  same  to  the  succession  of  foreign  nationals.  For  it  has  specifically  chosen   ALG:  What  needs  to  be  stipulated  for  a  will  to  be  joint?  “Our  property..  be  given  
to   leave,  inter   alia,   the  amount  of   successional   rights,   to   the   decedent's   and  transmitted  to  anyone  or  either  of  us,  who  may  survive  the  other,  or  who  
national  law.  Specific  provisions  must  prevail  over  general  ones.  A  provision  in   may  remain  the  surviving  spouse  of  the  other."    
a   foreigner's   will   to   the   effect   that   his   properties   shall   be   distributed   in    
accordance  with  Philippine  law  and  not  with  his  national  law,  is  illegal  and  void,   N.B.:  
for   his   national   law   cannot   be   ignored   in   regard   to   those   matters   that   Article   Art.   818.   Two   or   more   persons   cannot   make   a   will   jointly,   or   in   the   same  
10—now  Article  16—of  the  Civil  Code  states  said  national  law  should  govern.   instrument,  either  for  their  reciprocal  benefit  or  for  the  benefit  of  a  third  person.  
  (669)    
In   this   regard,   the   parties   do   not   submit   the   case   on,   nor   even   discuss,   the    
doctrine  of  renvoi,  applied  by  this  Court  in  Aznar  v.  Christensen.  Said  doctrine  is   Art.   819.   Wills,   prohibited   by   the   preceding   article,   executed   by   Filipinos   in   a  
usually   pertinent   where   the   decedent   is   a   national   of   one   country,   and   a   foreign  country  shall  not  be  valid  in  the  Philippines,  even  though  authorized  by  
domicile  of  another.  In  the  present  case,  it  is  not  disputed  that  the  decedent   the  laws  of  the  country  where  they  may  have  been  executed.  (733a)    
 
was  both  a  national  of  Texas  and  a  domicile  thereof  at  the  time  of  his  death.  
So   that   even   assuming   Texas   has   a   conflict   of   law   rule   providing   that   the   FACTS:  
domiciliary   system   (law   of   the   domicile)   should   govern,   the   same   would   not   Victor   Bilbao   jointly   with   his   wife   Ramona   M.   Navarro,   executed   the   will   in  
result   in   a   reference   back   (renvoi)   to   Philippine   law,   but   would   still   refer   to   question   on   October   6,   1931,   on   a   single   page   or   sheet.   The   two   testators   in  
Texas   law.   Nonetheless,  if  Texas  has  a  conflicts  rule  adopting  the  situs  theory   their   testament   directed   that   "all   of   our   respective   private   properties   both   real  
(lex   rei   sitae)   calling   for   the   application   of   the   law   of   the   place   where   the   and   personal,   and   all   of   our   conjugal   properties,   and   any   other   property  
properties  are  situated,  renvoi  would  arise,  since  the  properties  here  involved   belonging  to  either  or  both  of  us,  be  given  and  transmitted  to  anyone  or  either  
are   found   in   the   Philippines.   In   the   absence,   however,   of   proof   as   to   the   of   us,   who   may   survive   the   other,   or   who   may   remain   the   surviving   spouse   of  
conflict   of   law   rule   of   Texas,   it   should   not   be   presumed   different   from   ours.   the  other."    
Appellants'  position  is  therefore  not  rested  on  the  doctrine  of  renvoi.  As  stated,    
they   never   invoked   nor   even   mentioned   it   in   their   arguments.   Rather,   they   The  petition  for  probate  was  opposed  by  one  Filemon  Abringe,  a  near  relative  
argue   that   their   case   falls   under   the   circumstances   mentioned   in   the   third   of  the  deceased,  among  other  grounds,  that  the  alleged  will  was  executed   by  
paragraph  of  Article  17  in  relation  to  Article  16  of  the  Civil  Code.   the  husband  and  wife  for  their  reciprocal  benefit  and  therefore  not  valid,  and  
  that  it  was  not  executed  and  attested  to  as  required  by  law.  After  hearing,  the  
93.  BILBAO  V.  BILBAO   trial   court   found   the   will   to   have   been   executed   conjointly   by   the   deceased  
G.R.  No.  L-­‐2200  |  August  2,  1950     husband   and   wife   for   their   reciprocal   benefit,   and   that   a   will   of   that   kind   is  
  neither  contemplated  by  Act  No.  190,  known  as  the  Code  of  Civil  Procedure  nor  
DOCTRINE:   The   reason   for   this   provision,   especially   as   regards   husbands   and   permitted  by  article  669  of  the  Civil  Code  which  provides:  Two  or  more  persons  
wife   is   that   when   a   will   is   made   jointly   or   in   the   same   instrument,   the   spouse   cannot   make   a   will   conjointly   or   in   the   same   instrument,   either   for   their  
who  is  more  aggressive,  stronger  in  will  or  character  and  dominant  is  liable  to   reciprocal  benefit  or  for  the  benefit  of  a  third  person.    
dictate   the   terms   of   the   will   for   his   or   her   own   benefit   or   for   that   of   third    

 
CONFLICT  OF  LAWS                                                                                    AV  DE  TORRES   156  
ATTY.  ARIS  L.  GULAPA                                            AY  2015-­‐2016  
The   thesis   of   the   appellant   is,   that   inasmuch   as   the   present   law   on   wills   as   law.   The   intrinsic   validity   of   the   will   normally   comes   only   after   the   court   has  
embodied  in  the  Code  of  Civil  Procedure  has  been  taken  from  American  law,  it   declared  that  the  will  has  been  duly  authenticated.  
should  be  interpreted  in  accordance  with  the  said  law,  and  because  joint  and    
reciprocal   wills   are   neither   regarded   as   invalid   nor   on   the   contrary   they   are   EXC:   Where   practical   considerations   demand   that   the   intrinsic   validity   of   the  
allowed,  then   article   669   of   the   Civil   Code   prohibiting   the   execution   of   joint   will   be   passed   upon,   even   before   it   is   probated,   the   court   should   meet   the  
wills   whether   reciprocal   or   for   the   benefit   of   a   third   party   should   be   issue.    
considered  as  having  been  repealed  and  superseded  by  the  new  law.      
  FACTS:  
ISSUE:   Adoracion  C.  Campos  died,  leaving  her  father,  petitioner  Hermogenes  Campos  
Whether   a   joint   and   reciprocal   will   particularly   between   husband   and   wife   is   and   her   sisters,   private   respondent   Nenita   C.   Paguia,   Remedios   C.   Lopez   and  
valid  under  the  present  law   Marieta   C.   Medina   as   the   surviving   heirs.   As   Hermogenes   Campos   was   the   only  
  compulsory   heir,   he   executed   an   Affidavit   of   Adjudication   under   Rule   74,  
HELD:   Section   I   of   the   Rules   of   Court   whereby   he   adjudicated   unto   himself   thex  
No.   The   provisions   of   the   Code   of   Civil   Procedure   regarding   wills   have   not   ownership   of   the   entire   estate   of   the   deceased   Adoracion   Campos.   Eleven  
repealed  all  the  articles  of  the  old  Civil  Code  on  the  same  subject  matter;  article   months   after,   Nenita   C.   Paguia   filed   a   petition   for   the   reprobate   of   a   will   of   the  
669   of   the   Civil   Code   is   not   incompatible   or   inconsistent   with   said   provisions   of   deceased,  Adoracion  Campos,  which  was  allegedly  executed  in  the  US  and  for  
the  Code  of  Civil  Procedure,  and  finally,  said  article  669  of  the  Civil  Code  is  still   her  appointment  as  administratrix  of  the  estate  of  the  deceased  testatrix.    
in  force.    
  In  her  petition,  Nenita  alleged  that  the  testatrix  was  an  American  citizen  at  the  
The  provision  of  article  669  of  the  Civil  Code  prohibiting  the  execution  of  a  will   time   of   her   death   and   was   a   permanent   resident   of   4633   Ditman   Street,  
by   two   or   more   persons   conjointly   or   in   the   same   instrument   either   for   their   Philadelphia,   Pennsylvania,   U.S.A.;   that   the   testatrix   died   in   Manila   while  
reciprocal  benefit  or  for  the  benefit  of  a  third  person,  is  not  unwise  and  is  not   temporarily   residing   with   her   sister   Malate,   Manila;   that   during   her   lifetime,  
against   public   policy.   The   reason   for   this   provision,   especially   as   regards   the  testatrix  made  her  last  will  and  testament  on  July  10,  1975,  according  to  
husbands  and  wife  is  that  when  a  will  is  made  jointly  or  in  the  same  instrument,   the  laws  of  Pennsylvania,  U.S.A.,  nominating  Wilfredo  Barzaga  of  New  Jersey  
the  spouse  who  is  more  aggressive,  stronger  in  will  or  character  and  dominant   as   executor;   that   after   the   testatrix   death,   her   last   will   and   testament   was  
is   liable   to   dictate   the   terms   of   the   will   for   his   or   her   own   benefit   or   for   that   of   presented,  probated,  allowed,  and  registered  with  the  Registry  of  Wins  at  the  
third  persons  whom  he  or  she  desires  to  favor.  And,  where  the  will  is  not  only   County   of   Philadelphia,   U.S.A.,  that  Clement  L.  McLaughlin,  the  administrator  
joint   but   reciprocal,   either   one   of   the   spouses   who   may   happen   to   be   who  was  appointed  after  Dr.  Barzaga  had  declined  and  waived  his  appointment  
unscrupulous,   wicked,   faithless,   or   desperate,   knowing   as   he   or   she   does   the   as  executor  in  favor  of  the  former,  is  also  a  resident  of  Philadelphia,  U.S.A.,  and  
terms  of  the  will  whereby  the  whole  property  of  the  spouses  both  conjugal  and   that   therefore,   there   is   an   urgent   need   for   the   appointment   of   an  
paraphernal   goes   to   the   survivor,   may   be   tempted   to   kill   or   dispose   of   the   administratrix   to   administer   and   eventually   distribute   the   properties   of   the  
other.   estate  located  in  the  Philippines.    
   
95.  CAYETANO  V.  LEONIDAS   An   opposition   to   the   reprobate   of   the   will   was   filed   by   herein   petitioner  
G.R.  No.  L-­‐54919  |  May  30,  1984     alleging  among  other  things,  that  he  has  every  reason  to  believe  that  the  will  
  in   question   is   a   forgery;   that   the   intrinsic   provisions   of   the   will   are   null   and  
DOCTRINE:   GR:   The   probate   court's   authority   is   limited   only   to   the   extrinsic   void;   and   that   even   if   pertinent   American   laws   on   intrinsic   provisions   are  
validity   of   the   will,   the   due   execution   thereof,   the   testatrix's   testamentary   invoked,  the  same  could  not  apply  inasmuch  as  they  would  work  injustice  and  
capacity   and   the   compliance   with   the   requisites   or   solemnities   prescribed   by   injury  to  him.  Later,  however,  the  petitioner  through  his  counsel,  Atty.  Franco  

 
CONFLICT  OF  LAWS                                                                                    AV  DE  TORRES   157  
ATTY.  ARIS  L.  GULAPA                                            AY  2015-­‐2016  
Loyola,   filed   a   Motion   to   Dismiss   Opposition   (With   Waiver   of   Rights   or   denied.   Petitioner   Cayetano   persists   with   the   allegations   that   the   respondent  
Interests)   stating   that   he   "has   been   able   to   verify   the   veracity   thereof   (of   the   judge  acted  without  or  in  excess  of  his  jurisdiction.    
will)  and  now  confirms  the  same  to  be  truly  the  probated  will  of  his  daughter    
Adoracion."   Hence,   an   ex-­‐parte   presentation   of   evidence   for   the   reprobate   of   ISSUE:    
the  questioned  will  was  made.     1.  Whether  the  respondent  judge  acted  without  or  in  excess  of  his  jurisdiction  
  2.   Whether   the   national   law   should   apply   as   regards   the   intrinsic   validity   of   the  
On   January   10,   1979,   the   respondent   judge   issued   an   order,   admitting   and   provisions  of  the  will  
allowing   probate   in   the   Philippines   of   Adoracion   Campos’   Last   Will   And   3.  Whether  CFI  of  Manila  acquired  jurisdiction  
Testament   and   appointing   Nenita   Campos   Paguia   as   Administratrix   of   the    
estate  of  said  decedent.  Subsequently,  Hermogenes  Campos  filed  a  petition  for   HELD:    
relief,  praying  that  the  order  allowing  the  will  be  set  aside  on  the  ground  that   1.   No.   The   first   issue   raised   by   the   petitioner   is   anchored   on   the   allegation   that  
the  withdrawal  of  his  opposition  to  the  same  was  secured  through  fraudulent   the   respondent   judge   acted   with   grave   abuse   of   discretion   when   he   allowed  
means.   According   to   him,   the   "Motion   to   Dismiss   Opposition"   was   inserted   the   withdrawal   of   the   petitioner's   opposition   to   the   reprobate   of   the   will.   No  
among   the   papers   which   he   signed   in   connection   with   two   Deeds   of   proof   was   adduced   to   support   petitioner's   contention   that   the   motion   to  
Conditional   Sales   which   he   executed   with   the   Construction   and   Development   withdraw  was  secured  through  fraudulent  means  and  that  Atty.  Franco  Loyola  
Corporation  of  the  Philippines  (CDCP).  He  also  alleged  that  the  lawyer  who  filed   was   not   his   counsel   of   record.   The   records   show   that   after   the   firing   of   the  
the   withdrawal   of   the   opposition   was   not   his   counsel-­‐of-­‐record   in   the   special   contested  motion,  the  petitioner  at  a  later  date,  filed  a  manifestation  wherein  
proceedings  case.     he  confirmed  that  the  Motion  to  Dismiss  Opposition  was  his  voluntary  act  and  
  deed.   Moreover,   at   the   time   the   motion   was   filed,   the   petitioner's   former  
The  petition  for  relief  was  set  for  hearing  but  the  petitioner  failed  to  appear.  He   counsel,  Atty.  Jose  P.  Lagrosa  had  long  withdrawn  from  the  case  and  had  been  
made   several   motions   for   postponement.   Petitioner   filed   another   motion   substituted   by   Atty.   Franco   Loyola   who   in   turn   filed   the   motion.   The   present  
entitled   "Motion   to   Vacate   and/or   Set   Aside   the   Order   of   January   10,   1979,   petitioner   cannot,   therefore,   maintain   that   the   old   man's   attorney   of   record  
and/or  dismiss  the  case  for  lack  of  jurisdiction.     was  Atty.  Lagrosa  at  the  time  of  filing  the  motion.  Since  the  withdrawal  was  in  
  order,  the  respondent  judge  acted  correctly  in  hearing  the  probate  of  the  will  
The  hearing  of  May  29,  1980  was  re-­‐set  by  the  court  for  June  19,  1980.  When   ex-­‐parte,  there  being  no  other  opposition  to  the  same.    
the  case  was  called  for  hearing  on  this  date,  the  counsel  for  petitioner  tried  to    
argue   his   motion   to   vacate   instead   of   adducing   evidence   in   support   of   the   2.  Yes.  As   a   general   rule,   the   probate   court's   authority   is   limited   only   to   the  
petition   for   relief.   Thus,   the   respondent   judge   issued   an   order   dismissing   the   extrinsic   validity   of   the   will,   the   due   execution   thereof,   the   testatrix's  
petition  for  relief  for  failure  to  present  evidence  in  support  thereof.  Petitioner   testamentary  capacity  and  the  compliance  with  the  requisites  or  solemnities  
filed  a  MR  but  the  same  was  denied.  In  the  same  order,  respondent  judge  also   prescribed  by  law.  The  intrinsic  validity  of  the  will  normally  comes  only  after  
denied  the  motion  to  vacate  for  lack  of  merit.  Hence,  this  petition.     the   court   has   declared   that   the   will   has   been   duly   authenticated.   However,  
  where   practical   considerations   demand   that   the   intrinsic   validity   of   the   will  
Meanwhile,   on   June   6,   1982,   petitioner   Hermogenes   Campos   died   and   left   a   be  passed  upon,  even  before  it  is  probated,  the  court  should  meet  the  issue.    
will,  which,  incidentally  has  been  questioned  by  the  respondent,  as  on  its  face,    
patently   null   and   void,   and   a   fabrication,   appointing   Polly   Cayetano   as   the   In   the   case   at   bar,   the   petitioner   maintains   that   since   the   respondent   judge  
executrix  of  his  last  will  and  testament.  Cayetano,  therefore,  filed  a  motion  to   allowed   the   reprobate   of   Adoracion's   will,   Hermogenes   C.   Campos   was  
substitute   herself   as   petitioner   in   the   instant   case   which   was   granted   by   the   divested   of   his   legitime   which   was   reserved   by   the   law   for   him.   This   contention  
court.   A   motion   to   dismiss   the   petition   on   the   ground   that   the   rights   of   the   is  without  merit.  
petitioner  Hermogenes  Campos  merged  upon  his  death  with  the  rights  of  the    
respondent   and   her   sisters,   only   remaining   children   and   forced   heirs   was   Although   on   its   face,   the   will   appeared   to   have   preterited   the   petitioner   and  
 
CONFLICT  OF  LAWS                                                                                    AV  DE  TORRES   158  
ATTY.  ARIS  L.  GULAPA                                            AY  2015-­‐2016  
thus,   the   respondent   judge   should   have   denied   its   reprobate   outright,   the   INTRINSIC  VALIDITY  
private   respondents   have   sufficiently   established   that   Adoracion   was,   at   the   Lex  loci  intentionis  (it  is  usually  the  choice  of  forum)  
time   of   her   death,   an   American   citizen   and   a   permanent   resident   of    
Philadelphia,   Pennsylvania,   U.S.A.   Therefore,   under   Article   16   par.   (2)   and   PHILIPPINES  
1039  of  the  Civil  Code,  the  law  which  governs  Adoracion  Campo's  will  is  the    
law   of   Pennsylvania,   U.S.A.,   which   is   the   national   law   of   the   decedent.   FORMS  
Although   the   parties   admit   that   the   Pennsylvania   law   does   not   provide   for     Lex  loci  celebrationis  (Art.  17)  
legitimes   and   that   all   the   estate   may   be   given   away   by   the   testatrix   to   a   CAPACITY    
complete   stranger,   the   petitioner   argues   that   such   law   should   not   apply   National  law  (Art.  15)  –  Gulapa:  Disagrees  with  Salonga  that  this  pertains  
because  it  would  be  contrary  to  the  sound  and  established  public  policy  and   only   to   family   relations.   The   word   “family”   does   not   qualify   the  
would  run  counter  to  the  specific  provisions  of  Philippine  Law.     subsequent   enumeration,   that   is,   “…status,   condition   and   legal  
  capacity  of  persons…”  
It  is  a  settled  rule  that  as  regards  the  intrinsic  validity  of  the  provisions  of  the   INTRINSIC/ESSENTIAL  VALIDITY  
will,  as  provided  for  by  Article  16(2)  and  1039  of  the  Civil  Code,  the  national  law     Lex  loci  intentionis  
of   the   decedent   must   apply.   This   was   squarely   applied   in   the   case   of   Bellis   v.   a. Express  (Lex  voluntatis  –  when  there  is  a  choice  of  law  clause)  
Bellis.   b. Implied  (Most  substantial  connection  with  the  transaction  and  the  
    parties/Points  of  contact)  
   
3.   Yes.   Finally,   we   find   the   contention   of   the   petition   as   to   the   issue   of   96.  MOLINA  V.  DELA  RIVA  
jurisdiction  utterly  devoid  of  merit.  Applying  Rule  73,  Section  1,  of  the  Rules  of   1906  
Court,   the   settlement   of   the   estate   of   Adoracion   Campos   was   correctly   filed    
with   the   CFI   of   Manila   where   she   had   an   estate   since   it   was   alleged   and   VENUE  
proven   that   Adoracion   at   the   time   of   her   death   was   a   citizen   and   permanent    
resident   of   Pennsylvania,   United   States   of   America   and   not   a   "usual   resident   of   Kyna’s  notes:  
Cavite"  as  alleged  by  the  petitioner.  Moreover,  petitioner  is  now  estopped  from   ALG:   This   was   the   ruling   before.   The   prevailing   rule   now   is   that   VENUE   (not  
questioning   the   jurisdiction   of   the   probate   court   in   the   petition   for   relief.   It   is   a   jurisdiction)  may  be  agreed  upon  by  the  parties  via  a  Choice  of  Forum  Clause  or  
settled   rule   that   a   party   cannot   invoke   the   jurisdiction   of   a   court   to   secure   Arbitration  Clause.  
affirmative   relief,   against   his   opponent   and   after   failing   to   obtain   such   relief,    
repudiate  or  question  that  same  jurisdiction.     How  do  you  formulate  an  Arbitration  Clause  regarding  venue?  
  Do  not  forget  to  indicate  “…to  the  exclusion  of  all  other  courts.”  
XI.  CONTRACTS    
  See  HSBC  v.  Sherman  (1989)  
  “Applying   the   foregoing   to   the   case   at   bar,   the   parties   did   not   thereby   stipulate  
CONTRACTS  IN  GENERAL   that   only   the   courts   of   Singapore,   to   the   exclusion   of   all   the   rest,   has  
  jurisdiction.   Neither   did   the   clause   in   question   operate   to   divest   Philippine  
FORMS   courts   of   jurisdiction.   In   International   Law,   jurisdiction   is   often   defined   as   the  
Lex   loci   celebrationis   (derived   from   the   broad   proposition   that   the   right   of   a   State   to   exercise   authority   over   persons   and   things   within   its  
place  governs  the  act,  locus  regit  actum)   boundaries   subject   to   certain   exceptions.   Thus,   a   State   does   not   assume  
CAPACITY   jurisdiction   over   travelling   sovereigns,   ambassadors   and   diplomatic  
Personal  law  
 
CONFLICT  OF  LAWS                                                                                    AV  DE  TORRES   159  
ATTY.  ARIS  L.  GULAPA                                            AY  2015-­‐2016  
representatives   of   other   States,   and   foreign   military   units   stationed   in   or   the  city  of  Manila,  it  is  apparent  that  the  CFI  of  that  city  had  jurisdiction  to  try  
marching  through  State  territory  with  the  permission  of  the  latter's  authorities.   and  determine  this  action.  
This  authority,  which  finds  its  source  in  the  concept  of  sovereignty,  is  exclusive    
within   and   throughout   the   domain   of   the   State.   A   State   is   competent   to   take   It  is  further  urged  in  support  of  the  alleged  want  of  jurisdiction  on  the  part  of  
hold  of  any  judicial  matter  it  sees  fit  by  making  its  courts  and  agencies  assume   the   court   below,   that   the   parties   had   mutually   designated   in   the   contract   in  
jurisdiction   over   all   kinds   of   cases   brought   before   them   (J.   Salonga,   Private   question   the   town   of   Bato,   Islands   of   Catanduanes,   as   the   place   where   all  
International  Law,  1981,  pp.  37-­‐38)   judicial   and   extrajudicial   acts   necessary   under   the   terms   thereof   should   take  
  place.  Paragraph  9  of  the  contract  contains  in  fact  a  stipulation  to  that  effect.  
FACTS:   This   the   appellant   claims   amounted   to   an   express   submission   by   the  
This  is  an  action  to  recover  a  debt  due  upon  a  contract  executed  July  27,  1903,   contracting  parties  to  jurisdiction  of  the  CFI  of  the  Province  of  Albay,  in  which  
whereby   plaintiff   transferred   to   the   defendant   the   abaca   and   copra   business   the  town  of  Bato  was  located.  
theretofore   carried   on   by   him   at   various   places   in   the   Island   of   Catanduanes,    
with   all   the   property   and   right   pertaining   to   the   said   business,   or   the   sum   of   We   are   of   the   opinion   that   the   designation   of   the   town   of   Bato   made   by   the  
134,636   pesos   and   12   cents,   payable   in   Mexican   currency   or   its   equivalent   in   parties  had  no  legal  force  and  could  not  have  the  effect  of  depriving  the  CFI  of  
local  currency.  Defendant  paid  at  the  time  of  the  execution  of  the  contract,  on   Manila   of   the   jurisdiction   conferred   on   it   by   law.   This   would   be   true   even  
account  of  the  purchase  price,  the  sum  of  P33,659  pesos  and  3  cents,  promising   though  it  may  be  granted  that  the  parties  actually  intended  to  waive  the  rights  
to  pay  the  balance  on  three  installments  P33,659  pesos  and  3  cents  each,  with   of   domicile   and   expressly   submit   themselves   to   the   exclusive   jurisdiction   of   the  
interest   at   the   rate   of   5   per   cent   per   annum   from   the   date   of   the   contract.   The   CFI  of  Albay,  contended  the  appellant,  all  of  which  it  may  be  said  seems  to  be  
first  installment  became  due  July  27,  1904.  It  was  for  the  recovery  of  this  first   very   doubtful,   judging   from   the   vague   and   uncertain   manner   in   which   the  
installment  that  their  action  was  brought  in  the  CFI  of  the  City  of  Manila.     designation   was   made.   The   jurisdiction   of   a   court   is   filed   by   law   and   not   by   the  
  will   of   the   parties.  As   a   matter   of   public   policy,   parties   can   only   stipulate   in  
Defendant   demurred   to   the   complaint   on   the   ground   that   the   court   had   no   regard  to  that  which  is  expressly  authorized  by  law.  Section  377  of  the  Code  
jurisdiction  of  the  subject  of  the  action.  The  court  overruled  the  demurrer  and   of   Civil   Procedure   provides   a   plain   and   definite   rule   for   the   purpose   of  
defendant   refused   to   and   did   not,   as   a   matter   of   fact,   answer   plaintiff’s   determining   the   jurisdiction   of   courts   according   to   the   nature   of   the   action.  
complaint.     Neither   that   section   nor   any   other   provision   of   law,   of   which   we   have   any  
  knowledge,   authorizes   the   parties   to   submit   themselves   by   an   express  
Judgment  having  been  rendered  in  favor  of  the  plaintiff  for  the  sum  of  33,659   stipulation   to   the   jurisdiction   of   a   particular   court   to   the   exclusion   of   the  
pesos  and  3  cents,  Mexican  currency,  equal  to  30,052  pesos  and  70  centavos,   court  duly  vested  with  such  jurisdiction.  
Philippine   currency,   an   interest   thereon   at   the   rate   of   5   per   cent   per   annum    
from  July  27,  1903  and  costs,  the  defendant  duly  excepted.     It   is   not   true   as   contended   by   the   appellant   that   the   right   which   litigants   had  
  under  the  Spanish  law  to  submit  themselves  to  the  jurisdiction  of  a  particular  
ISSUES:   court   was   governed   by   the   provisions   of   the   Civil   Code.   Such   right   was  
Whether  the  court  had  jurisdiction  of  the  subject  of  the  action   recognized   and   governed   by   the   provisions   of   the   Law   of   Procedure   and   not   by  
  the   substantive   law.   The   right   to   contract,   recognized   in   the   Civil   Code   and  
HELD:     referred  to  by  appellant,  has  nothing  to  do  with  the  right  to  establish  and  fix  
Yes.  A  personal  action  like  this  for  the  record  of  a  debt  may  be  brought,  under   the  jurisdiction  of  a  court.  This  right  can  only  be  exercised  by  the  legislative  
section   377   of   the   Code   of   the   Civil   Procedure,   in   the   CFI   of   the   province   branch  of  the  Government,  the  only  one  vested  with  the  necessary  power  to  
where   the   plaintiff   resides   or   in   the   province   where   the   defendant   may   make  rules  governing  the  subject.  In  this  connection  it  may  be  said  that  the  
reside,  at  the  election  of  the  plaintiff.  Both  parties  to  this  case  being  residents   jurisdiction  of  a  court  can  not  be  the  subject-­‐matter  of  a  contract.  
 
 
CONFLICT  OF  LAWS                                                                                    AV  DE  TORRES   160  
ATTY.  ARIS  L.  GULAPA                                            AY  2015-­‐2016  
97.  INSULAR  GOVERNMENT  V.  FRANK   On  the  otherhand,  in  Insular  Government  v.  Frank,  an  obiter  dictum  favored  the  
G.  R.  No.    2935  |  March  23,  1909   compulsory  approach.    
     
CAPACITY  TO  ENTER  INTO  A  CONTRACT   “....  Matters  bearing  upon  the  execution,  the  interpretation  and  the  validity  of  
  the  contract  are  determined  by  the  law  where  the  contract  is  made.”    
Kyna’s   notes:   Under   Art.   17,   the   Philippines   follow   the   lex   loci   contractus   rule.    
Does  the  Philippine  law  follow  the  compulsory  or  optional  approach?   ALG:  The  result  would  have  been  the  same  if  they  applied  the  Nationality  Rule.  
   
Compulsory  approach  –  When  a  person  enters  into  a  contract  on  State  A,  he  has   Although  generally,  capacity  of  the  parties  to  enter  into  a  contract  is  governed  
to   obey   its   sovereign   and,   necessarily,   State   A’s   laws   with   respect   to   the   by  national  law.  This  is  one  case  not  involving  real  property  which  was  decided  
formalities   of   the   transaction.   Thus,   the   law   of   the   place   of   contracting   by   our   SC,   where   instead   of   national   law,   what   should   determine   capacity   to  
determines  the  validity  of  the  contract.  The  law,  lex  loci  contractus,  determines   enter  into  a  contract  is  the  lex  loci  celebrationis.  According  to  Conflict  of  Laws  
whether   the   agreement   should   be   sealed   and   delivered,   or   whether   it   should   be   writer  Edgardo  Paras,  Frank’s  capacity  should  be  judged  by  his  national  law  and  
acknowledged  before  a  notary  public.   not  by  the  law  of  the  place  where  the  contract  was  entered  into.  In  the  instant  
  case  whether  it  is  the  place  where  the  contract  was  made  or  Frank’s  nationality,  
Optional  approach  –  the  form  is  merely  the  dress  of  the  transaction,  and  being   the  result  would  be  the  same.  However,  as  suggested    by  the  mentioned  author,  
part   of   it,   should   be   governed   by   the   same   law   that   governs   the   entire   for   the   conflicts   rule   in   capacity   in   general,   national   law   of   the   parties   is  
transaction.  Moreover,  the  place  of  the  execution  of  the  contract  may  have  only   controlling.  
a  casual  or  incidental  connection  with  the  contract.      
  N.B.:   Article   15.   Laws   relating   to   family   rights   and   duties,   or   to   the   status,  
Thus  the  rules  are:   condition   and   legal   capacity   of   persons   are   binding   upon   citizens   of   the  
(1)   The   formalities   required   to   make   a   valid   contract   are   determined   by   the   Philippines,  even  though  living  abroad.  
local  (internal)  law  of  the  State  chosen  by  the  parties  to  govern  their  contract    
or,   in   default   thereof,   by   the   local   law   of   the   State   which   has   the   most   FACTS:  
significant  relationship  to  the  transaction  and   In  1903  in  the  state  of  Illinois,  Mr.  Frank,  a  US  citizen  and  a  representative  of  
(2)   Formalities   which   meet   the   requirements   of   the   place   where   the   parties   the   Insular   Government   of   the   Philippines   entered   into   a   contract   whereby   the  
execute  the  contract  will  usually  be  acceptable.     former   shall   serve   as   stenographer   in   the   Philippines   for   a   period   of   2   years.  
  The   contract   contained   a   provision   that   in   case   of   violation   of   its   terms,   Mr.  
This   optional,   alternate   approach   protects   the   rational,   justified   expectations   of   Frank  shall  be  liable  for  the  amount  incurred  by  the  Philippine  Government  for  
the  contracting  parties.     his  travel  from  Chicago  to  Manila  and  one-­‐half  salary  paid  during  such  period.  
  After   serving   for   6   months,   defendant   left   the   service   and   refused   to   make  
In   Philippine   law,   the   Civil   Code   follows   the   optional   approach   with   respect   to   further  compliance  with  the  terms  of  the  contract,  therefore  the  Government  
the  formal  validity  of  wills,  in  an  effort  to  uphold  the  wishes  of  the  testator.       sued   him   to   recover   the   amount   of   $269.23   plus   damages.   The   lower   court  
  ruled   in   favor   of   the   plaintiff,   hence   the   defendant   appealed   presenting  
Article   816.  The   will   of   an   alien   who   is   abroad   produces   effect   in   the   Philippines   minority  as  his  special  defense.  By  reason  of  the  fact  that  under  the  laws  of  the  
if   made   with   the   formalities   prescribed   by   the   law   of   the   place   in   which   he   Philippines,  contracts  made  by  person  who  did  not  reach  majority  age  of  23  are  
resides,   or   according   to   the   formalities   observed   in   his   country,   or   in   conformity   unenforceable.   Defendant   claim   that   he   is   an   adult   when   he   left   Chicago   but  
with  those  which  this  Code  prescribes.   was  a  minor  when  he  arrived  in  Manila  and  at  the  time  the  plaintiff  attempted  
  to  enforce  the  contract.    

 
CONFLICT  OF  LAWS                                                                                    AV  DE  TORRES   161  
ATTY.  ARIS  L.  GULAPA                                            AY  2015-­‐2016  
  went   before   a   notary   public   and   executed   two   instruments,   wherein   and  
ISSUE:     whereby  she  emancipated  her  two  sons,  with  their  consent  and  acceptance.  No  
Whether  the  contract  is  valid   guardian  of  the  person  or  property  of  these  two  sons  had  ever  been  applied  for  
  or  appointed  under  or  by  virtue  of  the  provisions  of  the  Code  of  Civil  Procedure  
HELD:   since   the   promulgation   of   the   Code   in   1901.   After   the   execution   of   Exhibit   T  
Yes.   Mr.   Frank   being   fully   qualified   to   enter   into   a   contract   at   the   place   and   and   U,   both   Joaquin   Ibañez   de   Aldecoa   and   Zoilo   Ibañez   de   Aldecoa  
time   the   contract   was   made,   he   cannot   therefore   plead   infancy   as   a   defense   at   participated   in   the   management   of   Aldecoa   and   Co,   as   partners   by   being  
the  place  where  the  contract  is  being  enforced.  Although  Mr.  Frank  was  still  a   present  and  voting  at  meetings  of  the  partners  of  the  company  upon  matters  
minor   under   Philippine   laws,   he   was   nevertheless   considered   an   adult   under   connected  with  its  affairs.    
the   laws   of   the   state   of   Illinois,   the   place   where   the   contract   was   made.  No    
rule   is   better   settled   in   law   than   that   matters   bearing   upon   the   execution,   On  the  23rd  of  February,  1906,  the  defendant  firm  of  Aldeco  and  Co.  obtained  
interpretation   and   validity   of   a   contract   are   determined   by   the   law   of   the   from  the  bank  a  credit  in  account  current  up  to  the  sum  of  P450,000  upon  the  
place  where  the  contract  is  made.  Matters  connected  to  its  performance  are   terms  and  conditions  set  forth  in  the  instrument  executed  on  that  date  (Exhibit  
regulated   by   the   law   prevailing   at   the   place   of   its   performance.   Matters   A).   Later   it   was   agreed   that   the   defendants,   Isabel   Palet   and   her   two   sons,  
respecting  a  remedy,  such  as  bringing  of  a  suit,  admissibility  of  evidence,  and   Joaquin   and   Zoilo,   should   mortgage,   in   addition   to   certain   securities   of   Aldecoa  
statutes   of   limitations,   depend   upon   the   law   of   the   place   where   the   suit   is   and   Co.,   as   set   forth   in   Exhibit   A,   certain   of   their   real   properties   as   additional  
brought.     security   for   the   obligations   of   Aldecoa   and   Co.   So,   on   March   23,   1906,   the  
  mortgage,   Exhibit   B,   was   executed   wherein   certain   corrections   in   the  
98.  IBANEZ  V.  HONGKONG  &  SHANGHAI  BANK   description   of   some   of   the   real   property   mortgaged   to   the   bank   by   Exhibit   A  
G.R.  No.  L-­‐6889  |  March  23,  1915   were  made  and  the  amount  for  which  each  of  the  mortgaged  properties  should  
  be   liable   was   set   forth.   These   two   mortgages,   Exhibits   A   and   B,   were   duly  
N.B.:  Not  discussed.   recorded  in  the  registry  of  property  of  the  city  of  Manila  on  March  23,  1906.    
   
FACTS:   On   the   31st   day   of   December,   1906,   the   firm   of   Aldecoa   and   Co.   went   into  
The   defendants,   Joaquin   Ibañez   de   Aldecoa,   Zoilo   Ibañez   de   Aldecoa,   and   liquidation   on   account   of   the   expiration   of   the   term   for   which   it   had   been  
Cecilia   Ibañez   de   Aldecoa,   were   born   in   the   Philippine   Islands   on   March   27,   organized,   and   the   intervener,   Urquhart,   was   duly   elected   by   the   parties   as  
1884,   July   4,   1885,   and   1887,   respectively,   the   legitimate   children   of   Zoilo   liquidator,   and   be   resolution   dated   January   24,   1907,   he   was   granted   the  
Ibañez   de   Alcoa   and   the   defendant,   Isabel   Palet.   Both   parents   were   native   of   authority  expressed  in  that  resolution.  
Spain.   The   father's   domicile   was   in   Manila,   and   he   died   here   on   October   4,    
1895.  The  widow,  still  retaining  her  Manila  domicile,  left  the  Philippine  Islands   On   June   30,   1907,   Aldeco   and   Co.   in   liquidation,   for   the   purposes   of   certain  
and  went  to  Spain  in  1897  because  of  her  health,  and  did  not  return  until  the   litigation  about  to  be  commenced  in  its  behalf,  required  an  injunction  bond  in  
latter  part  of  1902.  The  firm  of  Aldecoa  &  Co.,  of  which  Zoilo  Ibañez  de  Aldecoa,   the  sum  of  P50,000,  which  was  furnished  by  the  bank  upon  the  condition  that  
deceased,   had   been   a   member   and   managing   director,   was   reorganized   in   any  liability  incurred  on  the  part  of  the  bank  upon  this  injunction  bond  would  
December,   1896,   and   the   widow   became   one   of   the   general   or   "capitalistic"   be  covered  by  the  mortgage  of  February  23,  1906.  An  agreement  to  this  effect  
partners   of   the   firm.   The   three   children,   above   mentioned,   appear   in   the   was   executed   by   Aldecoa   and   Co.   in   liquidation,   by   Isabel   Palet,   by   Joaquin  
articles  of  agreement  as  industrial  partners.     Ibañez   de   Aldecoa,   who   had   then   attained   his   full   majority,   and   by   Zoilo   Ibañez  
`   de   Aldecoa,   who   was   not   yet   twenty-­‐three   years   of   age.   In   1908,   Joaquin  
On   July   31,   1903,   Isabel   Palet,   the   widowed   mother   of   Joaquin   Ibañez   de   Ibañez   de   Aldecoa,   Zoilo   Ibañez   de   Aldecoa,   and   Cecilia   Ibañez   de   Aldecoa  
Aldecoa  and  Zoilo  Ibañez  de  Aldecoa,  who  were  then  over  the  age  of  18  years,   commenced  an  action  against  their  mother,  Isabel  Palet,  and  Aldecoa  and  Co.,  
in  which  the  bank  was  not  a  party,  and  in  September  of  that  year  procured  a  
 
CONFLICT  OF  LAWS                                                                                    AV  DE  TORRES   162  
ATTY.  ARIS  L.  GULAPA                                            AY  2015-­‐2016  
judgment  of  the  Court  of  First  Instance  annulling  the  articles  of  copartnership   March   13,   1907,   Exhibit   V),   and   on   the   30th   day   of   August   of   that   year  
of  Aldecoa  and  Co.,  in  so  far  as  they  were  concerned,  and  decreeing  that  they   Urquhart,   as   liquidator,   under   the   authority   vested   in   him   as   such,   and   in  
were  creditors  and  not  partners  of  that  firm.     compliance  with  the  terms  of  the  contract  of  June  13,  1907,  mortgaged  to  the  
  plaintiff,   by   way   of   additional   security   for   the   performance   of   the   obligations  
The   real   property   of   the   defendant   Isabel   Palet,   mortgaged   to   the   plaintiff,   set   forth   in   Exhibits   A   and   B,   the   312   shares   of   the   Pasay   Estate   Company,  
corporation  by  the  instrument  of  March  23,  1906,  was,  at  the  instance  of  the   Limited,  acquired  by  Aldecoa  and  Co.    
defendant,  registered  under  the  provisions  of  the  Land  Registration  Act,  subject    
to  the  mortgage  thereon  in  favor  of  the  plaintiff,  by  decree,  of  the  land  court   On   the   31st   day   of   March,   1907,   Aldecoa   and   Co.   mortgaged,   as   additional  
dated  March  8,  1907.     security   for   the   performance   of   those   obligations,   to   the   plaintiff   the   right   of  
  mortgage,  pertaining  to  the  firm  of  Aldecoa  and  Co.,  upon  certain  real  estate  in  
On   the   6th   of   November,   1906,   the   defendants,   Isabel   Palet   and   her   three   that   Province   of   Ambos   Camarines,   mortgaged   to   Aldecoa   and   Co.   by   one  
children,  Joaquin  Ibañez  de  Aldecoa,  Zoilo  Ibañez  de  Aldecoa,  and  Cecilia  Ibañez   Andres   Garchitorena   to   secure   a   balance   of   indebtedness   to   that   firm   of   the  
de   Aldecoa,   applied   to   the   land   court   for   the   registration   of   their   title   to   the   sum   of   P20,280.19.   The   mortgage   thus   created   in   favor   of   the   bank   was   duly  
real   property   described   in   paragraph   4   of   the   instrument   of   March   23,   1906   recorded   in   the   registry   of   deeds   of   that   province.   On   the   31st   day   of   March,  
(Exhibit   B),   in   which   application   they   stated   that   the   undivided   three-­‐fourths   of   1907,   Aldecoa   and   Co.   mortgaged   as   further   additional   security   for   the  
said   properties   belonging   to   the   defendants,   Isabel   Palet,   Joaquin   Ibañez   de   performance  of  the  obligations,  the  right  of  mortgage  pertaining  to  the  firm  of  
Aldecoa,   and   Zoilo   Ibañez   de   Aldecoa,   were   subject   to   the   mortgage   in   favor   of   Aldecoa  and  Co.  upon  other  real  property  in  the  same  province,  mortgaged  by  
the   plaintiff   to   secure   the   sum   of   P203,985.97   under   the   terms   of   the   the   firm   of   Tremoya   Hermanos   and   Liborio   Tremoya,   to   secure   the  
instrument  dated  March  22,  1906.  Pursuant  to  this  petition  the  Court  of  Land   indebtedness   of   that   firm   to   the   firm   of   Aldecoa   and   Co.   of   P43,117.40   and   the  
Registration,   by   decree   dated   September   8,   1907,   registered   the   title   to   the   personal  debt  of  the  latter  of  P75,463.54.  The  mortgage  thus  created  in  favor  
undivided   three-­‐fourths   interest   therein   pertaining   to   the   defendants,   Isabel   of  the  bank  was  filed  for  record  with  the  registrar  of  deeds  of  that  province.    
Palet   and   her   two   sons,   Joaquin   and   Zoilo,   to   the   mortgage   in   favor   of   the    
plaintiff  to  secure  the  sun  of  P203,985.97.     On   the   30th   day   of   January,   1907,   Aldecoa   and   Co.   duly   authorized   the   bank   to  
  collect   from   certain   persons   and   firms,   named   in   the   instrument   granting   this  
On   December   22,   1906,   Aldecoa   and   Co.,   by   a   public   instrument   executed   authority,  any  and  all  debts  owing  by  them  to  Aldecoa  and  Co.  and  to  apply  all  
before   a   notary   public,   as   additional   security   for   the   performance   of   the   amounts   so   collected   to   the   satisfaction,   pro   tanto,   of   any   indebtedness   of  
obligations  in  favor  of  the  plaintiff  under  the  terms  of  the  contracts  Exhibits  A   Aldecoa  and  Co.  to  the  bank.    
and  B,  mortgaged  to  the  bank  the  right  of  mortgage  pertaining  to  Aldecoa  and    
Co.   upon   certain   real   property   in   the   Province   of   Albay,   mortgaged   to   said   By   a   public   instrument   dated   February   18,   1907,   Aldecoa   and   Co.  
company  by  one  Zubeldia  to  secure  an  indebtedness  to  that  firm.  Subsequent   acknowledged   as   indebtedness   to   Joaquin   Ibañez   de   Aldecoa   in   the   sum   of  
to  the  execution  of  this  instrument,  Zubeldia  caused  his  title  to  the  mortgaged   P154,589.20,   a   like   indebtedness   to   Zoilo   Ibañez   de   Aldecoa   in   the   sum   of  
property   to   be   registered   under   the   provisions   of   the   Land   Registration   Act,   P89,177.07.   On   September   30,   1908,   Joaquin,   Zoilo,   and   Cecilia   recovered   a  
subject  to  a  mortgage  of  Aldecoa  and  Co.  to  secure  the  sum  of  P103,943.84  and   judgment  in  the  Court  of  First  Instance  of  Manila  for  the  payment  to  them  of  
to  the  mortgage  of  the  mortgage  right  of  Aldecoa  and  Co.  to  the  plaintiff.     the   sum   of   P155,127.31,   as   the   balance   due   them   upon   the   indebtedness  
  acknowledged  in  the  public  instrument  dated  February  18,  1907.    
As  the  result  of  the  litigation  Aldecoa  and  Co.  and  A.  S.  Macleod,  wherein  the   On   November   30,   1907,   Joaquin,   Zoilo,   and   Cecilia   instituted   an   action   in   the  
injunction  bond  for  P50,000  was  made  by  the  bank  in  the  manner  and  for  the   Court  of  First  Instance  of  the  city  of  the  Manila  against  the  plaintiff  bank  for  the  
purpose   above   set   forth,   Aldecoa   and   Co.   became   the   owner,   through   a   purpose   of   obtaining   a   judicial   declaration   to   the   effect   that   the   contract  
compromise  agreement  executed  in  Manila  on  the  14th  of  August,  1907,  of  the   whereby  Aldecoa  and  Co.  mortgaged  to  the  bank  the  shares  of  the  Pasay  Estate  
shares   of   the   Pasay   Estate   Company   Limited   (referred   to   in   the   contract   of   Company   recovered   from   Alejandro   S.   Macleod,   was   null   and   void,   and   for   a  
 
CONFLICT  OF  LAWS                                                                                    AV  DE  TORRES   163  
ATTY.  ARIS  L.  GULAPA                                            AY  2015-­‐2016  
judgment   of   that   these   shares   be   sold   and   applied   to   the   satisfaction   of   their   induced,   or   attempted   to   induce,   any   customer   of   Aldecoa   and   Co.   to  
judgment   obtained   on   September   30,   1908.   Judgment   was   rendered   by   the   discontinue   business   relations   with   that   company.   The   court   further   found   that  
lower   court   in   favor   of   the   plaintiffs   in   that   action   in   accordance   with   their   Urquhart   had   failed   to   show   that   he   had   any   legal   interest   in   the   matter   in  
prayer,   but   upon   appeal   this   court   reversed   that   judgment   and   declared   that   litigadtion  between  plaintiff  and  defendants,  or  in  the  success  of  either  of  the  
the   mortgage   of   the   shares   of   stock   in   the   Pasay   Estate   Co.   to   the   bank   was   parties,  or  an  interest  against  both,  as  required  by  section  121  of  the  Code  of  
valid.     Civil  Procedure.    
   
In   October,   1908,   Joaquin   and   Zoilo   Ibañez   de   Aldecoa   instituted   an   action   ISSUES:    
against   the   plaintiff   bank   for   the   purpose   of   obtaining   a   judgment   annulling   1.   W/N   the   court   erred   in   overruling   the   defendant's   demurrer   based   upon   the  
the   mortgages   created   by   them   upon   their   interest   in   the   properties   alleged  ambiguity  and  vagueness  of  the  complaint  
described  in  Exhibits  A  and  B,  upon  the  ground  that  the  emancipation  by  their   2.  W/N  the  court  erred  in  ruling  that  there  was  no  competent  evidence  that  the  
mother   was   void   and   of   no   effect,   and   that,   therefore,   they   were   minors   plaintiff   had   induced   Aldecoa   and   Co.'s   provincial   debtors   to   cease   making  
incapable  of  creating  a  valid  mortgage  upon  their  real  property.  The  Court  of   consignments  to  that  firm  
First   Instance   dismissed   the   complaint   as   to   Joaquin   upon   the   ground   that   he   3.  W/N  the  court  erred  in  rendering  a  judgment  in  a  special  proceeding  for  the  
had   ratified   those   mortgages   after   becoming   of   age,   but   entered   a   judgment   foreclosure   of   a   mortgage,   Aldecoa   and   Co.   not   having   mortgaged   any   real  
annulling   said   mortgages   with   respect   to   Zoilo.   Both   parties   appealed   from   this   estate  of  any  kind  within  the  jurisdiction  of  the  trial  court,  and  the  obligation  of  
decision  and  the  case  was  given  registry  No.  6889  in  the  Supreme  Court.     the   persons   who   had   signed   the   contract   of   suretyship   in   favor   of   the   bank  
  having  been  extinguished  by  operation  of  law.    
On   the   31st   day   of   December,   1906,   on   which   date   the   defendant   Aldecoa   and    
Co.   went   into   liquidation,   the   amount   of   indebtedness   to   the   bank   upon   the   HELD:  
overdraft   created   by   the   terms   of   the   contract,   Exhibit   A,   was   P516,517.98.   With   respect   to   the   contention   set   forth   in   the   s   assignment   of   error   to   the  
Neither  the  defendant  Aldecoa  and  Co.,  nor  any  of  the  defendants  herein,  have   effect   that   the   bank   has   prejudiced   Aldecoa   and   Co.   by   having   induced  
paid   or   caused   to   be   paid   to   the   bank   the   yearly   partial   payments   due   under   customers  of  the  latter  to  cease  their  commercial  relations  with  this  defendant,  
the   terms   of   the   contract,   Exhibit   A.   But   from   time   to   time   the   bank   has   the   ruling   of   the   court   that   there   is   no   evidence   to   show   that   there   was   any  
collected  and  received  from  provincial  debtors  of  Aldecoa  and  Co.  the  various   such   inducement   is   fully   supported   by   the   record.   It   may   be   possible   that   some  
sums  shown  in  Exhibit  Q,  all  of  which  sums  so  received  have  been  placed  to  the   of   Aldecoa   and   Co.'s   customers   ceased   doing   business   with   that   firm   after   it  
credit   of   Aldecoa   and   Co.   and   notice   duty   given.   Also,   the   bank,   from   time   to   went   into   liquidation.   This   is   the   ordinary   effect   of   a   commercial   firm   going  
time,   since   the   date   upon   which   Aldecoa   and   Co.   went   into   liquidation,   has   consideration,  for  the  reason  that  it  was  a  well  known  fact  that  Aldecoa  and  Co.  
received   various   other   sums   from,   or   for   the   account   of,   Aldecoa   and   Co.,   all   of   was  insolvent.  It  is  hardly  probable  that  the  bank,  with  so  large  a  claim  against  
which   have   been   duly   placed   to   the   credit   of   that   firm,   including   the   sum   of   Aldecoa  and  Co.  and  with  unsatisfactory  security  for  the  payment  of  its  claim,  
P22,552.63,  the  amount  of  the  credit  against  one  Achaval,  assigned  to  the  bank   would   have   taken   any   action   whatever   which   might   have   had   the   effect   of  
by   Aldecoa   and   Co.   The   balance   to   the   credit   of   the   bank   on   the   31st   day   of   diminishing   Aldecoa   and   Co.'s   ability   to   discharge   their   claim.   The   contention  
December,  1911,  as  shown  on  the  books  of  Aldecoa  and  Co.,  was  for  the  sum  of   that  the  customers  of  Aldecoa  and  Co.  included  in  the  list  of  debtors  ceased  to  
P416.853.46.   It   appeared   that   an   error   had   been   committed   by   the   bank   in   make   consignments   to   the   firm   because   they   had   been   advised   by   the   bank  
liquidating   the   interest   charged   to   Aldecoa   and   Co.,   and   this   error   was   that  Aldecoa  and  Co.  had  authorized  the  bank  to  collect  these  credits  from  the  
corrected   so   that   the   actual   amount   of   the   indebtedness   of   Aldecoa   and   Co.   to   defendant's   provincial   customers   and   apply   the   amounts   so   collected   to   the  
the  plaintiff  on  the  15th  of  February,  1912,  with  interest  to  December  10,  1912,   partial   discharge   of   the   indebtedness   of   the   defendant   to   the   bank.  
the  date  of  the  judgment,  the  amount  was  P344,924.23.     Furthermore,   the   bank   was   expressly   empowered   to   take   any   steps   which  
  might   be   necessary,   judicially   or   extrajudicially,   for   the   collection   of   these  
The   trial   court   found   that   there   was   no   competent   evidence   that   the   bank   credits.  The  real  reason  which  caused  the  defendant's  provincial  customers  to  
 
CONFLICT  OF  LAWS                                                                                    AV  DE  TORRES   164  
ATTY.  ARIS  L.  GULAPA                                            AY  2015-­‐2016  
cease  making  shipments  was  due  to  the  fact  that  the  defendant,  being  out  of    
funds,  could  not  give  its  customers  any  further  credit.  It  is  therefore  clear  that   *N.B.:  
the  bank,  having  exercised  the  authority  conferred  upon  it  by  the  company  in  a   What  if  the  plaintiff  was  a  corporation?    
legal   manner,   is   not   responsible   for   any   damages   which   might   have   resulted   DOING  BUSINESS  
from   the   failure   of   the   defendant's   provincial   customers   to   continue   doing     WITH  LICENSE  –MAY  SUE  AND  BE  SUED  
business  with  that  firm.       WITHOUT  LICENSE  –MAY  NOT  SUE  BUT  MAY  BE  SUED  
   
In  the  third  assignments  of  errors  two  propositions  are  insisted  upon:  (1)  that  in   ISOLATED  TRANSACTION  
these   foreclosure   proceedings   the   court   was   without   jurisdiction   to   render     MAY   SUE   BUT   MAY   NOT   BE   SUED   (NO   JURISDICTION   OVER   FOREIGN  
judgment  against  Aldecoa  and  Co.  for  the  reason  that  firm  had  mortgaged  no   CORPORAITONS)   EXCEPT   IF   IT   CONSENTED   THROUGH   CONTRACTUAL  
real   property   within   the   city   of   Manila   to   the   plaintiff;   and   (2)   that   the   STIPULATION  
mortgages   given   by   this   defendant   have   been   extinguished   by   reason   of   the    
fact   that   the   bank   extended   the   time   within   which   the   defendant's   provincial   Corporation   A   is   a   foreign   corporation   doing   business   in   the   Philippines   but  
debtors  might  make  their  payments.     without   a   license.   It   entered   into   a   contract   with   a   Philippine   domestic  
  corporation.   Before   suing   the   domestic   corporation,   it   obtained   a   license.   Can   it  
We   understand   that   the   bank   is   not   seeking   to   exercise   its   mortgages   rights   sue?  Yes.    
upon   the   mortgages   which   the   defendant   firm   holds   upon   certain   real    
properties   in   the   Provinces   of   Albay   and   Ambros   Camarines   and   to   sell   these   Subsequent  acquisition  of  the  license  will  cure  the  lack  of  capacity  to  sue  at  the  
properties  at  public  auction  in  these  proceedings.  Nor  do  we  understand  that   time   of   the   execution   of   the   contract.   It   can   then   re-­‐file   the   suit   on   the   same  
the  judgment  of  the  trial  courts  directs  that  this  be  done.  Before  that  property   contract  or  transaction.    
can  be  sold  the  original  mortgagors  will  have  to  be  made  parties.  The  bank  is    
not  trying  to  foreclose,  in  this  section,  any  mortgages  on  real  property  executed   Home  Insurance  Company  v.  ESL,  G.R.  No.  L-­‐34382,  July  20,  1983  
by   Aldecoa   and   Co.   It   is   true   that   the   bank   sought   and   obtained   a   money    
judgment   against   that   firm,   and   at   the   same   time   and   in   the   same   action   Eriks  PTE.  Ltd  v.  CA,  G.R.  No.  118843,  February  6,  1997  
obtained  a  foreclosure  judgment  against  the  other  defendants.  If  two  or  more    
persons  are  in  solidum  the  debtors  mortgage  any  of  their  real  property  situate   FACTS:  
in   the   jurisdiction   of   the   court,   the   creditor,   in   case   of   the   solidary   debtors   in   This   is   an   action   to   collect   P59,082.92,   together   with   lawful   interests   from   14  
the  same  suit  and  secure  a  joint  and  several  judgment  against  them,  as  well  as   October   1947,   the   date   of   the   written   demand   for   payment,   and   costs.   The  
judgments  of  foreclosure  upon  the  respective  mortgages.     claim  arises  out  of  a  shipment  of  1,000  tons  of  coconut  oil  emulsion  sold  by  the  
  plaintiff,   as   agent   of   the   defendant,   to   Jas.   Maxwell   Fassett,   who   in   turn  
100.  KING  MAU  WU  V.  SYCIP   assigned  it  to  Fortrade  Corporation.  Under  an  agency  agreement  set  forth  in  a  
G.R.  No.  L-­‐5897  |  April  23,  1954   letter   dated   7   November   1946   in   New   York   addressed   to   the   defendant   and  
  accepted   by   the   latter   on   the   22nd   day   of   the   same   month,   the   plaintiff   was  
DOCTRINE:   A   non-­‐resident   may   sue   a   resident   in   the   courts   of   this   country   made   the   exclusive   agent   of   the   defendant   in   the   sale   of   coconut   oil   and   its  
where   the   defendant   may   be   summoned   and   his   property   leviable   upon   derivatives   outside   the   Philippines   and   was   to   be   paid   2   1/2   per   cent   on   the  
execution  in  the  case  of  a  favorable,  final  and  executory  judgment.  There  is  no   total  actual  sale  price  of  sales  obtained  through  his  efforts  in  addition  thereto  
conflict  of  laws  involved  in  the  case,  because  it  is  only  a  question  of  enforcing  an   50  per  cent  of  the  difference  between  the  authorized  sale  price  and  the  actual  
obligation   created   by   or   arising   from   contract;   and   unless   the   enforcement   of   sale  price.    
the  contract  be  against  public  policy  of  the  forum,  it  must  be  enforced.    

 
CONFLICT  OF  LAWS                                                                                    AV  DE  TORRES   165  
ATTY.  ARIS  L.  GULAPA                                            AY  2015-­‐2016  
After  the  trial  where  the  depositions  of  the  plaintiff  and  of  Jas,  Maxwell  Fassett   101.  SHEWARAM  V.  PHILIPPINE  AIRLINES  
and   several   letters   in   connection   therewith   were   introduced   and   the   testimony   G.R.  No.  L-­‐20099  |  July  7,  1966  
of  the  defendant  was  heard,  the  Court  rendered  judgment  as  prayed  for  in  the    
complaint.  A  motion  for  reconsideration  was  denied.  A  motion  for  a  new  trial   DOCTRINE:  The  fact  that  those  conditions  are  printed  at  the  back  of  the  ticket  
was   filed,   supported   by   the   defendant's   affidavit,   based   on   newly   discovered   stub   in   letters   so   small   that   they   are   hard   to   read   would   not   warrant   the  
evidence   which   consists   of   a   duplicate   original   of   a   letter   dated   16   October   presumption   that   the   appellee   was   aware   of   those   conditions   such   that   he   had  
1946  covering  the  sale  of  1,000  tons  of  coconut  oil  soap  emulsion  signed  by  Jas.   "fairly  and  freely  agreed"  to  those  conditions.  
Maxwell  Fassett  assigned  by  the  latter  to  the  defendant;  the  letter  of  credit  No.    
20122  of  the  Chemical  Bank  &  Trust  Company  in  favor  of  Jas.  Maxwell  Fassett   *N.B.:  Art.  1750.  A  contract  fixing  the  sum  that  may  be  recovered  by  the  owner  
assigned  by  the  latter  to  the  defendant;  and  a  letter  dated  16  December  1946   or   shipper   for   the   loss,   destruction,   or   deterioration   of   the   goods   is   valid,   if   it   is  
by   the   Fortrade   Corporation   to   Jas.   Maxwell   Fassett   accepted   it   on   24   reasonable   and   just   under   the   circumstances,   and   has   been   fairly   and   freely  
December  1946,  all  of  which  documents,  according  to  the  defendant,  could  not   agreed  upon.  [RJFF]  
be   produced   at   the   trial,   despite   the   use   of   reasonable   diligence,   and   if    
produced   they   would   alter   the   result   of   the   controversy.   The   motion   for   new   FACTS:  
trial  was  denied.  The  defendant  is  appealing  from  said  judgment.     Parmanand   Shewaram   was   a   paying   passenger   on   PAL’s   aircraft   from  
  Zamboanga   to   Manila   and   checked   in   a   suitcase   and   2   other   pieces.   The  
ISSUE:   suitcase   was   mistagged   by   PAL’s   personnel   in   Zamboanga   as   IGN   (for   Iligan)  
Whether   a   contract   executed   in   a   foreign   country   is   cognizable   by   Philippine   instead  of  MNL  (for  Manila),  thus  Shewaram  was  unable  to  receive  his  luggage  
courts   in   Manila.   He   made   a   claim   with   PAL’s   personnel   in   Manila   and   another  
  suitcase  similar  to  his  own  which  was  the  only  baggage  left  for  that  flight,  was  
HELD:   given  to  the  plaintiff  for  him  to  take  delivery,  but  he  refused  on  the  ground  that  
The   contention   that   as   the   contract   was   executed   in   New   York,   the   Court   of   it   was   not   his.   When   the   suitcase   was   located,   PAL   personnel   delivered   it   to  
First   Instance   of   Manila   has   no   jurisdiction   over   this   case,   is   without   merit,   plaintiff,   but   upon   checking,   Shewaram’s   Transistor   Radio   7   and   Rollflex  
because  a  non-­‐resident  may  sue  a  resident  in  the  courts  of  this  country  where   Camera  were  missing.  
the  defendant  may  be  summoned  and  his  property  leviable  upon  execution  in    
the   case   of   a   favorable,   final   and   executory   judgment.  It  is  a  personal  action   The   PAL   ticket,   on   the   reverse   side,   stated   in   fine   print   that:   The   liability,   if   any,  
for   the   collection   of   a   sum   of   money   which   the   Courts   of   First   Instance   have   for   loss   or   damage   to   checked   baggage   or   for   delay   in   the   delivery   thereof   is  
jurisdiction  to  try  and  decide.  There  is  no  conflict  of  laws  involved  in  the  case,   limited   to   its   value   and,   unless   the   passenger   declares   in   advance   a   higher  
because  it  is  only  a  question  of  enforcing  an  obligation  created  by  or  arising   valuation   and   pay   an   additional   charge   therefor,   the   value   shall   be   conclusively  
from  contract;  and  unless  the  enforcement  of  the  contract  be  against  public   deemed  not  to  exceed  P100.00  for  each  ticket.  
policy  of  the  forum,  it  must  be  enforced.    
   
The   plaintiff   is   entitled   to   collect   P7,589.88   for   commission   and   P50,000   for   ISSUE:  
one-­‐half   of   the   overprice,   or   a   total   of   P57,589.88,   lawful   interests   thereon   Whether  the  limited  liability  rule  shall  apply  in  the  case  at  bar?    
from  the  date  of  the  filing  of  the  complaint,  and  costs  in  both  instances.      
  HELD:  
NO.  The  law  that  may  be  invoked,  in  this  connection  is  Article  1750  of  the  New  
Civil  Code  which  provides  as  follows:  
 

 
CONFLICT  OF  LAWS                                                                                    AV  DE  TORRES   166  
ATTY.  ARIS  L.  GULAPA                                            AY  2015-­‐2016  
A  contract  fixing  the  sum  that  may  be  recovered  by  the  owner  or  shipper    
for   the   loss,   destruction,   or   deterioration   of   the   goods   is   valid,   if   it   is   While   it   may   be   true   that   Ong   Yiu   had   not   signed   the   plane   ticket,   he   is  
reasonable   and   just   under   the   circumstances,   and   has   been   fairly   and   nevertheless  bound  by  the  provisions  thereof.  “Such  provisions  have  been  held  
freely  agreed  upon.   to   be   a   part   of   the   contract   of   carriage,   and   valid   and   binding   upon   the  
  passenger   regardless   of   the   latter’s   lack   of   knowledge   or   assent   to   the  
In   accordance   with   the   above-­‐quoted   provision   of   Article   1750   of   the   New   Civil   regulation”.  It  is  what  is  known  as  a  contract  of  “adhesion”.  
Code,  the  pecuniary  liability  of  a  common  carrier  may,  by  contract,  be  limited    
to   a   fixed   amount.   It   is   required,   however,   that   the   contract   must   be   Kyna’s  notes:  
"reasonable  and  just  under  the  circumstances  and  has  been  fairly  and  freely   Compare  Shewaram  and  Ong  Yiu  Case.  In  Shewaram,  the  stipulation  cannot  be  
agreed  upon."   said  to  have  been  freely  agreed  upon  because  they  were  in  small  hard  to  read  
  font.  
In   the   case   before   us   We   believe   that   the   requirements   of   said   article   have   not    
been   met.   It   cannot   be   said   that   the   appellee   had   actually   entered   into   a   Do  other  stipulations  fall  under  this  pronouncement?  
contract  with  the  appellant,  embodying  the  conditions  as  printed  at  the  back   No.  Limitation  of  liability  of  this  kind  is  specially  covered  by  Article  1750.  It  must  
of  the  ticket  stub  that  was  issued  by  the  appellant  to  the  appellee.  The  fact   be  FAIRLY  and  FREELY  agreed  upon.    
that   those   conditions   are   printed   at   the   back   of   the   ticket   stub   in   letters   so    
small   that   they   are   hard   to   read   would   not   warrant   the   presumption   that   the   FACTS:  
appellee   was   aware   of   those   conditions   such   that   he   had   "fairly   and   freely   Ong   Yiu   was   scheduled   to   travel   from   Cebu   to   Butuan   on   board   a   PAL   flight.   He  
agreed"   to   those   conditions.   The   trial   court   has   categorically   stated   in   its   checked   in   one   piece   of   baggage   (a   blue   maleta)   containing   important  
decision   that   the   "Defendant   admits   that   passengers   do   not   sign   the   ticket,   documents   that   he   needed   for   the   trials   he   was   scheduled   to   attend.   Upon  
much   less   did   plaintiff   herein   sign   his   ticket   when   he   made   the   flight   on   arrival   at   Butuan,   he   discovered   that   the   luggage   was   missing   and   demanded  
November  23,  1959."  We  hold,  therefore,  that  the  appellee  is  not,  and  cannot   that  PAL  deliver  it  to  him  the  following  day.  PAL,  despite  their  extensive  efforts,  
be,   bound   by   the   conditions   of   carriage   found   at   the   back   of   the   ticket   stub   failed  to  deliver  the  baggage  on  time,  causing  Ong  Yiu  to  postpone  the  hearing.  
issued  to  him  when  he  made  the  flight  on  appellant's  plane  on  November  23,   Ong  Yiu  sued  PAL  for  damages  for  breach  of  contract  of  transportation.    
1959.    
  The   pertinent   Condition   of   Carriage   printed   at   the   back   of   the   plane   ticket  
It  having  been  clearly  found  by  the  trial  court  that  the  transistor  radio  and  the   reads:  
camera  of  the  appellee  were  lost  as  a  result  of  the  negligence  of  the  appellant    
as   a   common   carrier,   the   liability   of   the   appellant   is   clear   —   it   must   pay   the   8.   BAGGAGE   LIABILITY   ...   The   total   liability   of   the   Carrier   for   lost   or  
appellee  the  value  of  those  two  articles.   damaged   baggage   of   the   passenger   is   LIMITED   TO   P100.00   for   each  
  ticket   unless   a   passenger   declares   a   higher   valuation   in   excess   of  
102.  ONG  YIU  V.  CA   P100.00,   but   not   in   excess,   however,   of   a   total   valuation   of   P1,000.00  
1979   and  additional  charges  are  paid  pursuant  to  Carrier's  tariffs.  
   
DOCTRINE:  The   validity   of   this   stipulation   is   not   questioned   by   the   plaintiff.   They    
are   printed   in   reasonably   and   fairly   big   letters,   and   are   easily   readable.   ISSUES:    
Moreover,  plaintiff  had  been  a  frequent  passenger  of  PAL  from  Cebu  to  Butuan   Whether   Ong   Yiu   is   entitled   to   damages;   does   the   doctrine   of   limited   liability  
City  and  back,  and  he,  being  a  lawyer  and  businessman,  must  be  fully  aware  of   apply    
these  conditions.    

 
CONFLICT  OF  LAWS                                                                                    AV  DE  TORRES   167  
ATTY.  ARIS  L.  GULAPA                                            AY  2015-­‐2016  
HELD:    
As  a  general  proposition,  the  plaintiff's  maleta  having  been  pilfered  while  in  the   EXC:   When   the   conditions   printed   at   the   back   of   the   ticket   were   so   small   and  
custody   of   the   defendant,   it   is   presumed   that   the   defendant   had   been   hard  to  read  that  they  would  not  warrant  the  presumption  that  the  passenger  
negligent.   The   liability,   however,   of   PAL   for   the   loss,   in   accordance   with   the   was  aware  of  the  conditions  and  that  he  had  freely  and  fairly  agreed  thereto.  
stipulation   written   on   the   back   of   the   ticket,   Exhibit   12,   is   limited   to   P100.00    
per  baggage,  plaintiff  not  having  declared  a  greater  value,  and  not  having  called   FACTS:  
the   attention   of   the   defendant   on   its   true   value   and   paid   the   tariff   therefor.   PanAm   Airlines   refused   to   accommodate   Respondent   Tinitigan   on   Pan   Am  
The   validity   of   this   stipulation   is   not   questioned   by   the   plaintiff.   They   are   Flight  No.  431  from  Sto.  Domingo,  Republica  Dominica  to  San  Juan,  Puerto  Rico  
printed   in   reasonably   and   fairly   big   letters,   and   are   easily   readable.   notwithstanding   that   she   possessed   a   confirmed   plane   ticket.   While   plaintiff  
Moreover,   plaintiff   had   been   a   frequent   passenger   of   PAL   from   Cebu   to   was  standing  in  line  to  board  the  aircraft,  a  Pan  Am  employee  ordered  her  in  a  
Butuan   City   and   back,   and   he,   being   a   lawyer   and   businessman,   must   be   fully   loud   voice   to   step   out   of   line   because   her   ticket   was   not   confirmed   to   her  
aware  of  these  conditions.   embarrassment   in   the   presence   of   several   people   who   heard   and   order.  
  Despite  her  pleas  she  was  not  allowed  to  board  the  aircraft.  And  her  seat  was  
There   is   no   dispute   that   Ong   Yiu   did   not   declare   any   higher   value   for   his   also  given  to  a  Caucasian.  The  plane  took  off  without  her  but  with  her  luggage  
luggage,  much  less  did  he  pay  any  additional  transportation  charge.  But  Ong  Yiu   on   board.   She   was   forced   to   return   to   her   hotel   without   any   luggage   much   less  
argues   that   he   had   not   actually   entered   into   a   contract   with   PAL   limiting   the   an  extra  dress.    
latter's  liability  for  loss  or  delay  of  the  baggage  of  its  passengers,  and  that  Art.    
1750  of  the  Civil  Code  has  not  been  complied  with.   While   in   Sto.   Domingo,   Tinitigan   is   expected   to   be   in   San   Juan   that   same   day   to  
  meet   a   client   to   sign   a   contract   or   lose   it.   She   was   expected   to   make   a   profit   of  
While   it   may   be   true   that   Ong   Yiu   had   not   signed   the   plane   ticket,   he   is   $1,000   in   said   contract   but   because   she   was   unable   to   board   the   flight,   said  
nevertheless   bound   by   the   provisions   thereof.   “Such   provisions   have   been   profit  was  lost.  She  also  lost  her  luggage.  
held  to  be  a  part  of  the  contract  of  carriage,  and  valid  and  binding  upon  the    
passenger   regardless   of   the   latter’s   lack   of   knowledge   or   assent   to   the   The   airline   ticket   contains   the   following   conditions:   “NOTICE   OF   BAGGAGE  
regulation”.  It  is  what  is  known  as  a  contract  of  “adhesion”,  in  regards  which   LIABILITY  LIMITATIONS:  Liability  for  loss,  delay,  or  damage  to  baggage  is  limited  
it   has   been   said   that   contracts   of   adhesion   wherein   one   party   imposes   a   as  follows  unless  a  higher  value  is  declared  in  advance  and  additional  charges  
ready  made  form  of  contract  on  the  other,  as  the  plane  ticket  in  the  case  at   are   paid:   (1)   for   most   international   travel   (including   domestic   portions   of  
bar,   are   contracts   not   entirely   prohibited.   The   one   who   adheres   to   the   international  journeys)  to  approximately  $9.07  per  pound  ($20.00  per  kilo)  for  
contract   is   in   reality   free   to   reject   it   entirely;   if   he   adheres,   he   gives   his   checked  baggage  and  $400  per  passenger  for  unchecked  baggage…”  
consent.  "a   contract   limiting   liability   upon   an   agreed   valuation   does   not    
offend  against  the  policy  of  the  law  forbidding  one  from  contracting  against    
his  own  negligence.   ISSUE:  
  Whether  or  not  the  award  of  damages  was  proper.  
103.  PAN  AMERICAN  WORLD  AIRWAYS  V.  IAC    
1988   HELD:  
  Yes,  on  the  basis  of  the  foregoing  stipulations  printed  at  the  back  of  the  ticket,  
DOCTRINE:  GR:  The  stipulations  in  a  plane  ticket  though   not   signed,   nevertheless   Panam’s  liability  for  the  lost  baggage  of  private  respondent  Pangan  is  limited  to  
binds   the   passenger.   Such   provisions   have   been   held   to   be   a   part   of   the   $600.00  ($20.00  x  30  kilos)  as  the  latter  did  not  declare  a  higher  value  for  his  
contract   of   carriage,   and   valid   and   binding   upon   the   passenger   regardless   of   baggage  and  pay  the  corresponding  additional  charges.  
the  latter's  lack  of  knowledge  or  assent  to  the  regulation.    

 
CONFLICT  OF  LAWS                                                                                    AV  DE  TORRES   168  
ATTY.  ARIS  L.  GULAPA                                            AY  2015-­‐2016  
The  ruling  in  Ong  Yiu  v.  CA  is  applicable  in  this  case,  where  it  was  stated  that    
while   it   may   be   true   that   passenger   had   not   signed   the   plane   ticket,   he   is   Kyna’s  notes:  
nevertheless   bound   by   the   provisions   thereof.   "Such   provisions   have   been   ALG:   This   approach   is   called   by   Salonga   as   the   Center   of   gravity   rule.   In   the  
held  to  be  a  part  of  the  contract  of  carriage,  and  valid  and  binding  upon  the   absence  of  an  effective  choice  of  law,  the  courts  have  usually  applied  the  local  
passenger   regardless   of   the   latter's   lack   of   knowledge   or   assent   to   the   law  of  the  State  of  departure,  sometimes  on  the  stated  ground  that  is  was  the  
regulation.   place  of  making  or  the  center  of  gravity  of  the  contract.  The  forum  has  a  sound  
  legitimate  basis  for  the  application  of  the  policy  found  on  its  own  internal  law  
On  the  other  hand,  the  ruling  in  Shewaram  v.  Philippine  Air  Lines,  Inc.,  provided   “when  it  is  the  center  of  gravity  of  the  contract  and  has  the  most  significant  
for  the  exception  to  the  rule,  where  the  Court  held  that  the  stipulation  limiting   relationship  to  the  parties  and  the  contract.”  
the   carrier's   liability   to   a   specified   amount   was   invalid,   finds   no   application   in    
the   instant   case.   It   was   held   that   the   conditions   printed   at   the   back   of   the   FACTS:  
ticket   were   so   small   and   hard   to   read   that   they   would   not   warrant   the   Pakistan   Intl   Airlines   (PIA)   executed   2   separate   contracts   of   employments   in  
presumption   that   the   passenger   was   aware   of   the   conditions   and   that   he   had   Manila,  one  with  Farrales  and  the  other  with  Mamasig.  The  pertinent  portions  
freely   and   fairly   agreed   thereto.   In   the   instant   case,   similar   facts   that   would   of  the  contract  state  that  (1)  the  agreement  is  for  a  period  of  3  years,  but  can  
make  the  case  fall  under  the  exception  have  not  been  alleged,  much  less  shown   be  extended  by  the  mutual  consent  of  the  parties;  (2)  notwithstanding  anything  
to  exist.   to   contrary   as   herein   provided,   PIA   reserves   the   right   to   terminate   this  
  agreement   at   any   time   by   giving   the   EMPLOYEE   notice   in   writing   in   advance  
The  Court  also  rectified  the  misconception  that  the  Warsaw  Convention  which   one   month   before   the   intended   termination   or   in   lieu   thereof,   by   paying   the  
limits  a  carrier's  liability  to  US$9.07  per  pound  or  US$20.00  per  kilo  in  cases  of   EMPLOYEE  wages  equivalent  to  one  month's  salary;  (3)   this   agreement   shall   be  
contractual  breach  of  carriage  is  against  public  policy.  It  is  a  valid  provision.   construed   and   governed   under   and   by   the   laws   of   Pakistan,   and   only   the  
  Courts  of  Karachi,  Pakistan  shall  have  the  jurisdiction  to  consider  any  matter  
104.  PAKISTAN  INTERNATIONAL  AIRLINES  V.  OPLE   arising  out  of  or  under  this  agreement.  
1990    
  1  year  and  4  months  before  the  expiration  of  the  contracts  of  employment,  PIA  
CHOICE  OF  LAW   sent  separate  letters  to  Mamasig  and  Farrales,  advising  them  that  their  services  
  as   flight   stewardesses   would   be   terminated.   Farrales   and   Mamasig   filed   a  
DOCTRINE:  GR:  Lex  loci  intentionis  governs  (choice  of  law  and  choice  of  forum).   complaint   for   illegal   dismissal   and   non-­‐payment   of   company   benefits   and  
EXC:   (1)   When   the   contract   is   affected   with   public   interest   such   as   a   labor   bonuses.   PIA   contended   that   F   &   M   were   habitual   absentees   and   had   the   habit  
contract   and   (2)   when   there   are   multiple   substantive   contacts   between   Phil.   of  bringing  in  from  abroad  sizeable  quantities  of  personal  effects.  
laws  and  courts  and  the  parties.    
  ISSUE:  
The  first  clause  cannot  be  invoked  to  prevent  the  application  of  Phil.  labor  laws   Whether   the   provisions   in   the   contract   that   the   agreement   shall   be   governed  
and   regulations   to   the   subject   matter   of   the   case.   The   ER-­‐EE   relationship   by  the  laws  of  Pakistan  and  that  only  the  courts  of  Karachi,  Pakistan  shall  have  
between   PIA   and   F&M   is   affected   with   public   interest   and   the   applicable   Phil.   jurisdiction   over   any   controversy   arising   out   of   the   agreement,   may   be   given  
laws  and  regulations  cannot  be  rendered  illusory  by  the  parties  agreeing  upon   effect  
some  other  law  to  govern  their  relationship.  The  second  clause  cannot  also  be    
invoked   because   the   circumstances   of   the   case   shows   multiple   substantive   HELD:  
contacts   between   Phil.   law   and   Phil.   courts   on   the   one   hand,   and   the   No.  The  first  clause  cannot  be  invoked  to  prevent  the  application  of  Phil.  labor  
relationship  between  the  parties  on  the  other.   laws  and  regulations  to  the  subject  matter  of  the  case.  The  ER-­‐EE  relationship  

 
CONFLICT  OF  LAWS                                                                                    AV  DE  TORRES   169  
ATTY.  ARIS  L.  GULAPA                                            AY  2015-­‐2016  
between  PIA  and  F&M  is  affected  with  public  interest  and  the  applicable  Phil.   *N.B.:  Case  is  very  similar  to  question  #8  part  2  in  the  midterms  but  the  ruling  
laws  and  regulations  cannot  be  rendered  illusory  by  the  parties  agreeing  upon   in  this  case  was  not  used  as  a  basis  as  an  answer.    
some  other  law  to  govern  their  relationship.  The  second  clause  cannot  also  be   What’s  the  distinguishing  fact?  
invoked   because   the   circumstances   of   the   case   shows   multiple   substantive    
contacts   between   Phil.   law   and   Phil.   courts   on   the   one   hand,   and   the   1.  There’s  a  recruiter,  which  is  a  domestic  corporation.  
relationship   between   the   parties   on   the   other:   contract   was   executed   and   2.   Subject   contract   was   approved   by   POEA.   See   Manila   Hotel   Corp   v.   NLRC  
partially  performed  in  the  Phil.,  F&M  are  Filipino  citizens  and  PIA  is  licensed  to   (2000)  
do   business   in   the   Phil.,   and   F&M   were   based   in   the   Phil.   in   between   their    
flights.  All  the  above  contacts  point  to  the  Philippine  courts  and  administrative   This   is   not   to   say   that   Philippine   courts   and   agencies   have   no   power   to   solve  
agencies  as  a  proper  forum  for  the  resolution  of  contractual  disputes  between   controversies  involving  foreign  employers.  Neither  are  we  saying  that  we  do  not  
the   parties.   The   challenged   portion   of   the   employment   agreement   cannot   be   have   power   over   an   employment   contract   executed   in   a   foreign   country.   If  
given   effect   so   as   to   oust   Philippine   agencies   and   courts   of   the   jurisdiction   Santos   were   an   “overseas   contract   worker”,   a   Philippine   forum,   specifically  
vested   upon   them   by   Philippine   law.   Finally,   and   in   any   event,   PIA   did   not   the   POEA,   not   the   NLRC,   would   protect   him.   He   is   not   an   “overseas   contract  
undertake   to   plead   and   prove   the   contents   of   Pakistan   law   on   the   matter;   it   worker”  a  fact  which  he  admits  with  conviction.  
must   therefore   be   presumed   that   the   applicable   provisions   of   the   law   of    
Pakistan  are  the  same  as  the  applicable  provisions  of  Philippine  law.   TRUE   or   FALSE?   As   long   as   a   Filipino   enters   into   an   employment   contract,  
  Philippine   public   policy   shall   apply.   FALSE.   There   must   be   sufficient   Philippine  
As  to  the  dismissal  of  Farrales  &  Mamasig,  they  were  illegally  dismissed  and  are   contacts  before  Philippine  public  policy  applies.    
entitled  to  3  years  backwages  without  qualification  or  deduction.  PIA’s  right  to    
procedural  due  process  was  observed  as  it  was  given  the  opportunity  to  submit   Av:  The  fact  that  an  employment  contract  concerns  a  Filipino  does  not  mandate  
a   position   paper   and   present   evidence.   Also,   the   provisions   of   the   employment   the   application   of   Phil.   Laws   and   regulations   automatically   (Manila   Hotel   v.  
contract   must   not   be   contrary   to   law,   morals,   good   customs,   public   order,   NLRC).   However,   if   in   applying   the   center   of   gravity   rule,   Phil.   Laws   and  
public   policy.   The   employment   contract   prevents   security   of   tenure   of   F&M   regulations  is  found  to  be  the  center  of  the  gravity  of  the  contract  and  has  the  
from  accruing.   most  significant  relationship  to  the  parties  and  the  contract  (PIA  v.  CA)  then  it  
  will   be   applied.   This   is   supported   by   the   ruling   that   despite   a   choice   of   law  
105.  TRIPLE  EIGHT  INTEGRATED  SERVICES  V.  NLRC   clause   (PIA   v.   CA)   or   in   the   absence   of   such   clause   (Triple   Eight),   the   courts   of  
G.R.  No.  129584  |  3  December  1998   the   forum   will   not   enforce   any   foreign   claim   obnoxious   to   the   forum’s   public  
  policy  (Triple  Eight).  
DOCTRINE:   The   rule   simply   prescribes   a   “certification   by   a   competent   public    
health   authority”   and   not   a   “Philippine   public   health   authority.”   The   GR:  Choice  of  law  governs  
requirement   for   a   medical   certificate   under   Article   284   of   the   Labor   Code   EXC:  1.  Center  of  gravity/sufficient  contacts  with  the  law.  
cannot   be   dispensed   with;   otherwise,   it   would   sanction   the   unilateral   and   2.  Obnoxious  to  public  policy.  
arbitrary   determination   by   the   employer   of   the   gravity   or   extent   of   the    
employee’s  illness  and  thus  defeat  the  public  policy  on  the  protection  of  labor.   FACTS:  
  In   1992,   Osdana   was   recruited   by   Triple   Eight   for   employment   with   Gulf  
The   law   of   the   place   where   the   contract   is   made   governs   in   this   jurisdiction.   Catering  Company  (GCC),  a  firm  based  in  the  Kingdom  of  Saudi  Arabia.  Osdana  
The  contract  was  perfected  here,  so  the  Labor  Code  applies.  The  courts  of  the   was  engaged  to  work  as  a  food  server  for  36  months  with  a  salary  of  SR550.  She  
forum  won’t  enforce  any  foreign  claim  obnoxious  to  the  forum’s  public  policy.     claims   that   she   was   required   by   Triple   Eight   to   pay   a   total   of   11,950   pesos   in  
  placement   fees,   without   receipts.   She   was   asked   to   undergo   a   medical  

 
CONFLICT  OF  LAWS                                                                                    AV  DE  TORRES   170  
ATTY.  ARIS  L.  GULAPA                                            AY  2015-­‐2016  
examination,   too.   It   also   asked   Osdana   to   sign   another   ‘Contractor   Employee   competent   public   health   authority   in   Saudi   Arabia,   thereby   heading   off   any  
Agreement’   which   provided   that   she   would   be   employed   for   12   months   as   a   complaint  for  illegal  dismissal.  
ND  
waitress  with  a  salary  of  $280.  This  2 employment  agreement  was  approved    
by  the  POEA.     The  requirement  for  a  medical  certificate  under  Article  284  of  the  Labor  Code  
  cannot   be   dispensed   with;   otherwise,   it   would   sanction   the   unilateral   and  
Osdana   left   for   Riyadh,   Saudi   and   commenced   working   for   GCC.   She   was   arbitrary   determination   by   the   employer   of   the   gravity   or   extent   of   the  
assigned  to  the  College  of  Public  Administration  of  the  Oleysha  University  and   employee’s   illness   and   thus   defeat   the   public   policy   on   the   protection   of  
was  made  to  wash  dishes,  cooking  pots,  etc.  which  were  in  relation  to  being  a   labor.    As   the   Court   observed   in   Prieto   v.   NLRC,   “The   Court   is   not   unaware   of  
‘waitress.’  She  was  made  to  work  a  12-­‐hour  shift,  without  overtime  pay.     the   many   abuses   suffered   by   our   overseas   workers   in   the   foreign   land   where  
  they   have   ventured,   usually   with   heavy   hearts,   in   pursuit   of   a   more   fulfilling  
Osdana  suffered  from  numbness  and  pain  in  her  arms,  and  had  to  be  confined   future.    Breach  of  contract,  maltreatment,  rape,  insufficient  nourishment,  sub-­‐
from   June   18   to   August   22,   and   wasn’t   paid   her   salaries.   She   was   allowed   to   human  lodgings,  insults  and  other  forms  of  debasement,  are  only  a  few  of  the  
resume   work   as   Food   Server,   where   she   worked   7   days   a   week,   but   was   not   inhumane   acts   to   which   they   are   subjected   by   their   foreign   employers,   who  
paid.  She  was  again  confined.     probably  feel  they  can  do  as  they  please  in  their  country.  While  these  workers  
  may   indeed   have   relatively   little   defense   against   exploitation   while   they   are  
She   was   re-­‐assigned   to   the   Oleysha   University,   like   her   first   assignment.   She   abroad,  that  disadvantage  must  not  continue  to  burden  them  when  they  return  
was   diagnosed   as   having   Bilateral   Carpal   Tunnel   Syndrome.   The   pain   then   to  their  own  territory  to  voice  their  muted  complaint.    There  is  no  reason  why,  
became   unbearable,   and   she   underwent   two   surgical   operations.   She   wasn’t   in  their  own  land,  the  protection  of  our  own  laws  cannot  be  extended  to  them  
nd  
given   any   work   assignments   in   between   operations.   After   her   2 operation,   in  full  measure  for  the  redress  of  their  grievances.”  
she   was   dismissed   from   work   allegedly   on   ground   of   illness,   even   after   being    
given  a  certificate  that  she  was  fit  to  work.     Petitioner  likewise  attempts  to  sidestep  the  medical  certificate  requirement  by  
  contending   that   since   Osdana   was   working   in   Saudi   Arabia,   her   employment  
Upon  her  return  to  the  Philippines,  Osdana  sought  the  help  of  Triple  Eight,  but   was   subject   to   the   laws   of   the   host   country.  Apparently,   petitioner   hopes   to  
to   no   avail.   She   was   constrained   to   file   a   complaint   before   the   POEA   for   the   make   it   appear   that   the   labor   laws   of   Saudi   Arabia   do   not   require   any  
unpaid  and  underpaid  salaries,  for  the  unexpired  portion,  and  damages.  Under   certification   by   a   competent   public   health   authority   in   the   dismissal   of  
the   Migrant   Workers   and   Overseas   Filipinos   Act,   the   case   was   transferred   to   employees  due  to  illness.  
the  arbitration  branch  of  the  NLRC  and  assigned  to  Labor  Arbiter  Canizares.  The    
labor  arbiter  ruled  in  favor  of  Osdana.  NLRC  affirmed.  MR  was  denied.     Again,  petitioner’s  argument  is  without  merit.  
   
ISSUES:   First,  established  is  the  rule  that  lex  loci  contractus  (the  law  of  the  place  where  
Whether   there   was   GAD   because   of   ruling   in   favor   of   Osdana   even   if   there   was   the   contract   is   made)   governs   in   this   jurisdiction.    There   is   no   question   that   the  
no  factual  or  legal  basis  for  the  award  (illegal  dismissal)     contract   of   employment   in   this   case   was   perfected   here   in   the   Philippines.  
Please  see  below  for  issues  regarding  Conflict  of  Laws     Therefore,   the   Labor   Code,   its   implementing   rules   and   regulations,   and   other  
  laws   affecting   labor   apply   in   this   case.    Furthermore,   settled   is   the   rule   that   the  
HELD:   courts  of  the  forum  will  not  enforce  any  foreign  claim  obnoxious  to  the  forum’s  
No.   Petitioner   entirely   misses   the   point,   as   counsel   for   private   respondent   public  policy.  Here  in  the  Philippines,  employment  agreements  are  more  than  
states   in   the   Comment.   The   rule   simply   prescribes   a   “certification   by   a   contractual   in   nature.    The   Constitution   itself,   in   Article   XIII   Section   3,  
competent   public   health   authority”   and   not   a   “Philippine   public   health   guarantees  the  special  protection  of  workers,  to  wit:  
authority.”  If,  indeed,  Osdana  was  physically  unfit  to  continue  her  employment,    
her   employer   could   have   easily   obtained   a   certification   to   that   effect   from   a  
 
CONFLICT  OF  LAWS                                                                                    AV  DE  TORRES   171  
ATTY.  ARIS  L.  GULAPA                                            AY  2015-­‐2016  
“The   State   shall   afford   full   protection   to   labor,   local   and   overseas,   organized   into  their  contract  “as  a  set  of  terms.”  By  such  reference  to  the  provisions  of  the  
and   unorganized,   and   promote   full   employment   and   equality   of   employment   foreign  law,  the  contract  does  not  become  a  foreign  contract  to  be  governed  by  
opportunities  for  all.   the   foreign   law.   The   said   law   does   not   operate   as   a   statute   but   as   a   set   of  
  contractual  terms  deemed  written  in  the  contract.  
It   shall   guarantee   the   rights   of   all   workers   to   self-­‐organization,   collective    
bargaining   and   negotiations,   and   peaceful   concerted   activities,   including   the   A   basic   policy   of   contract   is   to   protect   the   expectation   of   the   parties.   Such  
right   to   strike   in   accordance   with   law.   They   shall   be   entitled   to   security   of   party   expectation   is   protected   by   giving   effect   to   the   parties’   own   choice   of  
tenure,   humane   conditions   of   work,   and   a   living   wage.    They   shall   also   the  applicable  law.  The  choice  of  law  must,  however,  bear  some  relationship  
participate   in   policy   and   decision-­‐making   processes   affecting   their   rights   and   the  parties  or  their  transaction.  There  is  no  question  that  the  contracts  sought  
benefits  as  may  be  provided  by  law.   to   be   enforced   by   claimants   have   a   direct   connection   with   the   Bahrain   law  
x  x  x                                                                                                x  x  x                                                                          x  x  x.”   because  the  services  were  rendered  in  that  country.  
   
This   public   policy   should   be   borne   in   mind   in   this   case   because   to   allow   foreign   FACTS:  
employers   to   determine   for   and   by   themselves   whether   an   overseas   contract   This   case   is   an   offshoot   of   a   service   contract   entered   into   by   a   Filipino  
worker   may   be   dismissed   on   the   ground   of   illness   would   encourage   illegal   or   construction   firm   with   the   Iraqi   Government   for   the   construction   of   the  
arbitrary  pre-­‐termination  of  employment  contracts.   Institute   of   Physical   Therapy-­‐Medical   Center,   Phase   II,   in   Baghdad,   Iraq,   at   a  
  time  when  the  Iran-­‐Iraq  war  was  ongoing.    
106.   PHIL.   EXPORT   AND   FOREIGN   LOAN   GUARANTEE   CORP.   V.   V.P.   EUSEBIO    
CONSTRUCTION  INC.   In   a   complaint   filed   with   the   RTC   Makati,   petitioner   Philippine   Export   and  
2004   Foreign   Loan   Guarantee   Corporation   (hereinafter   Philguarantee)   sought  
  reimbursement   from   the   respondents   of   the   sum   of   money   it   paid   to   Al   Ahli  
Kyna’s  notes:   Bank  of  Kuwait  pursuant  to  a  guarantee  it  issued  for  respondent  V.P.  Eusebio  
A   and   B   entered   into   a   contract   and   was   agreed   by   both   parties   that   NY   law   Construction,  Inc.  (VPECI).    
should  apply.    
  On   November   8,   1980:   State   Organization   of   Buildings   (SOB),   Ministry   of  
Important  factors  to  consider:   Housing   and   Construction,   Baghdad,   Iraq,   awarded   the   construction   of   the  
1.  NY  law  should  not  be  against  public  policy  of  the  Philippines  (not  obnoxious   Institute   of   Physical   Therapy–Medical   Rehabilitation   Center,   Phase   II,   in  
to  public  policy)   Baghdad,   Iraq,   (Project)   to   Ajyal   Trading   and   Contracting   Company   (Ajyal),   a  
2.  There  must  be  significant  contact  with  the  law   firm  duly  licensed  with  the  Kuwait  Chamber  of  Commerce  for  ID5,416,089/046  
See  Cadalin  v.  POEA’s  Administrator   (or  about  US$18,739,668)    
   
We   read   the   overseas   employment   contracts   in   question   as   adopting   the   March  7,  1981:  3-­‐Plex  International,  Inc.  represented  by  Spouses  Eduardo  and  
provisions  of  the  Amiri  Decree  No.  23  of  1976  as  part  and  parcel  thereof.  The   Iluminada  Santos  a  local  contractor  engaged  in  construction  business,  entered  
parties   to   a   contract   may   select   the   law   by   which   it   is   to   be   governed.   In   such   a   into  a  joint  venture  agreement  with  Ajyal.  However  since  it  was  not  accredited  
case,  the  foreign  law  is  adopted  as  a  “system”  to  regulate  the  relations  of  the   under  the  Philippine  Overseas  Construction  Board  (POCB),  it  had  to  assign  and  
parties,   including   questions   of   their   capacity   to   enter   into   the   contract,   the   transfer   all   its   right   to   VPECI.   VPECI   entered   into   an   agreement   that   the  
formalities   to   be   observed   by   them,   matters   of   performance,   and   so   forth.   execution  of  the  project  will  be  under  their  joint  management.    
Instead   of   adopting   the   entire   mass   of   the   foreign   law,   the   parties   may   just    
agree   that   specific   provisions   of   a   foreign   statute   shall   be   deemed   incorporated   To  comply  with  the  requirements  of  performance  bond  of  ID271,808/610  and  

 
CONFLICT  OF  LAWS                                                                                    AV  DE  TORRES   172  
ATTY.  ARIS  L.  GULAPA                                            AY  2015-­‐2016  
an  advance  payment  bond  of  ID541,608/901,  3-­‐Plex  and  VPECI  applied  for  the   otherwise,   the   Philippine   government   ,   through   the   Philguarantee   and   the  
issuance  of  a  guarantee  with  Philguarantee,  a  government  financial  institution   Central   Bank,   would   become   instruments   of   the   Iraqi   Government   in  
empowered  to  issue  guarantees  for  qualified  Filipino  contractors  to  secure  the   consummating   a   clear   act   of   injustice   and   inequity   committed   against   a   Filipino  
performance  of  approved  service  contracts  abroad.     contractor.    
   
Subsequently,   letters   of   guarantee   were   issued   by   Philguarantee   to   the   Central  Bank  authorized  the  remittance  to  Al  Ahli  Bank.  Philguarantee  informed  
Rafidain   Bank   of   Baghdad.   Al   Ahli   Bank   of   Kuwait   was,   therefore,   engaged   to   VPECI  that  it  would  remit  US$876,564  to  Al  Ahli  Bank,  and  reiterated  the  joint  
provide  a  counter-­‐guarantee  to  Rafidain  Bank,  but  it  required  a  similar  counter-­‐ and  solidary  obligation  of  the  respondents  to  reimburse  the  Philguarantee  for  
guarantee  in  its  favor  from  the  Philguarantee.     the   advances   made   on   its   counter-­‐guarantee   but   they   failed   to   pay   so   a   case  
  was  filed  in  the  RTC.    
The  Surety  Bond  was  later  amended  to  increase  the  amount  of  coverage  from    
P6.4   million   to   P6.967   million   and   to   change   the   bank   in   whose   favor   the   RTC  and  CA:  Against  Philguarantee  since  no  cause  of  action  since  it  was  expired  
petitioner's   guarantee   was   issued,   from   Rafidain   Bank   to   Al   Ahli   Bank   of   Kuwait   because  VPECI.  Inequity  to  allow  the  Philguarantee  to  pass  on  its  losses  to  the  
SOB  and  the  joint  venture  VPECI  and  Ajyal  executed  the  service  contract  for  the   Filipino   contractor   VPECI   which   had   sternly   warned   against   paying   the   Al   Ahli  
construction   of   the   Institute   of   Physical   Therapy   –   Medical   Rehabilitation   Bank   and   constantly   apprised   it   of   the   developments   in   the   Project  
Center,   Phase   II,   in   Baghdad,   Iraq.   It   commenced   only   on   the   last   week   of   implementation.    
August  1981  instead  of  the  June  2  1981      
  ISSUE:  
Prior   to   the   deadline,   upon   foreseeing   the   impossibility   to   meet   it,   the   surety   W/N  the  Philippine  laws  should  be  applied  in  determining  VPECI's  default  in  the  
bond   was   also   extended   for   more   than   12   times   until   May   1987   and   the   performance  of  its  obligations  under  the  service  contract    
Advance   Payment   Guarantee   was   extended   three   times   more   until   it   was    
cancelled  for  reimbursement.     HELD:  
  Yes.  A  corollary  issue  is  what  law  should  be  applied  in  determining  whether  the  
On  26  October  1986,  Al  Ahli  Bank  of  Kuwait  sent  a  telex  call  to  the  petitioner   respondent   contractor   has  defaulted  in   the   performance   of   its   obligations  
demanding   full   payment   of   its   performance   bond   counter-­‐guarantee   VPECI   under   the   service   contract.    The   question   of   whether   there   is   a   breach   of   an  
requested   Iraq   Trade   and   Economic   Development   Minister   Mohammad   Fadhi   agreement,   which   includes  default  or  mora,  pertains   to   the   essential   or  
Hussein  to  recall  the  telex  call  on  the  performance  guarantee  for  being  a  drastic   intrinsic  validity  of  a  contract.  
action   in   contravention   of   its   mutual   agreement   that   (1)   the   imposition   of    
penalty  would  be  held  in  abeyance  until  the  completion  of  the  project;  and  (2)   No   conflicts   rule   on   essential   validity   of   contracts   is   expressly   provided   for   in  
the   time   extension   would   be   open,   depending   on   the   developments   on   the   our   laws.    The   rule   followed   by   most   legal   systems,   however,   is   that   the  
negotiations  for  a  foreign  loan  to  finance  the  completion  of  the  project.     intrinsic   validity   of   a   contract   must   be   governed   by   the  lex  
  contractus  or  “proper  law  of  the  contract.”  This  is  the  law  voluntarily  agreed  
VPECI   advised   the   Philguarantee   not   to   pay   yet   Al   Ahli   Bank   because   efforts   upon   by   the   parties   (the  lex   loci   voluntatis)   or   the   law   intended   by   them  
were  being  exerted  for  the  amicable  settlement  of  the  Project.  VPECI  received   either   expressly   or   implicitly   (the  lex   loci   intentionis).  The  law  selected  may  be  
another   telex   message   from   Al   Ahli   Bank   stating   that   it   had   already   paid   to   implied  from  such  factors  as  substantial  connection  with  the  transaction,  or  the  
Rafidain   Bank   the   sum   of   US$876,564   under   its   letter   of   guarantee,   and   nationality  or  domicile  of  the  parties.  Philippine  courts  would  do  well  to  adopt  
demanding  reimbursement  by  Philguarantee.     the   first   and   most   basic   rule   in   most   legal   systems,   namely,  to   allow   the   parties  
  to  select  the  law  applicable  to  their  contract,  subject  to  the  limitation  that  it  is  
VPECI   requested   the   Central   Bank   to   hold   in   abeyance   the   payment   by   the   not  against  the  law,  morals,  or  public  policy  of  the  forum  and  that  the  chosen  
Philguarantee   "to   allow   the   diplomatic   machinery   to   take   its   course,   for   law  must  bear  a  substantive  relationship  to  the  transaction.  
 
CONFLICT  OF  LAWS                                                                                    AV  DE  TORRES   173  
ATTY.  ARIS  L.  GULAPA                                            AY  2015-­‐2016  
  particular  issue,  has  the  most  significant  relationship  to  the  occurrence  
It   must   be   noted   that   the   service   contract   between   SOB   and   VPECI   contains   no   and  the  parties.  
express   choice   of   the   law   that   would   govern   it.    In   the  United    
States  and  Europe,  the  two  rules  that  now  seem  to  have  emerged  as  “kings  of   3.  STATE-­‐INTEREST  ANALYSIS  
the   hill”   are   (1)   the   parties   may   choose   the   governing   law;   and   (2)   in   the   Where  the  forum  can  reasonably  assert  an  interest  in  the  application  
absence  of  such  a  choice,  the  applicable  law  is  that  of  the  State  that  “has  the   of   its   law   and   policy,   as   against   the   interest   of   another   State,   the  
most   significant   relationship   to   the   transaction   and   the   parties.”   Another   forum  should  apply  its  own  internal  law.  
authority  proposed  that  all  matters  relating  to  the  time,  place,  and  manner  of    
performance  and  valid  excuses  for  non-­‐performance  are  determined  by  the  law   4.  CAVER’S  PRINCIPLES  OF  PREFERENCE    
of   the   place   of   performance   or  lex   loci   solutionis,  which   is   useful   because   it   is   When  a  state  has  no  codal  or  statutory  provision  as  to  the  law  that  should  
undoubtedly  always  connected  to  the  contract  in  a  significant  way.   regulate  the  question  of  tort  liability  in  conflicts  cases,  its  courts  should  be  
  guided  by  certain  rules  or  principles  in  determining  which  of  the  conflicting  
In   this   case,   the   laws   of  Iraq  bear   substantial   connection   to   the   transaction,   rules   of   two   or   more   affected   states   should   apply   to   an   alleged   tort,   in  
since  one  of  the  parties  is  the  Iraqi  Government  and  the  place  of  performance   every  case  where  there  is  a  true  unavoidable  conflict.  
is   in  Iraq.    Hence,   the   issue   of   whether   respondent   VPECI   defaulted   in   its    
obligations  may  be  determined  by  the  laws  of  Iraq.    However,  since  that  foreign   THE   PHILIPPINES   FOLLOWS   THE   “STATE   OF   THE   MOST   SIGNIFICANT  
law   was   not   properly   pleaded   or   proved,   the   presumption   of   identity   or   RELATIONSHIP”  RULE  AS  ENUNCIATED  IN  SAUDI  ARABIAN  AIRLINES  V.  CA.  
similarity,   otherwise   known   as   the  processual   presumption,  comes   into    
play.    Where  foreign  law  is  not  pleaded  or,  even  if  pleaded,  is  not  proved,  the   111.  SAUDI  ARABIAN  AIRLINES  V.  CA,  supra  
presumption  is  that  foreign  law  is  the  same  as  ours.    
  DOCTRINES:   In  applying  the  “State  of  the  most  significant  relationship”  rule,  the  
XII.  TORT  AND  CRIMES   following   contacts   are   to   be   taken   into   account   and   evaluated   according   to  
  their   relative   importance   with   respect   to   the   particular   issue:   (a)   the   place  
  where  the  injury  occurred;  (b)  the  place  where  the  conduct  causing  the  injury  
LIABILITY  AND  DAMAGES  FOR  TORTS  IN  GENERAL  –  LEX  LOCI  DELICTI  COMMISSI   occurred;   (c)   the   domicile,   residence,   nationality,   place   of   incorporation   and  
(LAW  OF  THE  PLACE  WHERE  THE  DELICT  WAS  COMMITTED)   place  of  business  of  the  parties;  and  (d)  the  place  where  the  relationship,  if  any,  
  between  the  parties  is  centered.  
MODERN  THEORIES  AND  RULES  ON  TORT  LIABILITY  [SECS]    
  That  certain  acts  or  parts  of  the  injury  allegedly  occurred  in  another  country  is  
1.  THE  GERMAN  RULE  OF  ELECTIVE  CONCURRENCE   of  no  moment.  What  is  important  here  is  the  place  where  the  over-­‐all  harm  or  
Tort   is   committed   in   both   the   place   where   the   actor   engages   in   his   the  totality  of  the  alleged  injury  to  the  person,  reputation,  social  standing  and  
conduct   and   in   the   place   where   the   effects   of   his   conduct   occur.   The   human  rights  of  complainant,  had  lodged,  according  to  Morada.  
injured  person  may  choose  to  sue  under  one  law  or  the  other;  he  can    
elect   the   law   most   advantageous   to   his   demand,   but   he   is   not   (1)   What   legal   system   should   control   a   given   situation   where   some   of   the  
permitted  to  cumulate  the  benefits  flowing  from  more  than  one  law.   significant  facts  occurred  in  two  or  more  states;  and  (2)  to  what  extent  should  
  the  chosen  legal  system  regulate  the  situation.  
2.  THE  “STATE  OF  THE  MOST  SIGNIFICANT  RELATIONSHIP”  RULE    
The  rights  and  liabilities  of  the  parties  with  respect  to  an  issue  in  tort   FACTS:  
are   determined   by   the   local   law   of   the   state   which,   with   respect   to   the   In   1988,   SAUDIA   hired   Milagros   Morada   as   a   Flight   Attendant   for   its   airlines  

 
CONFLICT  OF  LAWS                                                                                    AV  DE  TORRES   174  
ATTY.  ARIS  L.  GULAPA                                            AY  2015-­‐2016  
based   in   Jeddah,   Saudi   Arabia.   In   1990,   while   on   a   lay-­‐over   in   Jakarta,   flight  and  took  away  her  passport  and  told  her  to  remain  in  Jeddah,  at  the  crew  
Indonesia,   Morada   went   to   a   disco   dance   with   fellow   crew   members   Thamer   quarters,   until   further   orders.   After   a   few   days,   a   SAUDIA   legal   officer   again  
Al-­‐Gazzawi   and   Allah   Al-­‐Gazzawi,   both   Saudi   nationals.   Because   it   was   almost   escorted  plaintiff  to  the  same  court  where  the  judge,  to  her  astonishment  and  
morning   when   they   returned   to   their   hotels,   they   agreed   to   have   breakfast   shock,   rendered   a   decision   sentencing   her   to   5   months   imprisonment   and   to  
together  at  the  room  of  Thamer.  When  they  were  in  the  room,  Allah  left  and   286   lashes.   Only   then   did   she   realize   that   the   Saudi   court   had   tried   her,  
shortly   after,   Thamer   attempted   to   rape   her.   Fortunately,   a   roomboy   and   together  with  Thamer  and  Allah,  for  what  happened  in  Jakarta.  The  court  found  
several  security  personnel  heard  her  cries  for  help  and  rescued  her.  Later,  the   Morada  guilty  of  (1)  adultery;  (2)  going  to  a  disco,  dancing  and  listening  to  the  
Indonesian  police  came  and  arrested  Thamer  and  Allah  Al-­‐Gazzawi,  the  latter  as   music   in   violation   of   Islamic   laws;   and   (3)   socializing   with   the   male   crew,   in  
an  accomplice.   contravention  of  Islamic  tradition.”  
   
When   plaintiff   returned   to   Jeddah   a   few   days   later,   SAUDIA   officials   She  was  denied  any  assistance  by  SAUDIA  thus,  she  then  asked  the  Philippine  
interrogated   her   about   the   Jakarta   incident.   They   then   requested   her   to   go   Embassy  in  Jeddah  to  help  her  while  her  case  is  on  appeal.  Meanwhile,  to  pay  
back   to   Jakarta   to   help   arrange   the   release   of   Thamer   and   Allah.   Morada   did   for   her   upkeep,   she   worked   on   the   domestic   flight   of   SAUDIA,   while   Thamer  
not   cooperate   in   the   negotiation   with   the   Indonesian   police   because   she   was   and   Allah   continued   to   serve   in   the   international   flights.   Because   she   was  
afraid  that  she  might  be  tricked  into  something  she  did  not  want  because  of  her   wrongfully  convicted,  the  Prince  of  Makkah  dismissed  the  case  against  her  and  
inability   to   understand   the   local   dialect.   She   also   declined   to   sign   a   blank   paper   allowed  her  to  leave  Saudi  Arabia.  Shortly  before  her  return  to  Manila,  she  was  
and   a   document   written   in   the   local   dialect.   However,   she   learned   that,   terminated   from   the   service   by   SAUDIA,   without   her   being   informed   of   the  
through   the   intercession   of   the   Saudi   Arabian   government,   the   Indonesian   cause.  
authorities   agreed   to   deport   Thamer   and   Allah   after   2   weeks   of   detention.    
Eventually,  they  were  again  put  in  service  by  defendant  SAUDIA.  In  September   In  November  1993,  Morada   filed   a   Complaint   for  damages   against   SAUDIA,   and  
1990,  SAUDIA  transferred  Morada  to  Manila.   Khaled   Al-­‐Balawi,   its   country   manager   in   the   trial   court.   SAUDIA   filed   an  
  Omnibus   MTD   which   raised   the   following   grounds,   to   wit:   (1)   that   the  
In  1992,  Morada’s  superiors  requested  her  to  see  Mr.  Ali  Meniewy,  Chief  Legal   Complaint   states   no   cause   of   action   against   SAUDIA;   (2)   that   defendant   Al-­‐
Officer  of  SAUDIA,  in  Jeddah,  Saudi  Arabia.  When  she  saw  him,  he  brought  her   Balawi  is  not  a  real  party  in  interest;  (3)  that  the  claim  or  demand  set  forth  in  
to   the   police   station   where   the   police   took   her   passport   and   questioned   her   the   Complaint   has   been   waived,   abandoned   or   otherwise   extinguished;   and   (4)  
about  the  Jakarta  incident.  Not  until  she  agreed  to  do  so  did  the  police  return   that  the  trial  court  has  no  jurisdiction  to  try  the  case  on  the  basis  of  Art.  21  of  
her  passport  and  allowed  her  to  catch  the  afternoon  flight  out  of  Jeddah.   the  NCC,  since  the  proper  law  applicable  is  the  law  of  the  KSA.  The  trial  court  
  denied  the  MTD.  
One   year   and   a   half   later,   in   Riyadh,   a   few   minutes   before   the   departure   of   her    
flight   to   Manila,   Morada   was   not   allowed   to   board   the   plane   and   instead   Consequently,   SAUDIA   filed   its   Petition   for   Certiorari   and   Prohibition   with  
ordered   to   take   a   later   flight   to   Jeddah   to   see   Mr.   Miniewy   again.   When   she   Prayer   for   Issuance   of   Writ   of   Preliminary   Injunction   and/or   TRO   with   the   CA,  
did,   she   was   brought   her   to   a   Saudi   court   where   she   was   asked   to   sign   a   which   ruled   that   the   Philippines   is   an   appropriate   forum   considering   that   the  
document  written  in  Arabic.  They  told  her  that  this  was  necessary  to  close  the   Amended  Complaint’s  basis  for  recovery  of  damages  is  Art.  21,  and  thus,  clearly  
case  against  Thamer  and  Allah.  As  it  turned  out,  plaintiff  signed  a  notice  to  her   within  the  jurisdiction  of  respondent  Court.  
to  appear  before  the  court  on  June  27,  1993.  She  then  returned  to  Manila.    
  ISSUE:  
Shortly   afterwards,   SAUDIA   summoned   plaintiff   to   report   to   Jeddah   once   again   Whether  the  Philippine  Law  should  govern.  
where  a  Saudi  judge  interrogated  her  through  an  interpreter  about  the  Jakarta    
incident.   After   one   hour   of   interrogation,   when   she   was   about   to   return   to   HELD:  
Manila,   a   SAUDIA   officer   told   her   that   the   airline   had   forbidden   her   to   take   Yes.   As   to   the   choice   of   applicable   law,   we   note   that   choice-­‐of-­‐law   problems  
 
CONFLICT  OF  LAWS                                                                                    AV  DE  TORRES   175  
ATTY.  ARIS  L.  GULAPA                                            AY  2015-­‐2016  
seek  to  answer  two  important  questions:  (1)  What  legal  system  should  control   or   done.   The   lex   fori—the   law   of   the   forum—is   particularly   important  
a  given  situation  where  some  of  the  significant  facts  occurred  in  two  or  more   because,  as  we  have  seen  earlier,  matters  of  ‘procedure’  not  going  to  the  
states;   and   (2)   to   what   extent   should   the   chosen   legal   system   regulate   the   substance  of  the  claim  involved  are  governed  by  it;  and  because  the  lex  
situation.   fori   applies   whenever   the   content   of   the   otherwise   applicable   foreign  
  law  is  excluded  from  application  in  a  given  case  for  the  reason  that  it  falls  
Before   a   choice   can   be   made,   it   is   necessary   for   us   to   determine   under   what   under  one  of  the  exceptions  to  the  applications  of  foreign  law;  and  
category   a   certain   set   of   facts   or   rules   fall.   This   process   is   known   as   (8)   the   flag   of   a   ship,   which   in   many   cases   is   decisive   of   practically   all  
“characterization,”   or   the   “doctrine   of   qualification.”   It   is   the   “process   of   legal  relationships  of  the  ship  and  of  its  master  or  owner  as  such.  It  also  
deciding  whether  or  not  the  facts  relate  to  the  kind  of  question  specified  in  a   covers  contractual  relationships  particularly  contracts  of  affreightment.  
conflicts   rule.”   The   purpose   of   “characterization”   is   to   enable   the   forum   to    
select  the  proper  law.   Considering  that  the  complaint  in  the  court  a  quo  is  one  involving  torts,  the  
  “connecting  factor”  or  “point  of  contact”  could  be  the  place  or  places  where  
Our   starting   point   of   analysis   here   is   not   a   legal   relation,   but   a   factual   the   tortious   conduct   or   lex   loci   actus   occurred.   And   applying   the   torts  
situation,  event,  or  operative  fact.  An  essential  element  of  conflict  rules  is  the   principle   in   a   conflicts   case,   we   find   that   the   Philippines   could   be   said   as   a  
indication  of  a  “test”  or  “connecting  factor”  or  “point  of  contact.”  Choice-­‐of-­‐ situs   of   the   tort   (the   place   where   the   alleged   tortious   conduct   took   place).  
law   rules   invariably   consist   of   a   factual   relationship   (such   as   property   right,   This   is   because   it   is   in   the   Philippines   where   SAUDIA   allegedly   deceived  
contract   claim)   and   a   connecting   factor   or   point   of   contact,   such   as   the   situs   of   Morada,   a   Filipina   residing   and   working   here.   According   to   her,   she   had  
the   res,   the   place   of   celebration,   the   place   of   performance,   or   the   place   of   honestly   believed   that   SAUDIA   would,   in   the   exercise   of   its   rights   and   in   the  
wrongdoing.   performance  of  its  duties,  “act  with  justice,  give  her  due  and  observe  honesty  
  and   good   faith.”   Instead,   SAUDIA   failed   to   protect   her.   That   certain   acts   or  
Note  that  one  or  more  circumstances  may  be  present  to  serve  as  the  possible   parts  of  the  injury  allegedly  occurred  in  another  country  is  of  no  moment.  What  
test  for  the  determination  of  the  applicable  law.  These  “test   factors”  or  “points   is   important   here   is   the   place   where   the   over-­‐all   harm   or   the   totality   of   the  
of  contact”  or  “connecting  factors”  could  be  any  of  the  following:   alleged  injury  to  the  person,  reputation,  social  standing  and  human  rights  of  
  complainant,  had  lodged,  according  to  Morada.  All  told,  it  is  not  without  basis  
(1)   the   nationality   of   a   person,   his   domicile,   his   residence,   his   place   of   to  identify  the  Philippines  as  the  situs  of  the  alleged  tort.  
sojourn,  or  his  origin;    
(2)  the  seat  of  a  legal  or  juridical  person,  such  as  a  corporation;   Moreover,  with  the  widespread  criticism  of  the  traditional  rule  of  lex  loci  delicti  
(3)  the  situs  of  a  thing,  that  is,  the  place  where  a  thing  is,  or  is  deemed  to   commissi,   modern   theories   and   rules   on   tort   liability   have   been   advanced   to  
be   situated.   In   particular,   the   lex   situs   is   decisive   when   real   rights   are   offer  fresh  judicial  approaches  to  arrive  at  just  results.  In  keeping  abreast  with  
involved;   the   modern   theories   on   tort   liability,   we   find   here   an   occasion   to   apply   the  
(4)   the   place   where   an   act   has   been   done,   the   locus   actus,   such   as   the   “State   of   the   most   significant   relationship”   rule,  which  in  our  view  should  be  
place   where   a   contract   has   been   made,   a   marriage   celebrated,   a   will   appropriate  to  apply  now,  given  the  factual  context  of  this  case.   In   determining  
signed   or   a  tort  committed.  The  lex  loci  actus  is   particularly   important  in   the  State  which  has  the  most  significant  relationship,  the  following  contacts  
contracts  and  torts;   are   to   be   taken   into   account   and   evaluated   according   to   their   relative  
(5)  the  place  where  an  act  is  intended  to  come  into  effect,  e.g.,  the  place   importance  with  respect  to  the  particular  issue:  (a)  the  place  where  the  injury  
of   performance   of   contractual   duties,   or   the   place   where   a   power   of   occurred;   (b)   the   place   where   the   conduct   causing   the   injury   occurred;   (c)   the  
attorney  is  to  be  exercised;   domicile,  residence,  nationality,  place  of  incorporation  and  place  of  business  
(6)   the   intention   of   the   contracting   parties   as   to   the   law   that   should   of  the  parties;  and  (d)  the  place  where  the  relationship,  if  any,  between  the  
govern  their  agreement,  the  lex  loci  intentionis;   parties  is  centered.  
(7)  the  place  where  judicial  or  administrative  proceedings  are  instituted    
 
CONFLICT  OF  LAWS                                                                                    AV  DE  TORRES   176  
ATTY.  ARIS  L.  GULAPA                                            AY  2015-­‐2016  
With  these  guidelines  in  mind,  the  trial  court  must  proceed  to  try  and  adjudge   clashes   between   our   laws   and   those   of   other   nations   which   could   result   in  
the   case   in   the   light   of   relevant   Philippine   law,   with   due   consideration   of   the   international   discord."   As   an   exception,   a   local   law   may   be   applied  
foreign  element  or  elements  involved.  Nothing  said  herein,  of  course,  should  be   extraterritorially,   when   a   statute   gives   a   clear   indication   of   an   extraterritorial  
construed  as  prejudging  the  results  of  the  case  in  any  manner  whatsoever.   application.    
   
108.  FILARTIGA  V.  PENS-­‐IRALA   Nothing   in   the   ATS's   text   evinces   a   clear   indication   of   extraterritorial   reach.  
630  F.  2d  876  (2d  Cir,  1980)   Violations   of   the   law   of   nations   affecting   aliens   can   occur   either   within   or  
  outside  the  United  States.  And  generic  terms,  like  "any"  in  the  phrase  "any  civil  
DOCTRINE:   Under   the   Alien   Tort   Statute,   which   grants   district   courts   original   action,"  do  not  rebut  the  presumption  against  extraterritoriality.  
jurisdiction  to  hear  tort  claims  brought  by  an  alien  that  have  been  "committed    
in  violation  of  the  law  of  nations  or  a  treaty  of  the  United  States."  *1.  Law  of   FACTS:  
nations  or  2.  Treaty  of  the  US   Dr.  Filartiga's  17y/o  son  Joelito  was  kidnapped  and  tortured  to  death  by  Pena,  
  former   Police   Inspector   General   in   Paraguay.   Filartiga   (P),   a   long   standing  
Official   torture   had   been   prohibited   by   the   law   of   nations.   The  prohibition  was   opponent  of  Pres.  Stroessner,  claims  this  was  done  in  retaliation  for  his  father's  
clear   and   unambiguous   and   admitted   no   distinction   between   treatment   of   political   activities   and   beliefs.   P   brought   a   criminal   case   in   Paraguayan   court,  
aliens   and   citizens.  The  court  determined  that  deliberate  torture  perpetuated   but   his   attorney   was   arrested,   threatened   with   death,   and   supposedly  
under   color   of   official   authority   violated   universally   accepted   norms   of   the   disbarred   without   just   cause.   Four   years   later,   another   man   confessed   to   the  
international  law  of  human  rights,  regardless  of  the  nationality  of  the  parties.   murder,  claiming  he  found  Joelito  and  his  wife  together,  and  said  the  crime  was  
  one  of  passion,  but  he  was  never  convicted,  and  also  the  evidence  showed  that  
Under   international   law,   any   state   that   engages   in   official   torture   violates   jus   Joelito's  death  "was  the  result  of  professional  methods  of  torture."    
cogens.    
  In  1978,  Dolly  Filártiga  and  (separately)  D  (Peña)  came  to  the  US.  Dolly  applied  
*N.B.:   for   political   asylum,   while   Peña   stayed   under   a   visitor's   visa.   Dolly   learned   of  
Torts  à  UN  Declaration  à  Customary  law   Peña's  presence  and  reported  it  to  the  Immigration  and  Naturalization  Service,  
  who   arrested   and   deported   Peña   for   staying   past   the   expiration   of   his   visa.  
Kyna’s   notes:   But   see   Kiobel   v.   Royal   Dutch.   The   latter   case   limited   the   When   Peña   was   taken   to   the   Brooklyn   Navy   Yard   pending   deportation,   Dolly  
application   of   Filartiga   case.   The   Court   in   Kiobel   held   that   corporate   liability   lodged   a   civil   complaint   in   U.S.   courts   for   Joelito's   wrongful   death   by   torture,  
claims   are   not   actionable   under   the   Alien   Tort   Statute.   In   this   regard,   the   asking  for  damages  in  the  amount  of  $10  million.    
presumption  against  extraterritoriality  applies  to  claims  under  the  ATS.    
  ISSUE:  
Av:   Filartiga   stated   that   official   torture   is   a   violation   of   jus   cogens   It   is   Whether   U.S.   courts   can   punish   non-­‐U.S.   citizens   for   tortious   acts   committed  
prohibited  by  the  law  of  nations,  without  discrimination  between  treatment  of   outside  the  U.S.  that  were  in  violation  of  the  law  of  nations  or  any  treaties  to  
aliens   and   citizens.   Thus,   torts   claims   committed   in   a   foreign   state   against   which  the  U.S.  is  a  party.    
aliens   may   be   brought   in   the   courts   of   a   domestic   state.   HOWEVER,   Kiobel   v.    
Royal   Dutch   stated   that   the   GR   is   a  presumption   against   extraterritoriality,   that   HELD:  
is   a   local   law   does   not   apply   extraterritorially.   In   other   words,   when   a   statute   Yes.  This  case  extended  the  jurisdiction  of  United  States  courts  to  tortious  acts  
gives  no  clear  indication  of  an  extraterritorial  application,  it  has  none.  There  is  a   committed  around  the  world.    
presumption   against   extraterritoriality,   that   is   a   local   law   does   not   apply    
extraterritorially.   The   presumption   "serves   to   protect   against   unintended   The  appellants  argued  that  Peña's  actions  had  violated  wrongful  death  statutes,  

 
CONFLICT  OF  LAWS                                                                                    AV  DE  TORRES   177  
ATTY.  ARIS  L.  GULAPA                                            AY  2015-­‐2016  
the   U.N.   Charter,   the   Universal   Declaration   of   Human   Rights,   the   American   109.  IN  RE  ESTATE  OF  FERDINAND  MARCOS  
Declaration  of  the  Rights  and  Duties  of  Man,  and  other  customary  international   1994  U.S.  App.  Lexis  14796  (9th  Circ.,  1994)  
law.  Petitioner  claimed  the  U.S.  courts  had  jurisdiction  to  hear  the  case  under    
the  Alien  Tort  Statute,  which  grants  district  courts  original  jurisdiction  to  hear   OVERVIEW:   During   the   ex-­‐President   of   Philippines'   tenure,   several   people   were  
tort   claims   brought   by   an   alien   that   have   been   "committed   in   violation   of   the   allegedly  tortured,  executed,  or  disappeared  at  the  hands  of  military  personnel.  
law   of   nations   or   a   treaty   of   the   United   States."   This   case   interpreted   that   The   ex-­‐President   and   his   family   fled   to   Hawaii.   Plaintiff   subjects   of   the   ex-­‐
statute   to   grant   jurisdiction   over   claims   for   torts   committed   both   within   the   president  filed  suit,  claiming  that  they  had  been  tortured,  or  were  the  families  
United  States  and  abroad.     of   people   tortured   and   executed.   A   preliminary   injunction   was   granted   to  
  prevent   defendant   from   transferring   or   dissipating   any   assets   in   order   to  
The   U.S.   courts   eventually   ruled   in   favor   of   the   Filártigas,   rewarding   them   preserve   the   possibility   of   collecting   a   judgment.   Defendant   appealed,  
roughly  $10.4  million.  Torture  was  clearly  a  violation  of  international  law  (aka   challenging   jurisdiction   over   plaintiffs'   action   under   the   Foreign   Sovereign  
"the  law  of  nations"),  and  the  U.S.  did  have  jurisdiction  over  the  case  since  the   Immunities   Act   (FSIA),   28   U.S.C.S.   §§   1330,   1602,   and   the   Alien   Tort   Act,   28  
claim   was   lodged   when   both   parties   were   inside   the   United   States.   U.S.C.S.  §  1350,  and  contending  that  any  cause  of  action  had  abated  upon  the  
Additionally,   Peña   had   sought   to   dismiss   the   case   based   on   forum   non   ex-­‐president's  death.  On  appeal,  the  court  affirmed  the  district  court's  decision,  
conveniens   (saying   that   Paraguay   was   a   more   convenient   location   for   the   trial),   finding   that   the   district   court   had   subject   matter   jurisdiction   because   the  
but  did  not  succeed.     alleged  acts  were  not  taken  within  any  official  mandate  and  were  therefore  not  
  acts   of   an   agency   of   foreign   state   within   FSIA.   The   court   stated   that   the   district  
Official   torture   had   been   prohibited   by   the   law   of   nations.   The   prohibition   court   applied   the   correct   law   and   correctly   ruled   that   money   damages   would  
was  clear  and  unambiguous  and  admitted  no  distinction  between  treatment   be  an  inadequate  remedy.    
of   aliens   and   citizens.   The   court   determined   that   deliberate   torture    
perpetuated   under   color   of   official   authority   violated   universally   accepted   FACTS:  
norms  of  the  international  law  of  human  rights,  regardless  of  the  nationality   Shortly   after   being   deposed   as   president   of   the   Philippines   in   February   1986,  
of  the  parties.     Ferdinand   Marcos   (“Marcos”)   and   his   wife   Imelda   fled   to   Hawai‘i,   taking   with  
  them   dozens   of   crates   filled   with   gold,   jewelry,   and   cash.   President   Corazon  
Accordingly,  we  must  conclude  that  the  dictum  in  Dreyfus  v.  von  Finck,  supra,   Aquino,   who   replaced   Marcos   as   president,   created   the   Presidential  
534  F.2d  at  31,  to  the  effect  that  "violations  of  international  law  do  not  occur   Commission  on  Good  Government,  an  official  agency  charged  with  recovering  
when  the  aggrieved  parties  are  nationals  of  the  acting  state,"  is  clearly  out  of   the  assets  of  the  Republic  from  the  Marcos  family  and  its  associates.    
tune  with  the  current  usage  and  practice  of  international  law.  The  treaties  and    
accords   cited   above,   as   well   as   the   express   foreign   policy   of   our   own   These   assets,   which   have   never   been   comprehensively   identified   in   any  
government,   all   make   it   clear   that   international   law   confers   fundamental   rights   litigation,   originally   included   U.S.   and   Philippine   real   estate   holdings,   valuable  
upon  all  people  vis-­‐a-­‐vis  their  own  governments.     art  works,  cash  and  other  property  seized  by  U.S.  Customs  officials  in  Hawai‘i,  
  and   funds   in   bank   accounts   in   California   and   Switzerland.   The   Republic  
Whenever  an  alleged  torturer  was  found  and  served  with  process  by  an  alien   contends  that  the  Marcoses  and  their  associates  obtained  these  assets  through  
within  the  borders  of  the  United  States,  federal  jurisdiction  was  appropriate.   misuse   of   Marcos'   official   position,   and   Philippine   law   provides   for   the  
The  court  determined  that  its  jurisdiction  was  appropriate.  The  court  reversed   forfeiture   to   the   national   treasury   of   property   unlawfully   obtained   by   public  
the   decision   of   the   district   court,   which   dismissed   appellants'   complaint   for   officials.    
want  of  subject  matter  jurisdiction.      
  A  number  of  lawsuits  were  filed  against  the  Marcos  family  in  American  courts.  
Among  them  were  five  suits  filed  in  the  Northern  District  of  California  and  the  
District   of   Hawai‘i   by   individuals   alleging   that   they   or   their   relatives   had   been  
 
CONFLICT  OF  LAWS                                                                                    AV  DE  TORRES   178  
ATTY.  ARIS  L.  GULAPA                                            AY  2015-­‐2016  
arrested,   tortured,   or   executed   by   military   intelligence   personnel   acting   Estate  in  the  Philippines  worth  $672  million,  as  well  as  $2  million  of  the  $409  
pursuant   to   martial   law   declared   by   Marcos   in   1971.   The   district   courts   million   in   cash   that   the   Estate   had   deposited   in   Swiss   banks.   It   claimed   that   the  
dismissed   all   five   suits   on   the   ground   that   the   Act   of   State   doctrine   precluded   Republic  had  sold  $481  million  worth  of  stock,  held  in  the  Meralco  Foundation  
liability.   for  the  benefit  of  the  Estate,  and  had  appropriated  the  proceeds  to  itself.  It  also  
  asserted  that  the  Republic  and  the  Estate  entered  into  two  agreements  on  26  
In  an  appeal  of  those  decisions  to  this  court,  the  Republic  filed  an  amicus  curiae   June   1992,   to   transfer   works   of   art   from   the   United   States   to   the   National  
brief   urging   the   U.S.   courts   to   exercise   jurisdiction   over   the   human   rights   Museum  of  the  Philippines,  and  to  divide  the  Estate's  other  assets  between  the  
 
claims.   This   court   reversed   in   two   unpublished   decisions. The   human   rights   Estate  and  the  Republic.    
cases   were   subsequently   consolidated   in   the   district   court   in   Hawai‘i   and    
certified  as  a  class  action  suit  against  the  Estate.   On  12  September  1994,  the  district  court  heard  argument  on  the  motion;    the  
  Republic   appeared   specially   and   asserted   its   sovereign   immunity.   The   district  
Meanwhile,   in   a   separate   action   filed   in   the   Central   District   of   California,   the   court   the   following   day   issued   an   order   identifying   the   Republic   as   a  
Republic   sued   the   Estate   and   Imelda   Marcos,   asserting   RICO   and   pendent   state   “representative,  agent,  aider  or  abettor”  of  the  Estate,  and  subjecting  it  to  the  
law  claims,  and  seeking  the  recovery  of  $1.55  billion  allegedly  plundered  from   injunction.  On  23  September  1994,  the  Republic  filed  an  appeal  from  this  order.    
the   Philippines   treasury.   On   25   June   1986,   the   district   court   enjoined   the    
Marcoses  and  their  associates  from  disposing  of  any  assets  anywhere  in  the   On   18   January   1995,   a   jury   in   Hawai‘i   awarded   Hilao   $766   million   in  
world.  We  affirmed  the  injunction.   compensatory  damages.  On  3  February  1995,  the  district  court  entered  a  final  
  judgment,   which   included   a   permanent   injunction   against   the   Estate   and   its  
The   Republic's   suit   against   the   Estate   was   settled   in   October   1991   and   “aiders   and   abettors”   and   a   finding   that   the   Republic   is   an   aider   and   abettor   of  
dismissed   on   4   November   1991.   As   part   of   the   settlement,   the   Estate   and   the  Estate.  On  6  February  1995,  the  Republic  filed  a  notice  of  appeal  (No.  95-­‐
Imelda  Marcos  agreed  to  transfer  the  Estate  assets  impounded  by  U.S.  Customs   15259)  from  this  final  judgment.    
 
officials   in   Hawai‘i, except   for   some   personal   items   and   the   cash   in   three    
   
accounts  at  a  Los  Angeles  bank, to  the  Republic. ISSUE:  
 
  Whether  such  acts  are  covered  by  sovereign  immunity    
The   assets   in   Swiss   banks   were   not   transferred   under   the   terms   of   the    
settlement,   perhaps   because   the   Swiss   courts   had   frozen   all   Marcos   assets   in   HELD:  
Switzerland   in   1986   at   the   request   of   the   Republic   and   had   agreed   that   the   The   Estate   argues   that   the   Alien   Tort   Act   is   a   purely   jurisdictional   statute   which  
assets   would   be   returned   to   the   Philippines   if   criminal   prosecutions   against   the   does  not  provide  the  plaintiffs  a  cause  of  action.  The  Estate  contends  that  Sec.  
 
Marcos  family  in  the  Philippines  succeeded. 1350,  like  the  Sec.  1331  "arising  under"  jurisdictional  provision,  does  not  grant  
  a   cause   of   action.   (the   "Judicial   Code,   in   vesting   jurisdiction   in   the   District  
The   injunction   freezing   the   Estate's   assets   was   dissolved   as   part   of   the   Courts,   does   not   create   causes   of   action,   but   only   confers   jurisdiction   to  
settlement.   However,   the   district   court   in   Hawai‘i   granted   Hilao's   request   to   adjudicate   those   arising   from   other   sources   which   satisfy   its   limiting  
have  the  injunction  reinstated  on  19  November  1991.  We  upheld  the  injunction   provisions").  
on   16   June   1994.   In   the   meantime,   Hilao   had   won   a   favorable   liability   verdict    
on   24   September   1992.   On   23   February   1994,   the   jury   awarded   Hilao   $1.2   However,   in   contrast   to   section   1331,   "which   requires   that   an   action   'arise  
billion  in  punitive  damages.   under'   the   laws   of   the   United   States,   section   1350   does   not   require   that   the  
  action  'arise  under'  the  law  of  nations,  but  only  mandates  a  'violation  of  the  law  
On   20   July   1994,   Hilao   filed   a   motion   to   modify   the   injunction   to   identify   the   of   nations'   in   order   to   create   a   cause   of   action."   It   is   unnecessary   that  
Republic  as  an  agent,  representative,  aider  or  abettor  of  the  Estate  subject  to   international   law   provide   a   specific   right   to   sue.   International   law   "does   not  
the   injunction.   Hilao   contended   that   the   Republic   had   seized   assets   of   the   require   any   particular   reaction   to   violations   of   law....   Whether   and   how   the  
 
CONFLICT  OF  LAWS                                                                                    AV  DE  TORRES   179  
ATTY.  ARIS  L.  GULAPA                                            AY  2015-­‐2016  
United   States   wished   to   react   to   such   violations   are   domestic   questions."   110.  TRAJANO  V.  MARCOS  
"[N]othing   more   than   a   violation   of   the   law   of   nations   is   required   to   invoke   113  S.  Ct.  2959  (1993)  
section  1350."      
  DOCTRINE:   All   states   believe   that   torture   is   wrong,   all   that   engage   in   torture  
Actionable   violations   of   international   law   must   be   of   a   norm   that   is   specific,   deny  it,  and  no  state  claims  a  sovereign  right  to  torture  its  own  citizens.  Under  
universal,   and   obligatory.   See   Filartiga,   630   F.2d   at   881   ("This   'international   international  law,  any  state  that  engages  in  official  torture  violates  jus  cogens.    
tort'   must   be   one   which   is   definable,   obligatory   (rather   than   horatory),   and    
universally  condemned")   OVERVIEW:  
  The   mother,   a   foreign   citizen,   claimed   that   the   official   had   orchestrated   the  
The   allegations   in   this   case   satisfy   the   specific,   universal   and   obligatory   torture  and  death  of  her  son  in  the  Philippines,  so  the  mother  filed  a  wrongful  
standard.  "Under  international  law,  ...  official  torture  violates  jus  cogens."     death  suit  in  the  district  court.  The  district  court  entered  a  default  judgment  for  
  the   mother.   On   appeal,   the   official   challenged   the   district   court's   subject  
[T]he  right  to  be  free  from  official  torture  is  fundamental  and  universal,  a  right   matter   jurisdiction   over   the   suit,   claiming   that   the   Foreign   Sovereign  
deserving  of  the  highest  stature  under  international  law,  a  norm  of  jus  cogens.   Immunities  Act  (FSIA)  made  her  immune  from  the  suit.  The  court  held  that  the  
The   crack   of   the   whip,   the   clamp   of   the   thumb   screw,   the   crush   of   the   iron   official   was   not   acting   within   any   official   mandate   when   the   tort   occurred,   so  
maiden,   and,   in   these   more   efficient   modern   times,   the   shock   of   the   electric   the   FSIA   was   not   triggered   and   could   not   provide   subject   matter   jurisdiction  
cattle   prod   are   forms   of   torture   that   the   international   order   will   not   tolerate.   over   the   suit.   The   court,   however,   held   that   the   district   court   properly   asserted  
To   subject   a   person   to   such   horrors   is   to   commit   one   of   the   most   egregious   jurisdiction  under  the  Alien  Tort  Statute  because  the  torture  of  the  victim  was  
violations  of  the  personal  security  and  dignity  of  a  human  being.   clearly   a   violation   of   international   law,   which   triggered   the   district   court's  
  exercise  of  jurisdiction  over  the  case.  The  court  rejected  the  official's  claim  that  
(torture  is  violation  of  customary  international  law)  ("the  proscription  of  official   the   extension   of   jurisdiction   to   the   district   court   violated   the   "arising   under"  
torture  [is]  a  principle  that  is  embodied  in  numerous  international  conventions   clause  of  U.S.  Const.  art.  III.    
and   declarations,   that   is   'clear   and   unambiguous'   ...   and   about   which   there   is    
universal  agreement  'in  the  modern  usage  and  practice  of  nations'  ");  Filartiga,   FACTS:  
630  F.2d  at  880-­‐84  (prohibition  against  official  torture  is  "universal,  obligatory,   In   August   of   1977,   Ferdinand   Marcos   was   President   of   the   Philippines,   Marcos-­‐
and  definable").  The  United  States  signed  the  Convention  Against  Torture  and   Manotoc   was   the   National   Chairman   of   the   Kabataang   Baranggay,   and   Fabian  
Other   Cruel,   Inhuman   or   Degrading   Treatment   or   Punishment,   39   U.N.   GAOR   Ver   was   in   charge   of   military   intelligence.   Archimedes   Trajano   was   a   student   at  
Supp.  (No.  51),  23  I.L.M.  1027  (1987),  to  which  the  United  States  Senate  gave   the  Mapua  Institute  of  Technology.  On  the  31st  of  August,  Trajano  went  to  an  
its   advice   and   consent.   Siderman,   965   F.2d   at   716.   The   prohibition   against   open  forum  discussion  at  which  Marcos-­‐Manotoc  was  speaking.  When  Trajano  
summary  execution  or  causing  "disappearance"  is  similarly  universal,  definable,   asked   a   question   about   her   appointment   as   director   of   an   organization,   he   was  
and  obligatory.     kidnapped,   interrogated,   and   tortured   to   death   by   military   intelligence  
  personnel   who   were   acting   under   Ver's   direction,   pursuant   to   martial   law  
We  thus  join  the  Second  Circuit  in  concluding  that  the  Alien  Tort  Act,  28  U.S.C.   declared   by   Marcos,   and   under   the   authority   of   Ver,   Marcos,   and   Marcos-­‐  
Sec.   1350,   creates   a   cause   of   action   for   violations   of   specific,   universal   and   Manotoc.    
obligatory   international   human   rights   standards   which   "confer[   ]   fundamental    
rights   upon   all   people   vis-­‐a-­‐vis   their   own   governments."   Filartiga,   630   F.2d   at   After   former   Philippine   President   Ferdinand   Marcos   and   his   daughter,   Imee  
885-­‐87.10  The  plaintiffs  state  a  cause  of  action.   Marcos-­‐Manotoc,   fled   to   Hawaii   in   1986,   they   were   sued   in   federal   court   by  
  Agapita   Trajano,   a   citizen   of   the   Philippines   who   then   lived   in   Hawaii,   for   the  
torture  and  wrongful  death  of  Trajano's  son,  Archimedes,  in  the  Philippines  on  

 
CONFLICT  OF  LAWS                                                                                    AV  DE  TORRES   180  
ATTY.  ARIS  L.  GULAPA                                            AY  2015-­‐2016  
August  31,  1977.  Marcos-­‐Manotoc  did  not  appear  and  a  default  judgment  was   Marcos-­‐Manotoc   argues   that   there   is   no   extraterritorial   jurisdiction   over   civil  
entered  against  her.     actions   based   on   torture.   She   urges   that   Filartiga   has   been   undermined   by  
  intervening   acts   of   the   legislative   and   executive   branches   which   indicate   that  
The   court   concluded   that   this   violation   of   fundamental   human   rights   the   United   States   is   not   obliged   to   open   its   courts   for   the   redress   of   torture  
constitutes  a  tort  in  violation  of  the  law  of  nations  under  28  U.S.C.  §  1350,  and   occurring   in   another   country.   First,   Marcos-­‐Manotoc   points   to   the   fact   that  
awarded   damages   of   $   4.16.   1350   provides   that:   "The  district   courts   shall   when   the   Senate   ratified   the   United   Nations   Convention   Against   Torture   and  
have  original   jurisdiction  of   any   civil   action   by   an  alien  for   a  tort  only,   Other   Cruel,   Inhuman   or   Degrading   Treatment   or   Punishment,   G.A.   Res.   39/46,  
committed  in  violation  of  the  law  of  nations  or  a  treaty  of  the  United  States."   39  U.N.  GAOR  Supp.  No.  51  at  197,  U.N.  Doc.  A/RES/39/708  (1984),  reprinted  in  
  23   I.L.M.   1027   (1984),   it   attached   an   understanding   to   Article   1416   that   a   state  
On   appeal,   she   contends   that   the   district   court   lacked   subject-­‐matter   is   required   to   provide   a   private   right   of   action   only   for   torture   committed   in  
jurisdiction   under   the   Alien   Tort   Statute,   28   U.S.C.   §   1350,   and   that   the   Foreign   territory   under   its   jurisdiction.   From   this   she   infers   that   it   is   inappropriate   to  
Sovereign   Immunities   Act,   28   U.S.C.   §§   1330,   1602-­‐11,   does   not   authorize   a   rely   on   principles   of   international   law   to   give   victims   of   torture   enforcement  
federal  court  to  assert  jurisdiction,  over  actions  taken  by  a  foreign  government   rights  outside  their  own  country.  Nothing  in  the  understanding,  however,  goes  
against  its  own  citizens.   so   far.   Even   if   it   could   be   read   to   reach   transitory   torts   such   as   wrongful   death,  
  the  understanding  does  not  prohibit  the  United  States  from  providing  a  forum  
ISSUE:   for   claims   by   aliens   for   torture   occurring   elsewhere.   The   understanding,  
Whether   a   United   States   district   court   had   subject   matter   jurisdiction   over   a   accordingly,  sheds  little  light  on  the  scope  of  §  1350.  
claim  of  official  acts  of  torture  committed  outside  the  United  States      
  The   same   is   true   of   the   fact   that   the   Department   of   Justice   has   changed   its  
HELD:   position   on   whether   a   plaintiff   such   as   Trajano   has   a   cause   of   action   cognizable  
Yes.   The   district   courts   shall   have   original   jurisdiction   of   any   civil   action   by   an   in   federal   court   for   a   violation   of   international   law   condemning   torture.  
alien  for  a  tort  only,  committed  in  violation  of  the  law  of  nations  or  a  treaty  of   Marcos-­‐Manotoc   suggests   that   the   executive   branch's   withdrawal   of   support  
the  United  States.   for  Filartiga  and  the  Senate's  refusal  to  obligate  federal  courts  to  hear  actions  
  such   as   Trajano's   demonstrate   that   the   United   States   does   not   recognize   a  
There  is  no  doubt,  as  the  district  court  found,  that  causing  Trajano's  death  was   private  right  of  action  for  torture  having  no  nexus  with  the  United  States.  We  
wrongful,  and  is  a  tort.  Nor,  in  view  of  Marcos-­‐Manotoc's  default,  is  there  any   do  not  read  the  executive  branch's  flip  on  this  issue  as  signifying  so  much;  its  
dispute  that  Trajano's  death  was  caused  by  torture.  And,  as  we  have  recently   change   of   position   in   different   cases   and   by   different   administrations   is   not   a  
held,   "it   would   be   unthinkable   to   conclude   other   than   that   acts   of   official   definitive  statement  by  which  we  are  bound  on  the  limits  of  §  1350.  Rather,  we  
torture   violate   customary   international   law."   We   believe,   therefore,   that   are   constrained   by   what   §   1350   shows   on   its   face:   no   limitations   as   to   the  
Trajano's  suit  as  an  alien  for  the  tort  of  wrongful  death,  committed  by  military   citizenship  of  the  defendant,  or  the  locus  of  the  injury.  
intelligence  officials  through  torture  prohibited  by  the  law  of  nations,  is  within    
the  jurisdictional  grant  of  §  1350.   Nor   do   these   acts   by   the   Senate   and   the   Department   of   Justice   support  
  Marcos-­‐   Manotoc's   argument   that   general   principles   of   international   law   may  
Marcos-­‐Manotoc   argues,   however,   that   the   district   court   erred   in   assuming   not   provide   a   basis   for   federal   court   jurisdiction   under   §   1350.   Regardless   of  
jurisdiction  of  a  tort  committed  by  a  foreign  state's  agents  against  its  nationals   the   extent   to   which   other   principles   may   appropriately   be   relied   upon,   the  
outside   of   the   United   States,   and   having   no   nexus   to   this   country.   If   §   1350   prohibition   against   official   torture   "carries   with   it   the   force   of   a   jus   cogens  
were  construed  to  confer  jurisdiction  under  these  circumstances,  she  asserts,  it   norm,"   which   "   'enjoy[s]   the   highest   status   within   international   law.'   As   our  
would   exceed   the   constitutional   limits   on   federal   court   jurisdiction   under   survey   of   the   scholarly   and   judicial   opinion   in   Siderman   reflects,   there   is  
Article  III  of  the  Constitution.  We  disagree.   widespread   agreement   on   this;   "all   states   believe   [torture]   is   wrong,   all   that  
  engage   in   torture   deny   it,   and   no   state   claims   a   sovereign   right   to   torture   its  
 
CONFLICT  OF  LAWS                                                                                    AV  DE  TORRES   181  
ATTY.  ARIS  L.  GULAPA                                            AY  2015-­‐2016  
own  citizens.  Under  international  law,  any  state  that  engages  in  official  torture   The   district   court's   approach   comports   with   the   view   that   the   First   Congress  
violates  jus  cogens."  We  therefore  conclude  that  the  district  court  did  not  err  in   enacted   the   predecessor   to   §   1350   to   provide   a   federal   forum   for   transitory  
founding   jurisdiction   on   a   violation   of   the   jus   cogens   norm   prohibiting   official   torts   (a   tort   action   which   follows   the   tortfeasor   wherever   he   goes),   see  
torture.   Filartiga,   630   F.2d   at   885   (tracing   transitory   tort   doctrine   to   1774   decision   of  
  Lord   Mansfield),   whenever   such   actions   implicate   the   foreign   relations   of   the  
Marcos-­‐Manotoc   finally   argues   that   the   district   court's   interpretation   of   §   1350   United  States.  See  Banco  Nacional  de  Cuba  v.  Sabbatino,  376  U.S.  398,  427  n.  
would   open   the   floodgates   to   "foreign"   cases   in   the   federal   courts.   She   also   25,  84  S.Ct.  923,  940  n.  25,  11  L.Ed.2d  804  (1964)  (citing  §  1350  as  example  of  
suggests  that,  contrary  to  the  original  purpose  behind  §  1350,  to  permit  cases   congressional   intent   to   make   claims   implicating   foreign   affairs   cognizable   in  
of   this   sort   would   invite,   rather   than   avoid,   controversy   with   foreign   nations.   federal  courts);  ("As  best  we  can  tell,  the  aim  of  section  1350  was  to  place  in  
We   do   not   share   these   concerns   in   this   case.   As   Siderman   makes   clear,   the   federal  court  actions  potentially  implicating  foreign  affairs.  The  intent  was  not  
prohibition   against   official   torture   occupies   a   uniquely   high   status   among   to   provide   a   forum   that   otherwise   would   not   exist   ...   but   to   provide   an  
norms   of   international   law.   The   Philippine   government   has   no   objection   to   a   alternative   forum   to   state   courts.").   The   district   court's   approach   also   allows  
United   States   District   Court's   entertaining   Trajano's   claim,   so   there   can   be   no   the  "law  of  nations"  and  "treaty"  prongs  of  §  1350  to  be  treated  consistently,  in  
unwarranted  interference  with  its  domestic  affairs.     that  the  cause  of  action  comes  from  municipal  tort  law  and  not  from  the  law  of  
  nations  or  treaties  of  the  United  States.  This  avoids  the  anomalous  result  which  
For   these   reasons,   subject-­‐matter   jurisdiction   was   not   inappropriately   troubled  Judge  Bork  in  Tel-­‐Oren,  that  whereas  Filartiga  found  a  private  right  of  
exercised   under   §   1350   even   though   the   actions   of   Marcos-­‐Manotoc   which   action   by   implying   it   from   principles   of   international   law,   no   private   cause   of  
caused  a  fellow  citizen  to  be  the  victim  of  official  torture  and  murder  occurred   action  can  ever  be  implied  from  a  non-­‐self-­‐executing  treaty.  
outside  of  the  United  States.    
  For   these   reasons   we   affirm   the   judgment   in   Trajano's   favor.   Her   suit   as   an  
At   most,   Marcos-­‐Manotoc   argues,   the   district   court   had   jurisdiction   under   §   alien  against  Marcos-­‐Manotoc  for  having  caused  the  wrongful  death  of  her  son,  
1350   to   determine   whether   Trajano   had   a   separate,   substantive   cause   of   by   official   torture   in   violation   of   a   jus   cogens   norm   of   international   law,  
action;   none   exists,   she   contends,   because   neither   the   treaties   set   out   in   the   properly   invokes   the   subject-­‐matter   jurisdiction   of   the   federal   courts   under   §  
complaint   nor   the   law   of   nations   provides   a   private   cause   of   action.   Thus,   to   1350.  
the  extent  the  court's  decision  relies  upon  either  treaties  or  international  law,    
Marcos-­‐Manotoc  submits  it  is  erroneous.   107.  SS  LOTUS  CASE  
  P.C.I.J.  SER.  A,  NO.  10,  P.  4  (1927)  
The  district  court  in  fact  agreed  with  Marcos-­‐Manotoc  that  §  1350  is  simply  a    
jurisdictional   statute   and   creates   no   cause   of   action   itself.   It   proceeded   to   OCCURENCES  IN  HIGH  SEAS  
determine   damages   on   default   under   Philippine   law.   From   this   we   assume   that    
the   court   did   not   rely   on   treaties   or   international   law   to   provide   the   cause   of   PREVAILING   DOCTRINE:   Only   the   flag   state   or   the   state   of   which   the   alleged  
action,  only  to  establish  federal  jurisdiction.  Indeed,  the  complaint  alleges  that   offender   was   a   national   had   jurisdiction   over   sailors   regarding   incidents  
Trajano's   claims   arise   under   wrongful   death   statutes,   as   well   as   international   occurring  in  high  seas.  (High  Seas  Convention)  
law.   Since   Marcos-­‐Manotoc's   appeal   is   only   to   the   extent   the   district   court    
founded  Trajano's  right  to  sue  on  treaties  or  the  law  of  nations,  it  lacks  merit   Kyna’s  Notes:  
because  the  tort  is  admitted.  That  it  was  committed  in  violation  of  international   Basic   Ratio   for   the   principle   upheld   in   this   case:   International   law   is   based   on  
law  supplies  the  jurisdictional  key  to  federal  court  under  §  1350.  We  cannot  say   the  consent  of  states.  
the  district  court  erred.   The  Lotus  principle  or  Lotus  approach,   usually   considered   a   foundation  
  of  international  law,  says  that  sovereign   states  may   act   in   any   way   they   wish  

 
CONFLICT  OF  LAWS                                                                                    AV  DE  TORRES   182  
ATTY.  ARIS  L.  GULAPA                                            AY  2015-­‐2016  
so   long   as   they   do   not   contravene   an   explicit   prohibition.  The  application  of   question   is   one   for   which   Turkish   law   prescribes   a   penalty   involving   loss   of  
this  principle  –  an  outgrowth  of  the  Lotus  case  –  to  future  incidents  raising  the   freedom   for   a   minimum   period   of   three   years;  "(2)   there   is   no   extradition  
issue  of  jurisdiction  over  people  on  the  high  seas  was  changed  by  article  11  of   treaty  or  that  extradition  has  not  been  accepted  either  by  the  government  of  
the  1958  High  Seas  Convention.  The  convention,  held  in  Geneva,  laid  emphasis   the   locality   where   the   guilty   person   has   committed   the   offence   or   by   the  
on  the  fact  that   only  the  flag  state  or  the  state  of  which  the  alleged  offender   government  of  his  own  country."    
was   a   national   had   jurisdiction   over   sailors   regarding   incidents   occurring   in    
high  seas.   The   French   government   protested   the   arrest   and   the   conviction   and   requested  
The   principle   has   also   been   used   in   arguments   against   the   reasons   of   that   the   case   be   transferred   to   a   French   court.   Turkey   proposed,   and   France  
the  United   States   of   America,   for   opposing   the   existence   of   the   International   agreed,  to  pose  the  following  question  to  the  Permanent  Court  of  International  
Criminal  Court  (ICC).   Justice   (PCIJ):   “(1)   Has   Turkey…   acted   in   conflict   with   the   principles   of  
  international   law—and   if   so,   what   principles—by   instituting…   criminal  
  proceedings  in  pursuance  of  Turkish  law  against  M.  Demons…?”  
FACTS:    
On  August  2,  1926,  the  S.S.  Lotus,  a  French  steamship,  collided  on  the  high  seas   The   French   government   invoked   the   1923   Convention   of   Lausanne   in   arguing  
with  the  Boz-­‐Kourt,  a  Turkish  collier.  The  Boz-­‐Kourt  split  in  two  and  sank,  and   against   Turkish   jurisdiction.   Article   15   of   the   Convention   indicated   that   “all  
eight   of   its   crew   members   were   killed.   The   Lotus   remained   to   assist   the   questions   of   jurisdiction   shall,   as   between   Turkey   and   the   other   contracting  
survivors   of   the   Boz-­‐Kourt,   including   its   captain,   Hassan   Bey,   and   then   Powers,   be   decided   in   accordance   with   the   principles   of   international   law.”  
continued   with   the   survivors   to   Constantinople.   Turkish   authorities   France   maintained   that   such   principles   precluded   criminal   jurisdiction   in   this  
subsequently   requested   that   Lieutenant   Demons,   the   officer   of   the   watch   on   case.   The   Court,   somewhat   significantly,   condensed   the   positions   of   the   parties  
board  the  Lotus  when  the  collision  occurred,  to  come  ashore  to  give  evidence.   in  the  following  way—Turkish  Courts,  in  order  to  have  jurisdiction,  should  be  
At   the   conclusion   of   the   questioning,   Turkish   authorities   placed   Demons   and   able   to   point   to   some   title   to   jurisdiction   recognized   by   international   law   in  
Hassan   Bey   under   arrest   pending   trial   on   charges   of   manslaughter.   At   trial,   favor  of  Turkey.  On  the  other  hand,  the  Turkish  Government  takes  the  view  
Demons   argued   that   the   Turkish   court   lacked   jurisdiction,   but   the   court   that  Article  15  allows  Turkey  jurisdiction  whenever  such  jurisdiction  does  not  
convicted   both   Demons   and   Hassan   Bey,   sentencing   each   to   a   term   of   come  into  conflict  with  a  principle  of  international  law.    
imprisonment.  The  decision  was  based  on  Article  6  of  the  Turkish  Penal  Code,    
Law  No.  765,  which  runs  as  follows:     ISSUE:  
  Whether  international  law  is  essentially  permissive  or  prohibitive  
[Translation]  "Any   foreigner   who,   apart   from   the   cases   contemplated   by   Article   whether  criminal  jurisdiction  does  or  does  not  exist  in  this  case.  whether  or  not  
4,  commits  an  offence  abroad  to  the  prejudice  of  Turkey  or  of  a  Turkish  subject,   the   principles   of   international   law   prevent   Turkey   from   instituting   criminal  
for  which  offence  Turkish  law  prescribes  a  penalty  involving  loss  of  freedom  for   proceedings  against  Lieutenant  Demons  under  Turkish  law.    
a   [p15]   minimum   period   of   not   less   than   one   year,   shall   be   punished   in    
accordance   with   the   Turkish   Penal   Code   provided   that   he   is   arrested   in   Turkey.   HELD:  
The   penalty   shall   however   be   reduced   by   one   third   and   instead   of   the   death   Turkey,   by   instituting   criminal   proceedings   against   Demons,   did   not   violate  
penalty,   twenty   years   of   penal   servitude   shall   be   awarded.  "Nevertheless,   in   international  law.  
such   cases,   the   prosecution   will   only   be   instituted   at   the   request   of   the    
Minister   of   Justice   or   on   the   complaint   of   the   injured   Party.  "If   the   offence   Relevant  Findings  of  the  Court:  
committed  injures  another  foreigner,  the  guilty  person  shall  be  punished  at  the   Establishing   Jurisdiction:   Does   Turkey   need   to   support   its   assertion   of  
request  of  the  Minister  of  Justice,  in  accordance  with  the  provisions  set  out  in   jurisdiction  using   an   existing   rule   of   international   law   or   is   the   mere   absence   of  
the   first   paragraph   of   this   article,   provided   however   that:  "(1)   the   article   in   a  prohibition  preventing  the  exercise  of  jurisdiction  enough?  
 
 
CONFLICT  OF  LAWS                                                                                    AV  DE  TORRES   183  
ATTY.  ARIS  L.  GULAPA                                            AY  2015-­‐2016  
The   first   principle   of   the   Lotus   case   said   that   jurisdiction   is   territorial:   A   State   cases  result  in  paralysing  the  action  of  the  courts,  owing  to  the  impossibility  of  
cannot   exercise   its   jurisdiction  outside   its   territory  unless   an   international   citing   a   universally   accepted   rule   on   which   to   support   the   exercise   of   their  
treaty   or   customary   law   permits   it  to   do   so.   This   is   what   we   called   the   first   [States’]  jurisdiction”  (para  48).  
Lotus  Principle.    
  The  PCIJ  based  this  finding  on  the  sovereign  will  of  States.  
“Now   the   first   and   foremost  restriction   imposed   by   international   law   upon   a    
State   is   that   –   failing   the   existence   of  a   permissive   rule   to   the   contrary   –   it   may   “International  law  governs  relations  between  independent  States.  The  rules  of  
not  exercise  its  power  in  any  form  in  the  territory  of  another  State.  In  this  sense   law   binding   upon   States   therefor  emanate   from   their   own   free   will   as  
jurisdiction  is  certainly  territorial;  it  cannot  be  exercised  by  a  State  outside  its   expressed   in   conventions   or   by   usages   generally   accepted   as   expressing  
territory   except   by   virtue   of   a   permissive   rule   derived   from   international   principles   of   law   and   established   in   order   to  regulate   the   relations   between  
custom  or  from  a  convention.”  (para  45)   these  co-­‐existing  independent  communities  or  with  a  view  to  the  achievement  
  of  common   aims.   Restrictions   upon   the   independence   of  States   cannot  
The   second   principle   of   the   Lotus   case:  Within   its   territory,   a   State   may   therefore  be  presumed”  
exercise   its   jurisdiction,   on   any   matter,   even   if   there   is   no   specific   rule   of    
international  law  permitting  it  to  do  so.  In  these  instances,  States  have  a  wide   [NB:  This  was  one  of  the  more  debated  aspects  of  the  judgement.  Some  argued  
measure   of   discretion,   which   is   only   limited   by   the   prohibitive   rules   of   that   the   Court   placed   too   much   emphasis   on   sovereignty   and   consent   of   States  
international  law.   (i.e.  took  a  strong  positivist  view)].  
   
“It   does   not,   however,   follow   that   international   law   prohibits   a   State   from   Criminal  Jurisdiction:  Territorial  Jurisdiction  
exercising  jurisdiction  in  its  own  territory,  in  respect  of  any  case  which  relates   France  alleged  that  the  flag  State  of  a  vessel  would  have  exclusive  jurisdiction  
to   acts   which   have   taken   place   abroad,   and   in   which   it   cannot   rely   on   some   over  offences  committed  on  board  the  ship  in  high  seas.  The  PCIJ  disagreed.  It  
permissive   rule   of   international   law.   Such   a   view   would   only   be   tenable   if   held  that  France,  as  the  flag  State,  did  not  enjoy  exclusive  territorial  jurisdiction  
international   law   contained   a   general   prohibition   to   States   to   extend   the   in   the   high   seas   in   respect   of   a   collision   with  a   vessel   carrying   the   flag   of  
application   of  their   laws   and   the   jurisdiction   of  their   courts   to   persons,   another  State  (paras  71  –  84).  The  Court  held  that  Turkey  and  France  both  have  
property  and  acts  outside  their  territory,  and  if,  as  an  exception  to  this  general   jurisdiction  in  respect  of  the  whole  incident:  i.e.  there  is  concurrent  jurisdiction.  
prohibition,   it   allowed   States   to   do   so   in   certain   specific   cases.   But   this   is   The   PCIJ  held   that   a   ship   in   the   high   seas   is   assimilated  to   the   territory   of   the  
certainly  not  the  case  under  international  law  as  it  stands  at  present.  Far  from   flag  State.  This  State  may  exercise  its  jurisdiction  over  the  ship,  in  the  same  way  
laying  down  a  general  prohibition  to  the  effect  that  States  may  not  extend  the   as  it  exercises  its  jurisdiction  over  its  land,  to  the  exclusion  of  all  other  States.  In  
application   of  their   laws   and   the   jurisdiction   of  their   courts   to   persons,   this   case,   the   Court   equated   the   Turkish   vessel   to   Turkish   territory.   In   this   case,  
property  and  acts  outside  their  territory,  it  leaves  them  in  this  respect  a  wide   the  PCIJ  held  that  the  “…  offence  produced  its  effects  on  the  Turkish  vessel  and  
measure   of   discretion,   which   is   only   limited   in   certain   cases   by   prohibitive   consequently  in  a  place  assimilated  to  Turkish  territory  in  which  the  application  
rules;  as  regards  other  cases,  every  State  remains  free  to  adopt  the  principles   of   Turkish   criminal   law   cannot   be   challenged,   even   in   regard   to   offences  
which   it   regards   as   best   and   most   suitable.   This   discretion   left   to   States   by   committed  there  by  foreigners.”  Turkey  had  jurisdiction  over  this  case.  
international  law  explains  the  great  variety  of  rules  which  they  have  been  able    
to  adopt  without  objections  or  complaints  on  the  part  of  other  States  …In  these    “If,  therefore,  a  guilty  act  committed  on  the  high  seas  produces  its  effects  on  a  
circumstances  all  that  can  be  required  of  a  State  is  that  it  should  not  overstep   vessel   flying   another   flag   or   in   foreign   territory,   the   same   principles   must  
the   limits   which   international   law   places   upon   its   jurisdiction;   within   these   be  applied  as  if  the  territories  of  two  different  States  were  concerned,  and  the  
limits,  its  title  to  exercise  jurisdiction  rests  in  its  sovereignty.”  (paras  46  and  47)   conclusion   must   therefore   be   drawn   that   there   is   no   rule   of   international   law  
This  applied  to  civil  and  criminal  cases.  If  the  existence  of  a  specific  rule  was  a   prohibiting  the  State  to  which  the  ship  on  which  the  effects  of  the  offence  have  
pre-­‐requisite   to  exercise   jurisdiction,   PCIJ   argued,   then  “it   would…in   many  
 
CONFLICT  OF  LAWS                                                                                    AV  DE  TORRES   184  
ATTY.  ARIS  L.  GULAPA                                            AY  2015-­‐2016  
taken  place  belongs,  from  regarding  the  offence  as  having  been  committed  in  
XIII.  RECOGNITION  AND  ENFORCEMENT  OF  FOREIGN  
its  territory  and  prosecuting,  accordingly,  the  delinquent.”  
   JUDGMENT  AND  FOREIGN  ARBITRAL  AWARDS  
 
The  Lotus  Case  was  also  significant  in  that  the  PCIJ  said  that  a  State  would  have    
territorial  jurisdiction,  even  if  the  crime  was  committed  outside  its  territory,  so   THE  LEX  FORI  ALWAYS  GOVERNS  RECOGNITION  AND  ENFORCEMENT.  
long  as  a  constitutive  element  of  the  crime  was  committed  in  that  State.  Today,    
we   call   this   subjective   territorial   jurisdiction.   In   order   for   subjective   territorial   RECOGNITION  
jurisdiction   to   be   established,   one   must   prove   that   the   element   of  the   crime   Foreign   judgment   is   given   the   same   effect   that   it   has   in   the   state  
and   the   actual   crime   are   entirely   inseparable;   i.e.,   if   the   constituent   element   where  it  was  rendered  with  respect  to  parties,  the  subject  matter,  and  
was  absent  –  the  crime  would  not  have  happened.   the  issues  involved.  
  Extension  to  another  state  of  the  res  judicata  effect  of  a  judgment.  
“The   offence   for   which   Lieutenant   Demons   appears   to   have   been    
prosecuted  was   an   act   –   of   negligence   or   imprudence   –   having   its   origin   on   ENFOCRMENT  
board   the   Lotus,   whilst   its   effects   made   themselves   felt   on   board   the   Boz-­‐   Firstly,  implies  the  recognition  of  a  foreign  judgment.  
Kourt.   These   two   elements   are,   legally,   entirely   inseparable,   so   much   so   that   In  addition,  affirmative  relief  is  given  to  a  party  entitled  to  it  because  of  
their  separation  renders  the  offence  non-­‐existent…  It  is  only  natural  that  each   the  foreign  judgment  
should  be  able  to  exercise  jurisdiction  and  to  do  so  in  respect  of  the  incident  as      
a  whole.  It  is  therefore  a  case  of  concurrent  jurisdiction.”   PHILIPPINES  
   
Customary  International  Law   RULE  39,  SEC.  48  
The   Lotus   case   gives   an   important   dictum  on  creating  customary   international   FOREIGN  JUDGMENTS  IN  REM  
law.   France   alleged   that   jurisdictional   questions   on  collision   cases   are   rarely   (Affects  the  personal  status,  Subject  is  property  within  the  Philippines,  
heard   in   criminal   cases   because   States   tend   to   prosecute   only   before   the   flag   Relief   demanded   is   for   excluding   a   party   from   any   interest   in   the  
State.  France  argued  that  this  absence  of  prosecutions  points  to  a  positive  rule   property)  
in   customary   law  on  collisions.The  Court   held   that   this  “…would   merely   show     =CONCLUSIVE  UPON  THE  TITLE  TO  THE  THING  
that   States   had   often,   in   practice,   abstained   from   instituting   criminal    
proceedings,  and  not  that  they  recognized  themselves  as  being  obliged  to  do  so;   FOREIGN  JUDGMENTS  IN  PERSONAM  
for  only  if  such  abstention  were  based  on  their  being  conscious  of  having  a  duty   (Where   the   complaint   does   not   involve   the   personal   status   of   the  
to   abstain   would   it   be   possible   to   speak   of   an   international   custom.   The   alleged   plaintiff  or  any  property  in  the  Philippines)  
fact  does  not  allow  one  to  infer  that  States  have  been  conscious  of  having  such   PRESUMPTIVE   EVIDENCE   OF   A   RIGHT   AS   BETWEEN   THE   PARTIES   AND  
a   duty;   on   the   other   hand,   as   will   presently   be   seen,   there   are   other   THEIR  =SUCCESSORS  IN  INTEREST  BY  SUBSEQUENT  TITLE  
circumstances   calculated   to   show   that   the   contrary   is   true.”  In   other   words,    
opinio  juris  is  reflected  in  acts  of  States  (Nicaragua  Case)  or  in  omissions  (Lotus   However,   such   foreign   judgments   may   be   repelled/impeached   by   evidence   of:  
case)   in   so   far   as   those   acts   or   omissions   are   done  following   a   belief   that   the   [JUN  COFC]  
said  State  is  obligated  by  law  to  act  or  refrain  from  acting  in  a  particular  way.   Want  of  Jurisdiction  of  the  court  
  Want  of  notice  to  the  party  
Collusion  
Fraud  
Clear  mistake  of  fact  or  law  

 
CONFLICT  OF  LAWS                                                                                    AV  DE  TORRES   185  
ATTY.  ARIS  L.  GULAPA                                            AY  2015-­‐2016  
    If   no   right   or   title   adjudged,   no   occasion   for   recognition   or  
Res  Judicata   enforcement  
A   rule   that   judgment   on   the   merits   by   a   court   having   jurisdiction   is     Demurrer  or  prescription  doesn’t  allow  r&e  
conclusive   between   the   parties   to   a   suit   as   to   all   matters   that   were   1. Final  judgment  
litigated  or  that  could  have  been  litigated  in  that  suit   Provisional  or  interlocutory  decrees  are  not  recognized  or  enforced  
  2. Fraud  
The  elements  of  res  judicata  are  as  follows:   Extrinsic   –   fraud   based   on   facts   not   controverted   or   resolved   in   the  
the  former  judgment  or  order  must  be  final   case   where   judgment   is   rendered,   or   that   which   would   go   to   the  
the  judgment  or  order  must  be  on  the  merits   jurisdiction   of   the   court   or   would   deprive   the   party   against   whom  
it  must  have  been  rendered  by  a  court  having  jurisdiction  over   judgment   is   rendered   a   chance   to   defend   the   action   to   which   he   has  
the  subject  matter  and  the  parties   meritorious  defense  
there   must   be,   between   the   first   and   the   second   action,   3. Clear  mistake  of  law  
identity  of  parties,  of  subject  matter  and  cause  of  action   Some   authors   does   not   consider   this   as   a   defense   since   there’s   nothing  
  to  show  that  the  forum  court  will  apply  the  law  better  than  the  foreign  
Policy  of  Preclusion   court  
Judgments  should  be  enforced  unless  there  are  reasons  for  denying  the   But  this  ground,  as  a  defense,  is  clearly  found  in  our  laws  so  we  have  to  
enforcement  (JUN  COFC)   follow  it  
  4. Contrary  to  public  policy  
Rationale   Should  only  be  used  under  exceptional  circumstances  
  protect  party  expectations  resulting  from  previous  litigation   Usually  intertwined  with  the  violation  of  some  law  
to  safeguard  against  the  harassment  of  defendants   5. No  court  available  in  the  forum  
to   insure   that   the   task   of   courts   not   be   increased   by   never-­‐ending   6. Penal  judgments  and  penalties  
litigation  of  the  same  disputes   Judgment  has  to  be  on  civil  or  commercial  matters  only  
to  promote  the  goal  of  all  law:  “rest  and  quietness.”   Judgment   on   collection   of   taxes   and   administrative   proceedings   are  
  also  not  allowed  
DEFENSES:  J4UN  COF2C  -­‐  PNPO   Test  of  penalties:  where  it  appears,  to  the  tribunal  which  is  called  upon  
  to  enforce  it,  to  be,  in  its  essential  character  and  effect,  a  punishment  
1.  Want  of  jurisdiction  over  the  subject  matter   of  an  offense  against  the  public,  or  a  grant  of  a  civil  right  to  a  private  
The   foreign   tribunal   must   have   had   jurisdiction   according   to   the   law   of   person  
the   forum   state   (so   if   enforcing   US   judgment,   the   American   court   must   7. Other  defenses  
have  had  jurisdiction  according  to  the  phil  law?)   The  form  state  should  only  be  involved  in  enforcing  or  recognizing  the  
2.  Want  of  jurisdiction  over  the  person   right/title,  not  fixing  the  amount  thereof  
Due   process   must   be   followed:   Valid   notice   and   Opportunity   to   be   The  judgment  must  be  for  a  sum  certain  
heard   Court  has  to  be  an  impartial  tribunal  
3.      The  action  must  be  a  judicial  or  quasi-­‐judicial  action    
Includes  arbitration  awards  because  1.  Voluntarily  submitted  to  by  the  
parties  and  2.  Enforceable  under  the  concept  of  res  judicata  (they  end  
litigations)  
4.      Judgments  on  the  merits  
  Sec.  39  =  evidence  of  a  right  or  title  
 
CONFLICT  OF  LAWS                                                                                    AV  DE  TORRES   186  
ATTY.  ARIS  L.  GULAPA                                            AY  2015-­‐2016  
4. the  award  deals  with  an   issue  not  contemplated  by  or  not  falling  within  the  
ARBITRATION  
  terms   of   the   submission   to   arbitration,   or   contains   matters   beyond   the  
  scope   of   the   arbitration   (subject   to   the   proviso   that   an   award   which  
1958   Convention   on   the   Recognition   and   Enforcement   of   Foreign   Arbitral   contains   decisions   on   such   matters   may   be   enforced   to   the   extent   that   it  
Awards  (The  New  York  Convention)   contains   decisions   on   matters   submitted   to   arbitration   which   can   be  
  separated  from  those  matters  not  so  submitted);  
Widely   considered   the   foundational   instrument   for  international   arbitration,   it   5. the   composition   of   the  arbitral   tribunal  was   not   in   accordance   with   the  
applies   to   the   recognition   and   enforcement   of   arbitral   awards   made   in   the   agreement   of   the   parties   or,   failing   such   agreement,   with   the   law   of   the  
territory  of  a  State  other  than  the  State  where  the  recognition  and  enforcement   place  where  the  hearing  took  place  (the  "lex  loci  arbitri");  
of   such   awards   are   sought,   and   arising   out   of   differences   between   persons,   6. the   award   has   not   yet   become   binding   upon   the   parties,   or   has   been   set  
whether   physical   or   legal.   It   also   applies   to   arbitrations   which   are   not   aside  or  suspended  by  a  competent  authority,  either  in  the  country  where  
considered  as  domestic  awards  in  the  state  where  recognition  and  enforcement   the   arbitration   took   place,   or   pursuant   to   the   law   of   the   arbitration  
is   sought.   Though   other   international  conventions  apply   to   the   cross-­‐border   agreement;  
enforcement   of   arbitration   awards,   the   New   York   Convention   is   by   far   the   most   7. the   subject   matter   of   the   award   was   not   capable   of   resolution   by  
important.   arbitration;  or  
  8. enforcement  would  be  contrary  to  "public  policy".  
OBJECTIVE    
PRINCIPAL:   Foreign   and   non-­‐domestic   arbitral   awards   will   not   be   THREE  TYPES  OF  RESERVATIONS:  
discriminated  against  and  it  obliges  Parties  to  ensure  such  awards  are   8. Conventional  Reservation  -­‐  some  countries  only  enforce  arbitration  awards  
recognized  and  generally  capable  of  enforcement  in  their  jurisdiction   issued  in  a  Convention  member  state  
in  the  same  way  as  domestic  awards.   9. Commercial  Reservation  –  some  countries  only  enforce  arbitration  awards  
ANCILLARY:  To  require  courts  of  Parties  to  give  full  effect  to  arbitration   that  are  related  to  commercial  transactions  
agreements   by   requiring   courts   to   deny   the   parties   access   to   court   in   10. Reciprocity  Reservation  –  some  countries  limit  application  to  awards  from  
contravention   of   their   agreement   to   refer   the   matter   to   an   arbitral   non-­‐contracting   states   such   that   they   will   only   apply   it   to   the   extent   to  
tribunal.   which  such  a  non-­‐contracting  state  grants  reciprocal  treatment.  
   
“Arbitral  awards"  shall  include  not  only  awards  made  by  arbitrators  appointed   Process  of  obtaining  recognition  or  award:  
for   each   case   but   also   those   made   by   permanent   arbitral   bodies   to   which   the   The   party   applying   for   recognition   and   enforcement   shall,   at   the   time   of   the  
parties  have  submitted.   application,  supply:  
  (a)   The   duly   authenticated   original   award   or   a   duly   certified   copy  
GR:   Under   the   Convention,   an  arbitration   award  issued   in   any   other   state   can   thereof;  
generally   be   freely   enforced   in   any   other   contracting   state,   only   subject   to   b)   The   original   agreement   referred   to   in   article   11   or   a   duly   certified  
certain,  limited  defenses.   copy  thereof.  
   
EXCS:  These  defenses  are:  [IVAN  TRIP]    
1. a   party   to   the   arbitration   agreement   was,   under   the   law   applicable   to   him,   Republic  Act  No.  9285  |  April  2,  2004  
under  some  incapacity;     ALTERNATIVE  DISPUTE  RESOLUTION  ACT  OF  2004  
2. the  arbitration  agreement  was  not  valid  under  its  governing  law;   ARTS.  42-­‐48  
3. a  party  was  not  given  proper  notice  of  the  appointment  of  the  arbitrator  or    
of  the  arbitration  proceedings,  or  was  otherwise  unable  to  present  its  case;   APPLICATION  OF  THE  NY  CONVENTION  
 
CONFLICT  OF  LAWS                                                                                    AV  DE  TORRES   187  
ATTY.  ARIS  L.  GULAPA                                            AY  2015-­‐2016  
The   New   York   Convention   shall   govern   the   recognition   and   The   losing   party   who   appeals   from   the   judgment   of   the   court  
enforcement  of  arbitral  awards  covered  by  the  said  Convention.   confirming   an   arbitral   award   shall   be   required   by   the   appellant   court  
The  recognition  and  enforcement  of  such  arbitral  awards  shall  be  filled   to  post  counterbond  executed  in  favor  of  the  prevailing  party  equal  to  
with   regional   trial   court   in   accordance   with   the   rules   of   procedure   to   the   amount   of   the   award   in   accordance   with   the   rules   to   be  
be  promulgated  by  the  Supreme  Court.   promulgated  by  the  Supreme  Court.  
   
APPLICANT  SHALL:  [TAPS]   VENUE  AND  JURISDICTION  
1. File  with  the  court  the  original  or  authenticated  copy  of  the  award   Deemed   as   special   proceedings   and   shall   be   filled   with   the   RTC   (i)  
and  arbitration  agreement   where  arbitration  proceedings  are  conducted;  (ii)  where  the  asset  to  be  
2. If   the   award   or   agreement   is   not   made   in   any   of   the   official   attached  or  levied  upon,  or  the  act  to  be  enjoined  is  located;  (iii)  where  
languages,   the   party   shall   supply   a   duly   certified   translation   any  of  the  parties  to  the  dispute  resides  or  has  his  place  of  business;  or  
thereof  into  any  of  such  languages.   (iv)   in   the   National   Judicial   Capital   Region,   at   the   option   of   the  
3. Establish  that  the  country  in  which  foreign  arbitration  award  was   applicant.  
made  is  a  party  to  the  New  York  Convention.    
4. Provide  appropriate  security  if  the  RTC  considers  it  proper   NOTICE  
  The  Court  shall  send  notice  to  the  parties  at  their  address  of  record  in  
  the  arbitration,  or  if  any  party  cannot  be  served  notice  at  such  address,  
NOT  COVERED  BY  THE  CONVENTION   at   such   party's   last   known   address.   The   notice   shall   be   sent   at   least  
The   recognition   and   enforcement   of   foreign   arbitral   awards   not   fifteen   (15)   days   before   the   date   set   for   the   initial   hearing   of   the  
covered  by  the  New  York  Convention  shall  be  done  in  accordance  with   application.  
procedural   rules   to   be   promulgated   by   the   Supreme   Court.   The   Court    
may,   on   grounds   of   comity   and   reciprocity,   recognize   and   enforce   a   99.  COMPAGNIE  DE  COMMERCE  V.  HAMBURG  AMERIKA  
non-­‐convention  award  as  a  convention  award.   G.R.  No.  L-­‐10986  |  March  31,  1917  
   
NOT  FOREIGN  JUDGMENT  EVEN  IF   Notwithstanding   the   fact   that   the   contract   was   executed   in   England,   in   the  
1. Confirmed  by  a  court  of  a  foreign  country   absence  of  averment  and  proof  that  under  the  law  of  England  compliance  with,  
2. Confirmed  by  the  RTC   or  an  offer  to  comply  with  such  a  stipulation  constitutes  a  condition  precedent  
  to  the  institution  of  judicial  proceeding  for  the  enforcement  of  the  contract.  
REJECTION    
A  party  to  a  foreign  arbitration  proceeding  may  oppose  an  application   N.B.:  
for   recognition   and   enforcement   of   the   arbitral   award   in   accordance   Ruling  in  this  case  is  no  longer  true  because  of   the  New  York  Convention  and  
with   the   procedural   rules   to   be   promulgated   by   the   Supreme   Court   Alternative   Dispute   Resolution   Act.   As   a   general   rule,   Philippine   courts   are  
only  on  those  grounds  enumerated  under  Article  V  [IVAN  TRIP]  of  the   obliged  to  recognize  and  enforce  foreign  arbitral  agreements  and  awards  only  
New   York   Convention.   Any   other   ground   raised   shall   be   disregarded   by   subject  to  certain,  limited  defenses.  [IVAN  TRIP]  
the  regional  trial  court.    
  FACTS:    
APPEAL  TO  THE  CA   HAMBURG  (German)  owned  a  steamship  named  SAMBIA,  which  proceeded  to  
A  decision  of  the  RTC  confirming,  vacating,  setting  aside,  modifying  or   the   port   of   Saigon   and   was   taking   the   cargo   belonging   to   COMPAGNIE.  
correcting   an   arbitral   award   may   be   appealed   to   the   CA   in   accordance   Apparently,   there   were   rumors   of   impending   war   between   Germany   and  
with  the  rules  of  procedure  to  be  promulgated  by  the  SC.  
 
CONFLICT  OF  LAWS                                                                                    AV  DE  TORRES   188  
ATTY.  ARIS  L.  GULAPA                                            AY  2015-­‐2016  
France  and  other  nations  of  Europe.  The  master  of  the  steamship  was  told  to   of   the   provision   in   the   charter   party   for   the   settlement   of   disputes   by   a  
take   refuge   at   a   neutral   port   (because   Saigon   was   a   French   port).   So,   to   stop   reference  to  arbitration  in  London  
that,   COMPAGNIE   asked   for   compulsory   detention   of   his   vessel   to   prevent   its    
property   from   leaving   Saigon.   However,   the   Governor   of   Saigon   refused   to   HELD:    
issue  an  order  because  he  had  not  been  officially  notified  of  the  declaration  of   No.  This  objection  to  the  jurisdiction  of  the  court  appears  for  the  first  time  in  
the  war.     defendant's   brief   on   appeal.   In   the   court   below   defendant   not   only   appeared  
  and   answered   without   objecting   to   the   court's   jurisdiction,   but   sought  
The   steamship   sailed   from   Saigon,   and   was   bound   for   Manila,   because   it   was   affirmative   relief;   and   it   is   very   clear   that   defendant   cannot   be   permitted   to  
issued   a   bill   of   health   by   the   US   consul   in   Saigon.   The   steamship   stayed   submit   the   issues   raised   by   the   pleadings   for   adjudication,   without   objection,  
continuously  in  Manila  and  where  it  contends  it  will  be  compelled  to  stay  until   and   then,   when   unsuccessful,   assail   the   court's   jurisdiction   in   reliance   upon   a  
the   war   ceases.   No   attempt   on   the   part   of   the   defendants   to   transfer   and   stipulation  in  the  charter  party  which  the  parties  were  at  entire  liberty  to  waive  
deliver   the   cargo   to   the   destinations   as   stipulated   in   the   charter   party.   That   if   they   so   desired.   We   do   not   stop   therefore   to   rule   upon   the   contention   of  
BEHN,   MEYER   and   COMPANY   (agent   of   HAMBURG   in   manila)   offered   to   opposing   counsel,   that   a   contractual   stipulation,   for   a   general   arbitration  
purchase  the  cargo  from  COMPAGNIE,  but  the  latter  never  received  the  cable   cannot  be  invoked  to  oust  our  courts  of  their  jurisdiction,  under  the  doctrine  
messages  so  they  never  answered.     announced   in   the   cases   of   Wahl   and   Wahl  v.  Donaldson,   Sims   &   Co.,   and  
  Cordoba  vs.  Conde;   and   that   this   doctrine   should   be   applied   in   the   case   at  
When   a   survey   was   done   on   the   ship,   it   was   found   that   the   cargo   was   “weevily   bar,   notwithstanding   the   fact   that   the   contract   was   executed   in   England,   in  
and  heating”  (sic),  so  BEHN  asked  for  court  authority  to  sell  the  cargo  and  the   the  absence  of  averment  and  proof  that  under  the  law  of  England  compliance  
balance  to  be  dumped  at  sea.  The  proceeds  of  the  sale  were  deposited  in  the   with,   or   an   offer   to   comply   with   such   a   stipulation   constitutes   a   condition  
court,  waiting  for  orders  as  to  what  to  do  with  it.     precedent   to   the   institution   of   judicial   proceeding   for   the   enforcement   of   the  
  contract.  
BEHN  wrote  COMPAGNIE  again  informing  the  latter  of  the  disposition  which  it  
made  upon  the  cargo.  COMPAGNIE  answered  that  it  was  still  waiting  for  orders  
as  to  what  to  do.    
 
COMPAGNIE  of  course  wanted  all  the  proceeds  of  the  sale  to  be  given  to  them  
(damages   for   the   defendants’   failure   to   deliver   the   cargo   to   the   destinations  
Dunkirk  and  Hamburg),  while  defendants  contend  that  they  have  a  lien  on  the  
proceeds   of   the   sale   (amount   due   to   them   because   of   the   upkeep   and  
maintenance  of  the  ship  crew  and  for  commissions  for  the  sale  of  the  cargo).    
 
The  trial  court  ruled  in  favor  of  the  plaintiffs.  On  appeal,  the  defendants  made  
the  ff:  assignments  on  appeal  (that  the  court  had  no  jurisdiction,  that  the  fear  
of   capture   was   not   force   majeure,   that   the   court   erred   in   concluding   that  
defendant  is  liable  for  damages  for  non-­‐delivery  of  cargo,  and  the  value  of  the  
award   of   damages).  On   appeal,   the   plaintiffs   also   contended   that   the   court  
erred  in  not  giving  the  full  value  of  damages.  
 
ISSUE:  
Whether  court  below  was  without  jurisdiction  of  the  subject-­‐matter  by  reason  
 
CONFLICT  OF  LAWS                                                                                    AV  DE  TORRES   189  
ATTY.  ARIS  L.  GULAPA                                            AY  2015-­‐2016  

Potrebbero piacerti anche